The Official Guide To GRE Revised General , 2nd Edition

The%20OFFICIAL%20Guide%20to%20REVISED%20GRE

User Manual:

Open the PDF directly: View PDF PDF.
Page Count: 577 [warning: Documents this large are best viewed by clicking the View PDF Link!]

This ebook was issued to HUAN-LIN CHANG, order #14803770223. Unlawful distribution of this ebook is prohibited.
Customer name HUAN-LIN CHANG , Order Id: 14803770223
The ETS® Personal Potential Index (ETS® PPI) is a convenient, web-based tool that
collects feedback from evaluators you select to provide information about you on
personal qualities that graduate deans and faculty have told us are important for
success, including:
6)*2'! #!) -!/%1%/3 6!(2*-& 6'))%)#) -#)%4/%*)
6*((0)%/%*)&%''. 6!.%'%!)! 6/$%.) )/!#-%/3
)!3*0-!1'0/*-.$1!+-*1% ! /$!%-"!! &3*0.!'!/(0'/%+'!!1'0/%*)."*-
%)'0.%*)%)0./*(%4! 1'0/%*)!+*-//$/2%''!.!)/*)5 !)/%''3/*/$!
institutions of your choice.
When used with GRE® scores, ETS PPI can help you stand out.
$!0)%,0!%)"*-(/%*)*)/%)! %)3*0-!.3/*0.!) %)"*-(/%1!1'0/%*)
!+*-/*(+'!(!)/./$!%)"*-(/%*)+-*1% ! 37.*-!.) /-).-%+/.) )
help graduate and business schools know youre up to the challenge.
Register for the GRE® General Test, and you can send four FREE ETS PPI Evaluation
Reports to the graduate programs of your choice!
To get started or to learn more, visit
www.ets.org/ppi/morecomplete
Show graduate programs a more
complete picture of your potential for success
Show graduate programs a
more
Sh d
This ebook was issued to HUAN-LIN CHANG, order #14803770223. Unlawful distribution of this ebook is prohibited.
Customer name HUAN-LIN CHANG , Order Id: 14803770223
The Official Guide to the GRE
®
revised General Test
The book you’re holding is a one-of-a-kind resource: the only GRE®guide created by
the ETS team that produces the actual exam. This book is uniquely suited to help you
do your best on this challenging test.
Here’s what you’ll find inside:
bTwo real, full-length GRE practice tests. Use actual tests to sharpen your skills
and build your confidence.
bTwo simulated, computer-based GRE practice tests. Experience test prep that
is as close to the actual test as you can get with the the POWERPREP®II,
Version 2.0 Software. Download the free software at www.ets.org/gre/
powerprep2/download.
bAuthentic GRE questions with answers and explanations. Practice with real
test questions created by the test makers.
bETS’s own test-taking strategies. Learn valuable hints and tips that can help
you get your best score.
bOfficial information on the GRE revised General Test. Get the facts about the
test content and structure—straight from ETS.
Welcome to
About ETS
At ETS, we advance quality and equity in education for people worldwide by creating
assessments based on rigorous research. ETS serves individuals, educational institu-
tions, and government agencies by providing customized solutions for teacher certi-
fication, English language learning, and elementary, secondary, and post-secondary
education, as well as conducting education research, analysis, and policy studies.
Founded as a nonprofit in 1947, ETS develops, administers and scores more than
50 million tests annually including the TOEFL®and TOEIC®tests, the GRE®tests,
and The Praxis Series™ assessments in more than 180 countries at over 9,000
locations worldwide. For more information, visit www.ets.org.
This ebook was issued to HUAN-LIN CHANG, order #14803770223. Unlawful distribution of this ebook is prohibited.
Customer name HUAN-LIN CHANG , Order Id: 14803770223
This page intentionally left blank
This ebook was issued to HUAN-LIN CHANG, order #14803770223. Unlawful distribution of this ebook is prohibited.
Customer name HUAN-LIN CHANG , Order Id: 14803770223
IMPORTANT
ETS makes available free test preparation materials for individuals planning to take
a GRE test. POWERPREP II software is available for individuals planning to take
the computer-based GRE revised General Test, and the Practice Book for the Paper-based
GRE revised General Test is available for individuals planning to take the paper-based test.
The information about how to prepare for the revised General Test, test-taking strategies,
question strategies, etc., that is included in the free test preparation is also included in
this Guide. The Guide also provides you with additional sets of practice questions and
two additional full-length practice tests with explanatory materials.
For more information about the GRE revised General Test, free and low-cost
GRE test preparation materials, and other GRE products and services,
please visit the GRE website at
www.ets.org/gre
Inquiries concerning the practice test questions in this book
should be sent to the GRE testing program at
GRETestQuestionInquiries@ets.org
This ebook was issued to HUAN-LIN CHANG, order #14803770223. Unlawful distribution of this ebook is prohibited.
Customer name HUAN-LIN CHANG , Order Id: 14803770223
This page intentionally left blank
This ebook was issued to HUAN-LIN CHANG, order #14803770223. Unlawful distribution of this ebook is prohibited.
Customer name HUAN-LIN CHANG , Order Id: 14803770223
revised General Test
Guide
to the
New York | Chicago | San Francisco | Lisbon | London | Madrid | Mexico City
Milan | New Delhi | San Juan | Seoul | Singapore | Sydney | Toronto
Official
GRE
The
®
2nd
Edition
This ebook was issued to HUAN-LIN CHANG, order #14803770223. Unlawful distribution of this ebook is prohibited.
Customer name HUAN-LIN CHANG , Order Id: 14803770223
Contents
How to Use This Book / Software Package . . . . . . . . . . . . . . . . . . . . . . . . . . . . . .xi
1About the
GRE
®revised General Test 1
Introduction . . . . . . . . . . . . . . . . . . . . . . . . . . . . . . . . . . . . . . . . . . . . . . . . . . . . . 1
Structure and Content of the Test . . . . . . . . . . . . . . . . . . . . . . . . . . . . . . . . . . . . 2
Preparing for the Test . . . . . . . . . . . . . . . . . . . . . . . . . . . . . . . . . . . . . . . . . . . . . . 4
Test-taking Strategies for the Computer-based Test . . . . . . . . . . . . . . . . . . . . . . 4
Test-taking Strategies for the Paper-based Test . . . . . . . . . . . . . . . . . . . . . . . . . 6
Understanding GRE Scoring . . . . . . . . . . . . . . . . . . . . . . . . . . . . . . . . . . . . . . . . 7
2
GRE
®Analytical Writing 11
Overview of the Analytical Writing Measure . . . . . . . . . . . . . . . . . . . . . . . . . . 11
Preparing for the Analytical Writing Measure . . . . . . . . . . . . . . . . . . . . . . . . . 12
General Strategies . . . . . . . . . . . . . . . . . . . . . . . . . . . . . . . . . . . . . . . . . . . . . . . 12
Analyze an Issue Task . . . . . . . . . . . . . . . . . . . . . . . . . . . . . . . . . . . . . . . . . . . . . 13
Analyze an Argument Task . . . . . . . . . . . . . . . . . . . . . . . . . . . . . . . . . . . . . . . . . 25
GRE Scoring Guide: Analyze an Issue . . . . . . . . . . . . . . . . . . . . . . . . . . . . . . . 37
GRE Scoring Guide: Analyze an Argument . . . . . . . . . . . . . . . . . . . . . . . . . . . 39
Score Level Descriptions . . . . . . . . . . . . . . . . . . . . . . . . . . . . . . . . . . . . . . . . . . 41
3
GRE
®Verbal Reasoning 43
Overview of the Verbal Reasoning Measure . . . . . . . . . . . . . . . . . . . . . . . . . . . 43
Verbal Reasoning Question Types . . . . . . . . . . . . . . . . . . . . . . . . . . . . . . . . . . . 43
This ebook was issued to HUAN-LIN CHANG, order #14803770223. Unlawful distribution of this ebook is prohibited.
Customer name HUAN-LIN CHANG , Order Id: 14803770223
4
GRE
® Verbal Reasoning
Practice Questions 143
SET 1. Discrete Questions: Easy . . . . . . . . . . . . . . . . . . . . . . . . . . . . . . . . . . . 145
SET 2. Discrete Questions: Medium . . . . . . . . . . . . . . . . . . . . . . . . . . . . . . . . 150
SET 3. Discrete Questions: Hard . . . . . . . . . . . . . . . . . . . . . . . . . . . . . . . . . . . 155
SET 4. Data Interpretation Sets . . . . . . . . . . . . . . . . . . . . . . . . . . . . . . . . . . . 161
Answer Key . . . . . . . . . . . . . . . . . . . . . . . . . . . . . . . . . . . . . . . . . . . . . . . . . . . . 164
Answers and Explanations . . . . . . . . . . . . . . . . . . . . . . . . . . . . . . . . . . . . . . . . 166
Practice Questions 53
SET 1. Discrete Questions: Easy . . . . . . . . . . . . . . . . . . . . . . . . . . . . . . . . . . . . 54
SET 2. Reading Comprehension Questions: Easy . . . . . . . . . . . . . . . . . . . . . . 56
SET 3. Discrete Questions: Medium . . . . . . . . . . . . . . . . . . . . . . . . . . . . . . . . . 60
SET 4. Reading Comprehension Questions: Medium . . . . . . . . . . . . . . . . . . . 63
SET 5. Discrete Questions: Hard . . . . . . . . . . . . . . . . . . . . . . . . . . . . . . . . . . . . 68
SET 6. Reading Comprehension Questions: Hard . . . . . . . . . . . . . . . . . . . . . . 71
Answer Key . . . . . . . . . . . . . . . . . . . . . . . . . . . . . . . . . . . . . . . . . . . . . . . . . . . . . 75
Answers and Explanations . . . . . . . . . . . . . . . . . . . . . . . . . . . . . . . . . . . . . . . . . 77
5
GRE
®Quantitative Reasoning 107
Overview of the Quantitative Reasoning Measure . . . . . . . . . . . . . . . . . . . . . 107
Quantitative Reasoning Question Types . . . . . . . . . . . . . . . . . . . . . . . . . . . . . 108
Using the Calculator . . . . . . . . . . . . . . . . . . . . . . . . . . . . . . . . . . . . . . . . . . . . . 129
Mathematical Conventions for the Quantitative Reasoning Measure
of the GRE revised General Test . . . . . . . . . . . . . . . . . . . . . . . . . . . . . . . . . 133
6
GRE
® Quantitative Reasoning
This ebook was issued to HUAN-LIN CHANG, order #14803770223. Unlawful distribution of this ebook is prohibited.
Customer name HUAN-LIN CHANG , Order Id: 14803770223
7
GRE
®Math Review 207
1. Arithmetic . . . . . . . . . . . . . . . . . . . . . . . . . . . . . . . . . . . . . . . . . . . . . . . . . . . 208
2. Algebra . . . . . . . . . . . . . . . . . . . . . . . . . . . . . . . . . . . . . . . . . . . . . . . . . . . . . 222
3. Geometry . . . . . . . . . . . . . . . . . . . . . . . . . . . . . . . . . . . . . . . . . . . . . . . . . . . . 248
4. Data Analysis . . . . . . . . . . . . . . . . . . . . . . . . . . . . . . . . . . . . . . . . . . . . . . . . 263
8
GRE
®Practice Test 1 303
Section 1: Analytical Writing . . . . . . . . . . . . . . . . . . . . . . . . . . . . . . . . . . . . . 304
Section 2: Analytical Writing . . . . . . . . . . . . . . . . . . . . . . . . . . . . . . . . . . . . . 306
Section 3: Verbal Reasoning . . . . . . . . . . . . . . . . . . . . . . . . . . . . . . . . . . . . . . 309
Section 4: Verbal Reasoning . . . . . . . . . . . . . . . . . . . . . . . . . . . . . . . . . . . . . . 318
Section 5: Quantitative Reasoning . . . . . . . . . . . . . . . . . . . . . . . . . . . . . . . . . 329
Section 6: Quantitative Reasoning . . . . . . . . . . . . . . . . . . . . . . . . . . . . . . . . . 337
Evaluating Your Performance . . . . . . . . . . . . . . . . . . . . . . . . . . . . . . . . . . . . . 345
Answer Key . . . . . . . . . . . . . . . . . . . . . . . . . . . . . . . . . . . . . . . . . . . . . . . . . . . . 347
Score Conversion Table . . . . . . . . . . . . . . . . . . . . . . . . . . . . . . . . . . . . . . . . . . 351
Analytical Writing Sample Responses and Reader Commentaries . . . . . . . . 353
Section 1: Analyze an Issue . . . . . . . . . . . . . . . . . . . . . . . . . . . . . . . . . . . .353
Section 2: Analyze an Argument . . . . . . . . . . . . . . . . . . . . . . . . . . . . . . . .360
Answers and Explanations . . . . . . . . . . . . . . . . . . . . . . . . . . . . . . . . . . . . . . . . 367
Section 3: Verbal Reasoning . . . . . . . . . . . . . . . . . . . . . . . . . . . . . . . . . . . 367
Section 4: Verbal Reasoning . . . . . . . . . . . . . . . . . . . . . . . . . . . . . . . . . . . 381
Section 5: Quantitative Reasoning . . . . . . . . . . . . . . . . . . . . . . . . . . . . . . 396
Section 6: Quantitative Reasoning . . . . . . . . . . . . . . . . . . . . . . . . . . . . . . 413
This ebook was issued to HUAN-LIN CHANG, order #14803770223. Unlawful distribution of this ebook is prohibited.
Customer name HUAN-LIN CHANG , Order Id: 14803770223
9
GRE
®Practice Test 2 431
Section 1: Analytical Writing . . . . . . . . . . . . . . . . . . . . . . . . . . . . . . . . . . . . . 432
Section 2: Analytical Writing . . . . . . . . . . . . . . . . . . . . . . . . . . . . . . . . . . . . . 434
Section 3: Verbal Reasoning . . . . . . . . . . . . . . . . . . . . . . . . . . . . . . . . . . . . . . 437
Section 4: Verbal Reasoning . . . . . . . . . . . . . . . . . . . . . . . . . . . . . . . . . . . . . . 446
Section 5: Quantitative Reasoning . . . . . . . . . . . . . . . . . . . . . . . . . . . . . . . . . 456
Section 6: Quantitative Reasoning . . . . . . . . . . . . . . . . . . . . . . . . . . . . . . . . . 464
Evaluating Your Performance . . . . . . . . . . . . . . . . . . . . . . . . . . . . . . . . . . . . . 473
Answer Key . . . . . . . . . . . . . . . . . . . . . . . . . . . . . . . . . . . . . . . . . . . . . . . . . . . . 475
Score Conversion Table . . . . . . . . . . . . . . . . . . . . . . . . . . . . . . . . . . . . . . . . . . 479
Analytical Writing Sample Responses and Reader Commentaries . . . . . . . . 481
Section 1: Analyze an Issue . . . . . . . . . . . . . . . . . . . . . . . . . . . . . . . . . . . .481
Section 2: Analyze an Argument . . . . . . . . . . . . . . . . . . . . . . . . . . . . . . . .489
Answers and Explanations . . . . . . . . . . . . . . . . . . . . . . . . . . . . . . . . . . . . . . . . 496
Section 3: Verbal Reasoning . . . . . . . . . . . . . . . . . . . . . . . . . . . . . . . . . . . 496
Section 4: Verbal Reasoning . . . . . . . . . . . . . . . . . . . . . . . . . . . . . . . . . . . 512
Section 5: Quantitative Reasoning . . . . . . . . . . . . . . . . . . . . . . . . . . . . . . 527
Section 6: Quantitative Reasoning . . . . . . . . . . . . . . . . . . . . . . . . . . . . . . 546
This ebook was issued to HUAN-LIN CHANG, order #14803770223. Unlawful distribution of this ebook is prohibited.
Customer name HUAN-LIN CHANG , Order Id: 14803770223
This book/software package provides a large amount of important information about
the GRE revised General Test, the kinds of questions it includes, and the knowledge
and skills that it measures. The book and software will help you:
bFamiliarize yourself with the test format and the test question types
bIdentify the test-taking skills you most need to focus on
bReview the math topics you need to know for the test
bCheck your progress with Verbal Reasoning and Quantitative Reasoning
practice questions
bPractice your test-taking skills using actual GRE tests
The following six-step program has been designed to help you make the best use of this
book and software.
STEP 1 Learn About the GRE revised General Test Format
Chapter 1 of this book provides official information about the structure of the GRE
revised General Test. Read this chapter to learn about the different test sections, the
number of questions in each section, and the section time limits. You’ll also find
valuable test-taking strategies from ETS and important information about how the test
is scored.
STEP 2 Study the Different GRE Question Types
Chapters 2, 3, and 5 of this book describe the types of questions you’ll encounter in
the three sections of the GRE revised General Test. Chapter 2 describes the Analytical
Writing section, which requires you to write essays in response to prompts that you
will be given. Chapter 3 describes the Verbal Reasoning question types. Chapter 5
describes the Quantitative Reasoning question types. In each case, you will learn what
the questions are designed to measure, and you will get tips for answering each ques-
tion type. You will also see samples of each question type, with helpful explanations.
STEP 3 Practice Answering GRE Verbal and Quantitative Reasoning
Questions
Chapters 4 and 6 offer sets of practice Verbal Reasoning and Quantitative Reasoning
questions in the format of the GRE revised General Test. The question sets are
arranged in order of increasing difficulty, from easy through medium to hard. Answer
the questions in each set, then read through the explanations to see which test topics
you found most challenging. Look for patterns. Did specific question formats give you
trouble? When did you need to guess at the answer? Use your results to identify your
weaknesses and to sharpen your test-taking skills.
How to Use This
Book/Software
Package
This ebook was issued to HUAN-LIN CHANG, order #14803770223. Unlawful distribution of this ebook is prohibited.
Customer name HUAN-LIN CHANG , Order Id: 14803770223
STEP 4 Use the Test Preview Tool in the
POWERPREP
®
II
Software
The Test Preview Tool includes a sample of each of the question types formatted as they
would be on the actual computer-based test. The Tool also contains the Help screens,
the Review screen, the word-processing software for the Analytical Writing section, and
the on-screen calculator, so that you can become familiar with all of the features of the
computer-based GRE revised General Test.
STEP 5 Review GRE Math Topics
Chapter 7 provides a review of math topics tested in the GRE revised General Test
Quantitative Reasoning sections. You do not necessarily need to tackle every topic in
the review, or to work through the review in the order in which it is presented. Skip
around if you like, but remember to focus on the topics that you know give you trouble.
Each section of the review ends with practice problems that you can use to see how
well you have mastered the material. If you get a problem wrong, go back into the
review section and re-read the corresponding instructional text.
STEP 6 Take the Practice Tests
Once you have completed your review, get ready for the real exam by taking the
authentic GRE Practice Tests in Chapters 8 and 9 of this book and in the POWER-
PREP®II software. When you take each test, try to simulate actual testing conditions. If
you are taking one of the tests in Chapters 8 and 9, sit in a quiet space, time yourself,
and work through as much of the test as time allows. If you are taking one of the tests
on the in the software, each test section will be timed for you. You’ll gain experience
with the test format, and you’ll learn to pace yourself so that you can earn your highest
score in the time allowed.
Please Note: for users of the software
If you need screen magnification or selectable colors to take the computer-based GRE
revised General Test, you can download a special version of ZoomText software from
the GRE website at www.ets.org/gre/powerprep2.
This ebook was issued to HUAN-LIN CHANG, order #14803770223. Unlawful distribution of this ebook is prohibited.
Customer name HUAN-LIN CHANG , Order Id: 14803770223
Introduction
The GRE®revised General Test—the most widely accepted graduate admissions
test worldwide—measures verbal reasoning, quantitative reasoning, critical
thinking, and analytical writing skills that are necessary for success in graduate
and business school.
Prospective graduate and business school applicants from all around the world
take the GRE revised General Test. Applicants come from varying educational and cul-
tural backgrounds, and the GRE revised General Test provides a common measure for
comparing candidates’ qualifications.
GRE scores are used by admissions committees and fellowship panels to supple-
ment your undergraduate records, recommendation letters, and other qualifications
for graduate-level study.
The GRE revised General Test is available at about 700 test centers in more than
160 countries. In most regions of the world, the computer-based test is available on a
continuous basis throughout the year. In Mainland China, Hong Kong, Taiwan, and
Korea, the computer-based test is available one to three times per month. In areas of
the world where computer-based testing is not available, the test is administered in
a paper-based format up to three times a year.
For the most up-to-date information about the GRE revised General Test, visit the
GRE website at www.ets.org/gre.
About the
GRE
®
revised General Test
gReview basic information on the structure of the
test, test preparation, and scoring
Your goal
for this
chapter
1
1
This ebook was issued to HUAN-LIN CHANG, order #14803770223. Unlawful distribution of this ebook is prohibited.
Customer name HUAN-LIN CHANG , Order Id: 14803770223
About the GRE revised General Test
Structure and Content of the Test
Content
The GRE revised General Test is composed of three measures—Analytical Writing,
Verbal Reasoning, and Quantitative Reasoning.
bThe Analytical Writing section measures the ability to articulate and support
complex ideas, examine claims and accompanying evidence, sustain a focused
and coherent discussion, and control the elements of standard written English.
The Analytical Writing section requires you to provide focused responses based
on the tasks presented, so you can accurately demonstrate your skill in directly
responding to a task.
bThe Verbal Reasoning section measures your ability to analyze and evaluate
written material and synthesize information obtained from it; understand the
meanings of words, sentences, and entire texts; and understand relationships
among words and among concepts. The Verbal Reasoning section measures
your ability to understand what you read and how you apply your reasoning
skills.
bThe Quantitative Reasoning section measures your basic mathematical skills
and your understanding of the elementary mathematical concepts of arith -
metic, algebra, geometry, and data analysis. The Quantitative Reasoning
section measures your ability to understand, interpret, and analyze quantitative
information and to solve problems using mathematical models.
The Computer-based GRE revised General Test
Typical Test Structure
Measure Number of Questions Allotted Time
Analytical Writing One “Analyze an Issue” task and 30 minutes per
(One section with two one “Analyze an Argument” task task
separately timed tasks)
Verbal Reasoning 20 questions per section 30 minutes per
(Two sections) section
Quantitative Reasoning 20 questions per section 35 minutes per
(Two sections) section
Unscored* (Position varies) Varies Varies
Research** (At the end Varies Varies
of the test)
* An unidentified, unscored section that does not count toward a score may be included and may
appear in any order after the Analytical Writing section. Questions in the unscored section are
being tried out either for possible use in future tests or to ensure that scores on new editions of
the test are comparable to scores from earlier editions.
** An identified research section may be included in place of the unscored section. The research
section will always appear at the end of the test. Questions in this section are included for ETS
research purposes and will not count toward your score.
2
This ebook was issued to HUAN-LIN CHANG, order #14803770223. Unlawful distribution of this ebook is prohibited.
Customer name HUAN-LIN CHANG , Order Id: 14803770223
The Analytical Writing section will always come first in the test. The Verbal Rea-
soning, Quantitative Reasoning, and unidentified/unscored sections may appear in any
order; therefore, you should treat each section as if it counts toward your score.
Total testing time is 3 hours and 45 minutes, including the unscored section. The
directions at the beginning of each Verbal Reasoning and Quantitative Reasoning sec-
tion specify the total number of questions in the section and the time allowed for the
section. For the Analytical Writing section, the timing for each task is shown when the
task is presented.
Test Design Features
The Verbal Reasoning and Quantitative Reasoning measures of the computer-based
GRE revised General Test are section-level adaptive. This means the computer selects
the second section of a measure based on your performance on the first section.
Additionally, the design of the revised test features advanced technology that allows
you to freely move forward and backward throughout an entire section. Other design
features include:
bPreview and review capabilities within a section
bA “mark and review” feature to tag questions, so you can skip them and return
later if you have time remaining in the section
bThe ability to change/edit answers within a section
bAn on-screen calculator for the Quantitative Reasoning measure
The Paper-based GRE revised General Test
Typical Test Structure
Measure Number of Questions Allotted Time
Analytical Writing Section One: “Analyze an Issue” task 30 minutes
(Two sections) Section Two: “Analyze an Argument” task per section
Verbal Reasoning 25 questions per section 35 minutes
(Two sections) per section
Quantitative Reasoning 25 questions per section 40 minutes
(Two sections) per section
The Analytical Writing sections will always be first, while the other four sections
may appear in any order.
Total testing time is approximately 3 hours and 30 minutes. The directions at the
beginning of each section specify the total number of questions in the section and the
time allowed for the section.
Test Design Features
bAnswers are entered into the test book, rather than a separate answer sheet.
bYou will be provided with an ETS calculator to use during the Quantitative
Reasoning section; you may not use your own calculator. Information about
using the calculator to help you answer questions appears in the free Practice
Book for the Paper-based GRE revised General Test, which is available at
www.ets.org/gre/prepare.
3
About the GRE revised General Test
This ebook was issued to HUAN-LIN CHANG, order #14803770223. Unlawful distribution of this ebook is prohibited.
Customer name HUAN-LIN CHANG , Order Id: 14803770223
About the GRE revised General Test
Preparing for the Test
Preparation for the test will depend on the amount of time you have available and
your personal preferences for how to prepare. At a minimum, before you take the
GRE®revised General Test, you should know what to expect from the test, including
the administrative procedures, types of questions and directions, approximate number
of questions, and amount of time for each section.
The administrative procedures include registration and appointment scheduling,
date, time, test center location, cost, score-reporting procedures, and availability of
special testing arrangements. You can find out about the administrative procedures for
the revised General Test in the GRE®Information and Registration Bulletin, which is
available at www.ets.org/gre/bulletinandforms.
Before taking the revised General Test, it is important to become familiar with the
content of each of the measures. In this publication, you’ll find information specific to
each measure of the test. You can use this information to understand the type of mate-
rial on which you’ll be tested and the question types within each measure.
It is also important to spend some time preparing for the Analytical Writing section
by reviewing the skills measured, scoring guides and score level descriptions, sample
topics, scored sample essay responses, and reader commentary.
Test-taking Strategies for the Computer-based Test
Analytical Writing Section
The Analytical Writing measure of the computer-based revised General Test uses an ele-
mentary word processor developed by ETS so that individuals familiar with a specific
commercial word processing software do not have an advantage or disadvantage. This
software contains the following functionality: insert text, delete text, cut and paste, and
undo the previous action. Tools such as a spelling checker and grammar checker are
not available in the ETS software, in large part to maintain fairness with those exami-
nees who must handwrite their essays at paper-based administrations.
It is important to budget your time. Within the 30-minute time limit for each task,
you’ll need to allow sufficient time to think about the topic, plan a response, and com-
pose your essay. Although GRE readers understand the time constraints under which
you write and will consider your response a first draft, you will still want to produce
the best possible example of your writing.
Save a few minutes at the end of each timed task to check for obvious errors.
Although an occasional typographical, spelling, or grammatical error will not affect
your score, severe or persistent errors will detract from the overall effectiveness of your
writing and lower your score.
Verbal Reasoning and Quantitative Reasoning Sections
The questions in the Verbal Reasoning and Quantitative Reasoning measures are pre-
sented in a variety of formats. Some require you to select a single answer choice; others
require you to select one or more answer choices, and yet others require you to enter
a numeric answer. Make sure when answering a question that you understand what
4
This ebook was issued to HUAN-LIN CHANG, order #14803770223. Unlawful distribution of this ebook is prohibited.
Customer name HUAN-LIN CHANG , Order Id: 14803770223
response is required. An on-screen calculator will be provided at the test center for use
during the Quantitative Reasoning sections.
When taking the computer-based GRE revised General Test, you are free to skip
questions that you might have difficulty answering within a section. The testing soft-
ware has a “mark and review” feature that enables you to mark questions you would
like to revisit during the time provided to work on that section. The testing software
also lets you view a complete list of all the questions in the section on which you’re
working, indicates whether you’ve answered each question, and identifies the questions
you’ve marked for review. Additionally, you can review questions you’ve already
answered and change your answers, provided you still have time remaining to work on
that section.
A sample review screen appears at the top of the page. The review screen is
intended to help you keep track of your progress on the test. Do not spend too much
time on the review screen, as this will take away from the time allotted to read and
answer the questions on the test.
Your Verbal Reasoning and Quantitative Reasoning scores will be determined by
the number of questions you answer correctly. Nothing is subtracted from a score if
you answer a question incorrectly. Therefore, to maximize your scores on the Verbal
Reasoning and Quantitative Reasoning measures, it is best to answer every question.
Work as rapidly as you can without being careless. Since no question carries
greater weight than any other, do not waste time pondering individual questions you
find extremely difficult or unfamiliar.
You may want to go through each of the Verbal Reasoning and Quantitative
Reason ing sections rapidly first, stopping only to answer questions you can answer
with certainty. Then go back and answer the questions that require greater thought,
concluding with the difficult questions if you have time.
During the actual administration of the revised General Test, you may work only
on one section at a time and only for the time allowed. Once you have completed a
section, you may not go back to it.
5
About the GRE revised General Test
This ebook was issued to HUAN-LIN CHANG, order #14803770223. Unlawful distribution of this ebook is prohibited.
Customer name HUAN-LIN CHANG , Order Id: 14803770223
About the GRE revised General Test
Scratch Paper
You will receive a supply of scratch paper before you begin the test. You can replenish
your supply of scratch paper as necessary throughout the test by asking the test
administrator.
Breaks
There is a 10-minute break following the third section and a one-minute break between
the other test sections. You might want to replenish your supply of scratch paper dur-
ing a scheduled break. Section timing will not stop if you take an unscheduled break,
so you should proceed with your test without interruption once it begins.
Test-taking Strategies for the Paper-based Test
Analytical Writing Sections
In the paper-based revised General Test, the topics in the Analytical Writing measure
will be presented in the test book, and you will handwrite your essay responses in the
test book in the space provided.
It is important to budget your time. Within the 30-minute time limit for each sec-
tion, you’ll need to allow sufficient time to think about the topic, plan a response, and
compose your essay. Although GRE readers understand the time constraints under
which you write and will consider your response a first draft, you will still want to pro-
duce the best possible example of your writing.
Save a few minutes at the end of each timed section to check for obvious errors.
Although an occasional spelling or grammatical error will not affect your score, severe
and persistent errors will detract from the overall effectiveness of your writing and
lower your score.
During the actual administration of the revised General Test, you may work only on
the particular writing section the test center supervisor designates and only for the
time allowed. You may not go back to an earlier section of the test after the supervisor
announces, “Please stop work” for that section. The supervisor is authorized to dismiss
you from the center for doing so.
Verbal Reasoning and Quantitative Reasoning Sections
The questions in the Verbal Reasoning and Quantitative Reasoning measures have a
variety of formats. Some require you to select a single answer choice; others require
you to select one or more answer choices, and yet others require you to enter a
numeric answer. Make sure when answering a question that you understand what
response is required. A calculator will be provided at the test center for use during the
Quantitative Reasoning sections.
When taking a Verbal Reasoning or Quantitative Reasoning section, you are free,
within that section, to skip questions that you might have difficulty answering and
come back to them later during the time provided to work on that section. Also during
that time you may change the answer to any question in that section by erasing it com-
pletely and filling in an alternative answer. Be careful not to leave any stray marks in
6
This ebook was issued to HUAN-LIN CHANG, order #14803770223. Unlawful distribution of this ebook is prohibited.
Customer name HUAN-LIN CHANG , Order Id: 14803770223
the answer area, as they may be interpreted as incorrect responses. You can, however,
safely make notes or perform calculations on other parts of the page. No additional
scratch paper will be provided.
Your Verbal Reasoning and Quantitative Reasoning scores will be determined by
the number of questions you answer correctly. Nothing is subtracted from a score if
you answer a question incorrectly. Therefore, to maximize your scores on the Verbal
Reasoning and Quantitative Reasoning measures, it is best to answer every question.
Work as rapidly as you can without being careless. Since no question carries
greater weight than any other, do not waste time pondering individual questions you
find extremely difficult or unfamiliar.
You may want to go through each of the Verbal Reasoning and Quantitative Rea-
soning sections rapidly first, stopping only to answer questions you can answer with
certainty. Then go back and answer the questions that require greater thought, con-
cluding with the difficult questions if you have time.
During the actual administration of the revised General Test, you may work only on
the section the test center supervisor designates and only for the time allowed. You
may not go back to an earlier section of the test after the supervisor announces, “Please
stop work” for that section. The supervisor is authorized to dismiss you from the cen-
ter for doing so. All answers must be recorded in the test book.
Breaks
There is a 10-minute break following the second Analytical Writing section.
Understanding GRE Scoring
Analytical Writing Measure
For the Analytical Writing measure, each essay receives a score from two readers using
a six-point holistic scale. In holistic scoring, readers are trained to assign scores based
on the overall quality of an essay in response to the assigned task. If the two scores dif-
fer by more than one point on the scale, the discrepancy is adjudicated by a third
GRE reader. Otherwise, the two scores on each essay are averaged.
The final scores on the two essays are then averaged and rounded to the nearest
half-point interval on the 0–6 score scale. A single score is reported for the Analytical
Writing measure. The primary emphasis in scoring the Analytical Writing section is on
your critical thinking and analytical writing skills. Scoring guides for the Issue and
Argument prompts are included in this publication, and they are available on the GRE
website at www.ets.org/gre/scores/how.
The GRE Program plans to implement e-rater®scoring technology in the scoring
process for the Analytical Writing measure of the computer-based GRE®revised
General Test. The e-rater scoring engine is a computerized natural language–processing
program developed by ETS. When e-rater scoring is implemented, information about it
will be available on the GRE website at www.ets.org/gre/scores/how.
7
About the GRE revised General Test
This ebook was issued to HUAN-LIN CHANG, order #14803770223. Unlawful distribution of this ebook is prohibited.
Customer name HUAN-LIN CHANG , Order Id: 14803770223
About the GRE revised General Test
Independent Intellectual Activity
During the scoring process for the revised Gene ral Test, your essay responses on the
Analytical Writing section will be reviewed by ETS essay-similarity-detection software
and by experienced raters. In light of the high value placed on independent intellectual
activity within graduate schools and universities, ETS reserves the right to cancel test
scores of any test taker when an essay response includes any of the following:
bText that is unusually similar to that found in one or more other GRE essay
responses
bQuoting or paraphrasing, without attribution, language that appears in pub-
lished or unpublished sources
bUnacknowledged use of work that has been produced through collaboration
with others without citation of the contribution of others
bEssays submitted as work of the test taker that appear to have been borrowed
in whole or in part from elsewhere or prepared by another person
When one or more of the above discrepancies occurs, ETS may conclude, in its
professional judgment, that the essay response does not reflect the independent writing
skills that this test seeks to measure. When ETS reaches that conclusion, it cancels the
Analytical Writing scores, and because Analytical Writing scores are an integral part of
GRE revised General Test scores, those scores are canceled as well.
Verbal Reasoning and Quantitative Reasoning Measures
of the Computer-based Test
For the Verbal Reasoning and Quantitative Reasoning measures of the computer-based
GRE revised General Test, the reported scores are based on the number of correct
responses to the questions included in the operational sections of the measure.
The Verbal Reasoning and Quantitative Reasoning measures are section-level adap-
tive. This means the computer selects the second section of a measure based on your
performance on the first section. Within each section, all questions contribute equally
to the final score. For each of the two measures, a raw score is computed. The raw
score is the number of questions you answered correctly.
The raw score is then converted to a scaled score through a process known as
equating. The equating process accounts for minor variations in difficulty from test to
test as well as the differences introduced by the section-level adaptation. Thus a given
scaled score for a particular measure reflects the same level of performance regardless
of which second section was selected and when the test was taken.
Verbal Reasoning and Quantitative Reasoning Measures
of the Paper-based Test
Scoring of the Verbal Reasoning and Quantitative Reasoning measures is essentially a
two-step process. First a raw score is computed for each measure. The raw score for
each measure is the number of questions answered correctly in the two sections for
that measure.
The Verbal Reasoning and Quantitative Reasoning raw scores are then converted to
scaled scores through a process known as equating. The equating process accounts for
8
This ebook was issued to HUAN-LIN CHANG, order #14803770223. Unlawful distribution of this ebook is prohibited.
Customer name HUAN-LIN CHANG , Order Id: 14803770223
minor variations in difficulty among the different test editions. Thus, a given scaled
score for a particular measure reflects the same level of performance regardless of
which edition of the test was taken.
Score Reporting
Three scores will be reported on the revised General Test:
ba Verbal Reasoning score reported on a 130–170 score scale, in 1-point
increments
ba Quantitative Reasoning score reported on a 130–170 score scale, in
1-point increments
ban Analytical Writing score reported on a 0–6 score scale, in half-point
increments
If you do not answer any questions at all for a measure (Verbal Reasoning, Quanti-
tative Reasoning, or Analytical Writing), you receive a No Score (NS) for that measure.
Descriptions of the analytical writing abilities characteristic of particular score
levels are available in this publication on page 41, and on the GRE website at www.
ets.org/gre/awscoredescriptions.
The
ScoreSelect
SM Option
Introduced in July 2012, the new ScoreSelectSM option is available for both the GRE
revised General Test and GRE Subject Tests and can be used by anyone with reportable
scores from the last five years. This option lets you decide which test scores to send to
the institutions you designate. You can send scores from your most recent test admin-
istration or scores for all of the times you’ve taken a GRE test as part of your four free
score reports. After test day, you can send scores from your Most Recent, All, or Any
specific test administration(s) for a fee when ordering Additional Score Reports. Just
remember, scores for a test administration must be reported in their entirety. For more
information, visit www.ets.org/gre/scoreselect.
Score Reporting Time Frames
Scores from computer-based GRE revised General Test administrations are reported
approximately 10 to 15 days after the test date. Scores from paper-based administra-
tions are reported within six weeks after the test date. If you are applying to a graduate
or business school program, be sure to review the appropriate admissions deadlines
and plan to take the test in time for your scores to reach the institution.
For more information on score reporting, visit the GRE website at www.ets.org/gre
/scores/get.
9
About the GRE revised General Test
This ebook was issued to HUAN-LIN CHANG, order #14803770223. Unlawful distribution of this ebook is prohibited.
Customer name HUAN-LIN CHANG , Order Id: 14803770223
This page intentionally left blank
This ebook was issued to HUAN-LIN CHANG, order #14803770223. Unlawful distribution of this ebook is prohibited.
Customer name HUAN-LIN CHANG , Order Id: 14803770223
Overview of the Analytical Writing Measure
The Analytical Writing measure assesses your critical thinking and analytical
writing skills. It assesses your ability to articulate and support complex ideas,
construct and evaluate arguments, and sustain a focused and coherent discus-
sion. It does not assess specific content knowledge.
The Analytical Writing measure consists of two separately timed analytical writing
tasks:
ba 30-minute “Analyze an Issue” task
ba 30-minute “Analyze an Argument” task
The Issue task presents an opinion on an issue of broad interest followed by spe-
cific instructions on how to respond to that issue. You are required to evaluate the
issue, considering its complexities, and develop an argument with reasons and exam-
ples to support your views.
The Argument task presents a different challenge from that of the Issue task: it
requires you to evaluate a given argument according to specific instructions. You will
need to consider the logical soundness of the argument rather than to agree or disagree
with the position it presents.
The two tasks are complementary in that one requires you to construct your own
argument by taking a position and providing evidence supporting your views on the
issue, whereas the other requires you to evaluate someone else’s argument by assessing
its claims and evaluating the evidence it provides.
gRecognize the two types of Analytical Writing
tasks
gStudy examples of each type of writing task
gLearn strategies for responding to the writing
tasks
gReview actual student responses and ETS reader
commentary
Your goals
for this
chapter
11
2
GRE
®
Analytical Writing
This ebook was issued to HUAN-LIN CHANG, order #14803770223. Unlawful distribution of this ebook is prohibited.
Customer name HUAN-LIN CHANG , Order Id: 14803770223
Preparing for the Analytical Writing Measure
Everyone even the most practiced and confident of writers should spend some
time preparing for the Analytical Writing measure before arriving at the test center. It
is important to review the skills measured and how the section is scored. It is also use-
ful to review the scoring guides and score level descriptions, sample topics, scored sam-
ple essay responses, and reader commentary for each task.
The tasks in the Analytical Writing measure relate to a broad range of subjects
from the fine arts and humanities to the social and physical sciences but no task
requires specific content knowledge. In fact, each task has been field-tested to ensure
that it possesses several important characteristics, including the following:
bGRE®test takers, regardless of their field of study or special interests, under-
stood the task and could easily respond to it.
bThe task elicited the kinds of complex thinking and persuasive writing that uni-
versity faculty consider important for success at the graduate level.
bThe responses were varied in content and in the way the writers developed
their ideas.
To help you prepare for the Analytical Writing measure, the GRE Program has pub-
lished the entire pool of tasks from which your test tasks will be selected. You might
find it helpful to review the Issue and Argument pools. You can view the published
pools at www.ets.org/gre/awtopics.
General Strategies
bIt is important to budget your time. Within the 30-minute time limit for the
Issue task, you will need to allow sufficient time to consider the issue and
the specific instructions, plan a response, and compose your essay. Within the
30-minute time limit for the Argument task, you will need to allow sufficient
time to consider the argument and the specific instructions, plan a response,
and compose your essay. Although GRE readers understand the time con-
straints under which you write and will consider your response a first draft,
you still want it to be the best possible example of your writing that you can
produce under the testing conditions.
bSave a few minutes at the end of each timed task to check for obvious errors.
Although an occasional spelling or grammatical error will not affect your score,
severe and persistent errors will detract from the overall effectiveness of your
writing and thus lower your score.
12
GRE Analytical Writing
This ebook was issued to HUAN-LIN CHANG, order #14803770223. Unlawful distribution of this ebook is prohibited.
Customer name HUAN-LIN CHANG , Order Id: 14803770223
Analyze an Issue Task
Understanding the Issue Task
The Analyze an Issue task assesses your ability to think critically about a topic of gen-
eral interest according to specific instructions and to clearly express your thoughts
about it in writing. Each issue topic makes a claim that test takers can discuss from
various perspectives and apply to many different situations or conditions. The issue
statement is followed by specific instructions. Your task is to present a compelling case
for your own position on the issue according to the specific instructions. Before begin-
ning your written response, be sure to read the issue and instructions carefully and
think about the issue from several points of view, considering the complexity of ideas
associated with those views. Then, make notes about the position you want to develop
and list the main reasons and examples that you could use to support that position.
It is important that you address the central issue according to the specific instruc-
tions. Each task is accompanied by one of the following sets of instructions.
bWrite a response in which you discuss the extent to which you agree or dis-
agree with the statement and explain your reasoning for the position you take.
In developing and supporting your position, you should consider ways in which
the statement might or might not hold true and explain how these considera-
tions shape your position.
bWrite a response in which you discuss the extent to which you agree or dis-
agree with the recommendation and explain your reasoning for the position
you take. In developing and supporting your position, describe specific cir-
cumstances in which adopting the recommendation would or would not be
advantageous and explain how these examples shape your position.
bWrite a response in which you discuss the extent to which you agree or dis-
agree with the claim. In developing and supporting your position, be sure to
address the most compelling reasons and/or examples that could be used to
challenge your position.
bWrite a response in which you discuss which view more closely aligns with
your own position and explain your reasoning for the position you take. In
developing and supporting your position, you should address both of the views
presented.
bWrite a response in which you discuss the extent to which you agree or dis-
agree with the claim and the reason on which that claim is based.
bWrite a response in which you discuss your views on the policy and explain
your reasoning for the position you take. In developing and supporting your
position, you should consider the possible consequences of implementing the
policy and explain how these consequences shape your position.
The GRE readers scoring your response are not looking for a “right” answer in
fact, there is no correct position to take. Instead, the readers are evaluating the skill
with which you address the specific instructions and articulate and develop an argu-
ment to support your evaluation of the issue.
13
GRE Analytical Writing
This ebook was issued to HUAN-LIN CHANG, order #14803770223. Unlawful distribution of this ebook is prohibited.
Customer name HUAN-LIN CHANG , Order Id: 14803770223
Understanding the Context for Writing: Purpose and Audience
The Issue task is an exercise in critical thinking and persuasive writing. The purpose of
this task is to determine how well you can develop a compelling argument supporting
your own evaluation of an issue and to effectively communicate that argument in writ-
ing to an academic audience. Your audience consists of GRE readers who are carefully
trained to apply the scoring criteria identified in the scoring guide for the Analyze an
Issue task (see pages 37–38).
To get a clearer idea of how GRE readers apply the Issue scoring criteria to actual
responses, you should review scored sample Issue essay responses and reader com-
mentary. The sample responses, particularly at the 5 and 6 score levels, will show you a
variety of successful strategies for organizing, developing, and communicating a per-
suasive argument. The reader commentary discusses specific aspects of evaluation and
writing, such as the use of examples, development and support, organization, language
fluency, and word choice. For each response, the reader commentary points out aspects
that are particularly persuasive as well as any that detract from the overall effectiveness
of the essay.
Preparing for the Issue Task
Because the Issue task is meant to assess the persuasive writing skills that you have
developed throughout your education, it has been designed neither to require any par-
ticular course of study nor to advantage students with a particular type of training.
Many college textbooks on composition offer advice on persuasive writing and
argumentation that you might find useful, but even this advice might be more techni-
cal and specialized than you need for the Issue task. You will not be expected to know
specific critical thinking or writing terms or strategies; instead, you should be able to
respond to the specific instructions and use reasons, evidence, and examples to support
your position on an issue. Suppose, for instance, that an Issue topic asks you to con-
sider a policy that would require government financial support for art museums and
the implications of implementing the policy. If your position is that government should
fund art museums, you might support your position by discussing the reasons art is
important and explain that government funding would make access to museums avail-
able to everyone. On the other hand, if your position is that government should not
support museums, you might point out that, given limited governmental funds, art
museums are not as deserving of governmental funding as are other, more socially
important, institutions, which would suffer if the policy were implemented. Or, if you
are in favor of government funding for art museums only under certain conditions, you
might focus on the artistic criteria, cultural concerns, or political conditions that you
think should determine how or whether art museums receive government funds. It
is not your position that matters so much as the critical thinking skills you display in
developing your position.
An excellent way to prepare for the Issue task is to practice writing on some of the
published topics. There is no “best” approach: some people prefer to start practicing
without regard to the 30-minute time limit; others prefer to take a “timed test” first and
practice within the time limit. No matter which approach you take when you practice
the Issue task, you should review the task directions, then
bcarefully read the claim and the specific instructions and make sure you under-
stand them; if they seem unclear, discuss them with a friend or teacher
14
GRE Analytical Writing
This ebook was issued to HUAN-LIN CHANG, order #14803770223. Unlawful distribution of this ebook is prohibited.
Customer name HUAN-LIN CHANG , Order Id: 14803770223
bthink about the claim and instructions in relation to your own ideas and expe-
riences, to events you have read about or observed, and to people you have
known; this is the knowledge base from which you will develop compelling rea-
sons and examples in your argument that reinforce, negate, or qualify the claim
in some way
bdecide what position on the issue you want to take and defend
bdecide what compelling evidence (reasons and examples) you can use to sup-
port your position
Remember that this is a task in critical thinking and persuasive writing. The most
successful responses will explore the complexity of the claim and instructions. As you
prepare for the Issue task, you might find it helpful to ask yourself the following ques-
tions:
bWhat precisely is the central issue?
bWhat precisely are the instructions asking me to do?
bDo I agree with all or with any part of the claim? Why or why not?
bDoes the claim make certain assumptions? If so, are they reasonable?
bIs the claim valid only under certain conditions? If so, what are they?
bDo I need to explain how I interpret certain terms or concepts used in the
claim?
bIf I take a certain position on the issue, what reasons support my position?
bWhat examples either real or hypothetical could I use to illustrate those
reasons and advance my point of view? Which examples are most compelling?
Once you have decided on a position to defend, consider the perspective of others
who might not agree with your position. Ask yourself:
bWhat reasons might someone use to refute or undermine my position?
bHow should I acknowledge or defend against those views in my essay?
To plan your response, you might want to summarize your position and make brief
notes about how you will support the position you’re going to take. When you’ve done
this, look over your notes and decide how you will organize your response. Then write
a response developing your position on the issue. Even if you don’t write a full
response, you should find it helpful to practice with a few of the Issue topics and to
sketch out your possible responses. After you have practiced with some of the topics,
try writing responses to some of the topics within the 30-minute time limit so that you
have a good idea of how to use your time in the actual test.
It would probably be helpful to get some feedback on your response from an
instructor who teaches critical thinking or writing or to trade papers on the same topic
with other students and discuss one another’s responses in relation to the scoring
guide. Try to determine how each paper meets or misses the criteria for each score
point in the guide. Comparing your own response to the scoring guide will help you see
how and where you might need to improve.
15
GRE Analytical Writing
This ebook was issued to HUAN-LIN CHANG, order #14803770223. Unlawful distribution of this ebook is prohibited.
Customer name HUAN-LIN CHANG , Order Id: 14803770223
16
GRE Analytical Writing
The Form of Your Response
You are free to organize and develop your response in any way that you think will effec-
tively communicate your ideas about the issue and the instructions. Your response
may, but need not, incorporate particular writing strategies learned in English compo-
sition or writing-intensive college courses. GRE readers will not be looking for a par-
ticular developmental strategy or mode of writing; in fact, when GRE readers are
trained, they review hundreds of Issue responses that, although highly diverse in con-
tent and form, display similar levels of critical thinking and persuasive writing. Read-
ers will see, for example, some Issue responses at the 6 score level that begin by briefly
summarizing the writer’s position on the issue and then explicitly announcing the main
points to be argued. They will see others that lead into the writer’s position by making
a prediction, asking a series of questions, describing a scenario, or defining critical
terms in the quotation. The readers know that a writer can earn a high score by giving
multiple examples or by presenting a single, extended example. Look at the sample
Issue responses, particularly at the 5 and 6 score levels, to see how other writers have
successfully developed and organized their arguments.
You should use as many or as few paragraphs as you consider appropriate for your
argument for example, you will probably need to create a new paragraph whenever
your discussion shifts to a new cluster of ideas. What matters is not the number of
examples, the number of paragraphs, or the form your argument takes but, rather, the
cogency of your ideas about the issue and the clarity and skill with which you commu-
nicate those ideas to academic readers.
Sample Issue Task
As people rely more and more on technology to solve problems, the ability of
humans to think for themselves will surely deteriorate.
Discuss the extent to which you agree or disagree with the statement and explain
your reasoning for the position you take. In developing and supporting your
position, you should consider ways in which the statement might or might not
hold true and explain how these considerations shape your position.
Strategies for This Topic
In this task, you are asked to discuss the extent to which you agree or disagree with the
statement. Thus, responses may range from strong agreement or strong disagreement,
to qualified agreement or qualified disagreement. You are also instructed to explain
your reasoning and consider ways in which the statement might or might not hold
true. A successful response need not comment on all or any one of the points listed
below and may well discuss other reasons or examples not mentioned here in support
of its position.
Although this topic is accessible to respondents of all levels of ability, for any
response to receive a top score, it is particularly important that you remain focused on
the task and provide clearly relevant examples and/or reasons to support the point of
view you are expressing. Lower level responses may be long and full of examples of
modern technology, but those examples may not be clearly related to a particular posi-
tion. For example, a respondent who strongly disagrees with the statement may choose
This ebook was issued to HUAN-LIN CHANG, order #14803770223. Unlawful distribution of this ebook is prohibited.
Customer name HUAN-LIN CHANG , Order Id: 14803770223
to use computer technology as proof that thinking ability is not deteriorating. The mere
existence of computer technology, however, does not adequately prove this point (per-
haps the ease of computer use inhibits our thinking ability). To receive a higher-level
score, the respondent should explain in what ways computer technology may call for or
require thinking ability.
This topic could elicit a wide variety of approaches, especially considering the dif-
ferent possible interpretations of the phrase “the ability of humans to think for them-
selves.” Although most respondents may take it to mean problem solving, others, with
equal effectiveness, could interpret it as emotional and social intelligence (i.e., the abil-
ity to communicate/connect with others). With any approach, it is possible to discuss
examples such as calculators, word processing tools such as spell and grammar check,
tax return software, Internet research, and a variety of other common household and
business technologies.
You may agree with the prompt and argue that:
breliance on technology leads to dependency; we come to rely on problem-
solving technologies to such a degree that when they fail, we are in worse shape
than if we didn’t have them in the first place
beveryday technologies such as calculators and cash registers have decreased
our ability to perform simple calculations, a “use it or lose it” approach to
thinking ability
Or you may take issue with the prompt and argue that technology facilitates and
improves our thinking skills, arguing that:
bdeveloping, implementing, and using technology requires problem solving
btechnology frees us from mundane problem solving (e.g., calculations) and
allows us to engage in more complex thinking
btechnology provides access to information otherwise unavailable
btechnology connects people at a distance and allows them to share ideas
btechnology is dependent on the human ability to think and make choices (every
implementation of and advance in technology is driven by human intelligence
and decision making)
On the other hand, you could decide to explore the middle ground in the debate
and point out that while technology may diminish some mental skill sets, it enables
other (perhaps more important) types of thinking to thrive. Such a response might dis-
tinguish between complex problem solving and simple “data maintenance” (i.e., per-
forming calculations and organizing information). Other approaches could include
taking a historical, philosophical, or sociological stance, or, with equal effectiveness,
using personal examples to illustrate a position. One could argue that the value or
detriment of relying on technology is determined by the individual (or society) using it
or that only those who develop technology (i.e., technical specialists) are maintaining
their problem-solving skills, while the rest of us are losing them.
Again, it is important for you to avoid overly general examples, or lists of examples
without expansion. It is also essential to do more than paraphrase the prompt. Please
keep in mind that what counts is the ability to clearly express a particular point of view
in relation to the issue and specific task instructions and to support that position with
relevant reasons and/or examples.
17
GRE Analytical Writing
This ebook was issued to HUAN-LIN CHANG, order #14803770223. Unlawful distribution of this ebook is prohibited.
Customer name HUAN-LIN CHANG , Order Id: 14803770223
*All responses in this publication are reproduced exactly as written, including errors, mis-
spellings, etc., if any.
Essay Responses and Reader Commentary
Score 6 Response *
The statement linking technology negatively with free thinking plays on recent human
experience over the past century. Surely there has been no time in history where the
lived lives of people have changed more dramatically. A quick reflection on a typical
day reveals how technology has revolutionized the world. Most people commute to
work in an automobile that runs on an internal combustion engine. During the
workday, chances are high that the employee will interact with a computer that
processes information on silicon bridges that are .09 microns wide. Upon leaving
home, family members will be reached through wireless networks that utilize satellites
orbiting the earth. Each of these common occurences would have been inconceivable
at the turn of the 19th century.
The statement attempts to bridge these dramatic changes to a reduction in the
ability for humans to think for themselves. The assumption is that an increased reliance
on technology negates the need for people to think creatively to solve previous
quandaries. Looking back at the introduction, one could argue that without a car,
computer, or mobile phone, the hypothetical worker would need to find alternate
methods of transport, information processing, and communication. Technology short
circuits this thinking by making the problems obsolete.
However, this reliance on technology does not necessarily preclude the creativity
that marks the human species. The prior examples reveal that technology allows for
convenience. The car, computer, and phone all release additional time for people to
live more efficiently. This efficiency does not preclude the need for humans to think for
themselves. In fact, technology frees humanity to not only tackle new problems, but
may itself create new issues that did not exist without technology. For example, the
proliferation of automobiles has introduced a need for fuel conservation on a global
scale. With increasing energy demands from emerging markets, global warming
becomes a concern inconceivable to the horse-and-buggy generation. Likewise
dependence on oil has created nation-states that are not dependent on taxation,
allowing ruling parties to oppress minority groups such as women. Solutions to these
complex problems require the unfettered imaginations of maverick scientists and
politicians.
In contrast to the statement, we can even see how technology frees the human
imagination. Consider how the digital revolution and the advent of the internet has
allowed for an unprecedented exchange of ideas. WebMD, a popular internet portal for
medical information, permits patients to self research symptoms for a more informed
doctor visit. This exercise opens pathways of thinking that were previously closed off
to the medical layman. With increased interdisciplinary interactions, inspiration can
arrive from the most surprising corners. Jeffrey Sachs, one of the architects of the UN
Millenium Development Goals, based his ideas on emergency care triage techniques.
The unlikely marriage of economics and medicine has healed tense, hyperinflation
environments from South America to Eastern Europe.
This last example provides the most hope in how technology actually provides hope
to the future of humanity. By increasing our reliance on technology, impossible goals
can now be achieved. Consider how the late 20th century witnessed the complete
elimination of smallpox. This disease had ravaged the human race since prehistorical
days, and yet with the technology of vaccines, free thinking humans dared to imagine a
18
GRE Analytical Writing
This ebook was issued to HUAN-LIN CHANG, order #14803770223. Unlawful distribution of this ebook is prohibited.
Customer name HUAN-LIN CHANG , Order Id: 14803770223
19
GRE Analytical Writing
world free of smallpox. Using technology, battle plans were drawn out, and smallpox
was systematically targeted and eradicated.
Technology will always mark the human experience, from the discovery of fire to the
implementation of nanotechnology. Given the history of the human race, there will be
no limit to the number of problems, both new and old, for us to tackle. There is no
need to retreat to a Luddite attitude to new things, but rather embrace a hopeful
posture to the possibilities that technology provides for new avenues of human
imagination.
Reader Commentary
The author of this essay stakes out a clear and insightful position on the issue and fol-
lows the specific instructions by discussing ways in which the statement might or
might not hold true, using specific reasons and examples to support that position. The
essay cogently argues that technology does not decrease our ability to think for our-
selves. It merely provides “additional time for people to live more efficiently.” In fact,
the problems that have developed alongside the growth of technology (pollution, polit-
ical unrest in oil-producing nations) actually call for more creative thinking, not less. In
further examples, the essay shows how technology allows for the linking of ideas that
may never have been connected in the past (like medicine and economic models),
pushing people to think in new ways. Examples are persuasive and fully developed; rea-
soning is logically sound and well supported.
Ideas in the essay are connected logically, with effective transitions used both
between paragraphs (“However,” or “In contrast to the statement”) and within para-
graphs. Sentence structure is varied and complex, and the essay clearly demonstrates
facility with the “conventions of standard written English (i.e., grammar, usage, and
mechanics)” (see Issue Scoring Guide, pages 37–38), with only minor errors appearing.
Thus, this essay meets all the requirements for receiving a top score, a 6.
Score 5 Response
Surely many of us have expressed the following sentiment, or some variation on it,
during our daily commutes to work: “People are getting so stupid these days!”
Surrounded as we are by striding and strident automatons with cell phones glued to
their ears, PDAs gripped in their palms, and omniscient, omnipresent CNN gleaming
in their eyeballs, it’s tempting to believe that technology has isolated and infantilized
us, essentally transforming us into dependent, conformist morons best equipped to
sideswip one another in our SUV’s.
Furthermore, hanging around with the younger, pre-commute generation, whom
tech-savviness seems to have rendered lethal, is even less reassuring. With “Teen
People” style trends shooting through the air from tiger-striped PDA to zebra-striped
PDA, and with the latest starlet gossip zipping from juicy Blackberry to teeny, turbo-
charged cell phone, technology seems to support young people’s worst tendencies to
follow the crowd. Indeed, they have seemingly evolved into intergalactic conformity
police. After all, today’s tech-aided teens are, courtesy of authentic, hands-on video
games, literally trained to kill; courtesy of chat and instant text messaging, they have
their own language; they even have tiny cameras to efficiently photodocument your
fashion blunders! Is this adolescence, or paparazzi terrorist training camp?
With all this evidence, it’s easy to believe that tech trends and the incorporation
This ebook was issued to HUAN-LIN CHANG, order #14803770223. Unlawful distribution of this ebook is prohibited.
Customer name HUAN-LIN CHANG , Order Id: 14803770223
of technological wizardry into our everyday lives have served mostly to enforce
conformity, promote dependence, heighten comsumerism and materialism, and
generally create a culture that values self-absorption and personal entitlement over
cooperation and collaboration. However, I argue that we are merely in the inchoate
stages of learning to live with technology while still loving one another. After all, even
given the examples provided earlier in this essay, it seems clear that technology hasn’t
impaired our thinking and problem-solving capacities. Certainly it has incapacitated our
behavior and manners; certainly our values have taken a severe blow. However, we are
inarguably more efficient in our badness these days. We’re effective worker bees of
ineffectiveness!
If technology has so increased our senses of self-efficacy that we can become
veritable agents of the awful, virtual CEO’s of selfishness, certainly it can be beneficial.
Harnessed correctly, technology can improve our ability to think and act for ourselves.
The first challenge is to figure out how to provide technology users with some direly-
needed direction.
Reader Commentary
The language of this essay clearly illustrates both its strengths and weaknesses. The
flowery and sometimes uncannily keen descriptions are often used to powerful effect,
but at other times, this descriptive language results in errors in syntax. See, for exam-
ple, the problems of parallelism in the second to last sentence of paragraph 2 (“After
all, today’s tech-aided teens . . . ”).
There is consistent evidence of facility with syntax and complex vocabulary (“Sur-
rounded as we are by striding and strident automatons with cell phones glued to their
ears, PDAs gripped in their palms, and omniscient, omnipresent CNN gleaming in their
eyeballs, it’s tempting to believe . . . ”). Such lucid prose, however, is often countered
with an over-reliance upon abstractions and tangential reasoning (what does the fact
that video games “literally train [teens] to kill” have to do with the use or deterioration
of thinking abilities, for example?).
Because this essay takes a complex approach to the issue (arguing, in effect, that
technology neither enhances nor reduces our ability to think for ourselves, but can be
used to do one or the other depending on the user) and because the author makes use
of “appropriate vocabulary and sentence variety” (see Issue Scoring Guide, pages
37–38), a score of 5 is appropriate.
Score 4 Response
In all actuality, I think it is more probable that our bodies will surely deteriorate long
before our minds do in any significant amount. Who can’t say that technology has
made us lazier, but that’s the key word, lazy, not stupid. The ever increasing amount of
technology that we incorporate into our daily lives makes people think and learn every
day, possibly more than ever before. Our abilities to think, learn, philosophize, etc. may
even reach limits never dreamed of before by average people. Using technology to
solve problems will continue to help us realize our potential as a human race.
If you think about it, using technology to solve more complicating problems gives
humans a chance to expand their thinking and learning, opening up whole new worlds
for many people. Many of these people are glad for the chance to expand their
horizons by learning more, going to new places, and trying new things. If it wasn’t for
20
GRE Analytical Writing
This ebook was issued to HUAN-LIN CHANG, order #14803770223. Unlawful distribution of this ebook is prohibited.
Customer name HUAN-LIN CHANG , Order Id: 14803770223
the invention of new technological devices, I wouldn’t be sitting at this computer trying
to philosophize about technology. It would be extremely hard for children in much
poorer countries to learn and think for themselves with out the invention of the
internet. Think what an impact the printing press, a technologically superior mackine at
the time, had on the ability of the human race to learn and think.
Right now we are seeing a golden age of technology, using it all the time during our
every day lives. When we get up there’s instant coffee and the microwave and all these
great things that help us get ready for our day. But we aren’t allowing our minds to
deteriorate by using them, we are only making things easier for ourselves and saving
time for other important things in our days. Going off to school or work in our cars
instead of a horse and buggy. Think of the brain power and genius that was used to
come up with that single invention that has changed the way we move across this
globe.
Using technology to solve our continually more complicated problems as a human
race is definately a good thing. Our ability to think for ourselves isn’t deteriorating, it’s
continuing to grow, moving on to higher though functions and more ingenious ideas.
The ability to use what technology we have is an example
Reader Commentary
This essay meets all the criteria of a 4-level essay. The writer develops a clear position
(“Using technology to solve problems will continue to help us realize our potential as a
human race”). The position is then developed with relevant reasons (“using technology
to solve more complicat[ed] problems gives humans a chance to expand their thinking
and learning . . . ” and “we are seeing a golden age of technology”). Point 1, “Using tech-
nology,” is supported with the simple, but relevant notions that technology allows us
access to information and abilities to which we would not normally have access. Simi-
larly, point 2, “the golden age,” is supported by the basic description of our technologi-
cally saturated social condition. Though the development and organization of the essay
does suffer from an occasional misstep (see paragraph 3’s abrupt progression from
coffeepots to the benefits of technology to cars), the essay as a whole flows smoothly
and logically from one idea to the next.
It is useful to compare this essay to the 3-level essay presented next. Though they
both utilize some very superficial discussion and often fail to probe deeply into the
issue, this writer does, however, take the analysis a step further. In paragraph 2, the dis-
tinction between this essay and the next one (the 3-level response) can most clearly be
seen. To support the notion that advances in technology actually help increase thinking
ability, the writer draws a clever parallel between the promise of modern, sophisticated
technology (computer) and the equally substantial/pervasive technology of the past
(printing press).
Like the analysis, the language in this essay clearly meets the requirements for a
score of 4. The writer displays sufficient control of language and the conventions of
standard written English. The preponderance of mistakes are of a cosmetic nature
(“using technology to solve more complicating problems”). There is a sentence frag-
ment (“Going off . . . ”) along with a comma splice (“Our ability . . . isn’t deteriorating,
it’s continuing to grow . . . ”) in paragraph 4. These errors, though, are minor and do
not interfere with the clarity of the ideas being presented.
21
GRE Analytical Writing
This ebook was issued to HUAN-LIN CHANG, order #14803770223. Unlawful distribution of this ebook is prohibited.
Customer name HUAN-LIN CHANG , Order Id: 14803770223
Score 3 Response
There is no current proof that advancing technology will deteriorate the ability of
humans to think. On the contrary, advancements in technology had advanced our vast
knowledge in many fields, opening opportunities for further understanding and
achievement. For example, the problem of dibilitating illnesses and diseases such as
alzheimer’s disease is slowing being solved by the technological advancements in stem
cell research. The future ability of growing new brain cells and the possibility to reverse
the onset of alzheimer’s is now becoming a reality. This shows our initiative as humans
to better our health demonstrates greater ability of humans to think.
One aspect where the ability of humans may initially be seen as an example of
deteriorating minds is the use of internet and cell phones. In the past humans had to
seek out information in many different enviroments and aspects of life. Now humans
can sit in a chair and type anything into a computer and get an answer. Our reliance on
this type of technology can be detrimental if not regulated and regularily substituted
for other information sources such as human interactions and hands on learning. I
think if humans understand that we should not have such a reliance on computer
technology, that we as a species will advance further by utilizing the opportunity of
computer technology as well as the other sources of information outside of a
computer. Supplementing our knowledge with internet access is surely a way for
technology to solve problems while continually advancing the human race.
Reader Commentary
This essay never moves beyond a superficial discussion of the issue. The writer
attempts to develop two points: that advancements in technology have progressed our
knowledge in many fields and that supplementing rather than relying on technology is
“surely a way for technology to solve problems while continually advancing the human
race.” Each point, then, is developed with relevant but insufficient evidence. In dis-
cussing the ability of technology to advance knowledge in many fields (a broad subject
rife with possible examples), the writer uses only one limited and very brief example
from a specific field (medicine and stem-cell research).
Development of the second point is hindered by a lack of specificity and organiza-
tion. The writer creates what might most be comparable to an outline. The writer cites
a need for regulation/supplementation and warns of the detriment of over-reliance
upon technology. However, the explanation of both the problem and the solution is
vague and limited (“Our reliance . . . can be detrimental . . . If humans understand that
we should not have such a reliance . . . we will advance further”). There is neither
explanation of consequences nor clarification of what is meant by “supplementing.”
This second paragraph is a series of generalizations, which are loosely connected and
lack a much needed grounding.
In the essay, there are some minor language errors and a few more serious flaws
(e.g., “The future ability of growing new brain cells” or “One aspect where the ability of
humans may initially be seen as an example of deteriorating minds…”). Despite the
accumulation of such flaws, though, meaning is generally clear. This essay earns a
score of 3, then, primarily for its limited development.
22
GRE Analytical Writing
This ebook was issued to HUAN-LIN CHANG, order #14803770223. Unlawful distribution of this ebook is prohibited.
Customer name HUAN-LIN CHANG , Order Id: 14803770223
Score 2 Response
In recent centuries, humans have developed the technology very rapidly, and you may
accept some merit of it, and you may see a distortion in society occured by it. To be
lazy for human in some meaning is one of the fashion issues in thesedays. There are
many symptoms and resons of it. However, I can not agree with the statement that the
technology make humans to be reluctant to thinkng thoroughly.
Of course, you can see the phenomena of human laziness along with developed
technology in some place. However, they would happen in specific condition, not
general. What makes human to be laze of thinking is not merely technology, but the
the tendency of human that they treat them as a magic stick and a black box. Not
understanding the aims and theory of them couses the disapproval problems.
The most important thing to use the thechnology, regardless the new or old, is to
comprehend the fundamental idea of them, and to adapt suit tech to tasks in need.
Even if you recognize a method as a all-mighty and it is extremely over-spec to your
needs, you can not see the result you want. In this procedure, humans have to consider
as long as possible to acquire adequate functions. Therefore, humans can not escape
from using their brain.
In addition, the technology as it is do not vain automatically, the is created by
humans. Thus, the more developed tech and the more you want a convenient life,
the more you think and emmit your creativity to breakthrough some banal method
sarcastically.
Consequently, if you are not passive to the new tech, but offensive to it, you would
not lose your ability to think deeply. Furthermore, you may improve the ability by
adopting it.
Reader Commentary
The language of this essay is what most clearly links it to the score point of 2. Amidst
sporadic moments of clarity, this essay is marred by serious errors in grammar, usage,
and mechanics that often interfere with meaning. It is unclear what the writer means
when he/she states, “To be lazy for human in some meaning is one of the fashion issues
in thesedays,” or “ . . . to adapt suit tech to tasks in need.” Despite such severe flaws, the
writer has made an obvious attempt to respond to the prompt (“I can not agree with
the statement that the technology make humans to be reluctant to thinking thor-
oughly”) as well as an unclear attempt to support such an assertion (“Not understand-
ing the aims and theory of them [technology] couses the disapproval problems” and
“The most important thing to use the thechnology . . . is to comprehend the fundamen-
tal idea of them”). Holistically, the essay displays a seriously flawed but not fundamen-
tally deficient attempt to develop and support its claims.
(Note: In this SPECIFIC case, the analysis is tied directly to the language. As the
language falters, so too does the analysis.)
23
GRE Analytical Writing
This ebook was issued to HUAN-LIN CHANG, order #14803770223. Unlawful distribution of this ebook is prohibited.
Customer name HUAN-LIN CHANG , Order Id: 14803770223
Score 1 Response
Humans have invented machines but they have forgot it and have started everything
technically so clearly their thinking process is deterioating.
Reader Commentary
The essay is clearly on topic, as evidenced by the writer’s usage of the more significant
terms from the prompt: “technically” (technologically), “humans”, “thinking” (think)
and “deterioating” (deteriorate). Such usage is the only clear evidence of understand-
ing. Meaning aside, the brevity of the essay (one sentence) clearly indicates the writer’s
inability to develop a response that addresses the specific instructions given (“Discuss
the extent to which you agree or disagree with the statement above and explain your
reasoning for the position you take”).
The language, too, is clearly one-level, as the sentence fails to achieve coherence.
The coherent phrases in this one-sentence response are those tied to the prompt:
“Humans have invented machines” and “their thinking process is deterioating.” Other-
wise, the point being made is unclear.
24
GRE Analytical Writing
This ebook was issued to HUAN-LIN CHANG, order #14803770223. Unlawful distribution of this ebook is prohibited.
Customer name HUAN-LIN CHANG , Order Id: 14803770223
Analyze an Argument Task
Understanding the Argument Task
The Analyze an Argument task assesses your ability to understand, analyze, and evalu-
ate arguments according to specific instructions and to clearly convey your evaluation
in writing. The task consists of a brief passage in which the author makes a case for
some course of action or interpretation of events by presenting claims backed by rea-
sons and evidence. Your task is to discuss the logical soundness of the author’s case
according to the specific instructions by critically examining the line of reasoning. This
task requires you to read the argument and instructions very carefully. You might want
to read them more than once and possibly make brief notes about points you want to
develop more fully in your response. In reading the argument, you should pay special
attention to
bwhat is offered as evidence, support, or proof
bwhat is explicitly stated, claimed, or concluded
bwhat is assumed or supposed, perhaps without justification or proof
bwhat is not stated, but necessarily follows from what is stated
In addition, you should consider the structure of the argument the way in which
these elements are linked together to form a line of reasoning; that is, you should rec-
ognize the separate, sometimes implicit steps in the thinking process and consider
whether the movement from each one to the next is logically sound. In tracing this line,
look for transition words and phrases that suggest that the author is attempting to
make a logical connection (e.g., however, thus, therefore, evidently, hence, in conclusion).
An important part of performing well on the Argument task is remembering what
you are not being asked to do. You are not being asked to discuss whether the state-
ments in the argument are true or accurate. You are not being asked to agree or dis-
agree with the position stated. You are not being asked to express your own views on
the subject being discussed (as you were in the Issue task). Instead, you are being asked
to evaluate the logical soundness of an argument of another writer according to spe-
cific instructions and, in doing so, to demonstrate the critical thinking, perceptive read-
ing, and analytical writing skills that university faculty consider important for success
in graduate school.
It is important that you address the argument according to the specific instruc-
tions. Each task is accompanied by one of the following sets of instructions.
bWrite a response in which you discuss what specific evidence is needed to eval-
uate the argument and explain how the evidence would weaken or strengthen
the argument.
bWrite a response in which you examine the stated and/or unstated assumptions
of the argument. Be sure to explain how the argument depends on these
assumptions and what the implications are for the argument if the assumptions
prove unwarranted.
bWrite a response in which you discuss what questions would need to be
answered in order to decide whether the recommendation and the argument on
which it is based are reasonable. Be sure to explain how the answers to these
questions would help to evaluate the recommendation.
25
GRE Analytical Writing
This ebook was issued to HUAN-LIN CHANG, order #14803770223. Unlawful distribution of this ebook is prohibited.
Customer name HUAN-LIN CHANG , Order Id: 14803770223
bWrite a response in which you discuss what questions would need to be
answered in order to decide whether the advice and the argument on which it
is based are reasonable. Be sure to explain how the answers to these questions
would help to evaluate the advice.
bWrite a response in which you discuss what questions would need to be
answered in order to decide whether the recommendation is likely to have the
predicted result. Be sure to explain how the answers to these questions would
help to evaluate the recommendation.
bWrite a response in which you discuss what questions would need to be
answered in order to decide whether the prediction and the argument on which
it is based are reasonable. Be sure to explain how the answers to these ques-
tions would help to evaluate the prediction.
bWrite a response in which you discuss one or more alternative explanations
that could rival the proposed explanation and explain how your explanation(s)
can plausibly account for the facts presented in the argument.
bWrite a response in which you discuss what questions would need to be
addressed in order to decide whether the conclusion and the argument on
which it is based are reasonable. Be sure to explain how the answers to the
questions would help to evaluate the conclusion.
“Analyze an Argument” is primarily a critical thinking task requiring a written
response. Consequently, the analytical skills displayed in your evaluation carry great
weight in determining your score; however, the clarity with which you convey ideas is
also important to your overall score.
Understanding the Context for Writing: Purpose and Audience
The purpose of the task is to see how well equipped you are to insightfully evaluate an
argument written by someone else and to effectively communicate your evaluation in
writing to an academic audience. Your audience consists of GRE readers carefully
trained to apply the scoring criteria identified in the scoring guide for the Analyze an
Argument task (see page 39–40).
To get a clearer idea of how GRE readers apply the Argument scoring criteria to
actual essays, you should review scored sample Argument essay responses and reader
commentary. The sample responses, particularly at the 5 and 6 score levels, will show
you a variety of successful strategies for organizing and developing an insightful evalu-
ation. The reader commentary discusses specific aspects of analytical writing, such as
cogency of ideas, development and support, organization, syntactic variety, and facility
with language. For each response, the reader commentary will point out aspects that
are particularly effective and insightful as well as any that detract from the overall
effectiveness of the responses.
Preparing for the Argument Task
Because the Argument task is meant to assess analytical writing and informal reason-
ing skills that you have developed throughout your education, it has been designed so
as not to require any specific course of study or to advantage students with a particular
type of training. Many college textbooks on rhetoric and composition have sections on
informal logic and critical thinking that might prove helpful, but even these might be
26
GRE Analytical Writing
This ebook was issued to HUAN-LIN CHANG, order #14803770223. Unlawful distribution of this ebook is prohibited.
Customer name HUAN-LIN CHANG , Order Id: 14803770223
27
GRE Analytical Writing
more detailed and technical than the task requires. You will not be expected to know
methods of analysis or technical terms. For instance, in one topic an elementary school
principal might conclude that the new playground equipment has improved student
attendance because absentee rates have declined since it was installed. You will not
need to see that the principal has committed the post hoc, ergo propter hoc fallacy; you
will simply need to see that there are other possible explanations for the improved
attendance, to offer some commonsense examples, and perhaps to suggest what would
be necessary to verify the conclusion. For instance, absentee rates might have de -
creased because the climate was mild. This would have to be ruled out in order for the
principal’s conclusion to be valid.
Although you do not need to know special analytical techniques and terminology,
you should be familiar with the directions for the Argument task and with certain key
concepts, including the following:
balternative explanation: a possible competing version of what might have
caused the events in question; an alternative explanation undercuts or qualifies
the original explanation because it too can account for the observed facts
banalysis: the process of breaking something (e.g., an argument) down into its
component parts in order to understand how they work together to make up
the whole
bargument: a claim or a set of claims with reasons and evidence offered as sup-
port; a line of reasoning meant to demonstrate the truth or falsehood of some-
thing
bassumption: a belief, often unstated or unexamined, that someone must hold in
order to maintain a particular position; something that is taken for granted but
that must be true in order for the conclusion to be true
bconclusion: the end point reached by a line of reasoning, valid if the reasoning
is sound; the resulting assertion
bcounterexample: an example, real or hypothetical, that refutes or disproves a
statement in the argument
bevaluation: an assessment of the quality of evidence and reasons in an argu-
ment and of the overall merit of an argument
An excellent way to prepare for the Analyze an Argument task is to practice writing
on some of the published Argument topics. There is no one way to practice that is best
for everyone. Some prefer to start practicing without adhering to the 30-minute time
limit. If you follow this approach, take all the time you need to evaluate the argument.
No matter which approach you take, you should
bcarefully read the argument and the specific instructions you might want to
read them over more than once
bidentify as many of the argument’s claims, conclusions, and underlying
assumptions as possible and evaluate their quality
bthink of as many alternative explanations and counterexamples as you can
bthink of what specific additional evidence might weaken or lend support to the
claims
bask yourself what changes in the argument would make the reasoning more
sound
This ebook was issued to HUAN-LIN CHANG, order #14803770223. Unlawful distribution of this ebook is prohibited.
Customer name HUAN-LIN CHANG , Order Id: 14803770223
Write down each of these thoughts as a brief note. When you’ve gone as far as you
can with your evaluation, look over the notes and put them in a good order for discus-
sion (perhaps by numbering them). Then write an evaluation according to the specific
instructions by fully developing each point that is relevant to those instructions. Even
if you choose not to write a full essay response, you should find it very helpful to prac-
tice evaluating a few of the arguments and sketching out your responses. When you
become quicker and more confident, you should practice writing some Argument
responses within the 30-minute time limit so that you will have a good sense of how to
pace yourself in the actual test. For example, you will not want to discuss one point so
exhaustively or to provide so many equivalent examples that you run out of time to
make your other main points.
You might want to get feedback on your response(s) from a writing instructor, a
philosophy teacher, or someone who emphasizes critical thinking in his or her course.
It can also be very informative to trade papers on the same topic with fellow students
and discuss one another’s responses in terms of the scoring guide. Focus not so much
on giving the “right scores” as on seeing how the papers meet or miss the performance
standards for each score point and what you therefore need to do in order to improve.
How to Interpret Numbers, Percentages, and Statistics in Argument Topics
Some arguments contain numbers, percentages, or statistics that are offered as evi-
dence in support of the argument’s conclusion. For example, an argument might claim
that a certain community event is less popular this year than it was last year because
only 100 people attended this year in comparison with 150 last year, a 33 percent
decline in attendance. It is important to remember that you are not being asked to do a
mathematical task with the numbers, percentages, or statistics. Instead you should
evaluate these as evidence intended to support the conclusion. In the example above,
the conclusion is that a community event has become less popular. You should ask
yourself: does the difference between 100 people and 150 people support that conclu-
sion? Note that, in this case, there are other possible explanations; for example, the
weather might have been much worse this year, this year’s event might have been held
at an inconvenient time, the cost of the event might have gone up this year, or there
might have been another popular event this year at the same time. Each of these could
explain the difference in attendance, and thus would weaken the conclusion that the
event was “less popular.” Similarly, percentages might support or weaken a conclusion
depending on what actual numbers the percentages represent. Consider the claim that
the drama club at a school deserves more funding because its membership has
increased by 100 percent. This 100 percent increase could be significant if there had
been 100 members and now there are 200 members, whereas the increase would be
much less significant if there had been 5 members and now there are 10. Remember
that any numbers, percentages, or statistics in Argument tasks are used only as evi-
dence in support of a conclusion, and you should always consider whether they actu-
ally support the conclusion.
The Form of Your Response
You are free to organize and develop your response in any way that you think will effec-
tively communicate your evaluation of the argument. Your response may, but need not,
incorporate particular writing strategies learned in English composition or writing-
intensive college courses. GRE readers will not be looking for a particular develop-
28
GRE Analytical Writing
This ebook was issued to HUAN-LIN CHANG, order #14803770223. Unlawful distribution of this ebook is prohibited.
Customer name HUAN-LIN CHANG , Order Id: 14803770223
mental strategy or mode of writing. In fact, when GRE readers are trained, they review
hundreds of Argument responses that, although highly diverse in content and form,
display similar levels of critical thinking and analytical writing. Readers will see, for
example, some essays at the 6 score level that begin by briefly summarizing the argu-
ment and then explicitly stating and developing the main points of the evaluation. The
readers know that a writer can earn a high score by developing several points in an
evaluation or by identifying a central feature in the argument and developing that eval-
uation extensively. You might want to look at the sample Argument responses, particu-
larly at the 5 and 6 score levels, to see how other writers have successfully developed
and organized their responses.
You should make choices about format and organization that you think support
and enhance the overall effectiveness of your evaluation. This means using as many
or as few paragraphs as you consider appropriate for your response for example, cre-
ating a new paragraph when your discussion shifts to a new point of evaluation. You
might want to organize your evaluation around the structure of the argument itself,
discussing the argument line by line. Or you might want to first point out a central
problem and then move on to discuss related weaknesses in the argument’s line of rea-
soning. Similarly, you might want to use examples if they help illustrate an important
point in your evaluation or move your discussion forward (remember, however, that, in
terms of your ability to perform the Argument task effectively, it is your critical think-
ing and analytical writing, not your ability to come up with examples, that is being
assessed). What matters is not the form the response takes, but how insightfully you
evaluate the argument and how articulately you communicate your evaluation to aca-
demic readers within the context of the task.
Sample Argument Task
In surveys Mason City residents rank water sports (swimming, boating, and fish -
ing) among their favorite recreational activities. The Mason River flowing through
the city is rarely used for these pursuits, however, and the city park department
devotes little of its budget to maintaining riverside recreational facilities. For years
there have been complaints from residents about the quality of the river’s water
and the river’s smell. In response, the state has recently announced plans to clean
up Mason River. Use of the river for water sports is, therefore, sure to increase. The
city government should for that reason devote more money in this year’s budget to
riverside recreational facilities.
Write a response in which you examine the stated and/or unstated assumptions of
the argument. Be sure to explain how the argument depends on the assumptions
and what the implications are if the assumptions prove unwarranted.
Strategies for This Topic
This argument cites a survey to support the prediction that the use of the Mason River
is sure to increase and thus recommends that the city government should devote more
money in this year’s budget to the riverside recreational facilities.
In developing your evaluation, you are asked to examine the argument’s stated
and/or unstated assumptions and discuss what the implications are if the assumptions
29
GRE Analytical Writing
This ebook was issued to HUAN-LIN CHANG, order #14803770223. Unlawful distribution of this ebook is prohibited.
Customer name HUAN-LIN CHANG , Order Id: 14803770223
prove unwarranted. A successful response, then, must discuss both the argument’s
assumptions AND the implications of these assumptions for the argument. A response
that does not address these aspects of the task will not receive a score of 4 or higher,
regardless of the quality of its other features.
Though responses may well raise other points not mentioned here and need not
mention all of these points, some assumptions of the argument, and some ways in
which the argument depends on those assumptions, include:
bThe assumption that people who rank water sports “among their favorite recre-
ational activities” are actually likely to participate in them. (It is possible that
they just like to watch them.) This assumption underlies the claim that use of
the river for water sports is sure to increase after the state cleans up the Mason
River and that the city should for that reason devote more money to riverside
recreational facilities.
bThe assumption that what residents say in surveys can be taken at face value.
(It is possible that survey results exaggerate the interest in water sports.) This
assumption underlies the claim that use of the river for water sports is sure to
increase after the state cleans up the Mason River and that the city should for
that reason devote more money to riverside recreational facilities.
bThe assumption that Mason City residents would actually want to do water
sports in the Mason River. (As recreational activities, it is possible that water
sports are regarded as pursuits for vacations and weekends away from the city.)
This assumption underlies the claim that use of the river for water sports is
sure to increase after the state cleans up the Mason River and that the city
should for that reason devote more money to riverside recreational facilities.
bThe assumption that the park department devoting little of its budget to main-
taining riverside recreational facilities means that these facilities are inade-
quately maintained. This assumption underlies the claim that the city should
devote more money in this year’s budget to riverside recreational facilities. If
current facilities are adequately maintained, then increased funding might not
be needed even if recreational use of the river does increase.
bThe assumption that the riverside recreational facilities are facilities designed
for people who participate in water sports and not some other recreational pur-
suit. This assumption underlies the claim that the city should devote more
money in this year’s budget to riverside recreational facilities.
bThe assumption that the dirtiness of the river is the cause of its being little used
and that cleaning up the river will be sufficient to increase recreational use of
the river. (Residents might have complained about the water quality and smell
even if they had no desire to boat, swim, or fish in the river.) This assumption
underlies the claim that the state’s plan to clean up the river will result in
increased use of the river for water sports.
bThe assumption that the complaints about the river are numerous and signifi-
cant. This assumption motivates the state’s plan to clean up the river and
underlies the claim that use of the river for water sports is sure to increase.
(Perhaps the complaints are coming from a very small minority; in which case
cleaning the river might be a misuse of state funds.)
bThe assumption that the state’s cleanup will occur soon enough to require
adjustments to this year’s budget. This assumption underlies the claim that the
30
GRE Analytical Writing
This ebook was issued to HUAN-LIN CHANG, order #14803770223. Unlawful distribution of this ebook is prohibited.
Customer name HUAN-LIN CHANG , Order Id: 14803770223
31
GRE Analytical Writing
city should devote more money in this year’s budget to riverside recreational
facilities.
bThe assumption that the cleanup, when it happens, will benefit those parts of
the river accessible from the city’s facilities. This assumption underlies the
claim that the city should devote more money to riverside recreational facili-
ties.
bThe assumption that the city government ought to devote more attention to
maintaining a recreational facility if demand for that facility increases.
bThe assumption that the city should finance the new project and not some
other agency or group (public or private).
Should any of the above assumptions prove unwarranted, the implications are:
bThat the logic of the argument falls apart/ is invalid/ is unsound.
bThat the state and city are spending their funds unnecessarily.
Essay Responses and Reader Commentary
Score 6 Response *
While it may be true that the Mason City government ought to devote more money to
riverside recreational facilities, this author’s argument does not make a cogent case
for increased resources based on river use. It is easy to understand why city residents
would want a cleaner river, but this argument is rife with holes and assumptions, and
thus, not strong enough to lead to increased funding.
Citing surveys of city residents, the author reports city resident’s love of water
sports. It is not clear, however, the scope and validity of that survey. For example, the
survey could have asked residents if they prefer using the river for water sports or
would like to see a hydroelectric dam built, which may have swayed residents toward
river sports. The sample may not have been representative of city residents, asking
only those residents who live upon the river. The survey may have been 10 pages long,
with 2 questions dedicated to river sports. We just do not know. Unless the survey is
fully representative, valid, and reliable, it can not be used to effectively back the
author’s argument.
Additionally, the author implies that residents do not use the river for swimming,
boating, and fishing, despite their professed interest, because the water is polluted and
smelly. While a polluted, smelly river would likely cut down on river sports, a concrete
connection between the resident’s lack of river use and the river’s current state is not
effectively made. Though there have been complaints, we do not know if there have
been numerous complaints from a wide range of people, or perhaps from one or two
individuals who made numerous complaints. To strengthen his/her argument, the
author would benefit from implementing a normed survey asking a wide range of
residents why they do not currently use the river.
Building upon the implication that residents do not use the river due to the quality
of the river’s water and the smell, the author suggests that a river clean up will result
in increased river usage. If the river’s water quality and smell result from problems
which can be cleaned, this may be true. For example, if the decreased water quality
*All responses in this publication are reproduced exactly as written, including errors, mis-
spellings, etc., if any.
This ebook was issued to HUAN-LIN CHANG, order #14803770223. Unlawful distribution of this ebook is prohibited.
Customer name HUAN-LIN CHANG , Order Id: 14803770223
and aroma is caused by pollution by factories along the river, this conceivably could be
remedied. But if the quality and aroma results from the natural mineral deposits in the
water or surrounding rock, this may not be true. There are some bodies of water which
emit a strong smell of sulphur due to the geography of the area. This is not something
likely to be afffected by a clean-up. Consequently, a river clean up may have no impact
upon river usage. Regardless of whether the river’s quality is able to be improved or
not, the author does not effectively show a connection between water quality and river
usage.
A clean, beautiful, safe river often adds to a city’s property values, leads to increased
tourism and revenue from those who come to take advantage of the river, and a better
overall quality of life for residents. For these reasons, city government may decide to
invest in improving riverside recreational facilities. However, this author’s argument is
not likely significantly persuade the city goverment to allocate increased funding.
Reader Commentary
This insightful response identifies important assumptions and thoroughly examines
their implications. The proposal to spend more on riverside recreational facilities rests
on a number of questionable assumptions, namely that:
bThe survey provides a reliable basis for budget planning;
bThe river’s pollution and odor are the only reasons for its limited recreational
use;
bEfforts to clean the water and remove the odor will be successful.
By showing that each assumption is highly suspect, this essay demonstrates the
weakness of the entire argument. For example, paragraph 2 points out that the survey
might not have used a representative sample, might have offered limited choices, and
might have contained very few questions on water sports. Paragraph 3 examines the
tenuous connection between complaints and limited use of the river for recreation.
Complaints about water quality and odor may be coming from only a few people, and
even if such complaints are numerous, other completely different factors may be much
more significant in reducing river usage. Finally, paragraph 4 explains that certain geo-
logic features may prevent effective river cleanup. Details such as these provide com-
pelling support.
In addition, careful organization insures that each new point builds upon the pre-
vious ones. Note, for example, the clear transitions at the beginning of paragraphs 3
and 4, as well as the logical sequence of sentences within paragraphs (specifically para-
graph 4).
Although this essay does contain minor errors, it still conveys ideas fluently. Note
the effective word choices (e.g.,“rife with . . . assumptions” and “may have swayed resi-
dents”). In addition, sentences are not merely varied; they also display skillful embed-
ding of subordinate elements. Note, for example, the sustained parallelism in the first
sentence of the concluding paragraph.
Since this response offers a cogent examination of the argument and also conveys
meaning skillfully, it earns a score of 6.
32
GRE Analytical Writing
This ebook was issued to HUAN-LIN CHANG, order #14803770223. Unlawful distribution of this ebook is prohibited.
Customer name HUAN-LIN CHANG , Order Id: 14803770223
Score 5 Response
The author of this proposal to increase the budget for Mason City riverside recreational
facilities offers an interesting argument but to move forward on the proposal would
definitely require more information and thought. While the correlations stated are
logical and probable, there may be hidden factors that prevent the City from diverting
resources to this project.
For example, consider the survey rankings among Mason City residents. The
thought is that such high regard for water sports will translate into usage. But, survey
responses can hardly be used as indicators of actual behavior. Many surveys con -
ducted after the winter holidays reveal people who list exercise and weight loss as a
top priority. Yet every profession does not equal a new gym membership. Even the
wording of the survey results remain ambiguous and vague. While water sports may
be among the residents’ favorite activities, this allows for many other favorites. What
remains unknown is the priorities of the general public. Do they favor these water
sports above a softball field or soccer field? Are they willing to sacrifice the municipal
golf course for better riverside facilities? Indeed the survey hardly provides enough
information to discern future use of improved facilities.
Closely linked to the surveys is the bold assumption that a cleaner river will result
in increased usage. While it is not illogical to expect some increase, at what level will
people begin to use the river? The answer to this question requires a survey to find out
the reasons our residents use or do not use the river. Is river water quality the primary
limiting factor to usage or the lack of docks and piers? Are people more interested in
water sports than the recreational activities that they are already engaged in? These
questions will help the city government forecast how much river usage will increase
and to assign a proportional increase to the budget.
Likewise, the author is optimistic regarding the state promise to clean the river.
We need to hear the source of the voices and consider any ulterior motives. Is this a
campaign year and the plans a campaign promise from the state representative? What
is the timeline for the clean-up effort? Will the state fully fund this project? We can
imagine the misuse of funds in renovating the riverside facilities only to watch the
new buildings fall into dilapidation while the state drags the river clean-up.
Last, the author does not consider where these additional funds will be diverted
from. The current budget situation must be assessed to determine if this increase can
be afforded. In a sense, the City may not be willing to draw money away from other
key projects from road improvements to schools and education. The author naively
assumes that the money can simply appear without forethought on where it will come
from.
Examining all the various angles and factors involved with improving riverside
recreational facilities, the argument does not justify increasing the budget. While the
proposal does highlight a possibility, more information is required to warrant any
action.
Reader Commentary
Each paragraph in the body of this perceptive essay identifies and examines an un -
stated assumption that is crucial to the argument. The major assumptions discussed are:
bThat a survey can accurately predict behavior,
bThat cleaning the river will, in itself, increase recreational usage,
bThat state plans to clean the river will actually be realized,
bThat Mason City can afford to spend more on riverside recreational facilities.
33
GRE Analytical Writing
This ebook was issued to HUAN-LIN CHANG, order #14803770223. Unlawful distribution of this ebook is prohibited.
Customer name HUAN-LIN CHANG , Order Id: 14803770223
Support within each paragraph is both thoughtful and thorough. Paragraph 2, for
example, points out vagueness in the wording of the survey: Even if water sports rank
among the favorite recreational activities of Mason City residents, other sports may still
be much more popular. Thus, if the first assumption proves unwarranted, the argument
to fund riverside facilities rather than soccer fields or golf courses becomes much
weaker. Paragraph 4 considers several reasons why river cleanup plans may not be suc-
cessful (the plans may be nothing more than campaign promises, or funding may not be
adequate). Thus, the weakness of the third assumption undermines the argument that
river recreation will increase and riverside improvements will be needed at all.
Instead of dismissing each assumption in isolation, this response places them in a
logical order and considers their connections. Note the appropriate transitions between
and within paragraphs, clarifying the links among the assumptions (e.g., Closely linked
to the surveys . . . ” or “The answer to this question requires . . . ”).
Along with strong development, this response also displays facility with language.
Minor errors in punctuation are present, but word choices are apt and sentences suit-
ably varied in pattern and length. The response uses a number of rhetorical questions,
but the implied answers are always clear enough to support the points being made.
Thus, the response satisfies all requirements for a score of 5, but its development is
not thorough or compelling enough for a 6.
Score 4 Response
The problem with the arguement is the assumption that if the Mason River were
cleaned up, that people would use it for water sports and recreation. This is not
necessarily true, as people may rank water sports among their favorite recreational
activities, but that does not mean that those same people have the financial ability,
time or equipment to pursue those interests.
However, even if the writer of the arguement is correct in assuming that the Mason
River will be used more by the city’s residents, the arguement does not say why the
recreational facilities need more money. If recreational facilities already exist along the
Mason River, why should the city allot more money to fund them? If the recreational
facilities already in existence will be used more in the coming years, then they will be
making more money for themselves, eliminating the need for the city government to
devote more money to them.
According to the arguement, the reason people are not using the Mason River for
water sports is because of the smell and the quality of water, not because the
recreational facilities are unacceptable.
If the city government alloted more money to the recreational facilities, then the
budget is being cut from some other important city project. Also, if the assumptions
proved unwarranted, and more people did not use the river for recreation, then much
money has been wasted, not only the money for the recreational facilities, but also the
money that was used to clean up the river to attract more people in the first place.
Reader Commentary
This competent response identifies some important unstated assumptions:
bThat cleaning up the Mason River will lead to increased recreational use,
bThat existing facilities along the river need more funding.
34
GRE Analytical Writing
This ebook was issued to HUAN-LIN CHANG, order #14803770223. Unlawful distribution of this ebook is prohibited.
Customer name HUAN-LIN CHANG , Order Id: 14803770223
35
GRE Analytical Writing
Paragraph 1 offers reasons why the first assumption is questionable (e.g., residents
may not have the necessary time or money for water sports). Similarly, paragraphs 2
and 3 explain that riverside recreational facilities may already be adequate and may, in
fact, produce additional income if usage increases. Thus, the response is adequately
developed and satisfactorily organized to show how the argument depends on ques-
tionable assumptions.
This essay does not, however, rise to a score of 5 because it fails to consider several
other unstated assumptions (e.g., that the survey is reliable or that the efforts to clean
the river will be successful). Furthermore, the final paragraph makes some extraneous,
unsupported assertions of its own. Mason City may actually have a budget surplus so
that cuts to other projects will not be necessary, and cleaning the river may provide
other real benefits even if it is not used more for water sports.
This response is generally free of errors in grammar and usage and displays suffi-
cient control of language to support a score of 4.
Score 3 Response
Surveys are created to speak for the people; however, surveys do not always speak
for the whole community. A survey completed by Mason City residents concluded that
the residents enjoy water sports as a form of recreation. If that is so evident, why has
the river not been used? The blame can not be soley be placed on the city park
department. The city park department can only do as much as they observe. The real
issue is not the residents use of the river, but their desire for a more pleasant smell and
a more pleasant sight. If the city government cleans the river, it might take years for the
smell to go away. If the budget is changed to accomodate the clean up of the Mason
River, other problems will arise. The residents will then begin to complain about other
issues in their city that will be ignored because of the great emphasis being placed on
Mason River. If more money is taken out of the budget to clean the river an assumption
can be made. This assumption is that the budget for another part of city maintenance
or building will be tapped into to. In addition, to the budget being used to clean up
Mason River, it will also be allocated in increasing riverside recreational facilites. The
government is trying to appease its residents, and one can warrant that the role of
the government is to please the people. There are many assumptions being made;
however, the government can not make the assumption that people want the river to
be cleaned so that they can use it for recreational water activities. The government has
to realize the long term effects that their decision will have on the monetary value of
their budget.
Reader Commentary
Even though much of this essay is tangential, it offers some relevant examination of the
argument’s assumptions. The early sentences mention a questionable assumption (that
the survey results are reliable) but do not explain how the survey might have been
flawed. Then the response drifts to irrelevant matters a defense of the city park de-
partment, a prediction of budget problems, and the problem of pleasing city residents.
Some statements even introduce unwarranted assumptions that are not part of the orig-
inal argument (e.g., “The residents will then begin to complain about other issues,” and
“This assumption is that the budget for another part of city maintenance or building will
be tapped into.”). Near the end, the response does correctly note that city government
This ebook was issued to HUAN-LIN CHANG, order #14803770223. Unlawful distribution of this ebook is prohibited.
Customer name HUAN-LIN CHANG , Order Id: 14803770223
should not assume that residents want to use the river for recreation. Hence, the pro-
posal to increase funding for riverside recreational facilities may not be justified.
In summary, the language in this response is reasonably clear, but its examination
of unstated assumptions remains limited, and therefore the essay earns a score of 3.
Score 2 Response
This statement looks like logical, but there are some wrong sentences in it which is not
logical.
First, this statement mentions raking water sports as their favorite recreational
activities at the first sentence. However, it seems to have a ralation between the first
sentence and the setence which mentions that increase the quality of the river’s water
and the river’s smell. This is a wrong cause and result to solve the problem.
Second, as a reponse to the complaints from residents, the state plan to clean up the
river. As a result, the state expects that water sports will increase. When you look at
two sentences, the result is not appropriate for the cause.
Third, the last statement is the conclusion. However, even though residents rank
water sports, the city government might devote the budget to another issue. This
statement is also a wrong cause and result.
In summary, the statement is not logical because there are some errors in it. The
supporting setences are not strong enough to support this issue.
Reader Commentary
Although this essay appears to be carefully organized, it does not follow the directions
for the assigned task. In his/her vague references to causal fallacies, the writer attempts
logical analysis but never refers explicitly or implicitly to any unusual assumptions.
Furthermore, several errors in grammar and sentence structure interfere with meaning
(e.g., “This statement looks like logical, but there are some wrong sentences in it which
is not logical.”).
Because this response “does not follow the directions for the assigned task” (see the
Argument Scoring Guide, pages 39–40) and contains errors in sentence structure and
logical development, it earns a score of 2.
Score 1 Response
The statement assumes that everyone in Mason City enjoys some sort of recreational
activity, which may not be necessarily true. They statement also assumes that if the
state cleans up the river, the use of the river for water sports will definitely increase.
Reader Commentary
The brevity of this two-sentence response makes it fundamentally deficient. Sentence
one states an assumption that is actually not present in the argument, and sentence
two correctly states an assumption but provides no discussion of its implications.
Although the response may begin to address the assigned task, it offers no develop-
ment. As such, it clearly “provides little evidence of the ability to develop an organized
response (i.e., is disorganized and/or extremely brief)” (see Argument Scoring Guide,
pages 39–40) and should earn a score of 1.
36
GRE Analytical Writing
This ebook was issued to HUAN-LIN CHANG, order #14803770223. Unlawful distribution of this ebook is prohibited.
Customer name HUAN-LIN CHANG , Order Id: 14803770223
GRE Scoring Guide: Analyze an Issue
Score 6
In addressing the specific task directions, a 6 response presents a cogent, well-articu-
lated analysis of the issue and conveys meaning skillfully.
A typical response in this category
barticulates a clear and insightful position on the issue in accordance with the
assigned task
bdevelops the position fully with compelling reasons and/or persuasive examples
bsustains a well-focused, well-organized analysis, connecting ideas logically
bconveys ideas fluently and precisely, using effective vocabulary and sentence
variety
bdemonstrates superior facility with the conventions of standard written English
(i.e., grammar, usage, and mechanics) but may have minor errors
Score 5
In addressing the specific task directions, a 5 response presents a generally thoughtful,
well-developed analysis of the issue and conveys meaning clearly.
A typical response in this category
bpresents a clear and well-considered position on the issue in accordance with
the assigned task
bdevelops the position with logically sound reasons and/or well-chosen examples
bis focused and generally well organized, connecting ideas appropriately
bconveys ideas clearly and well, using appropriate vocabulary and sentence
variety
bdemonstrates facility with the conventions of standard written English but may
have minor errors
Score 4
In addressing the specific task directions, a 4 response presents a competent analysis of
the issue and conveys meaning with acceptable clarity.
A typical response in this category
bpresents a clear position on the issue in accordance with the assigned task
bdevelops the position with relevant reasons and/or examples
bis adequately focused and organized
bdemonstrates sufficient control of language to express ideas with acceptable
clarity
bgenerally demonstrates control of the conventions of standard written English
but may have some errors
37
GRE Analytical Writing
This ebook was issued to HUAN-LIN CHANG, order #14803770223. Unlawful distribution of this ebook is prohibited.
Customer name HUAN-LIN CHANG , Order Id: 14803770223
Score 3
A 3 response demonstrates some competence in addressing the specific task directions,
in analyzing the issue, and in conveying meaning but is obviously flawed.
A typical response in this category exhibits ONE OR MORE of the following character-
istics:
bis vague or limited in addressing the specific task directions and/or in present-
ing or developing a position on the issue
bis weak in the use of relevant reasons or examples or relies largely on unsup-
ported claims
bis limited in focus and/or organization
bhas problems in language and sentence structure that result in a lack of clarity
bcontains occasional major errors or frequent minor errors in grammar, usage,
or mechanics that can interfere with meaning
Score 2
A 2 response largely disregards the specific task directions and/or demonstrates serious
weaknesses in analytical writing.
A typical response in this category exhibits ONE OR MORE of the following character-
istics:
bis unclear or seriously limited in addressing the specific task directions and/or
in presenting or developing a position on the issue
bprovides few, if any, relevant reasons or examples in support of its claims
bis poorly focused and/or poorly organized
bhas serious problems in language and sentence structure that frequently inter-
fere with meaning
bcontains serious errors in grammar, usage, or mechanics that frequently obscure
meaning
Score 1
A 1 response demonstrates fundamental deficiencies in analytical writing.
A typical response in this category exhibits ONE OR MORE of the following character-
istics:
bprovides little or no evidence of understanding the issue
bprovides little evidence of the ability to develop an organized response (e.g., is
disorganized and/or extremely brief)
bhas severe problems in language and sentence structure that persistently inter-
fere with meaning
bcontains pervasive errors in grammar, usage, or mechanics that result in inco-
herence
Score 0
Off topic (i.e., provides no evidence of an attempt to respond to the assigned topic), is
in a foreign language, merely copies the topic, consists of only keystroke characters, or
is illegible or nonverbal.
38
GRE Analytical Writing
This ebook was issued to HUAN-LIN CHANG, order #14803770223. Unlawful distribution of this ebook is prohibited.
Customer name HUAN-LIN CHANG , Order Id: 14803770223
GRE Scoring Guide: Analyze an Argument
Score 6
In addressing the specific task directions, a 6 response presents a cogent, well-articu-
lated examination of the argument and conveys meaning skillfully.
A typical response in this category
bclearly identifies aspects of the argument relevant to the assigned task and
examines them insightfully
bdevelops ideas cogently, organizes them logically, and connects them with clear
transitions
bprovides compelling and thorough support for its main points
bconveys ideas fluently and precisely, using effective vocabulary and sentence
variety
bdemonstrates superior facility with the conventions of standard written English
(i.e., grammar, usage, and mechanics) but may have minor errors
Score 5
In addressing the specific task directions, a 5 response presents a generally thoughtful,
well-developed examination of the argument and conveys meaning clearly.
A typical response in this category
bclearly identifies aspects of the argument relevant to the assigned task and
examines them in a generally perceptive way
bdevelops ideas clearly, organizes them logically, and connects them with appro-
priate transitions
boffers generally thoughtful and thorough support for its main points
bconveys ideas clearly and well, using appropriate vocabulary and sentence
variety
bdemonstrates facility with the conventions of standard written English but may
have minor errors
Score 4
In addressing the specific task directions, a 4 response presents a competent examina-
tion of the argument and conveys meaning with acceptable clarity.
A typical response in this category
bidentifies and examines aspects of the argument relevant to the assigned task
but may also discuss some extraneous points
bdevelops and organizes ideas satisfactorily but may not connect them with
transitions
bsupports its main points adequately but may be uneven in its support
bdemonstrates sufficient control of language to convey ideas with acceptable
clarity
bgenerally demonstrates control of the conventions of standard written English
but may have some errors
39
GRE Analytical Writing
This ebook was issued to HUAN-LIN CHANG, order #14803770223. Unlawful distribution of this ebook is prohibited.
Customer name HUAN-LIN CHANG , Order Id: 14803770223
Score 3
A 3 response demonstrates some competence in addressing the specific task directions,
in examining the argument, and in conveying meaning but is obviously flawed.
A typical response in this category exhibits ONE OR MORE of the following character-
istics:
bdoes not identify or examine most of the aspects of the argument relevant to the
assigned task, although some relevant examination of the argument is present
bmainly discusses tangential or irrelevant matters, or reasons poorly
bis limited in the logical development and organization of ideas
boffers support of little relevance and value for its main points
bhas problems in language and sentence structure that result in a lack of clarity
bcontains occasional major errors or frequent minor errors in grammar, usage,
or mechanics that can interfere with meaning
Score 2
A 2 response largely disregards the specific task directions and/or demonstrates serious
weaknesses in analytical writing.
A typical response in this category exhibits ONE OR MORE of the following character-
istics:
bdoes not present an examination based on logical analysis, but may instead
present the writer’s own views on the subject
bdoes not follow the directions for the assigned task
bdoes not develop ideas, or is poorly organized and illogical
bprovides little, if any, relevant or reasonable support for its main points
bhas serious problems in language and sentence structure that frequently inter-
fere with meaning
bcontains serious errors in grammar, usage, or mechanics that frequently ob scure
meaning
Score 1
A 1 response demonstrates fundamental deficiencies in analytical writing.
A typical response in this category exhibits ONE OR MORE of the following character-
istics:
bprovides little or no evidence of understanding the argument
bprovides little evidence of the ability to develop an organized response (e.g., is
disorganized and/or extremely brief)
bhas severe problems in language and sentence structure that persistently inter-
fere with meaning
bcontains pervasive errors in grammar, usage, or mechanics that result in inco-
herence
Score 0
Off topic (i.e., provides no evidence of an attempt to respond to the assigned topic), is
in a foreign language, merely copies the topic, consists of only keystroke char acters, or
is illegible, or nonverbal.
40
GRE Analytical Writing
This ebook was issued to HUAN-LIN CHANG, order #14803770223. Unlawful distribution of this ebook is prohibited.
Customer name HUAN-LIN CHANG , Order Id: 14803770223
Score Level Descriptions
Although the GRE Analytical Writing measure contains two discrete analytical writing
tasks, a single combined score is reported because it is more reliable than is a score for
either task alone. The reported score, the average of the scores for the two tasks, ranges
from 0 to 6, in half-point increments.
The statements below describe, for each score level, the overall quality of critical
thinking and analytical writing demonstrated across both the Issue and Argument
tasks. The Analytical Writing section is designed to assess both critical thinking skills
and writing ability. Thus, many aspects of analytical writing, including reasoning skills,
organization, and degree of control of the conventions of standard written English are
taken into consideration in the determination of a final score. For a full description of
how these criteria are used to assess essay responses, please refer to the scoring guides
for the Issue and Argument tasks, which are available on the GRE website at
www.ets.org/gre/revised/scoreguides.
Scores 6 and 5.5: Sustains insightful, in-depth analysis of complex ideas; develops and
supports main points with logically compelling reasons and/or highly persuasive exam-
ples; is well focused and well organized; skillfully uses sentence variety and precise
vocabulary to convey meaning effectively; demonstrates superior facility with sentence
structure and language usage but may have minor errors that do not interfere with
meaning.
Scores 5 and 4.5: Provides generally thoughtful analysis of complex ideas; develops and
supports main points with logically sound reasons and/or well-chosen examples; is gen-
erally focused and well organized; uses sentence variety and vocabulary to convey
meaning clearly; demonstrates good control of sentence structure and language usage
but may have minor errors that do not interfere with meaning.
Scores 4 and 3.5: Provides competent analysis of ideas; develops and supports main
points with relevant reasons and/or examples; is adequately organized; conveys mean-
ing with reasonable clarity; demonstrates satisfactory control of sentence structure and
language usage but may have some errors that affect clarity.
Scores 3 and 2.5: Displays some competence in analytical writing, although the writing
is flawed in at least one of the following ways: limited analysis or development; weak
organization; weak control of sentence structure or language usage, with errors that
often result in vagueness or lack of clarity.
Scores 2 and 1.5: Displays serious weaknesses in analytical writing. The writing is seri-
ously flawed in at least one of the following ways: serious lack of analysis or develop-
ment; lack of organization; serious and frequent problems in sentence structure or
language usage, with errors that obscure meaning.
Scores 1 and 0.5: Displays fundamental deficiencies in analytical writing. The writing is
fundamentally flawed in at least one of the following ways: content that is extremely
confusing or mostly irrelevant to the assigned tasks; little or no development; severe
and pervasive errors that result in incoherence.
41
GRE Analytical Writing
This ebook was issued to HUAN-LIN CHANG, order #14803770223. Unlawful distribution of this ebook is prohibited.
Customer name HUAN-LIN CHANG , Order Id: 14803770223
Score 0: The examinee’s analytical writing skills cannot be evaluated because the
responses do not address any part of the assigned tasks, are merely attempts to copy
the assignments, are in a foreign language, or display only indecipherable text.
Score NS: The examinee produced no text whatsoever.
42
GRE Analytical Writing
This ebook was issued to HUAN-LIN CHANG, order #14803770223. Unlawful distribution of this ebook is prohibited.
Customer name HUAN-LIN CHANG , Order Id: 14803770223
Overview of the Verbal Reasoning Measure
The Verbal Reasoning measure assesses your ability to analyze and evaluate writ-
ten material and synthesize information obtained from it, to analyze relation-
ships among component parts of sentences, and to recognize relationships
among words and concepts.
Verbal Reasoning questions appear in several formats, each of which is discussed
in detail below. About half of the measure requires you to read passages and answer
questions on those passages. The other half requires you to read, interpret, and com-
plete existing sentences, groups of sentences, or paragraphs. Many, but not all, of the
questions are standard multiple-choice questions, in which you are required to select a
single correct answer; others ask you to select multiple correct answers; and still others
ask you to select a sentence from the passage. The number of choices varies depending
on the type of question.
Verbal Reasoning Question Types
The Verbal Reasoning measure contains three types of questions:
bReading Comprehension
bText Completion
bSentence Equivalence
In this section you will study each of these question types one by one, and you’ll
learn valuable strategies for answering each type. Turn the page to begin.
GRE
®
Verbal Reasoning
gLearn the three types of
GRE
®Verbal Reasoning
questions
gGet tips for answering each question type
gStudy examples of GRE Verbal Reasoning
questions
Your goals
for this
chapter
43
3
This ebook was issued to HUAN-LIN CHANG, order #14803770223. Unlawful distribution of this ebook is prohibited.
Customer name HUAN-LIN CHANG , Order Id: 14803770223
Reading Comprehension Questions
Reading Comprehension questions are designed to test a wide range of abilities
required to read and understand the kinds of prose commonly encountered in graduate
school. Those abilities include
bunderstanding the meaning of individual words
bunderstanding the meaning of individual sentences
bunderstanding the meaning of paragraphs and larger bodies of text
bdistinguishing between minor and major points
bsummarizing a passage
bdrawing conclusions from the information provided
breasoning from incomplete data, inferring missing information
bunderstanding the structure of a text, how the parts relate to one another
bidentifying the author’s perspective
bidentifying the author’s assumptions
banalyzing a text and reaching conclusions about it
bidentifying strengths and weaknesses
bdeveloping and considering alternative explanations
As this list implies, reading and understanding a piece of text requires far more
than a passive understanding of the words and sentences it contains it requires active
engagement with the text, asking questions, formulating and evaluating hypotheses,
and reflecting on the relationship of the particular text to other texts and information.
Each Reading Comprehension question is based on a passage, which may range in
length from one paragraph to several paragraphs. The test contains approximately ten
passages; the majority of the passages in the test are one paragraph in length, and only
one or two are several paragraphs long. Passages are drawn from the physical sciences,
the biological sciences, the social sciences, the arts and humanities, and everyday top-
ics, and are based on material found in books and periodicals, both academic and
nonacademic.
Typically, about half of the questions on the test will be based on passages, and the
number of questions based on a given passage can range from one to six. Questions
can cover any of the topics listed above, from the meaning of a particular word to
assessing evidence that might support or weaken points made in the passage. Many,
but not all, of the questions are standard multiple-choice questions, in which you are
required to select a single correct answer; others ask you to select multiple correct
answers, and still others ask you to select a sentence from the passage. These question
types are presented in more detail below, and you should make sure that you are famil-
iar with the differences among them.
General Advice
Reading passages are drawn from many different disciplines and sources, so you may
encounter material with which you are not familiar. Do not be discouraged when this
happens; all the questions can be answered on the basis of the information provided in
the passage, and you are not expected to rely on any outside knowledge. If, however,
you encounter a passage that seems particularly hard or unfamiliar, you may want to
save it for last.
44
GRE Verbal Reasoning
This ebook was issued to HUAN-LIN CHANG, order #14803770223. Unlawful distribution of this ebook is prohibited.
Customer name HUAN-LIN CHANG , Order Id: 14803770223
bRead and analyze the passage carefully before trying to answer any of the ques-
tions and pay attention to clues that help you understand less explicit aspects
of the passage.
bTry to distinguish main ideas from supporting ideas or evidence.
bTry to distinguish ideas that the author is advancing from those he or she is
merely reporting.
bSimilarly, try to distinguish ideas that the author is strongly committed to
from those he or she advances as hypothetical or speculative.
bTry to identify the main transitions from one idea to the next.
bTry to identify the relationship between different ideas. For example:
Are they contrasting? Are they consistent?
Does one support the other?
Does one spell another out in greater detail?
Is one an application of another to a particular circumstance?
bRead each question carefully and be certain that you understand exactly what
is being asked.
bAnswer each question on the basis of the information provided in the passage
and do not rely on outside knowledge. Sometimes your own views or opinions
may conflict with those presented in a passage; if this happens, take special
care to work within the context provided by the passage. You should not expect
to agree with everything you encounter in the reading passages.
Reading Comprehension Multiple-choice Questions:
Select One Answer Choice
Description
These are the traditional multiple-choice questions with five answer choices of which
you must select one.
Tips for Answering
bRead
all
the answer choices before making your selection, even if you think
you know what the answer is in advance.
bDon’t be misled by answer choices that are only partially true or only partially
answer the question. The correct answer is the one that most accurately and
most completely answers the question posed. Be careful also not to pick an
answer choice simply because it is a true statement.
bPay attention to context. When the question asks about the meaning of a
word in the passage, be sure that the answer choice you select correctly
represents the way the word is being used in the passage. Many words have
quite different meanings in different contexts.
45
GRE Verbal Reasoning
This ebook was issued to HUAN-LIN CHANG, order #14803770223. Unlawful distribution of this ebook is prohibited.
Customer name HUAN-LIN CHANG , Order Id: 14803770223
Reading Comprehension Multiple-choice Questions:
Select One or More Answer Choices
Description
These provide three answer choices and ask you to select all that are correct; one, two,
or all three of the answer choices may be correct. To gain credit for these questions,
you must select all the correct answers, and only those; there is no credit for partially
correct answers. These questions are marked with square boxes beside the answer
choices, not circles or ovals.
Tips for Answering
bEvaluate each answer choice separately on its own merits. When evaluating
one answer choice, do not take the others into account.
bMake sure the answer choice you pick accurately and completely answers the
question posed. Be careful not to be misled by answer choices that are only
partially true or only partially answer the question. Be careful also not to pick
an answer choice simply because it is a true statement.
bDo not be disturbed if you think all three answer choices are correct.
Questions of this type can have three correct answer choices.
Reading Comprehension Questions: Select-in-Passage
Description
The question asks you to click on the sentence in the passage that meets a certain
description. To answer the question, choose one of the sentences and click on it; click-
ing anywhere on a sentence will highlight it. In longer passages, the question will usu-
ally apply to only one or two specified paragraphs, marked by an arrow (
>
); clicking
on a sentence elsewhere in the passage will not highlight it.
Note.Because this type of question requires the use of the computer, it does not appear
in the paper-based General Test. Similar multiple-choice questions are used in its place.
Tips for Answering
bBe careful to evaluate each of the relevant sentences in the passage
separately before selecting your answer. Do not evaluate any sentences that
are outside the paragraphs under consideration.
bDo not select a sentence if the description given in the question only partially
applies. A correct answer choice must accurately match the description in the
question. Note, however, that the description need not be complete, that is,
there may be aspects of the sentence that are not fully described in the
question.
46
GRE Verbal Reasoning
This ebook was issued to HUAN-LIN CHANG, order #14803770223. Unlawful distribution of this ebook is prohibited.
Customer name HUAN-LIN CHANG , Order Id: 14803770223
Sample Question Set
Questions 1 to 3 are based on the following reading passage.
Reviving the practice of using elements of popular music in classical composition, an
approach that had been in hibernation in the United States during the 1960s, composer
Philip Glass (born 1937) embraced the ethos of popular music in his compositions.
Glass based two symphonies on music by rock musicians David Bowie and Brian Eno,
but the symphonies’ sound is distinctively his. Popu lar elements do not appear out of
place in Glass’s classical music, which from its early days has shared certain harmonies
and rhythms with rock music. Yet this use of popular elements has not made Glass a
composer of popular music. His music is not a version of popular music packaged to
attract classical listeners; it is high art for listeners steeped in rock rather than the
classics.
Select only one answer choice.
1. The passage addresses which of the following issues related to Glass’s use of
popular elements in his classical compositions?
A
How it is regarded by listeners who prefer rock to the classics
B
How it has affected the commercial success of Glass’s music
C
Whether it has contributed to a revival of interest among other composers
in using popular elements in their compositions
D
Whether it has had a detrimental effect on Glass’s reputation as a
composer of classical music
E
Whether it has caused certain of Glass’s works to be derivative in quality
Consider each of the three choices separately and select all that apply.
2. The passage suggests that Glass’s work displays which of the following qualities?
A
A return to the use of popular music in classical compositions
B
An attempt to elevate rock music to an artistic status more closely
approximating that of classical music
C
A long-standing tendency to incorporate elements from two apparently
disparate musical styles
3. Select the sentence that distinguishes two ways of integrating rock and classical
music.
Explanation
The passage describes in general terms how Philip Glass uses popular music in his
classical compositions and explores how Glass can do this without being imitative.
Note that there are no opposing views discussed; the author is simply presenting his or
her views.
Question 1:One of the important points that the passage makes is that when Glass
uses popular elements in his music, the result is very much his own crea tion (it is “dis-
tinctively his”). In other words, the music is far from being derivative. Thus one issue
47
GRE Verbal Reasoning
This ebook was issued to HUAN-LIN CHANG, order #14803770223. Unlawful distribution of this ebook is prohibited.
Customer name HUAN-LIN CHANG , Order Id: 14803770223
that the passage addresses is the one referred to in answer Choice E—it answers it in
the negative. The passage does not discuss the impact of Glass’s use of popular ele-
ments on listeners, on the commercial success of his music, on other composers, nor
on Glass’s reputation, so none of Choices A through D is correct.
The correct answer is Choice E.
Question 2:To answer this question, it is important to assess each answer choice
independently. Since the passage says that Glass revived the use of popular music in
classical compositions, answer Choice A is clearly correct. On the other hand, the pas-
sage also denies that Glass composes popular music or packages it in a way to elevate
its status, so answer Choice B is incorrect. Finally, since Glass’s style has always mixed
elements of rock with classical elements, Choice C is correct.
Thus the correct answer is Choice A and Choice C.
Question 3:Almost every sentence in the passage refers to incorporating rock
music in classical compositions, but only the last sentence distinguishes two ways of
doing so. It distinguishes between writing rock music in a way that will make it attrac-
tive to classical listeners and writing classical music that will be attractive to listeners
familiar with rock.
Thus the correct answer is the last sentence of the passage.
Text Completion Questions
Description
As mentioned above, skilled readers do not simply absorb the information presented on
the page; instead, they maintain a constant attitude of interpretation and evaluation,
reasoning from what they have read so far to create a picture of the whole and revising
that picture as they go. Text Completion questions test this ability by omitting crucial
words from short passages and asking the test taker to use the remaining information
in the passage as a basis for selecting words or short phrases to fill the blanks and cre-
ate a coherent, meaningful whole.
Question Structure
bPassage composed of one to five sentences
bOne to three blanks
bThree answer choices per blank (five answer choices in the case of a single
blank)
bThe answer choices for different blanks function independently; that is, select-
ing one answer choice for one blank does not affect what answer choices you
can select for another blank
bSingle correct answer, consisting of one choice for each blank; no credit for
partially correct answers
48
GRE Verbal Reasoning
This ebook was issued to HUAN-LIN CHANG, order #14803770223. Unlawful distribution of this ebook is prohibited.
Customer name HUAN-LIN CHANG , Order Id: 14803770223
49
GRE Verbal Reasoning
Tips for Answering
Do not merely try to consider each possible combination of answers; doing so
will take too long and is open to error. Instead, try to analyze the passage in the
following way:
bRead through the passage to get an overall sense of it.
bIdentify words or phrases that seem particularly significant, either because
they emphasize the structure of the passage (words like
although
or
moreover
) or because they are central to understanding what the passage
is about.
bThink up your own words for the blanks. Try to fill in the blanks with words
or phrases that seem to you to fit and then see if similar words are offered
among the answer choices.
bDo not assume that the first blank is the one that should be filled first.
Perhaps one of the other blanks is easier to fill first. Select your choice for that
blank, and then see whether you can complete another blank. If none of the
choices for the other blank seem to make sense, go back and reconsider your
first selection.
bDouble-check your answers. When you have made your selection for each
blank, check to make sure that the passage is logically, grammatically, and
stylistically coherent.
Sample Questions
For each blank select one entry from the corresponding column of choices. Fill all
blanks in the way that best completes the text.
1. It is refreshing to read a book about our planet by an author who does not allow
facts to be (i)__________ by politics: well aware of the political disputes about
the effects of human activities on climate and biodiversity, this author does not
permit them to (ii)__________ his comprehensive description of what we know
about our biosphere. He emphasizes the enormous gaps in our knowledge, the
sparseness of our observations, and the (iii)__________, calling attention to the
many aspects of planetary evolution that must be better understood before we
can accurately diagnose the condition of our planet.
Blank (i) Blank (ii) Blank (iii)
A
overshadowed
D
enhance
G
plausibility of our
hypotheses
B
invalidated
E
obscure
H
certainty of our
entitlement
C
illuminated
F
underscore
I
superficiality of
our theories
Explanation
The overall tone of the passage is clearly complimentary. To understand what the
author of the book is being complimented on, it is useful to focus on the second blank.
This ebook was issued to HUAN-LIN CHANG, order #14803770223. Unlawful distribution of this ebook is prohibited.
Customer name HUAN-LIN CHANG , Order Id: 14803770223
50
GRE Verbal Reasoning
Here, we must determine what word would indicate something that the author is
praised for not permitting. The only answer choice that fits the case is “obscure,” since
enhancing and underscoring are generally good things to do, not things one should
refrain from doing. Choosing “obscure” clarifies the choice for the first blank; the only
choice that fits well with “obscure” is “overshadowed.” Notice that trying to fill blank
(i) without filling blank (ii) first is very hard—each choice has at least some initial plau-
sibility. Since the third blank requires a phrase that matches “enormous gaps” and
“sparseness of our observations,” the best choice is “superficiality of our theories.”
Thus the correct answer is overshadowed (Choice A), obscure (Choice E), and
superficiality of our theories (Choice I).
2. Vain and prone to violence, Caravaggio could not handle success: the more his
(i)__________ as an artist increased, the more (ii)__________ his life became.
Blank (i) Blank (ii)
A
temperance
D
tumultuous
B
notoriety
E
providential
C
eminence
F
dispassionate
Explanation
In this sentence, what follows the colon must explain or spell out what precedes it. So
roughly what the second part must say is that as Caravaggio became more successful,
his life got more out of control. When one looks for words to fill the blanks, it becomes
clear that “tumultuous” is the best fit for blank (ii), since neither of the other choices
suggests being out of control. And for blank (i), the best choice is “eminence,” since
to increase in eminence is a consequence of becoming more successful. It is true that
Cara vaggio might also increase in notoriety, but an increase in notoriety as an artist is
not as clear a sign of success as an increase in eminence.
Thus the correct answer is eminence (Choice C) and tumultuous (Choice D).
3. In parts of the Arctic, the land grades into the landfast ice so _______ that you
can walk off the coast and not know you are over the hidden sea.
A
permanently
B
imperceptibly
C
irregularly
D
precariously
E
relentlessly
Explanation
The word that fills the blank has to characterize how the land grades into the ice in a
way that explains how you can walk off the coast and over the sea without knowing it.
The word that does that is “imperceptibly”; if the land grades imperceptibly into the
ice, you might well not know that you had left the land. Describing the shift from land
to ice as permanent, irregular, precarious, or relentless would not help to explain how
you would fail to know.
Thus the correct answer is imperceptibly (Choice B).
This ebook was issued to HUAN-LIN CHANG, order #14803770223. Unlawful distribution of this ebook is prohibited.
Customer name HUAN-LIN CHANG , Order Id: 14803770223
Sentence Equivalence Questions
Description
Like Text Completion questions, Sentence Equivalence questions test the ability to
reach a conclusion about how a passage should be completed on the basis of partial
information, but to a greater extent they focus on the meaning of the completed whole.
Sentence Equivalence questions consist of a single sentence with just one blank, and
they ask you to find two choices that both lead to a complete, coherent sentence and
that produce sentences that mean the same thing.
Question Structure
bConsists of:
ba single sentence
bone blank
bsix answer choices
bRequires you to select two of the answer choices; no credit for partially correct
answers.
These questions are marked with square boxes beside the answer choices, not circles or
ovals.
Tips for Answering
Do not simply look among the answer choices for two words that mean the
same thing. This can be misleading for two reasons. First, the answer choices
may contain pairs of words that mean the same thing but do not fit coherently
into the sentence, and thus do not constitute a correct answer. Second, the pair
of words that do constitute the correct answer may not mean exactly the same
thing, since all that matters is that the resultant sentences mean the same thing.
bRead the sentence to get an overall sense of it.
bIdentify words or phrases that seem particularly significant, either because
they emphasize the structure of the sentence (words like
although
or
moreover
) or because they are central to understanding what the sentence
is about.
bThink up your own words for the blanks. Try to fill in the blank with a word
that seems to you to fit and then see if two similar words are offered among
the answer choices. If you find some word that is similar to what you are
expecting but cannot find a second one, do not become fixated on your
interpretation; instead, see whether there are other words among the answer
choices that can be used to fill the blank coherently.
bDouble-check your answers. When you have selected your pair of answer
choices for the blank, check to make sure that each one produces a sentence
that is logically, grammatically, and stylistically coherent, and that the two
sentences mean the same thing.
51
GRE Verbal Reasoning
This ebook was issued to HUAN-LIN CHANG, order #14803770223. Unlawful distribution of this ebook is prohibited.
Customer name HUAN-LIN CHANG , Order Id: 14803770223
Sample Questions
Select the two answer choices that, when used to complete the sentence, fit the
meaning of the sentence as a whole and produce completed sentences that are alike
in meaning.
1. Although it does contain some pioneering ideas, one would hardly characterize
the work as __________.
A
orthodox
B
eccentric
C
original
D
trifling
E
conventional
F
innovative
Explanation
The word “Although” is a crucial signpost here. The work contains some pioneering
ideas, but apparently it is not overall a pioneering work. Thus the two words that could
fill the blank appropriately are “original” and “innovative.” Note that “orthodox” and
“conventional” are two words that are very similar in meaning, but neither one com-
pletes the sentence sensibly.
Thus the correct answer is original (Choice C) and innovative (Choice F).
2. It was her view that the country’s problems had been _______ by foreign
technocrats, so that to ask for such assistance again would be counterproductive.
A
ameliorated
B
ascertained
C
diagnosed
D
exacerbated
E
overlooked
F
worsened
Explanation
The sentence relates a piece of reasoning, as indicated by the presence of “so that”:
asking for the assistance of foreign technocrats would be counterproductive because of
the effects such technocrats have had already. This means that the technocrats must
have bad effects; that is, they must have “exacerbated” or “worsened” the country’s
problems.
Thus the correct answer is exacerbated (Choice D) and worsened (Choice F).
52
GRE Verbal Reasoning
This ebook was issued to HUAN-LIN CHANG, order #14803770223. Unlawful distribution of this ebook is prohibited.
Customer name HUAN-LIN CHANG , Order Id: 14803770223
This chapter contains six sets of GRE Verbal Reasoning practice questions. Three
of the practice sets consist of examples of the discrete question types, Text Com-
pletion and Sentence Equivalence; and the other three sets consist of Reading
Comprehension questions. The sets are arranged in order of increasing difficulty. The
first two are easy, the next two are medium, and the final two are hard.
Following the last set is an answer key for quick reference. Then, at the end of the
chapter, you will find complete explanations for every question. Each explanation is
presented with the corresponding question, so that you can easily see what was asked
and what the various answer choices were.
Sharpen your GRE Verbal Reasoning skills by working your way through these
question sets. Begin with the easy sets and then move on to the medium-difficulty and
hard sets. Review the answer explanations carefully, paying particular attention to the
explanations for questions that you answered incorrectly. Turn the page to begin.
gPractice answering
GRE
®Verbal Reasoning
questions on your own
gStudy answer explanations, particularly for
questions you answered incorrectly
Your goals
for this
chapter
53
4
GRE
®
Verbal Reasoning
Practice Questions
This ebook was issued to HUAN-LIN CHANG, order #14803770223. Unlawful distribution of this ebook is prohibited.
Customer name HUAN-LIN CHANG , Order Id: 14803770223
SET 1. Discrete Questions: Easy
For Questions 1 to 5, select one entry for each blank from the corresponding column
of choices. Fill all blanks in the way that best completes the text.
1. Dominant interests often benefit most from _________ of governmental
interference in business, since they are able to take care of themselves if left
alone.
A
intensification
B
authorization
C
centralization
D
improvisation
E
elimination
2. Kagan maintains that an infant’s reactions to its first stressful experiences are
part of a natural process of development, not harbingers of childhood
unhappiness or _________ signs of adolescent anxiety.
A
prophetic
B
normal
C
monotonous
D
virtual
E
typical
3. An investigation that is _________ can occasionally yield new facts, even notable
ones, but typically the appearance of such facts is the result of a search in a
definite direction.
A
timely
B
unguided
C
consistent
D
uncomplicated
E
subjective
4. It is (i)_________ that so many portrait paintings hang in art museums, since the
subject matter seems to dictate a status closer to pictures in the family photo -
graph album than to high art. But perhaps it is the artistic skill with which the
portraits are painted that (ii)_________ their presence in art museums.
Blank (i) Blank (ii)
A
surprising
D
challenges
B
understandable
E
justifies
C
irrelevant
F
changes
54
GRE Verbal Reasoning Practice Questions
This ebook was issued to HUAN-LIN CHANG, order #14803770223. Unlawful distribution of this ebook is prohibited.
Customer name HUAN-LIN CHANG , Order Id: 14803770223
5. In stark contrast to his later (i)_________, Simpson was largely (ii)_________
politics during his college years, despite the fact that the campus he attended was
rife with political activity.
Blank (i) Blank (ii)
A
activism
D
devoted to
B
apathy
E
indifferent to
C
affability
F
shaped by
For Questions 6 to 8, select the two answer choices that, when used to complete
the sentence, fit the meaning of the sentence as a whole and produce completed
sentences that are alike in meaning.
6. As my eyesight began to _________, I spent a lot of time writing about it both
poems and “eye journals” describing what I saw as I looked out through
damaged eyes.
A
deteriorate
B
sharpen
C
improve
D
decline
E
recover
F
adjust
7. The judge’s standing in the legal community, though shaken by phony allegations
of wrongdoing, emerged, at long last, _________ .
A
unqualified
B
undiminished
C
undecided
D
undamaged
E
unresolved
F
unprincipled
8. Modern agricultural practices have been extremely successful in increasing the
productivity of major food crops, yet despite heavy use of pesticides, _________
losses to diseases and insect pests are sustained each year.
A
incongruous
B
reasonable
C
significant
D
considerable
E
equitable
F
fortuitous
55
GRE Verbal Reasoning Practice Questions
This ebook was issued to HUAN-LIN CHANG, order #14803770223. Unlawful distribution of this ebook is prohibited.
Customer name HUAN-LIN CHANG , Order Id: 14803770223
SET 2. Reading Comprehension Questions: Easy
For each of Questions 1 to 9, select one answer choice unless otherwise instructed.
1. A person who agrees to serve as mediator between two warring factions at the
request of both abandons by so agreeing the right to take sides later. To take sides
at a later point would be to suggest that the earlier presumptive impartiality was
a sham.
The passage above emphasizes which of the following points about mediators?
A
They should try to form no opinions of their own about any issue that is
related to the dispute.
B
They should not agree to serve unless they are committed to maintaining
a stance of impartiality.
C
They should not agree to serve unless they are equally acceptable to all
parties to a dispute.
D
They should feel free to take sides in the dispute right from the start,
provided that they make their biases publicly known.
E
They should reserve the right to abandon their impartiality so as not to be
open to the charge of having been deceitful.
56
GRE Verbal Reasoning Practice Questions
This ebook was issued to HUAN-LIN CHANG, order #14803770223. Unlawful distribution of this ebook is prohibited.
Customer name HUAN-LIN CHANG , Order Id: 14803770223
Questions 2 to 5 are based on the following reading passage.
Was Felix Mendelssohn (1809–1847) a great composer? On its face, the question seems
absurd. One of the most gifted prodigies in the history of music, he produced his first
masterpiece at sixteen. From then on, he was recognized as an artist of preternatural
abilities, not only as a composer but also as a pianist and conductor. But Mendelssohn’s
enduring popularity has often been at odds sometimes quite sharply with his
criti cal standing. Despite general acknowledgment of his genius, there has been a
noticeable reluctance to rank him with, say, Schumann or Brahms. As Haggin put it,
Men delssohn, as a composer, was a “minor master . . . working on a small scale of emo-
tion and texture.”
2. Select a sentence in the passage whose function is to indicate the range of
Mendelssohn’s musical talents.
3. The passage suggests that anyone attempting to evaluate Mendelssohn’s career
must confront which of the following dichotomies?
A
The tension between Mendelssohn’s career as a composer and his career
as a pianist and conductor
B
The contrast between Mendelssohn’s popularity and that of Schumann
and Brahms
C
The discrepancy between Mendelssohn’s popularity and his standing
among critics
D
The inconsistency between Mendelssohn’s reputation during his lifetime
and his reputation since his death
E
The gap between Mendelssohn’s prodigious musical beginnings and his
decline in later years
4. It can be inferred that the “reluctance” mentioned in the passage is being
ascribed to
A
most composers since Mendelssohn
B
Schumann and Brahms
C
the music-listening public
D
music critics generally
E
Haggin exclusively
5. The author mentions Schumann and Brahms primarily in order to
A
provide examples of composers who are often compared with
Mendelssohn
B
identify certain composers who are more popular than Mendelssohn
C
identify composers whom Mendelssohn influenced
D
establish the milieu in which Mendelssohn worked
E
establish a standard of comparison for Mendelssohn as a composer
57
GRE Verbal Reasoning Practice Questions
This ebook was issued to HUAN-LIN CHANG, order #14803770223. Unlawful distribution of this ebook is prohibited.
Customer name HUAN-LIN CHANG , Order Id: 14803770223
Questions 6 and 7 are based on the following reading passage.
While most scholarship on women’s employment in the United States recognizes that
the Second World War (1939–1945) dramatically changed the role of women in the
workforce, these studies also acknowledge that few women remained in manufacturing
jobs once men returned from the war. But in agriculture, unlike other industries where
women were viewed as temporary workers, women’s employment did not end with the
war. Instead, the expansion of agriculture and a steady decrease in the number of male
farmworkers combined to cause the industry to hire more women in the postwar years.
Consequently, the 1950s saw a growing number of women engaged in farm labor, even
though rhetoric in the popular media called for the return of women to domestic life.
6. It can be inferred from the passage that the manufacturing and agricultural
sectors in the United States following the Second World War differed in which of
the following respects?
A
The rate of expansion in each sector
B
The percentage of employees in each sector who were men
C
The trend in the wages of men employed in each sector
D
The attitude of the popular media toward the employment of women in
each sector
E
The extent to which women in each sector were satisfied with their jobs
7. Which of the following statements about women’s employment in the United
States during and after the Second World War is most clearly supported by the
passage?
A
Most women who joined the workforce during the Second World War
wanted to return to domestic life when the war ended.
B
The great majority of women who joined the workforce during the Second
World War were employed in manufacturing jobs.
C
The end of the Second World War was followed by a large-scale transfer of
women workers from manufacturing to agriculture.
D
The increase in women’s employment that accompanied the Second World
War was longer lasting in agriculture than it was in manufacturing.
E
The popular media were more forceful in calling for women to join the
workforce during the Second World War than in calling for women to
return to domestic life after the war.
58
GRE Verbal Reasoning Practice Questions
This ebook was issued to HUAN-LIN CHANG, order #14803770223. Unlawful distribution of this ebook is prohibited.
Customer name HUAN-LIN CHANG , Order Id: 14803770223
line
5
10
Questions 8 and 9 are based on the following reading passage.
Since the Hawaiian Islands have never been connected to other land masses, the great
variety of plants in Hawaii must be a result of the long-distance dispersal of seeds, a
process that requires both a method of transport and an equivalence between the ecol-
ogy of the source area and that of the recipient area.
There is some dispute about the method of transport involved. Some biologists
argue that ocean and air currents are responsible for the transport of plant seeds to
Hawaii. Yet the results of flotation experiments and the low temperatures of air cur-
rents cast doubt on these hypotheses. More probable is bird transport, either externally,
by accidental attachment of the seeds to feathers, or internally, by the swallowing of
fruit and subsequent excretion of the seeds. While it is likely that fewer varieties of
plant seeds have reached Hawaii externally than internally, more varieties are known
to be adapted to external than to internal transport.
8. The author of the passage is primarily concerned with
A
discussing different approaches biologists have taken to testing theories
about the distribution of plants in Hawaii
B
discussing different theories about the transport of plant seeds to Hawaii
C
discussing the extent to which air currents are responsible for the
dispersal of plant seeds to Hawaii
D
resolving a dispute about the adaptability of plant seeds to bird transport
E
resolving a dispute about the ability of birds to carry plant seeds long
distances
9. The author mentions the results of flotation experiments on plant seeds (lines
7–8) most probably in order to
A
support the claim that the distribution of plants in Hawaii is the result of
the long-distance dispersal of seeds
B
lend credibility to the thesis that air currents provide a method of
transport for plant seeds to Hawaii
C
suggest that the long-distance dispersal of seeds is a process that requires
long periods of time
D
challenge the claim that ocean currents are responsible for the transport
of plant seeds to Hawaii
E
refute the claim that Hawaiian flora evolved independently from flora in
other parts of the world
59
GRE Verbal Reasoning Practice Questions
This ebook was issued to HUAN-LIN CHANG, order #14803770223. Unlawful distribution of this ebook is prohibited.
Customer name HUAN-LIN CHANG , Order Id: 14803770223
SET 3. Discrete Questions: Medium
For Questions 1 to 5, select one entry for each blank from the corresponding column
of choices. Fill all blanks in the way that best completes the text.
1. It comes as no surprise that societies have codes of behavior; the character of the
codes, on the other hand, can often be _________ .
A
predictable
B
unexpected
C
admirable
D
explicit
E
confusing
2. Like Béla Bartók, Ruth Crawford not only brought a composer’s acumen to the
notation of folk music, she also had a marked (i)_________ the task. This was
clear in her agonizing over how far to try to represent the minute details of a
performance in a written text, and this (ii)_________ makes her work a landmark
in ethnomusicology.
Blank (i) Blank (ii)
A
reverence for
D
fastidiousness
B
detachment from
E
didacticism
C
curiosity about
F
iconoclasm
3. Political advertising may well be the most (i)_________ kind of advertising:
political candidates are usually quite (ii)_________, yet their campaign
advertisements often hide important differences behind smoke screens of smiles
and empty slogans.
Blank (i) Blank (ii)
A
polemical
D
interchangeable
B
effective
E
dissimilar
C
deceptive
F
vocal
60
GRE Verbal Reasoning Practice Questions
This ebook was issued to HUAN-LIN CHANG, order #14803770223. Unlawful distribution of this ebook is prohibited.
Customer name HUAN-LIN CHANG , Order Id: 14803770223
4. Richard M. Russell said 52 percent of the nation’s growth since the Second World
War had (i)_________ invention. He said, (ii)_________ research, the government’s
greatest role in assuring continuing innovation is promoting a strong, modern
patent office. “Unless we can (iii)_________ original ideas, we will not have
invention,” Mr. Russell said. Speculating on the state of innovation over the next
century, several inventors agreed that the future lay in giving children the tools to
think creatively and the motivation to invent.
Blank (i) Blank (ii) Blank (iii)
A
been at the
D
in addition to
G
evaluate
expense of restricting
B
no bearing on
E
aside from
H
protect
supporting
C
come through
F
far from
I
disseminate
exaggerating
5. Statements presented as fact in a patent application are (i)_________ unless a
good reason for doubt is found. The invention has only to be deemed “more likely
than not” to work in order to receive initial approval. And, although thousands of
patents are challenged in court for other reasons, no incentive exists for anyone
to expend effort (ii)_________ the science of an erroneous patent. For this reason
the endless stream of (iii)_________ devices will continue to yield occasional
patents.
Blank (i) Blank (ii) Blank (iii)
A
presumed
D
corroborating
G
novel
verifiable
B
carefully
E
advancing
H
bogus
scrutinized
C
considered
F
debunking
I
obsolete
capricious
61
GRE Verbal Reasoning Practice Questions
This ebook was issued to HUAN-LIN CHANG, order #14803770223. Unlawful distribution of this ebook is prohibited.
Customer name HUAN-LIN CHANG , Order Id: 14803770223
62
GRE Verbal Reasoning Practice Questions
For Questions 6 to 8, select the two answer choices that, when used to complete
the sentence, fit the meaning of the sentence as a whole and produce completed
sentences that are alike in meaning.
6. Ever a demanding reader of the fiction of others, the novelist Chase was likewise
often the object of _________ analyses by his contemporaries.
A
exacting
B
copious
C
respectful
D
acerbic
E
scathing
F
meticulous
7. Her _________ should not be confused with miserliness; as long as I have known
her, she has always been willing to assist those who are in need.
A
stinginess
B
diffidence
C
frugality
D
illiberality
E
intolerance
F
thrift
8. A misconception frequently held by novice writers is that sentence structure
mirrors thought: the more convoluted the structure, the more _________ the
ideas.
A
complicated
B
engaged
C
essential
D
fanciful
E
inconsequential
F
involved
This ebook was issued to HUAN-LIN CHANG, order #14803770223. Unlawful distribution of this ebook is prohibited.
Customer name HUAN-LIN CHANG , Order Id: 14803770223
63
GRE Verbal Reasoning Practice Questions
SET 4. Reading Comprehension Questions: Medium
For each of Questions 1 to 9, select one answer choice unless otherwise instructed.
Questions 1 and 2 are based on the following reading passage.
I enjoyed A Dream of Light & Shadow: Portraits of Latin American Women Writers for
the same reasons that, as a child, I avidly consumed women’s biographies: the fascina-
tion with how the biographical details of another female’s life are represented and
interpreted.
A Dream offers a rich read, varied in both the lives and texts of the women por-
trayed, and the perspectives and styles of the sixteen essayists. Yet, as an adult, I have
come to demand of any really “great” book a self-consciousness about the tenuous
nature of representations of reality, a critical contextualization of florid detail, and a
self-awareness of the role of ideology in our lives. In these critical senses, A Dream is
inadequate.
For the following question, consider each of the choices separately and select all that
apply.
1. The author of the passage suggests that A Dream falls short in which of the
following respects?
A
It does not appear to recognize that representations of reality can be
unreliable.
B
It seems to focus on stylistic variety at the expense of accuracy of detail.
C
It offers a wealth of detail without sufficient critical examination of that
detail.
2. Which of the following best describes the function of the second sentence
(“A Dream . . . essayists”) in the context of the passage as a whole?
A
To give examples of how A Dream presents fascinating portraits that
display awareness of the tenuous nature of representations of reality
B
To elaborate on how A Dream fulfills the author’s childhood criteria for a
pleasurable book
C
To suggest that the author enjoyed A Dream for reasons more
sophisticated than the reasons she enjoyed certain books as a child
D
To illustrate ways in which the author finds A Dream to be inadequate in
certain critical senses
E
To imply that A Dream is too varied in focus to provide a proper
contextualization of the biographical details it offers
This ebook was issued to HUAN-LIN CHANG, order #14803770223. Unlawful distribution of this ebook is prohibited.
Customer name HUAN-LIN CHANG , Order Id: 14803770223
64
GRE Verbal Reasoning Practice Questions
3. During the day in Lake Constance, the zooplankton D. hyalina departs for the
depths where food is scarce and the water cold. D. galeata remains near the
warm surface where food is abundant. Even though D. galeata grows and
reproduces much faster, its population is often outnumbered by D. hyalina.
Which of the following, if true, would help resolve the apparent paradox
presented above?
A
The number of species of zooplankton living at the bottom of the lake is
twice that of species living at the surface.
B
Predators of zooplankton, such as whitefish and perch, live and feed near
the surface of the lake during the day.
C
In order to make the most of scarce food resources, D. hyalina matures
more slowly than D. galeata.
D
D. galeata clusters under vegetation during the hottest part of the day to
avoid the Sun’s rays.
E
D. galeata produces twice as many offspring per individual in any given
period of time as does D. hyalina.
This ebook was issued to HUAN-LIN CHANG, order #14803770223. Unlawful distribution of this ebook is prohibited.
Customer name HUAN-LIN CHANG , Order Id: 14803770223
Questions 4 and 5 are based on the following reading passage.
Tocqueville, apparently, was wrong. Jacksonian America was not a fluid, egalitarian
society where individual wealth and poverty were ephemeral conditions. At least so
argues E. Pessen in his iconoclastic study of the very rich in the United States between
1825 and 1850.
Pessen does present a quantity of examples, together with some refreshingly intelli-
gible statistics, to establish the existence of an inordinately wealthy class. Though
active in commerce or the professions, most of the wealthy were not self-made but had
inherited family fortunes. In no sense mercurial, these great fortunes survived the
financial panics that destroyed lesser ones. Indeed, in several cities the wealthiest one
percent constantly increased its share until by 1850 it owned half of the community’s
wealth. Although these observations are true, Pessen overestimates their importance by
concluding from them that the undoubted progress toward inequality in the late eigh-
teenth century continued in the Jacksonian period and that the United States was a
class-ridden, plutocratic society even before industrialization.
4. According to the passage, Pessen indicates that all of the following were true of
the very wealthy in the United States between 1825 and 1850 EXCEPT:
A
They formed a distinct upper class.
B
Many of them were able to increase their holdings.
C
Some of them worked as professionals or in business.
D
Most of them accumulated their own fortunes.
E
Many of them retained their wealth in spite of financial upheavals.
5. Which of the following best states the author’s main point?
A
Pessen’s study has overturned the previously established view of the social
and economic structure of early-nineteenth-century America.
B
Tocqueville’s analysis of the United States in the Jacksonian era remains
the definitive account of this period.
C
Pessen’s study is valuable primarily because it shows the continuity of the
social system in the United States throughout the nineteenth century.
D
The social patterns and political power of the extremely wealthy in the
United States between 1825 and 1850 are well documented.
E
Pessen challenges a view of the social and economic systems in the United
States from 1825 to 1850, but he draws conclusions that are incorrect.
65
GRE Verbal Reasoning Practice Questions
This ebook was issued to HUAN-LIN CHANG, order #14803770223. Unlawful distribution of this ebook is prohibited.
Customer name HUAN-LIN CHANG , Order Id: 14803770223
66
GRE Verbal Reasoning Practice Questions
Questions 6 to 9 are based on the following reading passage.
The evolution of intelligence among early large mammals of the grasslands was due in
great measure to the interaction between two ecologically synchronized groups of
these animals, the hunting carnivores and the herbivores that they hunted. The inter-
action resulting from the differences between predator and prey led to a general
improvement in brain functions; however, certain components of intelligence were
improved far more than others.
The kind of intelligence favored by the interplay of increasingly smarter catchers
and increasingly keener escapers is defined by attention that aspect of mind carrying
consciousness forward from one moment to the next. It ranges from a passive, free-
floating awareness to a highly focused, active fixation. The range through these states
is mediated by the arousal system, a network of tracts converging from sensory systems
to integrating centers in the brain stem. From the more relaxed to the more vigorous
levels, sensitivity to novelty is increased. The organism is more awake, more vigilant;
this increased vigilance results in the apprehension of ever more subtle signals as the
organism becomes more sensitive to its surroundings. The processes of arousal and
concentration give attention its direction. Arousal is at first general, with a flooding of
impulses in the brain stem; then gradually the activation is channeled. Thus begins
concentration, the holding of consistent images. One meaning of intelligence is the way
in which these images and other alertly searched information are used in the context of
previous experience. Consciousness links past attention to the present and permits the
integration of details with perceived ends and purposes.
The elements of intelligence and consciousness come together marvelously to pro-
duce different styles in predator and prey. Herbivores and carnivores develop different
kinds of attention related to escaping or chasing. Although in both kinds of animal,
arousal stimulates the production of adrenaline and norepinephrine by the adrenal
glands, the effect in herbivores is primarily fear, whereas in carnivores the effect is
primarily aggression. For both, arousal attunes the animal to what is ahead. Perhaps it
does not experience forethought as we know it, but the animal does experience some-
thing like it. The predator is searchingly aggressive, inner-directed, tuned by the
nervous system and the adrenal hormones, but aware in a sense closer to human con-
sciousness than, say, a hungry lizard’s instinctive snap at a passing beetle. Using past
events as a framework, the large mammal predator is working out a relationship
between movement and food, sensitive to possibilities in cold trails and distant sounds
and yesterday’s unforgotten lessons. The herbivore prey is of a different mind. Its
mood of wariness rather than searching and its attitude of general expectancy instead
of anticipating are silk-thin veils of tranquillity over an explosive endocrine system.
6. The author refers to a hungry lizard (line 31) primarily in order to
A
demonstrate the similarity between the hunting methods of mammals and
those of nonmammals
B
broaden the application of the argument by including an insectivore as an
example
C
make a distinction between higher and lower levels of consciousness
D
provide an additional illustration of the brutality characteristic of
predators
E
offer an objection to suggestions that all animals lack consciousness
line
5
10
15
20
25
30
35
This ebook was issued to HUAN-LIN CHANG, order #14803770223. Unlawful distribution of this ebook is prohibited.
Customer name HUAN-LIN CHANG , Order Id: 14803770223
67
GRE Verbal Reasoning Practice Questions
7. It can be inferred from the passage that in animals less intelligent than the
mammals discussed in the passage
A
past experience is less helpful in ensuring survival
B
attention is more highly focused
C
muscular coordination is less highly developed
D
there is less need for competition among species
E
environment is more important in establishing the proper ratio of prey
to predator
8. According to the passage, improvement in brain function among early large
mammals resulted primarily from which of the following?
A
Interplay of predator and prey
B
Persistence of free-floating awareness in animals of the grasslands
C
Gradual dominance of warm-blooded mammals over cold-blooded reptiles
D
Interaction of early large mammals with less intelligent species
E
Improvement of the capacity for memory among herbivores and
carnivores
9. According to the passage, as the process of arousal in an organism continues,
all of the following may occur EXCEPT
A
the production of adrenaline
B
the production of norepinephrine
C
a heightening of sensitivity to stimuli
D
an increase in selectivity with respect to stimuli
E
an expansion of the range of states mediated by the brain stem
This ebook was issued to HUAN-LIN CHANG, order #14803770223. Unlawful distribution of this ebook is prohibited.
Customer name HUAN-LIN CHANG , Order Id: 14803770223
SET 5. Discrete Questions: Hard
For Questions 1 to 6, select one entry for each blank from the corresponding column
of choices. Fill all blanks in the way that best completes the text.
1. For some time now, _________ has been presumed not to exist: the cynical
conviction that everybody has an angle is considered wisdom.
A
rationality
B
flexibility
C
diffidence
D
disinterestedness
E
insincerity
2. Human nature and long distances have made exceeding the speed limit a
(i)_________ in the state, so the legislators surprised no one when, acceding to
public demand, they (ii)_________ increased penalties for speeding.
Blank (i) Blank (ii)
A
controversial habit
D
endorsed
B
cherished tradition
E
considered
C
disquieting ritual
F
rejected
3. Serling’s account of his employer’s reckless decision making (i)_________ that
company’s image as (ii)_________ bureaucracy full of wary managers.
Blank (i) Blank (ii)
A
belies
D
an injudicious
B
exposes
E
a disorganized
C
overshadows
F
a cautious
68
GRE Verbal Reasoning Practice Questions
This ebook was issued to HUAN-LIN CHANG, order #14803770223. Unlawful distribution of this ebook is prohibited.
Customer name HUAN-LIN CHANG , Order Id: 14803770223
69
GRE Verbal Reasoning Practice Questions
4. No other contemporary poet’s work has such a well-earned reputation for
(i)_________, and there are few whose moral vision is so imperiously unsparing.
Of late, however, the almost belligerent demands of his severe and densely
forbidding poetry have taken an improbable turn. This new collection is the
poet’s fourth book in six years an ample output even for poets of sunny
disposition, let alone for one of such (ii)_________ over the previous 50 years.
Yet for all his newfound (iii)_________, his poetry is as thorny as ever.
Blank (i) Blank (ii) Blank (iii)
A
patent
D
penitential
G
taciturnity
accessibility austerity
B
intrinsic frivolity
E
intractable
H
volubility
prolixity
C
near
F
impetuous
I
pellucidity
impenetrability prodigality
5. Managers who think that strong environmental performance will (i)_________
their company’s financial performance often (ii)_________ claims that systems
designed to help them manage environmental concerns are valuable tools. By
contrast, managers who perceive environmental performance to be (iii)_________
to financial success may view an environmental management system as
extraneous. In either situation, and whatever their perceptions, it is a manager’s
commitment to achieving environmental improvement rather than the mere
presence of a system that determines environmental performance.
Blank (i) Blank (ii) Blank (iii)
A
eclipse
D
uncritically
G
complementary
accept
B
bolster
E
appropriately
H
intrinsic
acknowledge
C
degrade
F
hotly dispute
I
peripheral
6. Philosophy, unlike most other subjects, does not try to extend our knowledge by
discovering new information about the world. Instead it tries to deepen our
understanding through (i)_________ what is already closest to us the
experiences, thoughts, concepts, and activities that make up our lives but that
ordinarily escape our notice precisely because they are so familiar. Philosophy
begins by finding (ii)_________ the things that are (iii) _________.
Blank (i) Blank (ii) Blank (iii)
A
attainment of
D
essentially
G
most prosaic
irrelevant
B
rumination on
E
utterly
H
somewhat
mysterious hackneyed
C
detachment from
F
thoroughly
I
refreshingly
commonplace novel
This ebook was issued to HUAN-LIN CHANG, order #14803770223. Unlawful distribution of this ebook is prohibited.
Customer name HUAN-LIN CHANG , Order Id: 14803770223
For Questions 7 to 9, select the two answer choices that, when used to complete
the sentence, fit the meaning of the sentence as a whole and produce completed
sentences that are alike in meaning.
7. The government’s implementation of a new code of ethics appeared intended to
shore up the ruling party’s standing with an increasingly _________ electorate at a
time when the party is besieged by charges that it trades favors for campaign
money.
A
aloof
B
placid
C
restive
D
skittish
E
tranquil
F
vociferous
8. Overlarge, uneven, and ultimately disappointing, the retrospective exhibition
seems too much like special pleading for a forgotten painter of real but _________
talents.
A
limited
B
partial
C
undiscovered
D
circumscribed
E
prosaic
F
hidden
9. Newspapers report that the former executive has been trying to keep a low profile
since his _________ exit from the company.
A
celebrated
B
mysterious
C
long-awaited
D
fortuitous
E
indecorous
F
unseemly
70
GRE Verbal Reasoning Practice Questions
This ebook was issued to HUAN-LIN CHANG, order #14803770223. Unlawful distribution of this ebook is prohibited.
Customer name HUAN-LIN CHANG , Order Id: 14803770223
71
GRE Verbal Reasoning Practice Questions
SET 6. Reading Comprehension Questions: Hard
For each of Questions 1 to 8, select one answer choice unless otherwise instructed.
1. In the United States between 1850 and 1880, the number of farmers continued to
increase, but at a rate lower than that of the general population.
Which of the following statements directly contradicts the information presented
above?
A
The number of farmers in the general population increased slightly in the
30 years between 1850 and 1880.
B
The rate of growth of the United States labor force and the rate of growth
of the general population rose simultaneously in the 30 years between
1850 and 1880.
C
The proportion of farmers in the United States labor force remained
constant in the 30 years between 1850 and 1880.
D
The proportion of farmers in the United States labor force decreased from
64 percent in 1850 to 49 percent in 1880.
E
The proportion of farmers in the general population increased from 68
percent in 1850 to 72 percent in 1880.
2. A ten-year comparison between the United States and the Soviet Union in terms
of crop yields per acre revealed that when only planted acreage is compared,
Soviet yields were equal to 68 percent of United States yields. When total
agricultural acreage (planted acreage plus fallow acreage) is compared, however,
Soviet yield was 114 percent of United States yield.
From the information above, which of the following can be most reliably inferred
about United States and Soviet agriculture during the ten-year period?
A
A higher percentage of total agricultural acreage was fallow in the United
States than in the Soviet Union.
B
The United States had more fallow acreage than planted acreage.
C
Fewer total acres of available agricultural land were fallow in the Soviet
Union than in the United States.
D
The Soviet Union had more planted acreage than fallow acreage.
E
The Soviet Union produced a greater volume of crops than the United
States produced.
This ebook was issued to HUAN-LIN CHANG, order #14803770223. Unlawful distribution of this ebook is prohibited.
Customer name HUAN-LIN CHANG , Order Id: 14803770223
72
GRE Verbal Reasoning Practice Questions
Questions 3 and 4 are based on the following reading passage.
For hot desert locations with access to seawater, a new greenhouse design generates
freshwater and cool air. Oriented to the prevailing wind, the front wall of perforated
cardboard, moistened and cooled by a trickle of seawater pumped in, cools and mois-
tens hot air blowing in. This cool, humidified air accelerates plant growth; little water
evaporates from leaves. Though greenhouses normally capture the heat of sunlight,
a double-layered roof, the inner layer coated to reflect infrared light outward, allows
visi ble sunlight in but traps solar heat between the two layers. This heated air, drawn
down from the roof, then mixes with the greenhouse air as it reaches a second
seawater-moistened cardboard wall at the back of the greenhouse. There the air
absorbs more moisture, which then condenses on a metal wall cooled by seawater, and
thus distilled water for irrigating the plants collects.
For the following question, consider each of the choices separately and select all that
apply.
3. It can be inferred that the process described in the passage makes use of which of
the following?
A
The tendency of hot air to rise
B
The directional movement of wind
C
The temperature differential between the sea and the desert
For the following question, consider each of the choices separately and select all that
apply.
4. It can be inferred that the greenhouse roof is designed to allow for which of the
following?
A
The avoidance of intense solar heat inside the greenhouse
B
The entry of sunlight into the greenhouse to make the plants grow
C
The mixture of heated air with greenhouse air to enhance the collection of
moisture
This ebook was issued to HUAN-LIN CHANG, order #14803770223. Unlawful distribution of this ebook is prohibited.
Customer name HUAN-LIN CHANG , Order Id: 14803770223
73
GRE Verbal Reasoning Practice Questions
Questions 5 to 8 are based on the following reading passage.
Many critics of Emily Brontë’s novel Wuthering Heights see its second part as a coun-
ter point that comments on, if it does not reverse, the first part, where a romantic read-
ing receives more confirmation. Seeing the two parts as a whole is encouraged by the
novel’s sophisticated structure, revealed in its complex use of narrators and time shifts.
Granted that the presence of these elements need not argue for an authorial awareness
of novelistic construction comparable to that of Henry James, their presence does
encourage attempts to unify the novel’s heterogeneous parts. However, any interpreta-
tion that seeks to unify all of the novel’s diverse elements is bound to be somewhat
unconvincing. This is not because such an interpretation necessarily stiffens into a the-
sis (although rigidity in any interpretation of this or of any novel is always a danger),
but because Wuthering Heights has recalcitrant elements of undeniable power that, ulti-
mately, resist inclusion in an all-encompassing interpretation. In this respect, Wuther-
ing Heights shares a feature of Hamlet.
5. According to the passage, which of the following is a true statement about the
first and second parts of Wuthering Heights?
A
The second part has received more attention from critics.
B
The second part has little relation to the first part.
C
The second part annuls the force of the first part.
D
The second part provides less substantiation for a romantic reading.
E
The second part is better because it is more realistic.
6. Which of the following inferences about Henry James’s awareness of novelistic
construction is best supported by the passage?
A
James, more than any other novelist, was aware of the difficulties of
novelistic construction.
B
James was very aware of the details of novelistic construction.
C
James’s awareness of novelistic construction derived from his reading of
Brontë.
D
James’s awareness of novelistic construction has led most commentators
to see unity in his individual novels.
E
James’s awareness of novelistic construction precluded him from violating
the unity of his novels.
7. The author of the passage would be most likely to agree that an interpretation
of a novel should
A
not try to unite heterogeneous elements in the novel
B
not be inflexible in its treatment of the elements in the novel
C
not argue that the complex use of narrators or of time shifts indicates a
sophisticated structure
D
concentrate on those recalcitrant elements of the novel that are outside
the novel’s main structure
E
primarily consider those elements of novelistic construction of which the
author of the novel was aware
This ebook was issued to HUAN-LIN CHANG, order #14803770223. Unlawful distribution of this ebook is prohibited.
Customer name HUAN-LIN CHANG , Order Id: 14803770223
74
GRE Verbal Reasoning Practice Questions
For the following question, consider each of the choices separately and select all that
apply.
8. The author of the passage suggests which of the following about Hamlet?
A
Hamlet has usually attracted critical interpretations that tend to stiffen
into theses.
B
Hamlet has elements that are not amenable to an all-encompassing critical
interpretation.
C
Hamlet is less open to an all-encompassing critical interpretation than is
Wuthering Heights.
This ebook was issued to HUAN-LIN CHANG, order #14803770223. Unlawful distribution of this ebook is prohibited.
Customer name HUAN-LIN CHANG , Order Id: 14803770223
ANSWER KEY
SET 1. Discrete Questions: Easy
1. Choice E: elimination
2. Choice A: prophetic
3. Choice B: unguided
4. Choice A: surprising; Choice E: justifies
5. Choice A: activism; Choice E: indifferent to
6. Choice A: deteriorate; AND Choice D: decline
7. Choice B: undiminished; AND Choice D: undamaged
8. Choice C: significant; AND Choice D: considerable
SET 2. Reading Comprehension Questions: Easy
1. Choice B: They should not agree to serve unless they are committed to
maintaining a stance of impartiality.
2. Sentence 4: From then on, he was recognized as an artist of preternatural
abilities, not only as a composer but also as a pianist and conductor.
3. Choice C: The discrepancy between Mendelssohn’s popularity and his standing
among critics
4. Choice D: music critics generally
5. Choice E: establish a standard of comparison for Mendelssohn as a composer
6. Choice B: The percentage of employees in each sector who were men
7. Choice D: The increase in women’s employment that accompanied the Second
World War was longer lasting in agriculture than it was in manufacturing.
8. Choice B: discussing different theories about the transport of plant seeds to
Hawaii
9. Choice D: challenge the claim that ocean currents are responsible for the
transport of plant seeds to Hawaii
SET 3. Discrete Questions: Medium
1. Choice B: unexpected
2. Choice A: reverence for; Choice D: fastidiousness
3. Choice C: deceptive; Choice E: dissimilar
4. Choice C: come through; Choice E: aside from supporting; Choice H: protect
5. Choice A: presumed verifiable; Choice F: debunking; Choice H: bogus
6. Choice A: exacting; AND Choice F: meticulous
7. Choice C: frugality; AND Choice F: thrift
8. Choice A: complicated; Choice F: involved
75
GRE Verbal Reasoning Practice Questions
This ebook was issued to HUAN-LIN CHANG, order #14803770223. Unlawful distribution of this ebook is prohibited.
Customer name HUAN-LIN CHANG , Order Id: 14803770223
SET 4. Reading Comprehension Questions: Medium
1. Choice A: It does not appear to recognize that representations of reality can be
unreliable; AND Choice C: It offers a wealth of detail without sufficient critical
examination of that detail.
2. Choice B: To elaborate on how A Dream fulfills the author’s childhood criteria for
a pleasurable book
3. Choice B: Predators of zooplankton, such as whitefish and perch, live and feed
near the surface of the lake during the day.
4. Choice D: Most of them accumulated their own fortunes.
5. Choice E: Pessen challenges a view of the social and economic systems in the
United States from 1825 to 1850, but he draws conclusions that are incorrect.
6. Choice C: make a distinction between higher and lower levels of consciousness
7. Choice A: past experience is less helpful in ensuring survival
8. Choice A: Interplay of predator and prey
9. Choice E: an expansion of the range of states mediated by the brain stem
SET 5. Discrete Questions: Hard
1. Choice D: disinterestedness
2. Choice B: cherished tradition; Choice F: rejected
3. Choice A: belies; Choice F: a cautious
4. Choice C: near impenetrability; Choice D: penitential austerity; Choice H:
volubility
5. Choice B: bolster; Choice D: uncritically accept; Choice I: peripheral
6. Choice B: rumination on; Choice E: utterly mysterious; Choice G: most prosaic
7. Choice C: restive; AND Choice D: skittish
8. Choice A: limited; AND Choice D: circumscribed
9. Choice E: indecorous; AND Choice F: unseemly
SET 6. Reading Comprehension Questions: Hard
1. Choice E: The proportion of farmers in the general population increased from
68 percent in 1850 to 72 percent in 1880.
2. Choice A: A higher percentage of total agricultural acreage was fallow in the
United States than in the Soviet Union.
3. Choice B: The directional movement of wind; AND Choice C: The temperature
differential between the sea and the desert.
4. Choice A: The avoidance of intense solar heat inside the greenhouse; AND
Choice B: The entry of sunlight into the greenhouse to make the plants grow;
AND Choice C: The mixture of heated air with greenhouse air to enhance the
collection of moisture.
5. Choice D: The second part provides less substantiation for a romantic reading.
6. Choice B: James was very aware of the details of novelistic construction.
7. Choice B: not be inflexible in its treatment of the elements in the novel
8. Choice B: Hamlet has elements that are not amenable to an all-encompassing
critical interpretation.
76
GRE Verbal Reasoning Practice Questions
This ebook was issued to HUAN-LIN CHANG, order #14803770223. Unlawful distribution of this ebook is prohibited.
Customer name HUAN-LIN CHANG , Order Id: 14803770223
77
GRE Verbal Reasoning Practice Questions
Answers and Explanations
SET 1. Discrete Questions: Easy
For Questions 1 to 5, select one entry for each blank from the corresponding column
of choices. Fill all blanks in the way that best completes the text.
1. Dominant interests often benefit most from _________ of governmental
interference in business, since they are able to take care of themselves if left
alone.
A
intensification
B
authorization
C
centralization
D
improvisation
E
elimination
Explanation
The sentence explains why dominant interests often benefit from a certain condition.
Since the explanation is that they are able to take care of themselves if left alone, it
follows that the condition is one in which interference is absent. Thus the best answer
is “elimination.” None of the other answer choices suggests an absence of interference
indeed “intensification,” “authorization,” and “centralization” suggest quite the
opposite.
Thus the correct answer is elimination (Choice E).
2. Kagan maintains that an infant’s reactions to its first stressful experiences are
part of a natural process of development, not harbingers of childhood
unhappiness or _________ signs of adolescent anxiety.
A
prophetic
B
normal
C
monotonous
D
virtual
E
typical
Explanation
The sentence contrasts the infant’s reactions, part of a normal developmental process,
with future unhappiness and anxiety. The missing word describes signs of adolescent
anxiety as they relate to the infant. Choice A is correct: “prophetic” signs, like har -
bingers, foretell future occurrences, and for the infant, adolescent anxiety is a future
occurrence. Since an infant cannot literally display signs of adolescent anxiety, “nor-
mal,” “monotonous,” and “typical” are all incorrect. And “virtual” is incorrect, because
virtual signs are not real signs, and what Kagan is denying is that the infant’s reactions
are not real signs of later unhappiness.
Thus the correct answer is prophetic (Choice A).
This ebook was issued to HUAN-LIN CHANG, order #14803770223. Unlawful distribution of this ebook is prohibited.
Customer name HUAN-LIN CHANG , Order Id: 14803770223
78
GRE Verbal Reasoning Practice Questions
3. An investigation that is _________ can occasionally yield new facts, even notable
ones, but typically the appearance of such facts is the result of a search in a
definite direction.
A
timely
B
unguided
C
consistent
D
uncomplicated
E
subjective
Explanation
As the words “can occasionally” and “but typically” indicate, the missing word de -
scribes an investigation that contrasts with a “search in a definite direction.” Among
the answer choices, only “unguided” provides a contrasting description; none of the
other choices suggests an appropriate contrast.
Thus the correct answer is unguided (Choice B).
4. It is (i)_________ that so many portrait paintings hang in art museums, since the
subject matter seems to dictate a status closer to pictures in the family photo -
graph album than to high art. But perhaps it is the artistic skill with which the
portraits are painted that (ii)_________ their presence in art museums.
Blank (i) Blank (ii)
A
surprising
D
challenges
B
understandable
E
justifies
C
irrelevant
F
changes
Explanation
In the part following “since,” the first sentence of the paragraph suggests that the sub-
ject matter of portraits might not seem to fit with the idea of “high art.” So the sugges-
tion is that the presence of portrait paintings in art museums is in that sense odd or
unfitting. Of the choices available for Blank (i), “surprising” is the one that expresses
this sense. The second sentence, in contrast to the first, offers a point in favor of por-
traits “artistic skill.” So the second sentence is offering a reason why portraits should
be in art museums. Of the choices for Blank (ii), “justifies” is the one that completes
that thought.
Thus the correct answer is surprising (Choice A) and justifies (Choice E).
5. In stark contrast to his later (i)_________, Simpson was largely (ii)_________
politics during his college years, despite the fact that the campus he attended was
rife with political activity.
Blank (i) Blank (ii)
A
activism
D
devoted to
B
apathy
E
indifferent to
C
affability
F
shaped by
This ebook was issued to HUAN-LIN CHANG, order #14803770223. Unlawful distribution of this ebook is prohibited.
Customer name HUAN-LIN CHANG , Order Id: 14803770223
Explanation
The sentence tells us that there is a contrast between the way Simpson related to poli-
tics in his college years and how he related to politics later in life. So the choices that
complete the blanks must contrast with each other. The part of the sentence beginning
with “despite” indicates that Simpson’s relation to politics in his college years did not
involve engagement in the political activity that was “rife.” Of the choices for Blank (ii),
only “indifferent to” conveys that nonengagement. And of the choices for Blank (i), only
“activism” supplies the required contrast with “indifferent to.”
Thus the correct answer is activism (Choice A) and indifferent to (Choice E).
For Questions 6 to 8, select the two answer choices that, when used to complete
the sentence, fit the meaning of the sentence as a whole and produce completed
sentences that are alike in meaning.
6. As my eyesight began to _________, I spent a lot of time writing about it both
poems and “eye journals” describing what I saw as I looked out through
damaged eyes.
A
deteriorate
B
sharpen
C
improve
D
decline
E
recover
F
adjust
Explanation
The author has “damaged” eyes, and any word that fills the blank must reflect that fact.
The words that best do so are “deteriorate” and “decline” (Choices A and D), which gen-
erate sentences alike in meaning. “Sharpen” and “improve” produce sentences alike in
meaning, but neither word makes sense when inserted into the blank. Though “adjust”
makes some sense when inserted into the blank, no other option produces a sentence
similar in meaning.
Thus the correct answer is deteriorate (Choice A) and decline (Choice D).
7. The judge’s standing in the legal community, though shaken by phony allegations
of wrongdoing, emerged, at long last, _________ .
A
unqualified
B
undiminished
C
undecided
D
undamaged
E
unresolved
F
unprincipled
Explanation
The use of the word “though” establishes a contrast between the blank, which requires
a description of the judge’s standing, and “phony allegations of wrongdoing.” Thus the
words that best complete the blank must indicate that the judge’s reputation was not
79
GRE Verbal Reasoning Practice Questions
This ebook was issued to HUAN-LIN CHANG, order #14803770223. Unlawful distribution of this ebook is prohibited.
Customer name HUAN-LIN CHANG , Order Id: 14803770223
adversely affected by these allegations. The only words that do so are “undiminished”
and “undamaged” (Choices B and D), which produce sentences alike in meaning.
“Undecided” and “unresolved” also produce sentences alike in meaning, but neither
word makes sense when inserted into the blank.
Thus the correct answer is undiminished (Choice B) and undamaged (Choice D).
8. Modern agricultural practices have been extremely successful in increasing the
productivity of major food crops, yet despite heavy use of pesticides, _________
losses to diseases and insect pests are sustained each year.
A
incongruous
B
reasonable
C
significant
D
considerable
E
equitable
F
fortuitous
Explanation
The word “despite” suggests the level of losses is somehow surprising given the heavy
use of pesticides. The only words that describe an appropriate level of losses are “sig-
nificant” and “considerable” (Choices C and D), which produce sentences alike in
meaning. “Reasonable” and “equitable” also produce sentences alike in meaning, but
neither word generates the contrast necessary for the sentence to make sense.
Thus the correct answer is significant (Choice C) and considerable (Choice D).
SET 2. Reading Comprehension Questions: Easy
For each of Questions 1 to 9, select one answer choice unless otherwise instructed.
1. A person who agrees to serve as mediator between two warring factions at the
request of both abandons by so agreeing the right to take sides later. To take sides
at a later point would be to suggest that the earlier presumptive impartiality was
a sham.
The passage above emphasizes which of the following points about mediators?
A
They should try to form no opinions of their own about any issue that is
related to the dispute.
B
They should not agree to serve unless they are committed to maintaining
a stance of impartiality.
C
They should not agree to serve unless they are equally acceptable to all
parties to a dispute.
D
They should feel free to take sides in the dispute right from the start,
provided that they make their biases publicly known.
E
They should reserve the right to abandon their impartiality so as not to be
open to the charge of having been deceitful.
80
GRE Verbal Reasoning Practice Questions
This ebook was issued to HUAN-LIN CHANG, order #14803770223. Unlawful distribution of this ebook is prohibited.
Customer name HUAN-LIN CHANG , Order Id: 14803770223
81
GRE Verbal Reasoning Practice Questions
Explanation
By pointing out the consequences of abandoning impartiality, the paragraph points out
the importance for mediators of maintaining impartiality at all times. This is the point
made in Choice B, which is therefore the correct answer. Choice A is incorrect,
because it goes further than anything asserted in the passage. The passage does not
rule out the possibility that one can have an opinion about issues related to a dispute
without taking sides in the actual dispute. Choice C is incorrect because it is a presup-
position on which the passage is based rather than the point of the passage; that is, the
fact that the mediator is acceptable to both parties is a given, since they both ask the
mediator to serve. Choices D and E are both inconsistent with the main point of the
passage, the importance of impartiality at all times, so both are incorrect.
Questions 2 to 5 are based on the following reading passage.
Was Felix Mendelssohn (1809–1847) a great composer? On its face, the question seems
absurd. One of the most gifted prodigies in the history of music, he produced his first
masterpiece at sixteen. From then on, he was recognized as an artist of preternatural
abilities, not only as a composer but also as a pianist and conductor. But Mendelssohn’s
enduring popularity has often been at odds sometimes quite sharply with his
criti cal standing. Despite general acknowledgment of his genius, there has been a
noticeable reluctance to rank him with, say, Schumann or Brahms. As Haggin put it,
Mendelssohn, as a composer, was a “minor master . . . working on a small scale of emo-
tion and texture.”
Description
The passage starts by outlining the popular view that Mendelssohn was a great com-
poser and then points out that critics do not generally accord him that status.
2. Select a sentence in the passage whose function is to indicate the range of
Mendelssohn’s musical talents.
Explanation
This question asks which sentence in the passage serves to indicate the range of
Mendelssohn’s musical talents. The correct answer is the fourth sentence (“From then
. . . conductor”), the only sentence in the passage that mentions Mendelssohn’s achieve-
ments across three different realms: composing, piano performance, and conducting.
All the other sentences can be eliminated because, while they consider the question of
Mendelssohn’s claim to greatness, they do not specifically discuss the broad range of
his musical talents.
This ebook was issued to HUAN-LIN CHANG, order #14803770223. Unlawful distribution of this ebook is prohibited.
Customer name HUAN-LIN CHANG , Order Id: 14803770223
3. The passage suggests that anyone attempting to evaluate Mendelssohn’s career
must confront which of the following dichotomies?
A
The tension between Mendelssohn’s career as a composer and his career
as a pianist and conductor
B
The contrast between Mendelssohn’s popularity and that of Schumann
and Brahms
C
The discrepancy between Mendelssohn’s popularity and his standing
among critics
D
The inconsistency between Mendelssohn’s reputation during his lifetime
and his reputation since his death
E
The gap between Mendelssohn’s prodigious musical beginnings and his
decline in later years
Explanation
The passage clearly presents the discrepancy between Mendelssohn’s popularity and
his critical standing as an interpretive problem. Therefore, Choice C is correct. The
other answer choices are incorrect because the passage never indicates that there was
any conflict among the different aspects of Mendelssohn’s professional life; never dis-
cusses Schumann’s and Brahms’s popularity; does not discuss any differences between
Mendelssohn’s reputation during his lifetime and after his death; and makes no men-
tion of a decline in Mendelssohn’s later life.
4. It can be inferred that the “reluctance” mentioned in the passage is being
ascribed to
A
most composers since Mendelssohn
B
Schumann and Brahms
C
the music-listening public
D
music critics generally
E
Haggin exclusively
Explanation
Choice D is correct. The “reluctance” is mentioned in the context of a discussion about
Mendelssohn’s critical standing and thus is being ascribed to music critics generally.
Choices A and B can be eliminated because the passage does not discuss any com-
posers’ views of Mendelssohn. Choice C is incorrect because the word “reluctance” is
mentioned only after the passage turns from discussing the popular view of
Mendelssohn to the critical view. Choice E is incorrect because the words “As Haggin
put it” indicate that Haggin is only one example of critics who have this reluctance.
5. The author mentions Schumann and Brahms primarily in order to
A
provide examples of composers who are often compared with
Mendelssohn
B
identify certain composers who are more popular than Mendelssohn
C
identify composers whom Mendelssohn influenced
D
establish the milieu in which Mendelssohn worked
E
establish a standard of comparison for Mendelssohn as a composer
82
GRE Verbal Reasoning Practice Questions
This ebook was issued to HUAN-LIN CHANG, order #14803770223. Unlawful distribution of this ebook is prohibited.
Customer name HUAN-LIN CHANG , Order Id: 14803770223
Explanation
Schumann and Brahms are mentioned as a way of explaining how critics rank
Mendelssohn that is, as less accomplished than some other composers who are
widely acknowledged as major. Therefore, Choice E is correct. Choice A might look
like a correct answer at first glance. However, careful consideration reveals that the
point the author is making when Schumann and Brahms are mentioned is not the fre-
quency of that comparison but the results of it. Therefore, Choice A can be eliminated.
Choices B, C, and D are incorrect because the passage does not discuss Schumann’s
and Brahms’s popularity, Mendelssohn’s influence on other composers, or the milieu in
which Mendelssohn worked.
Questions 6 and 7 are based on the following reading passage.
While most scholarship on women’s employment in the United States recognizes that
the Second World War (1939–1945) dramatically changed the role of women in the
workforce, these studies also acknowledge that few women remained in manufacturing
jobs once men returned from the war. But in agriculture, unlike other industries where
women were viewed as temporary workers, women’s employment did not end with the
war. Instead, the expansion of agriculture and a steady decrease in the number of male
farmworkers combined to cause the industry to hire more women in the postwar years.
Consequently, the 1950s saw a growing number of women engaged in farm labor, even
though rhetoric in the popular media called for the return of women to domestic life.
Description
The first sentence states that the Second World War led to significant changes in
women’s employment, but that these changes were largely reversed in manufacturing
after the war. The second sentence discusses the fact that unlike in other industries,
employment of women in agriculture was more permanent; the third provides more
detail regarding the trend in agriculture and the reasons for it; and the fourth summa-
rizes the consequences of the trend.
6. It can be inferred from the passage that the manufacturing and agricultural
sectors in the United States following the Second World War differed in which of
the following respects?
A
The rate of expansion in each sector
B
The percentage of employees in each sector who were men
C
The trend in the wages of men employed in each sector
D
The attitude of the popular media toward the employment of women in
each sector
E
The extent to which women in each sector were satisfied with their jobs
Explanation
The correct choice for this question is Choice B. We are told that few women remained
in the manufacturing sector once men returned from the war, while the number of
women who worked in agriculture increased after the war as the number of men in
agriculture decreased. It is therefore inferable that the percentage of employees work-
ing in manufacturing who were men increased while the percentage of employees
working in agriculture who were men decreased. Choices A, C, and E are incorrect
83
GRE Verbal Reasoning Practice Questions
This ebook was issued to HUAN-LIN CHANG, order #14803770223. Unlawful distribution of this ebook is prohibited.
Customer name HUAN-LIN CHANG , Order Id: 14803770223
because the passage provides no information about rates of expansion, wage trends, or
women’s job satisfaction. Choice D is incorrect because the only mention of the popu-
lar media occurs in the final sentence, and no distinction is made between the sectors
there.
7. Which of the following statements about women’s employment in the United
States during and after the Second World War is most clearly supported by the
passage?
A
Most women who joined the workforce during the Second World War
wanted to return to domestic life when the war ended.
B
The great majority of women who joined the workforce during the Second
World War were employed in manufacturing jobs.
C
The end of the Second World War was followed by a large-scale transfer of
women workers from manufacturing to agriculture.
D
The increase in women’s employment that accompanied the Second World
War was longer lasting in agriculture than it was in manufacturing.
E
The popular media were more forceful in calling for women to join the
workforce during the Second World War than in calling for women to
return to domestic life after the war.
Explanation
The correct choice for this question is Choice D. We are told in the passage that
women’s employment in manufacturing fell quickly after men returned from the war.
However, not only did women’s employment in agriculture not decline after the end of
the war, it actually increased. The other choices are incorrect because the passage pro-
vides no information about what women who joined the workforce wanted to do; about
the distribution of women across industries; about what happened to women who left
manufacturing; nor about media appeals for women to join the wartime workforce.
Questions 8 and 9 are based on the following reading passage.
Since the Hawaiian Islands have never been connected to other land masses, the great
variety of plants in Hawaii must be a result of the long-distance dispersal of seeds, a
process that requires both a method of transport and an equivalence between the ecol-
ogy of the source area and that of the recipient area.
There is some dispute about the method of transport involved. Some biologists
argue that ocean and air currents are responsible for the transport of plant seeds to
Hawaii. Yet the results of flotation experiments and the low temperatures of air cur-
rents cast doubt on these hypotheses. More probable is bird transport, either externally,
by accidental attachment of the seeds to feathers, or internally, by the swallowing of
fruit and subsequent excretion of the seeds. While it is likely that fewer varieties of
plant seeds have reached Hawaii externally than internally, more varieties are known
to be adapted to external than to internal transport.
Description
The passage raises the question of how seeds reached the Hawaiian Islands. It intro-
duces one possible method ocean and air currents but refers to evidence that casts
doubt on that method. It then introduces a second method bird transport and dis-
cusses two ways in which that might occur.
84
GRE Verbal Reasoning Practice Questions
line
5
10
This ebook was issued to HUAN-LIN CHANG, order #14803770223. Unlawful distribution of this ebook is prohibited.
Customer name HUAN-LIN CHANG , Order Id: 14803770223
85
GRE Verbal Reasoning Practice Questions
8. The author of the passage is primarily concerned with
A
discussing different approaches biologists have taken to testing theories
about the distribution of plants in Hawaii
B
discussing different theories about the transport of plant seeds to Hawaii
C
discussing the extent to which air currents are responsible for the
dispersal of plant seeds to Hawaii
D
resolving a dispute about the adaptability of plant seeds to bird transport
E
resolving a dispute about the ability of birds to carry plant seeds long
distances
Explanation
Given the description of the passage above, it is clear that Choice B is correct: the pas-
sage focuses on “different theories about the transport of plant seeds to Hawaii.”
Choice A can be eliminated: while the passage does refer to flotation experiments, it
does not elaborate on experimental methods. Choice C identifies an idea that is part of
the passage’s main concern, but since this is only one of the competing theories dis-
cussed in the passage, not the primary focus, Choice C is incorrect. Choices D and E
are incorrect because the passage does not resolve any disputes.
9. The author mentions the results of flotation experiments on plant seeds (lines
7–8) most probably in order to
A
support the claim that the distribution of plants in Hawaii is the result of
the long-distance dispersal of seeds
B
lend credibility to the thesis that air currents provide a method of
transport for plant seeds to Hawaii
C
suggest that the long-distance dispersal of seeds is a process that requires
long periods of time
D
challenge the claim that ocean currents are responsible for the transport
of plant seeds to Hawaii
E
refute the claim that Hawaiian flora evolved independently from flora in
other parts of the world
Explanation
This question asks why the author mentions flotation experiments. Flotation experi-
ments are mentioned in the passage in order to show that some evidence casts doubt
on the claim that ocean currents were the means by which seeds were transported to
Hawaii. Thus, Choice D is correct. Choice A is incorrect since the claim that plant dis-
tribution in Hawaii is the result of long-distance dispersal of seeds is a given in the pas-
sage, not an idea that the author feels a need to substantiate. Choice B is eliminable
since the flotation experiments are introduced at a point where the author is challeng-
ing, rather than lending credibility to, the air current hypothesis and because flotation
experiments would more likely reflect on ocean currents than air currents. Choice C is
eliminable since the passage does not address the length of time required for long-
distance seed dispersal. Finally, Choice E is eliminable since it too describes an idea
that is not discussed in the passage.
This ebook was issued to HUAN-LIN CHANG, order #14803770223. Unlawful distribution of this ebook is prohibited.
Customer name HUAN-LIN CHANG , Order Id: 14803770223
86
GRE Verbal Reasoning Practice Questions
SET 3. Discrete Questions: Medium
For Questions 1 to 5, select one entry for each blank from the corresponding column
of choices. Fill all blanks in the way that best completes the text.
1. It comes as no surprise that societies have codes of behavior; the character of the
codes, on the other hand, can often be _________ .
A
predictable
B
unexpected
C
admirable
D
explicit
E
confusing
Explanation
The words “on the other hand” indicate that while the existence of societal codes of
behavior is no surprise, their character may be quite surprising. Thus the correct
answer is Choice B, unexpected, which means the same as surprising. “Predictable” is
the very opposite of surprising, and none of the other answer choices means “surpris-
ing.”
Thus the correct answer is unexpected (Choice B).
2. Like Béla Bartók, Ruth Crawford not only brought a composer’s acumen to the
notation of folk music, she also had a marked (i)_________ the task. This was
clear in her agonizing over how far to try to represent the minute details of a
performance in a written text, and this (ii)_________ makes her work a landmark
in ethnomusicology.
Blank (i) Blank (ii)
A
reverence for
D
fastidiousness
B
detachment from
E
didacticism
C
curiosity about
F
iconoclasm
Explanation
In this example, both blanks can be filled by focusing on the statement that Crawford
agonized over the details in her representations of folk music performances. The only
choice for blank (ii) that matches this description is “fastidiousness”; neither “didacti-
cism” nor “iconoclasm” reflects the notion of agonizing over details. Similarly, only
“reverence for” fits in Blank (i), since neither “detachment from” nor “curiosity about”
reflects the degree of care Crawford took.
Thus the correct answer is reverence for (Choice A) and fastidiousness (Choice D).
This ebook was issued to HUAN-LIN CHANG, order #14803770223. Unlawful distribution of this ebook is prohibited.
Customer name HUAN-LIN CHANG , Order Id: 14803770223
87
GRE Verbal Reasoning Practice Questions
3. Political advertising may well be the most (i)_________ kind of advertising:
political candidates are usually quite (ii)_________, yet their campaign
advertisements often hide important differences behind smoke screens of smiles
and empty slogans.
Blank (i) Blank (ii)
A
polemical
D
interchangeable
B
effective
E
dissimilar
C
deceptive
F
vocal
Explanation
Looking at Blank (i), it is hard to select a correct answer, since all three answer choices
fit the immediate context well. Looking to the second part of the sentence, however, we
can see such expressions as “hide” and “smoke screens,” both of which suggest that the
correct answer for Blank (i) is “deceptive.” Making that assumption, we can go on to
see that the answer for Blank (ii) is “dissimilar,” since what is deceptive about political
advertisements is that they hide important differences. Reading the sentence again
with “deceptive” and “dissimilar” in place confirms those choices.
Thus the correct answer is deceptive (Choice C) and dissimilar (Choice E).
4. Richard M. Russell said 52 percent of the nation’s growth since the Second World
War had (i)_________ invention. He said, (ii)_________ research, the government’s
greatest role in assuring continuing innovation is promoting a strong, modern
patent office. “Unless we can (iii)_________ original ideas, we will not have
invention,” Mr. Russell said. Speculating on the state of innovation over the next
century, several inventors agreed that the future lay in giving children the tools to
think creatively and the motivation to invent.
Blank (i) Blank (ii) Blank (iii)
A
been at the
D
in addition to
G
evaluate
expense of restricting
B
no bearing on
E
aside from
H
protect
supporting
C
come through
F
far from
I
disseminate
exaggerating
Explanation
A quick overview of the paragraph shows that its topic is the encouragement of inven-
tion and innovation. This implies that Blank (i) should be filled with “come through,”
which emphasizes the importance of invention; the other choices suggest that inven-
tion is irrelevant or somehow harmed by growth. Again, the only one of the choices for
Blank (ii) that continues the theme of encouraging invention is “aside from support-
ing.” Finally, the second sentence emphasizes the importance for innovation of a strong
patent office, and this thought is reaffirmed in the following quotation from Mr. Rus-
sell, which requires “protect” in Blank (iii).
Thus the correct answer is come through (Choice C), aside from supporting
(Choice E), and protect (Choice H).
This ebook was issued to HUAN-LIN CHANG, order #14803770223. Unlawful distribution of this ebook is prohibited.
Customer name HUAN-LIN CHANG , Order Id: 14803770223
88
GRE Verbal Reasoning Practice Questions
5. Statements presented as fact in a patent application are (i)_________ unless a
good reason for doubt is found. The invention has only to be deemed “more likely
than not” to work in order to receive initial approval. And, although thousands of
patents are challenged in court for other reasons, no incentive exists for anyone
to expend effort (ii)_________ the science of an erroneous patent. For this reason
the endless stream of (iii)_________ devices will continue to yield occasional
patents.
Blank (i) Blank (ii) Blank (iii)
A
presumed
D
corroborating
G
novel
verifiable
B
carefully
E
advancing
H
bogus
scrutinized
C
considered
F
debunking
I
obsolete
capricious
Explanation
The paragraph appears to be explaining some odd or unexpected aspect of the patent
process. The third sentence helps to clarify what this aspect is; it discusses challenges
to patents. The only choice for Blank (ii) that is concerned with challenging a patent is
“debunking,” since “corroborating” and “advancing” suggest support instead. This in
turn provides the answer for the third blank, since the preceding sentence does explain
how “bogus” devices may nonetheless get a patent. And we can also now better under-
stand the first sentence it too must help explain how bogus devices get patents,
which it can do only if the blank is filled with “presumed verifiable,” suggesting that
patent applications are taken at face value and not dismissed out of hand nor subjected
to careful scrutiny.
Thus the correct answer is presumed verifiable (Choice A), debunking (Choice
F), and bogus (Choice H).
For Questions 6 to 8, select the two answer choices that, when used to complete
the sentence, fit the meaning of the sentence as a whole and produce completed
sentences that are alike in meaning.
6. Ever a demanding reader of the fiction of others, the novelist Chase was likewise
often the object of _________ analyses by his contemporaries.
A
exacting
B
copious
C
respectful
D
acerbic
E
scathing
F
meticulous
Explanation
The use of the word “likewise” indicates that the analyses of Chase’s work by contem-
poraries were like the readings he gave the fiction of others. Since he is described as a
“demanding reader,” the words that best fit the blank will be similar in meaning to
This ebook was issued to HUAN-LIN CHANG, order #14803770223. Unlawful distribution of this ebook is prohibited.
Customer name HUAN-LIN CHANG , Order Id: 14803770223
89
GRE Verbal Reasoning Practice Questions
“demanding.” The words that meet this requirement are “exacting” (Choice A) and
“meticulous” (Choice F), and they produce sentences that are alike in meaning.
Although “acerbic analyses” means close to the same thing as “scathing analyses,” both
“acerbic” and “scathing” have meanings that are quite different from “demanding,” so
neither fits well in the blank.
Thus the correct answer is exacting (Choice A) and meticulous (Choice F).
7. Her _________ should not be confused with miserliness; as long as I have known
her, she has always been willing to assist those who are in need.
A
stinginess
B
diffidence
C
frugality
D
illiberality
E
intolerance
F
thrift
Explanation
The sentence explains that the person spoken of is not miserly, since she is quite pre-
pared to be generous. So for the sentence to make sense, the word filling the blank has
to be something that is consistent with generosity and yet might, by those without a
full understanding of her behavior, be mistaken for miserliness. The words “frugality”
and “thrift” fulfill this requirement and yield two sentences that are alike in meaning,
so that pair forms the correct answer. Neither “stinginess” nor “illiberality” makes
sense in the sentence, since they are synonymous with “miserliness” and inconsistent
with generosity. Other choices, such as “diffidence,” might perhaps make a sensible
sentence if placed in the blank but do not form part of the correct answer since they
have no companion word that would make a sentence of similar meaning.
Thus the correct answer is frugality (Choice C) and thrift (Choice F).
8. A misconception frequently held by novice writers is that sentence structure
mirrors thought: the more convoluted the structure, the more _________ the
ideas.
A
complicated
B
engaged
C
essential
D
fanciful
E
inconsequential
F
involved
Explanation
Because the second half of the sentence illustrates the idea that “structure mirrors
thought,” any word that fills the blank must be similar in meaning to “convoluted.” The
two words that are similar to “convoluted” are “complicated” and “involved” (Choices
A and F), which produce sentences alike in meaning. “Fanciful,” while somewhat sim-
ilar in meaning to “convoluted,” is not as similar to either “complicated” or “involved”
as those words are to each other. The other answer choices are not similar in meaning
to “convoluted,” and thus do not produce coherent sentences.
Thus the correct answer is complicated (Choice A) and involved (Choice F).
This ebook was issued to HUAN-LIN CHANG, order #14803770223. Unlawful distribution of this ebook is prohibited.
Customer name HUAN-LIN CHANG , Order Id: 14803770223
SET 4. Reading Comprehension Questions: Medium
For each of Questions 1 to 9, select one answer choice unless otherwise instructed.
Questions 1 and 2 are based on the following reading passage.
I enjoyed A Dream of Light & Shadow: Portraits of Latin American Women Writers for
the same reasons that, as a child, I avidly consumed women’s biographies: the fascina-
tion with how the biographical details of another female’s life are represented and
interpreted.
A Dream offers a rich read, varied in both the lives and texts of the women por-
trayed, and the perspectives and styles of the sixteen essayists. Yet, as an adult, I have
come to demand of any really “great” book a self-consciousness about the tenuous
nature of representations of reality, a critical contextualization of florid detail, and a
self-awareness of the role of ideology in our lives. In these critical senses, A Dream is
inadequate.
Description
The passage follows the following structure: the first sentence discusses a collection of
biographical sketches and what the author found particularly appealing about similar
works as a child; the second sentence describes several positive aspects of this particu-
lar collection and how it satisfies the author’s early interests; the third sentence
describes a demanding set of criteria that the author now applies when assessing such
work; and in the fourth sentence the author says the collection being discussed does
not meet those criteria.
For the following question, consider each of the choices separately and select all that
apply.
1. The author of the passage suggests that A Dream falls short in which of the
following respects?
A
It does not appear to recognize that representations of reality can be
unreliable.
B
It seems to focus on stylistic variety at the expense of accuracy of detail.
C
It offers a wealth of detail without sufficient critical examination of that
detail.
Explanation
Choices A and C are correct. We know from the final sentence that the collection falls
short of several criteria established by the author.
Choice A is correct: the book does not demonstrate sufficient awareness of the
“tenuous nature of representations of reality.”
Choice B is incorrect: there is no mention in the passage of any concern on the part
of the author about the accuracy of detail.
Choice C is correct: the book does not offer an adequate “critical contextualization
of florid detail.”
90
GRE Verbal Reasoning Practice Questions
This ebook was issued to HUAN-LIN CHANG, order #14803770223. Unlawful distribution of this ebook is prohibited.
Customer name HUAN-LIN CHANG , Order Id: 14803770223
2. Which of the following best describes the function of the second sentence
(“A Dream . . . essayists”) in the context of the passage as a whole?
A
To give examples of how A Dream presents fascinating portraits that
display awareness of the tenuous nature of representations of reality
B
To elaborate on how A Dream fulfills the author’s childhood criteria for a
pleasurable book
C
To suggest that the author enjoyed A Dream for reasons more
sophisticated than the reasons she enjoyed certain books as a child
D
To illustrate ways in which the author finds A Dream to be inadequate in
certain critical senses
E
To imply that A Dream is too varied in focus to provide a proper
contextualization of the biographical details it offers
Explanation
This question asks about the function of the second sentence. The correct choice is
Choice B. As discussed in the description of the passage, that sentence describes what
is appealing about the collection in the context of the author’s childhood tastes. Choice
A is incorrect both because the sentence does not provide examples and because the col-
lection does not display an awareness of the tenuous nature of representations of real-
ity. Choice C is not correct: although one might suspect that the author’s enjoyment of
collections as an adult would be on a more sophisticated level than when she was young,
there is no discussion or even suggestion of that in the passage. Choice D is incorrect be-
cause the sentence describes the virtues of the collection. The aspects of the collection
that the author finds inadequate are not addressed until later. Choice E is incorrect be-
cause, according to the passage, the fact that the collection is varied makes it a “rich”
read. There is no suggestion that the variety hinders proper contextualization.
3. During the day in Lake Constance, the zooplankton D. hyalina departs for the
depths where food is scarce and the water cold. D. galeata remains near the
warm surface where food is abundant. Even though D. galeata grows and
reproduces much faster, its population is often outnumbered by D. hyalina.
Which of the following, if true, would help resolve the apparent paradox
presented above?
A
The number of species of zooplankton living at the bottom of the lake is
twice that of species living at the surface.
B
Predators of zooplankton, such as whitefish and perch, live and feed near
the surface of the lake during the day.
C
In order to make the most of scarce food resources, D. hyalina matures
more slowly than D. galeata.
D
D. galeata clusters under vegetation during the hottest part of the day to
avoid the Sun’s rays.
E
D. galeata produces twice as many offspring per individual in any given
period of time as does D. hyalina.
Description
The paragraph presents an apparent paradox: the zooplankton that spends the day in
less hospitable conditions often outnumbers the one that stays in more hospitable con-
ditions.
91
GRE Verbal Reasoning Practice Questions
This ebook was issued to HUAN-LIN CHANG, order #14803770223. Unlawful distribution of this ebook is prohibited.
Customer name HUAN-LIN CHANG , Order Id: 14803770223
92
GRE Verbal Reasoning Practice Questions
Explanation
The presence of predators of zooplankton feeding near the surface during the day
would suggest that D. galeata is consumed at a higher rate than D. hyalina: this would
explain why D. hyalina is often more numerous, so Choice B is correct. Choices C and
E are incorrect because although they help to explain why the two zooplankton repro-
duce at different rates, they do not help to resolve the apparent paradox. Choices A and
D are incorrect because nothing is said in the paragraph to show the relevance of the
presence of other species of zooplankton, nor of the habit of clustering under vegeta-
tion, to the relative population size of the two species.
Questions 4 and 5 are based on the following reading passage.
Tocqueville, apparently, was wrong. Jacksonian America was not a fluid, egalitarian
society where individual wealth and poverty were ephemeral conditions. At least so
argues E. Pessen in his iconoclastic study of the very rich in the United States between
1825 and 1850.
Pessen does present a quantity of examples, together with some refreshingly intelli-
gible statistics, to establish the existence of an inordinately wealthy class. Though
active in commerce or the professions, most of the wealthy were not self-made but had
inherited family fortunes. In no sense mercurial, these great fortunes survived the
financial panics that destroyed lesser ones. Indeed, in several cities the wealthiest one
percent constantly increased its share until by 1850 it owned half of the community’s
wealth. Although these observations are true, Pessen overestimates their importance by
concluding from them that the undoubted progress toward inequality in the late eigh-
teenth century continued in the Jacksonian period and that the United States was a
class-ridden, plutocratic society even before industrialization.
Description
The passage describes Pessen’s argument that Jacksonian America was not fluid and
egalitarian but class-ridden and plutocratic, and criticizes it for leaping to an unjusti-
fied conclusion.
4. According to the passage, Pessen indicates that all of the following were true of
the very wealthy in the United States between 1825 and 1850 EXCEPT:
A
They formed a distinct upper class.
B
Many of them were able to increase their holdings.
C
Some of them worked as professionals or in business.
D
Most of them accumulated their own fortunes.
E
Many of them retained their wealth in spite of financial upheavals.
Explanation
For this question, you are to identify the one statement that CANNOT be correctly
attributed to Pessen. Therefore, you must first determine which of the statements given
can be attributed to Pessen. According to the passage, Pessen maintains all of the fol-
lowing: there was a class of “inordinately wealthy” Americans (Choice A); in some
places that class “constantly increased its share” (Choice B); its members were “active
in commerce or the professions” (Choice C); and “these great fortunes survived the
financial panics that destroyed lesser ones” (Choice E). However, Pessen also main-
tains, in contradiction to Choice D, that “most of the wealthy were not self-made but
had inherited family fortunes.” Therefore, Choice D is correct.
This ebook was issued to HUAN-LIN CHANG, order #14803770223. Unlawful distribution of this ebook is prohibited.
Customer name HUAN-LIN CHANG , Order Id: 14803770223
5. Which of the following best states the author’s main point?
A
Pessen’s study has overturned the previously established view of the social
and economic structure of early-nineteenth-century America.
B
Tocqueville’s analysis of the United States in the Jacksonian era remains
the definitive account of this period.
C
Pessen’s study is valuable primarily because it shows the continuity of the
social system in the United States throughout the nineteenth century.
D
The social patterns and political power of the extremely wealthy in the
United States between 1825 and 1850 are well documented.
E
Pessen challenges a view of the social and economic systems in the United
States from 1825 to 1850, but he draws conclusions that are incorrect.
Explanation
It is important to realize that although most of the passage is devoted to describing
Pessen’s study, the author’s main point is to criticize the conclusion Pessen draws.
Choices A, C, and D omit any reference to the author’s critical evaluation of Pessen’s
study, and hence are not statements of the author’s main point. Choice B is also incor-
rect. Because Pessen criticizes Tocqueville and the author criticizes Pessen, it might
seem that the author’s main point is to defend Tocqueville’s analysis. However, the pas-
sage does not indicate that Tocqueville’s analysis is definitive. Choice E is correct.
According to the first paragraph, Pessen challenges Tocqueville’s view, but according to
the second paragraph, Pessen’s conclusions are incorrect.
Questions 6 to 9 are based on the following reading passage.
The evolution of intelligence among early large mammals of the grasslands was due in
great measure to the interaction between two ecologically synchronized groups of
these animals, the hunting carnivores and the herbivores that they hunted. The inter-
action resulting from the differences between predator and prey led to a general
improvement in brain functions; however, certain components of intelligence were
improved far more than others.
The kind of intelligence favored by the interplay of increasingly smarter catchers
and increasingly keener escapers is defined by attention that aspect of mind carrying
consciousness forward from one moment to the next. It ranges from a passive, free-
floating awareness to a highly focused, active fixation. The range through these states
is mediated by the arousal system, a network of tracts converging from sensory systems
to integrating centers in the brain stem. From the more relaxed to the more vigorous
levels, sensitivity to novelty is increased. The organism is more awake, more vigilant;
this increased vigilance results in the apprehension of ever more subtle signals as the
organism becomes more sensitive to its surroundings. The processes of arousal and
concentration give attention its direction. Arousal is at first general, with a flooding of
impulses in the brain stem; then gradually the activation is channeled. Thus begins
concentration, the holding of consistent images. One meaning of intelligence is the way
in which these images and other alertly searched information are used in the context of
previous experience. Consciousness links past attention to the present and permits the
integration of details with perceived ends and purposes.
The elements of intelligence and consciousness come together marvelously to pro-
duce different styles in predator and prey. Herbivores and carnivores develop different
kinds of attention related to escaping or chasing. Although in both kinds of animal,
arousal stimulates the production of adrenaline and norepinephrine by the adrenal
93
GRE Verbal Reasoning Practice Questions
line
5
10
15
20
25
This ebook was issued to HUAN-LIN CHANG, order #14803770223. Unlawful distribution of this ebook is prohibited.
Customer name HUAN-LIN CHANG , Order Id: 14803770223
94
GRE Verbal Reasoning Practice Questions
glands, the effect in herbivores is primarily fear, whereas in carnivores the effect is
primarily aggression. For both, arousal attunes the animal to what is ahead. Perhaps it
does not experience forethought as we know it, but the animal does experience some-
thing like it. The predator is searchingly aggressive, inner-directed, tuned by the
nervous system and the adrenal hormones, but aware in a sense closer to human con-
sciousness than, say, a hungry lizard’s instinctive snap at a passing beetle. Using past
events as a framework, the large mammal predator is working out a relationship
between movement and food, sensitive to possibilities in cold trails and distant sounds
and yesterday’s unforgotten lessons. The herbivore prey is of a different mind. Its
mood of wariness rather than searching and its attitude of general expectancy instead
of anticipating are silk-thin veils of tranquillity over an explosive endocrine system.
Description
The passage describes improvements in certain components of intelligence among
early large mammals of the grasslands. The second paragraph focuses on attention as
a primary area of improvement, and the third paragraph outlines how attention differs
in predator and prey species.
6. The author refers to a hungry lizard (line 31) primarily in order to
A
demonstrate the similarity between the hunting methods of mammals and
those of nonmammals
B
broaden the application of the argument by including an insectivore as an
example
C
make a distinction between higher and lower levels of consciousness
D
provide an additional illustration of the brutality characteristic of
predators
E
offer an objection to suggestions that all animals lack consciousness
Explanation
Choice C is correct. The “hungry lizard’s instinctive snap” is contrasted with the mam-
mal’s higher level of awareness. Choices A and B are incorrect. The example of the hun-
gry lizard provides a contrast; it does not demonstrate a similarity or extend the
author’s argument. Choices D and E are incorrect. Brutality is not mentioned in the
passage as a characteristic of predators, and there is no suggestion that all animals lack
consciousness.
7. It can be inferred from the passage that in animals less intelligent than the
mammals discussed in the passage
A
past experience is less helpful in ensuring survival
B
attention is more highly focused
C
muscular coordination is less highly developed
D
there is less need for competition among species
E
environment is more important in establishing the proper ratio of prey
to predator
Explanation
Choice A is correct. In lines 18–20, the author defines intelligence in terms of an ani-
mal’s use of past experience. In the context of the entire passage, it can be inferred that
line
30
35
This ebook was issued to HUAN-LIN CHANG, order #14803770223. Unlawful distribution of this ebook is prohibited.
Customer name HUAN-LIN CHANG , Order Id: 14803770223
more intelligent animals, such as the grassland mammals discussed, are better able to
use past experience to help them survive than less intelligent animals are. Choice B is
incorrect. The second paragraph of the passage indicates that attention is more highly
focused in animals of greater, rather than less, intelligence. Choices C, D, and E are
incorrect. The author does not discuss muscular coordination as an element in intelli-
gence, gives no indication that in less intelligent species there is less need for competi-
tion, and does not discuss how a proper ratio of prey to predator is established.
8. According to the passage, improvement in brain function among early large
mammals resulted primarily from which of the following?
A
Interplay of predator and prey
B
Persistence of free-floating awareness in animals of the grasslands
C
Gradual dominance of warm-blooded mammals over cold-blooded reptiles
D
Interaction of early large mammals with less intelligent species
E
Improvement of the capacity for memory among herbivores and
carnivores
Explanation
Choice A is correct. It directly paraphrases the statement in lines 3–5, which describes
the author’s view of the development of improved brain function in early mammals.
Choice B is incorrect. It is likely that the persistence of “free-floating awareness” played
a part in the animals’ survival, but there is no indication in the passage that brain func-
tion improved because of it. Choices C and D are incorrect: the passage does not dis-
cuss the relationship between mammals and reptiles or the interaction between large
mammals and less intelligent species. Choice E is incorrect. Improved capacity for
memory is an improvement in brain function, rather than a reason for improved brain
function.
9. According to the passage, as the process of arousal in an organism continues,
all of the following may occur EXCEPT
A
the production of adrenaline
B
the production of norepinephrine
C
a heightening of sensitivity to stimuli
D
an increase in selectivity with respect to stimuli
E
an expansion of the range of states mediated by the brain stem
Explanation
This question asks you what does NOT occur during arousal. To answer the question,
you must first determine what does occur. According to the passage, arousal does stim-
ulate the production of adrenaline and norepinephrine (lines 24–26); does increase sen-
sitivity to stimuli (lines 12–13); and does increase concentration on specific stimuli
(lines 16–18). Thus Choices A through D all describe consequences of arousal. Only
Choice E is correct. There is no indication in the passage that the range of states medi-
ated by the brain stem expands during arousal.
95
GRE Verbal Reasoning Practice Questions
This ebook was issued to HUAN-LIN CHANG, order #14803770223. Unlawful distribution of this ebook is prohibited.
Customer name HUAN-LIN CHANG , Order Id: 14803770223
96
GRE Verbal Reasoning Practice Questions
SET 5. Discrete Questions: Hard
For Questions 1 to 6, select one entry for each blank from the corresponding column
of choices. Fill all blanks in the way that best completes the text.
1. For some time now, _________ has been presumed not to exist: the cynical
conviction that everybody has an angle is considered wisdom.
A
rationality
B
flexibility
C
diffidence
D
disinterestedness
E
insincerity
Explanation
The colon indicates that the second part of the sentence will explain the first part. The
missing word will describe the opposite of the cynical conviction that “everybody has
an angle,” that is, that each person is concerned primarily with his or her own inter-
ests. Since “disinterestedness” means lack of self-interest, Choice D is correct. None of
the other answer choices means something that is contrasted with or opposed to being
primarily concerned with one’s own interests.
Thus the correct answer is disinterestedness (Choice D).
2. Human nature and long distances have made exceeding the speed limit a
(i)_________ in the state, so the legislators surprised no one when, acceding to
public demand, they (ii)_________ increased penalties for speeding.
Blank (i) Blank (ii)
A
controversial habit
D
endorsed
B
cherished tradition
E
considered
C
disquieting ritual
F
rejected
Explanation
The reference to human nature and long distances suggest that it is rather routine for
drivers to exceed the speed limit in this state. “Cherished tradition” best fits this con-
text for Blank (i), since there is nothing in the sentence to suggest that speeding here is
“controversial” or “disquieting.” In Blank (ii) we need to consider what the legislature
would do that would surprise no one with regard to increased penalties for speeding.
Given what we have learned so far, “rejected” is the best answer; it would be surprising
if the legislature “endorsed” or even “considered” increased penalties for speeding.
Thus the correct answer is cherished tradition (Choice B) and rejected (Choice F).
This ebook was issued to HUAN-LIN CHANG, order #14803770223. Unlawful distribution of this ebook is prohibited.
Customer name HUAN-LIN CHANG , Order Id: 14803770223
97
GRE Verbal Reasoning Practice Questions
3. Serling’s account of his employer’s reckless decision making (i)_________ that
company’s image as (ii)_________ bureaucracy full of wary managers.
Blank (i) Blank (ii)
A
belies
D
an injudicious
B
exposes
E
a disorganized
C
overshadows
F
a cautious
Explanation
The correct answer for Blank (i) must support, or at least be consistent with, the con-
trast between Serling’s account, which emphasizes the recklessness of the company’s
decision making, and the company’s image, that of a bureaucracy full of wary man-
agers. For Blank (i), “belies” is the best choice since Serling’s account would certainly
belie or contradict the company’s image. “Exposes” makes little sense since the image
presumably is already out in the open, and there is nothing in the sentence that sug-
gests Serling’s account “overshadows” the company’s image. As for Blank (ii), “a cau-
tious” is the most logical choice. Neither “an injudicious” nor “a disorganized” makes
sense in Blank (ii) as they both go against the notion of wariness.
Thus the correct answer is belies (Choice A) and a cautious (Choice F).
4. No other contemporary poet’s work has such a well-earned reputation for
(i)_________, and there are few whose moral vision is so imperiously unsparing.
Of late, however, the almost belligerent demands of his severe and densely
forbidding poetry have taken an improbable turn. This new collection is the
poet’s fourth book in six years an ample output even for poets of sunny
disposition, let alone for one of such (ii)_________ over the previous 50 years.
Yet for all his newfound (iii)_________, his poetry is as thorny as ever.
Blank (i) Blank (ii) Blank (iii)
A
patent
D
penitential
G
taciturnity
accessibility austerity
B
intrinsic frivolity
E
intractable
H
volubility
prolixity
C
near
F
impetuous
I
pellucidity
impenetrability prodigality
Explanation
Since the author of the paragraph has described the poet’s reputation as “well-earned,”
the correct completion for Blank (i) must be something that is consistent with what the
rest of the passage says about the poet’s work. Only “near impenetrability” fulfills this
requirement, since the next sentence tells us that the poet’s work is “severe” and
“densely forbidding,” which rule out both accessibility and frivolity. The Blank (ii) com-
pletion must contrast with “ample output,” and of the available options, only “peniten-
tial austerity” does so. Finally, the word in Blank (iii), since it is preceded by
“new found,” must refer to the change that has occurred in the poet’s work. The change
the paragraph has described is an increase in output, so “volubility” is the correct
choice.
This ebook was issued to HUAN-LIN CHANG, order #14803770223. Unlawful distribution of this ebook is prohibited.
Customer name HUAN-LIN CHANG , Order Id: 14803770223
98
GRE Verbal Reasoning Practice Questions
Thus the correct answer is near impenetrability (Choice C), penitential austerity
(Choice D), and volubility (Choice H).
5. Managers who think that strong environmental performance will (i)_________
their company’s financial performance often (ii)_________ claims that systems
designed to help them manage environmental concerns are valuable tools. By
contrast, managers who perceive environmental performance to be (iii)_________
to financial success may view an environmental management system as
extraneous. In either situation, and whatever their perceptions, it is a manager’s
commitment to achieving environmental improvement rather than the mere
presence of a system that determines environmental performance.
Blank (i) Blank (ii) Blank (iii)
A
eclipse
D
uncritically
G
complementary
accept
B
bolster
E
appropriately
H
intrinsic
acknowledge
C
degrade
F
hotly
I
peripheral
dispute
Explanation
The first two sentences introduce two contrasting sets of managers. The managers
identified in the second sentence view systems designed to help manage environmental
concerns as “extraneous,” suggesting that they would view environmental performance
to be “peripheral” (Choice I) to financial performance. The other options for Blank (iii)
“complementary” and “intrinsic” are not consistent with the idea that environ-
mental management systems are extraneous. With Blank (iii) filled in, we can go back
to Blanks (i) and (ii) with greater confidence: “bolster” works best in Blank (i), since
the two sets of managers have contrasting views. Blank (ii) is not straightforward
clearly these managers would not “hotly dispute” this claim, but “appropriately
acknowledge” is less easily ruled out. “Uncritically accept” makes sense and is con-
firmed when we look at the final sentence in which the author warns that, in either
situation, “the mere presence of a system” is not enough to achieve environmental
improvement. In fact, a system is not even necessary. Thus the author of the paragraph
does not regard the systems as particularly valuable, ruling out “appropriately acknowl-
edge.”
Thus the correct answer is bolster (Choice B), uncritically accept (Choice D), and
peripheral (Choice I).
This ebook was issued to HUAN-LIN CHANG, order #14803770223. Unlawful distribution of this ebook is prohibited.
Customer name HUAN-LIN CHANG , Order Id: 14803770223
99
GRE Verbal Reasoning Practice Questions
6. Philosophy, unlike most other subjects, does not try to extend our knowledge by
discovering new information about the world. Instead it tries to deepen our
understanding through (i)_________ what is already closest to us the
experiences, thoughts, concepts, and activities that make up our lives but that
ordinarily escape our notice precisely because they are so familiar. Philosophy
begins by finding (ii)_________ the things that are (iii) _________.
Blank (i) Blank (ii) Blank (iii)
A
attainment of
D
essentially
G
most prosaic
irrelevant
B
rumination on
E
utterly
H
somewhat
mysterious hackneyed
C
detachment from
F
thoroughly
I
refreshingly
commonplace novel
Explanation
The first two sentences present a contrast between extending our knowledge by discov-
ering “new information about the world” which we are told philosophy does not do
and extending knowledge through some activity involving “things that are closest to
us.” The first blank asks us to identify that activity, and although “attainment” makes
little sense in context, both “rumination on” and “detachment from” have some appeal.
However, the clear implication that philosophy attends to things that ordinarily escape
our notice eliminates “detachment from” as a correct answer. Blank (ii) requires some-
thing that suggests the importance of familiar things as subjects of philosophical rumi-
nation, and “utterly mysterious” does just that. “Essentially irrelevant” and “thoroughly
commonplace” do not fit logically since they suggest that these “familiar” things are
unimportant. Similarly, Blank (iii) needs to be consistent with the description of those
things as familiar and close. “Most prosaic” fits that idea while “refreshingly novel”
goes in the other direction. “Somewhat hackneyed” has some plausibility but is too
negative given the overall tone of the sentence; there is no indication that those things
are in any way trite.
Thus the correct answer is rumination on (Choice B), utterly mysterious (Choice
E), and most prosaic (Choice G).
For Questions 7 to 9, select the two answer choices that, when used to complete
the sentence, fit the meaning of the sentence as a whole and produce completed
sentences that are alike in meaning.
7. The government’s implementation of a new code of ethics appeared intended to
shore up the ruling party’s standing with an increasingly _________ electorate at a
time when the party is besieged by charges that it trades favors for campaign
money.
A
aloof
B
placid
C
restive
D
skittish
E
tranquil
F
vociferous
This ebook was issued to HUAN-LIN CHANG, order #14803770223. Unlawful distribution of this ebook is prohibited.
Customer name HUAN-LIN CHANG , Order Id: 14803770223
100
GRE Verbal Reasoning Practice Questions
Explanation
The words filling the blank must be consistent with the idea that the ruling party
needs to “shore up” its standing with the electorate. In their own way, Choices A,
C, D, and F are consistent with that idea, but only two of these when taken
together “restive” and “skittish” produce sentences that are alike in meaning.
“Aloof” fits the blank reasonably well, but there is no other word offered that is
nearly alike in meaning. The same holds for “vociferous.” “Placid” and “tranquil”
are similar in meaning but do not fit the context of the sentence.
Thus the correct answer is restive (Choice C) and skittish (Choice D).
8. Overlarge, uneven, and ultimately disappointing, the retrospective exhibition
seems too much like special pleading for a forgotten painter of real but
_________ talents.
A
limited
B
partial
C
undiscovered
D
circumscribed
E
prosaic
F
hidden
Explanation
The sentence is explaining why the exhibition of the painter’s work was unsatis-
factory, and since it says that the painter’s talents were real, the word in the blank
has to indicate why those talents were not, in the opinion of the author of the sen-
tence, good enough. The words “limited” and “circumscribed” do so and also pro-
duce sentences that are alike in meaning, so this pair forms the correct answer.
Although “undiscovered” and “hidden” are similar in meaning, they do not make
sense in the context of the sentence, since they do not indicate why the painter’s
talents were not adequate. Other choices, such as “partial” and “prosaic” might
make sense in context, but none of the other choices that meets that criterion also
has a companion choice that would produce another sentence alike in meaning.
Thus the correct answer is limited (Choice A) and circumscribed (Choice D).
9. Newspapers report that the former executive has been trying to keep a low
profile since his _________ exit from the company.
A
celebrated
B
mysterious
C
long-awaited
D
fortuitous
E
indecorous
F
unseemly
Explanation
The sentence needs to be completed with a word that suggests a reason for the
executive to wish to keep a low profile. The words “indecorous” and “unseemly”
both suggest such a reason, and the sentences completed with those two choices
This ebook was issued to HUAN-LIN CHANG, order #14803770223. Unlawful distribution of this ebook is prohibited.
Customer name HUAN-LIN CHANG , Order Id: 14803770223
101
GRE Verbal Reasoning Practice Questions
are alike in meaning. Therefore, that pair forms the correct answer. Although one
might get a sensible sentence by filling the blank with another choice, such as “long-
awaited,” none of the other choices that meets that criterion also has a companion
choice that would produce another sentence alike in meaning.
Thus the correct answer is indecorous (Choice E) and unseemly (Choice F).
SET 6. Reading Comprehension Questions: Hard
For each of Questions 1 to 8, select one answer choice unless otherwise instructed.
1. In the United States between 1850 and 1880, the number of farmers continued to
increase, but at a rate lower than that of the general population.
Which of the following statements directly contradicts the information presented
above?
A
The number of farmers in the general population increased slightly in the
30 years between 1850 and 1880.
B
The rate of growth of the United States labor force and the rate of growth
of the general population rose simultaneously in the 30 years between
1850 and 1880.
C
The proportion of farmers in the United States labor force remained
constant in the 30 years between 1850 and 1880.
D
The proportion of farmers in the United States labor force decreased from
64 percent in 1850 to 49 percent in 1880.
E
The proportion of farmers in the general population increased from 68
percent in 1850 to 72 percent in 1880.
Explanation
The given sentence indicates that the proportion of farmers in the general population
decreased from 1850 to 1880. Choice E says exactly the opposite that this proportion
increased and therefore it contradicts the passage and is the correct response. Choice
A is incorrect because it agrees with the given sentence, and Choices B, C, and D are all
incorrect because they refer to the labor force, about which the given sentence says
nothing.
This ebook was issued to HUAN-LIN CHANG, order #14803770223. Unlawful distribution of this ebook is prohibited.
Customer name HUAN-LIN CHANG , Order Id: 14803770223
2. A ten-year comparison between the United States and the Soviet Union in terms
of crop yields per acre revealed that when only planted acreage is compared,
Soviet yields were equal to 68 percent of United States yields. When total
agricultural acreage (planted acreage plus fallow acreage) is compared, however,
Soviet yield was 114 percent of United States yield.
From the information above, which of the following can be most reliably inferred
about United States and Soviet agriculture during the ten-year period?
A
A higher percentage of total agricultural acreage was fallow in the United
States than in the Soviet Union.
B
The United States had more fallow acreage than planted acreage.
C
Fewer total acres of available agricultural land were fallow in the Soviet
Union than in the United States.
D
The Soviet Union had more planted acreage than fallow acreage.
E
The Soviet Union produced a greater volume of crops than the United
States produced.
Explanation
If crop yield per planted acre was less in the Soviet Union than it was in the United
States, yet crop yield per total (planted plus fallow) agricultural acreage was greater in
the Soviet Union than it was in the United States, the percentage of the total acreage
that was left fallow must have been lower in the Soviet Union than in the United States.
Therefore, Choice A is the correct answer. Since the information provided in the para-
graph is given in terms of yield per acre, no conclusion can be drawn about actual
acreage, so Choices B, C, and D are all incorrect. Similarly, it is impossible to deter-
mine the total volume of crops produced in the Soviet Union, so Choice E is incorrect.
Questions 3 and 4 are based on the following reading passage.
For hot desert locations with access to seawater, a new greenhouse design generates
freshwater and cool air. Oriented to the prevailing wind, the front wall of perforated
cardboard, moistened and cooled by a trickle of seawater pumped in, cools and mois-
tens hot air blowing in. This cool, humidified air accelerates plant growth; little water
evaporates from leaves. Though greenhouses normally capture the heat of sunlight,
a double-layered roof, the inner layer coated to reflect infrared light outward, allows
visible sunlight in but traps solar heat between the two layers. This heated air, drawn
down from the roof, then mixes with the greenhouse air as it reaches a second
seawater-moistened cardboard wall at the back of the greenhouse. There the air
absorbs more moisture, which then condenses on a metal wall cooled by seawater, and
thus distilled water for irrigating the plants collects.
Description
The passage describes a greenhouse design and the process by which the design gener-
ates freshwater and cool air in a desert environment lacking in these things.
102
GRE Verbal Reasoning Practice Questions
This ebook was issued to HUAN-LIN CHANG, order #14803770223. Unlawful distribution of this ebook is prohibited.
Customer name HUAN-LIN CHANG , Order Id: 14803770223
103
GRE Verbal Reasoning Practice Questions
For the following question, consider each of the choices separately and select all that
apply.
3. It can be inferred that the process described in the passage makes use of which of
the following?
A
The tendency of hot air to rise
B
The directional movement of wind
C
The temperature differential between the sea and the desert
Explanation
Choices B and C are correct. This question asks the reader which of the three phe-
nomena listed in the answer choices is used in the process described in the passage.
Choice A is incorrect: the passage does not indicate that the tendency of hot air to
rise is used in the process, and in fact says that heated air is drawn down, not up, as
part of the greenhouse design.
Choice B is correct: the second sentence describes the orientation of a perforated
cardboard wall toward the prevailing wind so that hot air blows in and is moistened.
Choice C is correct: the passage describes the use of seawater to cool hot desert
air and to provide moisture that is absorbed by heated air and then condensed on a
seawater-cooled surface for the purpose of irrigating the plants.
For the following question, consider each of the choices separately and select all that
apply.
4. It can be inferred that the greenhouse roof is designed to allow for which of the
following?
A
The avoidance of intense solar heat inside the greenhouse
B
The entry of sunlight into the greenhouse to make the plants grow
C
The mixture of heated air with greenhouse air to enhance the collection of
moisture
Explanation
All three choices are correct. This question asks the reader which of the three effects
listed in the answer choices are intended as part of the design of the greenhouse roof.
Choice A is correct: the purpose of the double-layered roof is to trap solar heat
before it gets inside the greenhouse proper.
Choice B is correct: the coating on the inner layer of the roof allows visible sun-
light into the greenhouse.
Choice C is correct: the last two sentences of the passage describe how heated air
from the roof is drawn down to mix with greenhouse air, resulting in the collection of
distilled water for irrigation purposes.
This ebook was issued to HUAN-LIN CHANG, order #14803770223. Unlawful distribution of this ebook is prohibited.
Customer name HUAN-LIN CHANG , Order Id: 14803770223
Questions 5 to 8 are based on the following reading passage.
Many critics of Emily Brontë’s novel Wuthering Heights see its second part as a coun-
terpoint that comments on, if it does not reverse, the first part, where a romantic read-
ing receives more confirmation. Seeing the two parts as a whole is encouraged by the
novel’s sophisticated structure, revealed in its complex use of narrators and time shifts.
Granted that the presence of these elements need not argue for an authorial awareness
of novelistic construction comparable to that of Henry James, their presence does
encourage attempts to unify the novel’s heterogeneous parts. However, any interpreta-
tion that seeks to unify all of the novel’s diverse elements is bound to be somewhat
unconvincing. This is not because such an interpretation necessarily stiffens into a the-
sis (although rigidity in any interpretation of this or of any novel is always a danger),
but because Wuthering Heights has recalcitrant elements of undeniable power that, ulti-
mately, resist inclusion in an all-encompassing interpretation. In this respect, Wuther-
ing Heights shares a feature of Hamlet.
Description
The passage discusses a critical view concerning the unity of structure of Wuthering
Heights, then, following the use of “However,” expresses a reservation about that view.
5. According to the passage, which of the following is a true statement about the
first and second parts of Wuthering Heights?
A
The second part has received more attention from critics.
B
The second part has little relation to the first part.
C
The second part annuls the force of the first part.
D
The second part provides less substantiation for a romantic reading.
E
The second part is better because it is more realistic.
Explanation
This question requires the reader to identify which of the given relationships between
the novel’s first and second parts is one that is described in the passage. According to
the first sentence, the first part of the novel tends to confirm the “romantic” reading
more strongly than the second. Therefore, Choice D is correct. Nothing in the passage
suggests that critics have paid more attention to the second part, that the two parts
have little relation, or that the second part is better. Therefore, Choices A, B, and E are
incorrect. Choice C is a more extreme statement than any found in the passage, and
therefore it is incorrect.
6. Which of the following inferences about Henry James’s awareness of novelistic
construction is best supported by the passage?
A
James, more than any other novelist, was aware of the difficulties of
novelistic construction.
B
James was very aware of the details of novelistic construction.
C
James’s awareness of novelistic construction derived from his reading of
Brontë.
D
James’s awareness of novelistic construction has led most commentators
to see unity in his individual novels.
E
James’s awareness of novelistic construction precluded him from violating
the unity of his novels.
104
GRE Verbal Reasoning Practice Questions
This ebook was issued to HUAN-LIN CHANG, order #14803770223. Unlawful distribution of this ebook is prohibited.
Customer name HUAN-LIN CHANG , Order Id: 14803770223
Explanation
This question focuses on the passage’s mention of Henry James and asks what can be
inferred from it. The third sentence implies that James represents a very high degree of
authorial awareness of novelistic construction and that no such claim is necessarily
being made for Brontë. Thus, Choice B is the correct answer. Choice A is incorrect,
since the passage does not imply that there are particular difficulties that James under-
stood uniquely among novelists. Choice C is incorrect because the passage does not
state or imply that James read Brontë. The passage also does not say anything about
commentators’ opinions of the unity of James’s works; therefore Choice D is incorrect.
Choice E is incorrect because the passage itself offers no information about the unity
of James’s novels.
7. The author of the passage would be most likely to agree that an interpretation
of a novel should
A
not try to unite heterogeneous elements in the novel
B
not be inflexible in its treatment of the elements in the novel
C
not argue that the complex use of narrators or of time shifts indicates a
sophisticated structure
D
concentrate on those recalcitrant elements of the novel that are outside
the novel’s main structure
E
primarily consider those elements of novelistic construction of which the
author of the novel was aware
Explanation
This question requires the reader to determine what can be inferred from the passage
about its author’s view of the interpretation of novels. Choice A may seem attractive
because in the passage the author says that Wuthering Heights has heterogeneous ele-
ments that resist inclusion in a unifying interpretive scheme. Choice A is incorrect,
however, because the author does not indicate that the unification of different elements
is to be avoided in interpretation generally. By contrast, the author’s parenthetical
statement about rigidity does present a general warning against inflexibility of inter-
pretation, and it is this that supports Choice B as the correct answer. Choice C is incor-
rect, as the author actually suggests the contrary of this view in the second sentence of
the passage. Although the author mentions recalcitrant elements of Wuthering Heights,
there is no suggestion by the author that such elements deserve a special focus in inter-
pretation. Therefore Choice D is incorrect. The author of the passage does not indicate
which elements, if any, of novelistic construction are most worthy of consideration.
Therefore Choice E is incorrect.
105
GRE Verbal Reasoning Practice Questions
This ebook was issued to HUAN-LIN CHANG, order #14803770223. Unlawful distribution of this ebook is prohibited.
Customer name HUAN-LIN CHANG , Order Id: 14803770223
For the following question, consider each of the choices separately and select all that
apply.
8. The author of the passage suggests which of the following about Hamlet?
A
Hamlet has usually attracted critical interpretations that tend to stiffen
into theses.
B
Hamlet has elements that are not amenable to an all-encompassing critical
interpretation.
C
Hamlet is less open to an all-encompassing critical interpretation than is
Wuthering Heights.
Explanation
Choice B is correct. This question asks the reader which of the three statements about
Hamlet listed in the answer choices are suggested by the author of the passage.
Choice A is incorrect: the passage does not provide information about the charac-
teristics of the usual critical interpretations of Hamlet.
Choice B is correct: Hamlet is mentioned only in the final sentence of the passage,
which refers to “this respect” in which Hamlet and Wuthering Heights are similar. The
previous sentence reveals the point of similarity referred to: Wuthering Heights has
elements that resist inclusion in an all-encompassing interpretive framework.
Choice C is incorrect: the passage mentions only a feature shared between Hamlet
and Wuthering Heights. It does not suggest anything about a difference in their open-
ness to a particular critical interpretation.
106
GRE Verbal Reasoning Practice Questions
This ebook was issued to HUAN-LIN CHANG, order #14803770223. Unlawful distribution of this ebook is prohibited.
Customer name HUAN-LIN CHANG , Order Id: 14803770223
5gLearn the four types of
GRE
®Quantitative
Reasoning questions
gGet tips for answering each question type
gStudy sample Quantitative Reasoning questions
with solutions
gLearn how to use the on-screen calculator
Your goals
for this
chapter
107
Overview of the Quantitative Reasoning Measure
The Quantitative Reasoning measure of the GRE revised General Test assesses your:
bbasic mathematical skills
bunderstanding of elementary mathematical concepts
bability to reason quantitatively and to model and solve problems with quantita-
tive methods
Some of the questions in the measure are posed in real-life settings, while others
are posed in purely mathematical settings. The skills, concepts, and abilities are
tested in the four content areas below.
Arithmetic topics include properties and types of integers, such as divisibility,
factorization, prime numbers, remainders, and odd and even integers; arithmetic
operations, exponents, and roots; and concepts such as estimation, percent, ratio,
rate, absolute value, the number line, decimal representation, and sequences of
numbers.
Algebra topics include operations with exponents; factoring and simplifying
algebraic expressions; relations, functions, equations, and inequalities; solving
linear and quadratic equations and inequalities; solving simultaneous equations
and inequalities; setting up equations to solve word problems; and coordinate
geometry, including graphs of functions, equations, and inequalities, intercepts,
and slopes of lines.
Geometry topics include parallel and perpendicular lines, circles, triangles—
including isosceles, equilateral, and 30°-60°-90° triangles—quadrilaterals, other
polygons, congruent and similar figures, three-dimensional figures, area, perimeter,
volume, the Pythagorean theorem, and angle measurement in degrees. The ability to
construct proofs is not tested.
Data analysis topics include basic descriptive statistics, such as mean, median,
mode, range, standard deviation, interquartile range, quartiles, and percentiles; inter-
pretation of data in tables and graphs, such as line graphs, bar graphs, circle graphs,
boxplots, scatterplots, and frequency distributions; elementary probability, such as
probabilities of compound events and independent events; random variables and
GRE
®
Quantitative Reasoning
This ebook was issued to HUAN-LIN CHANG, order #14803770223. Unlawful distribution of this ebook is prohibited.
Customer name HUAN-LIN CHANG , Order Id: 14803770223
GRE Quantitative Reasoning
108
probability distributions, including normal distributions; and counting methods,
such as combinations, permutations, and Venn diagrams. These topics are typically
taught in high school algebra courses or introductory statistics courses. Inferential
statistics is not tested.
The content in these areas includes high school mathematics and statistics at a
level that is generally no higher than a second course in algebra; it does not include
trigonometry, calculus, or other higher-level mathematics. The publication Math Re-
view for the GRE revised General Test, which is available at www.ets.org/gre/prepare,
provides detailed information about the content of the Quantitative Reasoning mea-
sure. The Math Review is Chapter 7 in this book.
The mathematical symbols, terminology, and conventions used in the Quantita-
tive Reasoning measure are those that are standard at the high school level. For
example, the positive direction of a number line is to the right, distances are non-
negative, and prime numbers are greater than 1. Whenever nonstandard notation is
used in a question, it is explicitly introduced in the question.
In addition to conventions, there are some assumptions about numbers and geo-
metric figures that are used in the Quantitative Reasoning measure. Two of these as-
sumptions are (i) all numbers used are real numbers and (ii) geometric figures are
not necessarily drawn to scale. More about conventions and assumptions appears in
the publication Mathematical Conventions for the GRE revised General Test, which is
available at www.ets.org/gre/prepare and at the end of this chapter.
Quantitative Reasoning Question Types
The Quantitative Reasoning measure has four types of questions:
Quantitative Comparison questions
Multiple-choice questionsSelect One Answer Choice
Multiple-choice questionsSelect One or More Answer Choices
Numeric Entry questions
Each question appears either independently as a discrete question or as part of a
set of questions called a Data Interpretation set. All of the questions in a Data Inter-
pretation set are based on the same data presented in tables, graphs, or other dis-
plays of data.
In the computer-based test, you are allowed to use a basic calculatorprovided
on-screenon the Quantitative Reasoning measure. Information about using the cal-
culator appears later in this chapter.
For those taking the paper-based test, handheld calculators will be provided at
the test center for use during the test. Information about using the handheld calcula-
tor to help you answer questions appears in the free Practice Book for the Paper-based
GRE revised General Test, which is available at www.ets.org/gre/prepare.
Quantitative Comparison Questions
Description
Questions of this type ask you to compare two quantitiesQuantity A and Quantity
Band then determine which of the following statements describes the comparison.
Quantity A is greater.
Quantity B is greater.
The two quantities are equal.
The relationship cannot be determined from the information given.
This ebook was issued to HUAN-LIN CHANG, order #14803770223. Unlawful distribution of this ebook is prohibited.
Customer name HUAN-LIN CHANG , Order Id: 14803770223
GRE Quantitative Reasoning
109
Tips for Answering
Become familiar with the answer choices. Quantitative Comparison questions
always have the same answer choices, so get to know them, especially the last
choice, “The relationship cannot be determined from the information given.”
Never select this last choice if it is clear that the values of the two quantities can
be determined by computation. Also, if you determine that one quantity is
greater than the other, make sure you carefully select the corresponding choice
so as not to reverse the first two choices.
Avoid unnecessary computations. Don’t waste time performing needless
computations in order to compare the two quantities. Simplify, transform, or
estimate one or both of the given quantities only as much as is necessary to
compare them.
Remember that geometric figures are not necessarily drawn to scale. If any
aspect of a given geometric figure is not fully determined, try to redraw the
figure, keeping those aspects that are completely determined by the given
information fixed but changing the aspects of the figure that are not
determined. Examine the results. What variations are possible in the relative
lengths of line segments or measures of angles?
Plug in numbers. If one or both of the quantities are algebraic expressions, you
can substitute easy numbers for the variables and compare the resulting
quantities in your analysis. Consider all kinds of appropriate numbers before
you give an answer: e.g., zero, positive and negative numbers, small and large
numbers, fractions and decimals. If you see that Quantity A is greater than
Quantity B in one case and Quantity B is greater than Quantity A in another
case, choose “The relationship cannot be determined from the information
given.”
Simplify the comparison. If both quantities are algebraic or arithmetic
expressions and you cannot easily see a relationship between them, you can try
to simplify the comparison. Try a step-by-step simplification that is similar to the
steps involved when you solve the equation for x, or similar to the5=4x+3
steps involved when you determine that the inequality is equivalent
3y+2<y
5
to the simpler inequality Begin by setting up a comparison involving the1<y.
two quantities, as follows:
Quantity A ? Quantity B
| |
where is a “placeholder” that could represent the relationship greater than?
| |
(>), less than (<), or equal to (=) or could represent the fact that the
relationship cannot be determined from the information given. Then try to
simplify the comparison, step by step, until you can determine a relationship
between simplified quantities. For example, you may conclude after the last step
that represents equal to (=). Based on this conclusion, you may be able to
?
| |
compare Quantities A and B. To understand this strategy more fully, see sample
questions 6 to 9.
This ebook was issued to HUAN-LIN CHANG, order #14803770223. Unlawful distribution of this ebook is prohibited.
Customer name HUAN-LIN CHANG , Order Id: 14803770223
GRE Quantitative Reasoning
110
Sample Questions
Compare Quantity A and Quantity B, using additional information centered
above the two quantities if such information is given, and select one of the
following four answer choices:
AQuantity A is greater.
BQuantity B is greater.
CThe two quantities are equal.
DThe relationship cannot be determined from the information given.
A symbol that appears more than once in a question has the same meaning
throughout the question.
1.
Quantity A Quantity B
The least prime number
greater than 24
The greatest prime number
less than 28
AQuantity A is greater.
BQuantity B is greater.
CThe two quantities are equal.
DThe relationship cannot be determined from the information given.
Explanation
For the integers greater than 24, note that 25, 26, 27, and 28 are not prime
numbers, but 29 is a prime number, as are 31 and many other greater integers.
Thus, 29 is the least prime number greater than 24, and Quantity A is 29. For
the integers less than 28, note that 27, 26, 25, and 24 are not prime numbers,
but 23 is a prime number, as are 19 and several other lesser integers. Thus, 23 is
the greatest prime number less than 28, and Quantity B is 23. The correct
answer is Choice A, Quantity A is greater.
2.
Lionel is younger than Maria.
Quantity A Quantity B
Twice Lionel’s age Maria’s age
AQuantity A is greater.
BQuantity B is greater.
CThe two quantities are equal.
DThe relationship cannot be determined from the information given.
Explanation
If Lionel’s age is 6 years and Maria’s age is 10 years, then Quantity A is greater,
but if Lionel’s age is 4 years and Maria’s age is 10 years, then Quantity B is
greater. Thus, the relationship cannot be determined.
The correct answer is Choice D, the relationship cannot be determined
from the information given.
This ebook was issued to HUAN-LIN CHANG, order #14803770223. Unlawful distribution of this ebook is prohibited.
Customer name HUAN-LIN CHANG , Order Id: 14803770223
GRE Quantitative Reasoning
111
3.
Quantity A Quantity B
54% of 360 150
AQuantity A is greater.
BQuantity B is greater.
CThe two quantities are equal.
DThe relationship cannot be determined from the information given.
Explanation
Without doing the exact computation, you can see that 54 percent of 360 is
greater than of 360, which is 180, and 180 is greater than Quantity B, 150.
1
2
Thus, the correct answer is Choice A, Quantity A is greater.
4.
Figure 1
R
S
P
Q
PQ = PR
Quantity A Quantity B
PS SR
AQuantity A is greater.
BQuantity B is greater.
CThe two quantities are equal.
DThe relationship cannot be determined from the information given.
Explanation
From Figure 1, you know that PQR is a triangle and that point Sis between
points Pand R, so and You are also given thatPS <PR SR <PR.PQ =PR.
However, this information is not sufficient to compare PS and SR. Furthermore,
because the figure is not necessarily drawn to scale, you cannot determine the
relative sizes of PS and SR visually from the figure, though they may appear to
be equal. The position of Scan vary along side PR anywhere between Pand R.
Following are two possible variations of Figure 1, each of which is drawn to be
consistent with the information PQ =PR.
This ebook was issued to HUAN-LIN CHANG, order #14803770223. Unlawful distribution of this ebook is prohibited.
Customer name HUAN-LIN CHANG , Order Id: 14803770223
GRE Quantitative Reasoning
112
Figure 2 Figure 3
R
S
P
Q
PQ = PR
R
S
P
Q
PQ = PR
Note that Quantity A is greater in Figure 2 and Quantity B is greater in Figure 3.
Thus, the correct answer is Choice D, the relationship cannot be
determined from the information given.
5.
2
y=2x+7x3
Quantity A Quantity B
xy
AQuantity A is greater.
BQuantity B is greater.
CThe two quantities are equal.
DThe relationship cannot be determined from the information given.
Explanation
If then so in this case, but if then
2
x=0, y=2(0 ) +7(0) 3=−3, x>y;x=1,
so in that case,
2
y=2(1 ) +7(1) 3=6, y>x.
Thus, the correct answer is Choice D, the relationship cannot be
determined from the information given.
Note that plugging numbers into expressions may not be conclusive. It is conclusive,
however, if you get different results after plugging in different numbers: the conclu-
sion is that the relationship cannot be determined from the information given. It is
also conclusive if there are only a small number of possible numbers to plug in and
all of them yield the same result, say, that Quantity B is greater.
Now suppose that there are an infinite number of possible numbers to plug in. If
you plug many of them in and each time the result is, for example, that Quantity A
is greater, you still cannot conclude that Quantity A is greater for every possible
number that could be plugged in. Further analysis would be necessary and should
focus on whether Quantity A is greater for all possible numbers or whether there are
numbers for which Quantity A is not greater.
This ebook was issued to HUAN-LIN CHANG, order #14803770223. Unlawful distribution of this ebook is prohibited.
Customer name HUAN-LIN CHANG , Order Id: 14803770223
GRE Quantitative Reasoning
113
The following sample questions focus on simplifying the comparison.
6.
y>4
Quantity A Quantity B
3y+2
5
y
AQuantity A is greater.
BQuantity B is greater.
CThe two quantities are equal.
DThe relationship cannot be determined from the information given.
Explanation
Set up the initial comparison:
3y+2?y
| |
5
Then simplify:
Step 1: Multiply both sides by 5 to get
3y+2?5y
| |
Step 2: Subtract 3yfrom both sides to get
2?2y
| |
Step 3: Divide both sides by 2 to get
1?y
| |
The comparison is now simplified as much as possible. In order to compare 1
and y, note that you are given the information (above Quantities A and B).y>4
It follows from that or so that in the comparison the
y>4y>1, 1 <y,1?y,
| |
placeholder represents less than (<):?1<y.
| |
However, the problem asks for a comparison between Quantity A and
Quantity B, not a comparison between 1 and y. To go from the comparison
between 1 and yto a comparison between Quantities A and B, start with the last
comparison, and carefully consider each simplification step in reverse1 <y,
order to determine what each comparison implies about the preceding
comparison, all the way back to the comparison between Quantities A and B if
possible. Since step 3 was “divide both sides by 2,” multiplying both sides of the
comparison by 2 implies the preceding comparison thus reversing1 <y2<2y,
step 3. Each simplification step can be reversed as follows:
Reverse step 3: multiply both sides by 2.
Reverse step 2: add 3yto both sides.
Reverse step 1: divide both sides by 5.
When each step is reversed, the relationship remains less than (<), so
Quantity A is less than Quantity B.
Thus, the correct answer is Choice B, Quantity B is greater.
This ebook was issued to HUAN-LIN CHANG, order #14803770223. Unlawful distribution of this ebook is prohibited.
Customer name HUAN-LIN CHANG , Order Id: 14803770223
GRE Quantitative Reasoning
114
While some simplification steps like subtracting 3 from both sides or dividing both
sides by 10 are always reversible, it is important to note that some steps, like squar-
ing both sides, may not be reversible.
Also, note that when you simplify an inequality, the steps of multiplying or divid-
ing both sides by a negative number change the direction of the inequality; for ex-
ample, if then So the relationship in the final, simplified inequalityx<y,x>−y.
may be the opposite of the relationship between Quantities A and B. This is another
reason to consider the impact of each step carefully.
7.
Quantity A Quantity B
30 29
22
2
28
2
AQuantity A is greater.
BQuantity B is greater.
CThe two quantities are equal.
DThe relationship cannot be determined from the information given.
Explanation
Set up the initial comparison:
30 29
22
28
?2
| |
2
Then simplify:
Step 1: Multiply both sides by 2 to get
30 29 29
22?2
| |
Step 2: Add to both sides to get
29
2
30 29 29
2?2+2
| |
Step 3: Simplify the right-hand side using the fact that to get
29 30
(2)(2 ) =2
30 30
2?2
| |
The resulting relationship is equal to (=). In reverse order, each simplification
step implies equal to in the preceding comparison. So Quantities A and B are
also equal.
Thus, the correct answer is Choice C, the two quantities are equal.
This ebook was issued to HUAN-LIN CHANG, order #14803770223. Unlawful distribution of this ebook is prohibited.
Customer name HUAN-LIN CHANG , Order Id: 14803770223
GRE Quantitative Reasoning
115
8.
Quantity A Quantity B
2
x+12x1
AQuantity A is greater.
BQuantity B is greater.
CThe two quantities are equal.
DThe relationship cannot be determined from the information given.
Explanation
Set up the initial comparison:
2
x+1?2x1
| |
Then simplify by noting that the quadratic polynomial can be
2
x2x+1
factored:
Step 1: Subtract 2xfrom both sides to get
2
x2x+1? 1
| |
Step 2: Factor the left-hand side to get
2
(x1) ? 1
| |
The left-hand side of the comparison is the square of a number. Since the square
of a number is always greater than or equal to 0, and 0 is greater than the1,
simplified comparison is the inequality and the resulting
2
(x1) >−1
relationship is greater than (>). In reverse order, each simplification step implies
the inequality greater than (>) in the preceding comparison. Therefore, Quantity
A is greater than Quantity B.
The correct answer is Choice A, Quantity A is greater.
9.
w>1
Quantity A Quantity B
7w42w+5
AQuantity A is greater.
BQuantity B is greater.
CThe two quantities are equal.
DThe relationship cannot be determined from the information given.
This ebook was issued to HUAN-LIN CHANG, order #14803770223. Unlawful distribution of this ebook is prohibited.
Customer name HUAN-LIN CHANG , Order Id: 14803770223
GRE Quantitative Reasoning
116
Explanation
Set up the initial comparison:
7w4?2w+5
| |
Then simplify:
Step 1: Subtract 2wfrom both sides and add 4 to both sides to get
5w?9
| |
Step 2: Divide both sides by 5 to get
9
w?
| |
5
The comparison cannot be simplified any further. Although you are given that
you still don’t know how wcompares to or 1.8. For example, if
9
w>1, , w=1.5,
5
then but if then In other words, the relationship betweenw<1.8, w=2, w>1.8.
wand cannot be determined. Note that each of these simplification steps is
9
5
reversible, so in reverse order, each simplification step implies that the
relationship cannot be determined in the preceding comparison. Thus, the
relationship between Quantities A and B cannot be determined.
The correct answer is Choice D, the relationship cannot be determined
from the information given.
The strategy of simplifying the comparison works most efficiently when you note
that a simplification step is reversible while actually taking the step. Here are some
common steps that are always reversible:
Adding any number or expression to both sides of a comparison
Subtracting any number or expression from both sides
Multiplying both sides by any nonzero number or expression
Dividing both sides by any nonzero number or expression
Remember that if the relationship is an inequality, multiplying or dividing both
sides by any negative number or expression will yield the opposite inequality. Be
aware that some common operations like squaring both sides are generally not re-
versible and may require further analysis using other information given in the ques-
tion in order to justify reversing such steps.
This ebook was issued to HUAN-LIN CHANG, order #14803770223. Unlawful distribution of this ebook is prohibited.
Customer name HUAN-LIN CHANG , Order Id: 14803770223
GRE Quantitative Reasoning
117
Multiple-choice QuestionsSelect One Answer Choice
Description
These questions are multiple-choice questions that ask you to select only one answer
choice from a list of five choices.
Tips for Answering
Use the fact that the answer is there. If your answer is not one of the five
answer choices given, you should assume that your answer is incorrect and do
the following:
Reread the question carefullyyou may have missed an important detail or
misinterpreted some information.
Check your computations—you may have made a mistake, such as mis-keying
a number on the calculator.
Reevaluate your solution methodyou may have a flaw in your reasoning.
Examine the answer choices. In some questions you are asked explicitly which
of the choices has a certain property. You may have to consider each choice
separately, or you may be able to see a relationship between the choices that
will help you find the answer more quickly. In other questions, it may be helpful
to work backward from the choices, say, by substituting the choices in an
equation or inequality to see which one works. However, be careful, as that
method may take more time than using reasoning.
For questions that require approximations, scan the answer choices to see how
close an approximation is needed. In other questions, too, it may be helpful to
scan the choices briefly before solving the problem to get a better sense of
what the question is asking. If computations are involved in the solution, it may
be necessary to carry out all computations exactly and round only your final
answer in order to get the required degree of accuracy. In other questions, you
may find that estimation is sufficient and will help you avoid spending time on
long computations.
Sample Questions
Select a single answer choice.
1. If what is the value of x?5x+32 =42x,
A4
B3
C4
D7
E12
Explanation
Solving the equation for x, you get and so The correct7x=−28, x=−4.
answer is Choice A, 4.
This ebook was issued to HUAN-LIN CHANG, order #14803770223. Unlawful distribution of this ebook is prohibited.
Customer name HUAN-LIN CHANG , Order Id: 14803770223
GRE Quantitative Reasoning
118
2. Which of the following numbers is farthest from the number 1 on the
number line?
A10
B5
C0
D5
E10
Explanation
Circling each of the answer choices in a sketch of the number line (Figure 4)
shows that of the given numbers, is the greatest distance from 1.10
4510–1–2–3–4–5 2 3 10 117689
–6–7–8–9–11 –10
Figure 4
Another way to answer the question is to remember that the distance
between two numbers on the number line is equal to the absolute value of the
difference of the two numbers. For example, the distance between and 1 is10
and the distance between 10 and 1 is The10 1=11, 10 1=9=9.
correct answer is Choice A, 10.
3.
O1
1
y
x
y = f(x)
Figure 5
The figure above shows the graph of the function fdefined by
for all numbers x. For which of the following functions g,f(x)=2x+4
defined for all numbers x, does the graph of gintersect the graph of f?
Ag(x)=x2
Bg(x)=x+3
Cg(x)=2x2
Dg(x)=2x+3
Eg(x)=3x2
This ebook was issued to HUAN-LIN CHANG, order #14803770223. Unlawful distribution of this ebook is prohibited.
Customer name HUAN-LIN CHANG , Order Id: 14803770223
GRE Quantitative Reasoning
119
Explanation
You can see that all five choices are linear functions whose graphs are lines with
various slopes and y-intercepts. The graph of Choice A is a line with slope 1 and
y-intercept shown in Figure 6.2,
O1
1
y
x
y = f (x)
Figure 6
It is clear that this line will not intersect the graph of fto the left of the y-axis.
To the right of the y-axis, the graph of fis a line with slope 2, which is greater
than slope 1. Consequently, as the value of xincreases, the value of yincreases
faster for fthan for g, and therefore the graphs do not intersect to the right of
the y-axis. Choice B is similarly ruled out. Note that if the y-intercept of either of
the lines in Choices A and B were greater than or equal to 4 instead of less than
4, they would intersect the graph of f.
Choices C and D are lines with slope 2 and y-intercepts less than 4. Hence,
they are parallel to the graph of f(to the right of the y-axis) and therefore will
not intersect it. Any line with a slope greater than 2 and a y-intercept less than 4,
like the line in Choice E, will intersect the graph of f(to the right of the y-axis).
The correct answer is Choice E, g(x)=3x2.
4. A car got 33 miles per gallon using gasoline that cost $2.95 per gallon.
Approximately what was the cost, in dollars, of the gasoline used in driving
the car 350 miles?
A$10
B$20
C$30
D$40
E$50
Explanation
Scanning the answer choices indicates that you can do at least some estimation
and still answer confidently. The car used gallons of gasoline, so the cost was
350
33
dollars. You can estimate the product by estimating
350 350 350
(2.95) (2.95)
冢冣 冢冣
33 33 33
a little low, 10, and estimating 2.95 a little high, 3, to get approximately
dollars. You can also use the calculator to compute a more exact(10)(3) =30
This ebook was issued to HUAN-LIN CHANG, order #14803770223. Unlawful distribution of this ebook is prohibited.
Customer name HUAN-LIN CHANG , Order Id: 14803770223
GRE Quantitative Reasoning
120
answer and then round the answer to the nearest 10 dollars, as suggested by the
answer choices. The calculator yields the decimal which rounds to 3031.287.. .,
dollars.
Thus, the correct answer is Choice C, $30.
5. A certain jar contains 60 jelly beans22 white, 18 green, 11 yellow, 5 red,
and 4 purple. If a jelly bean is to be chosen at random, what is the
probability that the jelly bean will be neither red nor purple?
A0.09
B0.15
C0.54
D0.85
E0.91
Explanation
Since there are 5 red and 4 purple jelly beans in the jar, there are 51 that are
neither red nor purple, and the probability of selecting one of these is Since
51 .
60
all of the answer choices are decimals, you must convert the fraction to its
decimal equivalent, 0.85.
Thus, the correct answer is Choice D, 0.85.
Multiple-choice QuestionsSelect One or More Answer Choices
Description
These questions are multiple-choice questions that ask you to select one or more an-
swer choices from a list of choices. A question may or may not specify the number
of choices to select. These questions are marked with square boxes beside the an-
swer choices, not circles or ovals.
Tips for Answering
Note whether you are asked to indicate a specific number of answer choices or
all choices that apply. In the latter case, be sure to consider all of the choices,
determine which ones are correct, and select all of those and only those
choices. Note that there may be only one correct choice.
In some questions that involve conditions that limit the possible values of
numerical answer choices, it may be efficient to determine the least and/or the
greatest possible value. Knowing the least and/or greatest possible value may
enable you to quickly determine all of the choices that are correct.
Avoid lengthy calculations by recognizing and continuing numerical patterns.
This ebook was issued to HUAN-LIN CHANG, order #14803770223. Unlawful distribution of this ebook is prohibited.
Customer name HUAN-LIN CHANG , Order Id: 14803770223
GRE Quantitative Reasoning
121
Sample Questions
Select one or more answer choices according to the specific question directions.
If the question does not specify how many answer choices to select, select all
that apply.
The correct answer may be just one of the choices or as many as all of the
choices, depending on the question.
No credit is given unless you select all of the correct choices and no others.
If the question specifies how many answer choices to select, select exactly that
number of choices.
1. Which two of the following numbers have a product that is between 1 and
0?
Indicate both of the numbers.
A20
B10
C
4
2
D
2
3
Explanation
For this question, you must select a pair of answer choices. The product of the
pair must be negative, so the possible products are
42
(20)(2 ), (20)(3 ),
and The product must also be greater than 1. The first
42
(10)(2 ), (10)(3 ).
product is , the second product is , and third
20 20 20 20
=− 1=− 1
42
216 3 9
product is , so you can stop there.
10 10
=− 1
4
216
The correct answer consists of Choices B (10) and C ( ).
4
2
2. Which of the following integers are multiples of both 2 and 3 ?
Indicate all such integers.
A8
B9
C12
D18
E21
F36
Explanation
You can first identify the multiples of 2, which are 8, 12, 18, and 36, and then
among the multiples of 2 identify the multiples of 3, which are 12, 18, and 36.
Alternatively, if you realize that every number that is a multiple of 2 and 3 is also
a multiple of 6, you can check which choices are multiples of 6.
The correct answer consists of Choices C (12), D (18), and F (36).
This ebook was issued to HUAN-LIN CHANG, order #14803770223. Unlawful distribution of this ebook is prohibited.
Customer name HUAN-LIN CHANG , Order Id: 14803770223
GRE Quantitative Reasoning
122
3. Each employee of a certain company is in either Department Xor
Department Y, and there are more than twice as many employees in
Department Xas in Department Y. The average (arithmetic mean) salary
is $25,000 for the employees in Department Xand $35,000 for the
employees in Department Y. Which of the following amounts could be
the average salary for all of the employees of the company?
Indicate all such amounts.
A$26,000
B$28,000
C$29,000
D$30,000
E$31,000
F$32,000
G$34,000
Explanation
One strategy for answering this kind of question is to find the least and/or
greatest possible value. Clearly the average salary is between $25,000 and
$35,000, and all of the answer choices are in this interval. Since you are told that
there are more employees with the lower average salary, the average salary of all
employees must be less than the average of $25,000 and $35,000, which is
$30,000. If there were exactly twice as many employees in Department Xas in
Department Y, then the average salary for all employees would be, to the nearest
dollar, the following weighted mean,
(2)(25,000) +(1)(35,000) 28,333 dollars
2+1
where the weight for $25,000 is 2 and the weight for $35,000 is 1. Since there
are more than twice as many employees in Department Xas in Department Y,
the actual average salary must be even closer to $25,000 because the weight for
$25,000 is greater than 2. This means that $28,333 is the greatest possible
average. Among the choices given, the possible values of the average are
therefore $26,000 and $28,000.
Thus, the correct answer consists of Choices A ($26,000) and B
($28,000).
Intuitively, you might expect that any amount between $25,000 and $28,333
is a possible value of the average salary. To see that $26,000 is possible, in the
weighted mean above, use the respective weights 9 and 1 instead of 2 and 1. To
see that $28,000 is possible, use the respective weights 7 and 3.
This ebook was issued to HUAN-LIN CHANG, order #14803770223. Unlawful distribution of this ebook is prohibited.
Customer name HUAN-LIN CHANG , Order Id: 14803770223
GRE Quantitative Reasoning
123
4. Which of the following could be the units digit of where nis a positive
n
57 ,
integer?
Indicate all such digits.
A0
B1
C2
D3
E4
F5
G6
H7
I8
J9
Explanation
The units digit of is the same as the units digit of for all positive integers
nn
57 7
n. To see why this is true for compute by hand and observe how its
2
n=2, 57
units digit results from the units digit of Because this is true for every
2
7.
positive integer n, you need to consider only powers of 7. Beginning with n=1
and proceeding consecutively, the units digits of 7, and are 7, 9, 3, 1,
234 5
7,7,7, 7
and 7, respectively. In this sequence, the first digit, 7, appears again, and the
pattern of four digits, 7, 9, 3, 1, repeats without end. Hence, these four digits are
the only possible units digits of and therefore of
nn
757.
The correct answer consists of Choices B (1), D (3), H (7), and J (9).
Numeric Entry Questions
Description
Questions of this type ask you either to enter your answer as an integer or a decimal
in a single answer box or to enter it as a fraction in two separate boxesone for the
numerator and one for the denominator. In the computer-based test, use the com-
puter mouse and keyboard to enter your answer.
Tips for Answering
Make sure you answer the question that is asked. Since there are no answer
choices to guide you, read the question carefully and make sure you provide
the type of answer required. Sometimes there will be labels before or after the
answer box to indicate the appropriate type of answer. Pay special attention to
units such as feet or miles, to orders of magnitude such as millions or billions,
and to percents as compared with decimals.
If you are asked to round your answer, make sure you round to the required
degree of accuracy. For example, if an answer of 46.7 is to be rounded to the
nearest integer, you need to enter the number 47. If your solution strategy
involves intermediate computations, you should carry out all computations
exactly and round only your final answer in order to get the required degree of
accuracy. If no rounding instructions are given, enter the exact answer.
This ebook was issued to HUAN-LIN CHANG, order #14803770223. Unlawful distribution of this ebook is prohibited.
Customer name HUAN-LIN CHANG , Order Id: 14803770223
GRE Quantitative Reasoning
124
Examine your answer to see if it is reasonable with respect to the information
given. You may want to use estimation or another solution path to double-check
your answer.
Sample Questions
Enter your answer as an integer or a decimal if there is a single answer box OR as a
fraction if there are two separate boxesone for the numerator and one for the
denominator.
To enter an integer or a decimal, either type the number in the answer box using the
keyboard or use the Transfer Display button on the calculator.
First, click on the answer boxa cursor will appear in the boxand then type the
number.
To erase a number, use the Backspace key.
For a negative sign, type a hyphen. For a decimal point, type a period.
To remove a negative sign, type the hyphen again and it will disappear; the
number will remain.
The Transfer Display button on the calculator will transfer the calculator display to
the answer box.
Equivalent forms of the correct answer, such as 2.5 and 2.50, are all correct.
Enter the exact answer unless the question asks you to round your answer.
To enter a fraction, type the numerator and the denominator in the respective boxes
using the keyboard.
For a negative sign, type a hyphen; to remove it, type the hyphen again. A decimal
point cannot be used in a fraction.
The Transfer Display button on the calculator cannot be used for a fraction.
Fractions do not need to be reduced to lowest terms, though you may need to
reduce your fraction to fit in the boxes.
1. One pen costs $0.25 and one marker costs $0.35. At those prices, what is the total
cost of 18 pens and 100 markers?
$
Explanation
Multiplying $0.25 by 18 yields $4.50, which is the cost of the 18 pens; and
multiplying $0.35 by 100 yields $35.00, which is the cost of the 100 markers. The
total cost is therefore Equivalent decimals, such as$4.50 +$35.00 =$39.50.
$39.5 or $39.500, are considered correct.
Thus, the correct answer is $39.50 (or equivalent).
Note that the dollar symbol is in front of the answer box, so the symbol $
does not need to be entered in the box. In fact, only numbers, a decimal point,
and a negative sign can be entered in the answer box.
This ebook was issued to HUAN-LIN CHANG, order #14803770223. Unlawful distribution of this ebook is prohibited.
Customer name HUAN-LIN CHANG , Order Id: 14803770223
GRE Quantitative Reasoning
125
2. Rectangle Rhas length 30 and width 10, and square Shas length 5. The
perimeter of Sis what fraction of the perimeter of R?
Explanation
The perimeter of Ris and the perimeter of Sis30 +10 +30 +10 =80, (4)(5) =20.
Therefore, the perimeter of Sis of the perimeter of R. To enter the answer
20
80
you should enter the numerator 20 in the top box and the denominator 80
20 ,
80
in the bottom box. Because the fraction does not need to be reduced to lowest
terms, any fraction that is equivalent to is also considered correct, as long as
20
80
it fits in the boxes. For example, both of the fractions and are considered
21
84
correct.
Thus, the correct answer is (or any equivalent fraction).
20
80
3.
RESULTS OF A USED-CAR AUCTION
Small Cars Large Cars
Number of cars offered 32 23
Number of cars sold 16 20
Projected sales total for cars offered (in thousands) $70 $150
Actual sales total (in thousands) $41 $120
Figure 7
For the large cars sold at an auction that is summarized in the table above,
what was the average sale price per car?
$
Explanation
From Figure 7, you see that the number of large cars sold was 20 and the sales
total for large cars was $120,000 (not $120). Thus the average sale price per car
was $120,000 =$6,000.
20
The correct answer is $6,000 (or equivalent).
(Note that the comma in 6,000 will appear automatically in the answer box
in the computer-based test.)
This ebook was issued to HUAN-LIN CHANG, order #14803770223. Unlawful distribution of this ebook is prohibited.
Customer name HUAN-LIN CHANG , Order Id: 14803770223
GRE Quantitative Reasoning
126
4. A merchant made a profit of $5 on the sale of a sweater that cost the
merchant $15. What is the profit expressed as a percent of the merchant’s
cost?
Give your answer to the nearest whole percent.
%
Explanation
The percent profit is percent, which is 33%, to the
5(100) =33.333.. . =33.3
冢冣
15
nearest whole percent.
Thus, the correct answer is 33% (or equivalent).
If you use the calculator and the Transfer Display button, the number that
will be transferred to the answer box is 33.333333, which is incorrect since it is
not given to the nearest whole percent. You will need to adjust the number in
the answer box by deleting all of the digits to the right of the decimal point
(using the Backspace key).
Also, since you are asked to give the answer as a percent, the decimal
equivalent of 33 percent, which is 0.33, is incorrect. The percent symbol next to
the answer box indicates that the form of the answer must be a percent.
Entering 0.33 in the box would give the erroneous answer 0.33%.
5. Working alone at its constant rate, machine Aproduces kcar parts in 10
minutes. Working alone at its constant rate, machine Bproduces kcar parts
in 15 minutes. How many minutes does it take machines Aand B, working
simultaneously at their respective constant rates, to produce kcar parts?
minutes
Explanation
Machine Aproduces parts per minute, and machine Bproduces parts per
kk
10 15
minute. So when the machines work simultaneously, the rate at which the parts
are produced is the sum of these two rates, which is kk 11
+=k+=
冢冣
10 15 10 15
parts per minute. To compute the time required to produce kparts at
25 k
k=
冢冣
150 6
this rate, divide the amount kby the rate to get
kk
=6.
6k
6
Therefore, the correct answer is 6 minutes (or equivalent).
One way to check that the answer of 6 minutes is reasonable is to observe
that if the slower rate of machine Bwere the same as machine As faster rate of
kparts in 10 minutes, then the two machines, working simultaneously, would
take half the time, or 5 minutes, to produce the kparts. So the answer has to be
greater than 5 minutes. Similarly, if the faster rate of machine Awere the same as
machine Bs slower rate of kparts in 15 minutes, then the two machines would
take half the time, or 7.5 minutes, to produce the kparts. So the answer has to
be less than 7.5 minutes. Thus, the answer of 6 minutes is reasonable compared
to the lower estimate of 5 minutes and the upper estimate of 7.5 minutes.
This ebook was issued to HUAN-LIN CHANG, order #14803770223. Unlawful distribution of this ebook is prohibited.
Customer name HUAN-LIN CHANG , Order Id: 14803770223
GRE Quantitative Reasoning
127
Data Interpretation Sets
Description
Data Interpretation questions are grouped together and refer to the same table,
graph, or other data presentation. These questions ask you to interpret or analyze
the given data. The types of questions may be Multiple-choice (both types) or
Numeric Entry.
Tips for Answering
Scan the data presentation briefly to see what it is about, but do not spend
time studying all of the information in detail. Focus on those aspects of the
data that are necessary to answer the questions. Pay attention to the axes and
scales of graphs; to the units of measurement or orders of magnitude (such as
billions) that are given in the titles, labels, and legends; and to any notes that
clarify the data.
Bar graphs and circle graphs, as well as other graphical displays of data, are
drawn to scale, so you can read or estimate data visually from such graphs.
For example, you can use the relative sizes of bars or sectors to compare the
quantities that they represent, but be aware of broken scales and of bars that do
not start at 0.
The questions are to be answered only on the basis of the data presented,
everyday facts (such as the number of days in a year), and your knowledge of
mathematics. Do not make use of specialized information you may recall from
other sources about the particular context on which the questions are based
unless the information can be derived from the data presented.
Sample Questions
Questions 1 to 3 are based on the following data.
ANNUAL PERCENT CHANGE IN DOLLAR AMOUNT OF SALES
AT FIVE RETAIL STORES FROM 2006 TO 2008
Store Percent Change
from 2006 to 2007
Percent Change
from 2007 to 2008
P10 10
Q20 9
R512
S715
T17 8
Figure 8
1. If the dollar amount of sales at Store Pwas $800,000 for 2006, what was
the dollar amount of sales at that store for 2008 ?
A$727,200
B$792,000
C$800,000
D$880,000
E$968,000
This ebook was issued to HUAN-LIN CHANG, order #14803770223. Unlawful distribution of this ebook is prohibited.
Customer name HUAN-LIN CHANG , Order Id: 14803770223
GRE Quantitative Reasoning
128
Explanation
According to Figure 8, if the dollar amount of sales at Store Pwas $800,000 for
2006, then it was 10 percent greater for 2007, which is 110 percent of that
amount, or $880,000. For 2008 the amount was 90 percent of $880,000, which is
$792,000.
The correct answer is Choice B, $792,000.
Note that an increase of 10 percent for one year and a decrease of 10 percent
for the following year does not result in the same dollar amount as the original
dollar amount because the base that is used in computing the percents is
$800,000 for the first change but $880,000 for the second change.
2. At Store T, the dollar amount of sales for 2007 was what percent of the
dollar amount of sales for 2008 ?
Give your answer to the nearest 0.1 percent.
%
Explanation
If Ais the dollar amount of sales at Store Tfor 2007, then 8 percent of A,or
is the amount of decrease from 2007 to 2008. Thus is
0.08A,A0.08A=0.92A
the dollar amount for 2008. Therefore, the desired percent can be obtained by
dividing Aby which equals Expressed as a
A1
0.92A,==1.0869565.. ..
0.92A0.92
percent and rounded to the nearest 0.1 percent, this number is 108.7%.
Thus, the correct answer is 108.7% (or equivalent).
3. Based on the information given, which of the following statements must be
true?
Indicate all such statements.
AFor 2008 the dollar amount of sales at Store Rwas greater than that at
each of the other four stores.
BThe dollar amount of sales at Store Sfor 2008 was 22 percent less
than that for 2006.
CThe dollar amount of sales at Store Rfor 2008 was more than 17
percent greater than that for 2006.
Explanation
For Choice A, since the only data given in Figure 8 are percent changes from
year to year, there is no way to compare the actual dollar amount of sales at the
stores for 2008 or for any other year. Even though Store Rhad the greatest
percent increase from 2006 to 2008, its actual dollar amount of sales for 2008
may have been much smaller than that for any of the other four stores, and
therefore Choice A is not necessarily true.
For Choice B, even though the sum of the two percent decreases would
suggest a 22 percent decrease, the bases of the percents are different. If Bis the
dollar amount of sales at Store Sfor 2006, then the dollar amount for 2007 is 93
percent of B, or and the dollar amount for 2008 is given by
0.93B, (0.85)(0.93)B,
which is Note that this represents a percent decrease of0.7905B.
percent, which is less than 22 percent, and so Choice B is100 79.05 =20.95
not true.
This ebook was issued to HUAN-LIN CHANG, order #14803770223. Unlawful distribution of this ebook is prohibited.
Customer name HUAN-LIN CHANG , Order Id: 14803770223
GRE Quantitative Reasoning
129
For Choice C, if Cis the dollar amount of sales at Store Rfor 2006, then the
dollar amount for 2007 is given by and the dollar amount for 2008 is given1.05C
by which is Note that this represents a 17.6 percent(1.12)(1.05)C, 1.176C.
increase, which is greater than 17 percent, so Choice C must be true.
Therefore, the correct answer consists of only Choice C (The dollar
amount of sales at Store Rfor 2008 was more than 17 percent greater than
that for 2006).
Using the Calculator
Sometimes the computations you need to do in order to answer a question in the
Quantitative Reasoning measure are somewhat time-consuming, like long division, or
involve square roots. For such computations, you can use the on-screen calculator
provided in the computer-based test. The on-screen calculator is shown in Figure 9.
Figure 9
Although the calculator can shorten the time it takes to perform computations,
keep in mind that the calculator provides results that supplement, but do not re-
place, your knowledge of mathematics. You must use your mathematical knowledge
to determine whether the calculator’s results are reasonable and how the results can
be used to answer a question.
Here are some general guidelines for calculator use in the Quantitative Reason-
ing measure:
Most of the questions don’t require difficult computations, so don’t use the
calculator just because it’s available.
Use it for calculations that you know are tedious, such as long division, square
roots, and addition, subtraction, or multiplication of numbers that have several
digits.
Avoid using it for simple computations that are quicker to do mentally, such as
and
4,300
2
10 490, (4)(70), , 25, 30 .
10
Avoid using it to introduce decimals if you are asked to give an answer as a
fraction.
Some questions can be answered more quickly by reasoning and estimating than
by using the calculator.
This ebook was issued to HUAN-LIN CHANG, order #14803770223. Unlawful distribution of this ebook is prohibited.
Customer name HUAN-LIN CHANG , Order Id: 14803770223
GRE Quantitative Reasoning
130
If you use the calculator, estimate the answer beforehand so that you can
determine whether the calculator’s answer is “in the ballpark.” This may help
you avoid key-entry errors.
The following guidelines are specific to the on-screen calculator in the computer-
based test:
When you use the computer mouse or the keyboard to operate the calculator,
take care not to mis-key a number or operation.
Note all of the calculator’s buttons, including Transfer Display.
The Transfer Display button can be used on Numeric Entry questions with a
single answer box. This button will transfer the calculator display to the answer
box. You should check that the transferred number has the correct form to
answer the question. For example, if a question requires you to round your
answer or convert your answer to a percent, make sure that you adjust the
transferred number accordingly.
Take note that the calculator respects order of operations, as explained below.
A mathematical convention called order of operations establishes which opera-
tions are performed before others in a mathematical expression that has more than
one operation. The order is as follows: parentheses, exponentiation (including square
roots), multiplications and divisions (from left to right), additions and subtractions
(from left to right). With respect to order of operations, the value of the expression
is 9 because the expression is evaluated by first multiplying 2 and 4 and1 +24
then by adding 1 to the result. This is how the on-screen calculator in the Quantita-
tive Reasoning measure performs the operations. (Note that many basic calculators
follow a different convention, whereby they perform multiple operations in the order
that they are entered into the calculator. For such calculators, the result of entering
is 12. To get this result, the calculator adds 1 and 2, displays a result of 3,1 +24
then multiplies 3 and 4, and displays a result of 12.)
In addition to parentheses, the on-screen calculator has one memory location
and three memory buttons that govern it: memory recall memory clear
MR ,
| |
and memory sum These buttons function as they normally do onMC , M+.
||||
most basic calculators.
Some computations are not defined for real numbers: for example, division by
zero or taking the square root of a negative number. If you enter the
60=,
||||
word Error will be displayed. Similarly, if you enter then Error will be1 ,
||| |
displayed. To clear the display, you must press the clear button C.
| |
The calculator displays up to eight digits. If a computation results in a number
greater than 99,999,999, then Error will be displayed. For example, the
calculation results in Error. The clear button must be
10,000,000 10 =C
||||||
used to clear the display. If a computation results in a positive number less than
0.00000001, or 10
8
, then 0 will be displayed.
This ebook was issued to HUAN-LIN CHANG, order #14803770223. Unlawful distribution of this ebook is prohibited.
Customer name HUAN-LIN CHANG , Order Id: 14803770223
GRE Quantitative Reasoning
131
Below are some examples of computations using the calculator.
1. Compute 6.73
4+.
2
Explanation
Enter to get 7.365. Alternatively, enter to get 3.365,4 +6.73 2=6.73 2=
||||||||||
and then enter to get 7.365.+4=
||||
2. Compute 8.4 +9.3
.
70
Explanation
Since division takes precedence over addition in the order of operations, you
need to override that precedence in order to compute this fraction. Here are two
ways to do that. You can use the parentheses for the addition in the numerator,
entering to get Or you can use the equals
( 8.4 +9.3 ) 70 =0.2528571.
||||||| |||| |
sign after 9.3, entering to get the same result. In the8.4 +9.3 =70 =
||||| |||| |
second way, note that pressing the first is essential, because without it,=
| |
would erroneously compute instead.
9.3
8.4 +9.3 70 =8.4 +
||||||| |
冢冣
70
Incidentally, the exact value of the expression is the repeating
8.4 +9.3
70
decimal where the digits 285714 repeat without end, but the
0.25285714,
calculator rounds the decimal to 0.2528571.
3. Find the length, to the nearest 0.01, of the hypotenuse of a right triangle
with legs of length 21 and 54; that is, use the Pythagorean theorem and
calculate
22
21 +54 .
Explanation
Enter to get 57.939624. Again, pressing the before21 21 +54 54 ==
||||||||| |||
the is essential because would erroneously compute21 21 +54 54 =
冪冪
||||||||||| |
This is because the square root would take precedence over the
2
21 +54 54.
multiplication in the order of operations. Note that parentheses could be used, as
in , but they are not necessary because the
(2121 ) +(5454 ) =
||||||| || |||||| || |
multiplications already take precedence over the addition.
Incidentally, the exact answer is a nonterminating, nonrepeating decimal, or
an irrational number, but the calculator rounds the decimal to 57.939624.
Finally, note that the problem asks for the answer to the nearest 0.01, so the
correct answer is 57.94.
4. Compute
3
(15) .
Explanation
Enter to get15 Ⳳ⳯15 Ⳳ⳯15 =−3,375.
||| |||| |||| |
This ebook was issued to HUAN-LIN CHANG, order #14803770223. Unlawful distribution of this ebook is prohibited.
Customer name HUAN-LIN CHANG , Order Id: 14803770223
GRE Quantitative Reasoning
132
5. Convert 6 miles per hour to feet per second.
Explanation
The solution to this problem uses the conversion factors feet and1 mile =5,280
as follows:1 hour =3,600 seconds
6 miles 5,280 feet 1 hour feet
=?
冢冣冢 冣
1 hour 1 mile 3,600 seconds second
Enter to get 8.8. Alternatively, enter to get the
65280 3600 =65280 =
||||||||||
result 31,680, and then enter to get 8.8 feet per second.3600 =
||||
6. At a fund-raising event, 43 participants donated $60 each, 21 participants
donated $80 each, and 16 participants donated $100 each. What was the
average (arithmetic mean) donation per participant, in dollars?
Explanation
The solution to this problem is to compute the weighted mean
You can use the memory buttons and parentheses
(43)(60) +(21)(80) +(16)(100).
43 +21 +16
for this computation as follows:
Enter 43 60 =M+21 80 =M+16 100 =M+MR
||||| |||||| |||||| || || |
to get 73.25, or $73.25 per participant.(43+21 +16 ) =
||||||||| |
When the button is first used, the number in the calculator display isM+
| |
stored in memory and an Mappears to the left of the display to show that the
memory function is in use. Each subsequent use of the button adds the
M+
| |
number in the current display to the number stored in memory and replaces the
number stored in memory by the sum. When the button is pressed in the
MR
| |
computation above, the current value in memory, 5,860, is displayed. To clear the
memory, use the button, and the Mnext to the display disappears.
MC
| |
This ebook was issued to HUAN-LIN CHANG, order #14803770223. Unlawful distribution of this ebook is prohibited.
Customer name HUAN-LIN CHANG , Order Id: 14803770223
133
Mathematical Conventions for the Quantitative
Reasoning Measure of the GRE revised General Test
The mathematical symbols and terminology used in the Quantitative Reasoning mea-
sure of the test are conventional at the high school level, and most of these appear
in the Math Review (Chapter 7). Whenever nonstandard or special notation or termi-
nology is used in a test question, it is explicitly introduced in the question. However,
there are some particular assumptions about numbers and geometric figures that are
made throughout the test. These assumptions appear in the test at the beginning of
the Quantitative Reasoning sections, and they are elaborated below.
Also, some notation and terminology, while standard at the high school level in
many countries, may be different from those used in other countries or from those
used at higher or lower levels of mathematics. Such notation and terminology are
clarified below. Because it is impossible to ascertain which notation and terminology
should be clarified for an individual test taker, more material than necessary may be
included.
Finally, there are some guidelines for how certain information given in test ques-
tions should be interpreted and used in the context of answering the questionsin-
formation such as certain words, phrases, quantities, mathematical expressions, and
displays of data. These guidelines appear at the end.
Numbers and Quantities
All numbers used in the test questions are real numbers. In particular, integers
and both rational and irrational numbers are to be considered, but imaginary
numbers are not. This is the main assumption regarding numbers. Also, all
quantities are real numbers, although quantities may involve units of
measurement.
Numbers are expressed in base 10 unless otherwise noted, using the 10 digits 0
through 9 and a period to the right of the ones digit, or units digit, for the
decimal point. Also, in numbers that are 1,000 or greater, commas are used to
separate groups of three digits to the left of the decimal point.
When a positive integer is described by the number of its digits, e.g., a two-digit
integer, the digits that are counted include the ones digit and all the digits
further to the left, where the left-most digit is not 0. For example, 5,000 is a four-
digit integer, whereas 031 is not considered to be a three-digit integer.
Some other conventions involving numbers: one billion means 1,000,000,000, or
(not as in some countries); one dozen means 12; the Greek letter
912
10 10 , p
represents the ratio of the circumference of a circle to its diameter and is
approximately 3.14.
When a positive number is to be rounded to a certain decimal place and the
number is halfway between the two nearest possibilities, the number should be
rounded to the greater possibility. For example, 23.5 rounded to the nearest
integer is 24, and 123.985 rounded to the nearest 0.01 is 123.99. When the
number to be rounded is negative, the number should be rounded to the lesser
possibility. For example, 36.5 rounded to the nearest integer is 37.
Repeating decimals are sometimes written with a bar over the digits that repeat,
as in and
25 1
=2.083 =0.142857.
12 7
If r,s, and tare integers and then rand sare factors,ordivisors,oft; also,rs =t,
tis a multiple of r(and of s) and tis divisible by r(and by s). The factors of an
integer include positive and negative integers. For example, is a factor of7
GRE Quantitative Reasoning
This ebook was issued to HUAN-LIN CHANG, order #14803770223. Unlawful distribution of this ebook is prohibited.
Customer name HUAN-LIN CHANG , Order Id: 14803770223
134
35, 8 is a factor of and the integer 4 has six factors: 1, 2, and40, 4, 2, 1,
4. The terms factor,divisor, and divisible are used only when r,s, and tare
integers. However, the term multiple can be used with any real numbers sand t
provided ris an integer. For example, 1.2 is a multiple of 0.4, and is a2p
multiple of .p
The least common multiple of two nonzero integers aand bis the least positive
integer that is a multiple of both aand b. The greatest common divisor (or greatest
common factor)ofaand bis the greatest positive integer that is a divisor of both a
and b.
If an integer nis divided by a nonzero integer dresulting in a quotient qwith
remainder r, then where Furthermore, if and only if nn =qd +r,0r<d.r=0
is a multiple of d. For example, when 20 is divided by 7, the quotient is 2 and the
remainder is 6; when 21 is divided by 7, the quotient is 3 and the remainder is 0;
and when is divided by 7, the quotient is and the remainder is 4.17 3
Aprime number is an integer greater than 1 that has only two positive divisors: 1
and itself. The first five prime numbers are 2, 3, 5, 7, and 11. A composite number
is an integer greater than 1 that is not a prime number. The first five composite
numbers are 4, 6, 8, 9, and 10.
Odd and even integers are not necessarily positive; for example, is odd, and
7
and 0 are even.
18
The integer 0 is neither positive nor negative.
Mathematical Expressions, Symbols, and Variables
As is common in algebra, italic letters like xare used to denote numbers,
constants, and variables. Letters are also used to label various objects, such as line
point P, function f, set S, list T, event E, random variable X, Brand X, City Y,
,
and Company Z. The meaning of a letter is determined by the context.
When numbers, constants, or variables are given, their possible values are all real
numbers unless otherwise restricted. It is common to restrict the possible values
in various ways. Here are some examples: nis a nonzero integer; and T
1x<p;
is the tens digits of a two-digit positive integer, so Tis an integer from 1 to 9.
Standard mathematical symbols at the high school level are used. These include
the arithmetic operations and though multiplication is usually denoted
+,,,,
by juxtaposition, often with parentheses, e.g., 2yand ; and division is
(3)(4.5)
usually denoted with a horizontal fraction bar, e.g., Sometimes mixed
w.
3
numbers, or mixed fractions, are used, like and These two
31
410 .
82
numbers are equal to and respectively. Exponents are also used, e.g.,
35 21
,
82
and for all nonzero numbers x.
1
10 20
2=1,024, 10 =,x=1
100
Mathematical expressions are to be interpreted with respect to order of operations,
which establishes which operations are performed before others in an expression.
The order is as follows: parentheses; exponentiation; negation; multiplication and
division (from left to right); addition and subtraction (from left to right). For
example, the value of the expression is 9, because the expression is1 +24
evaluated by first multiplying 2 and 4 and then adding 1 to the result. Also,
2
3
means “the negative of ‘3 squared’ ” because exponentiation takes precedence over
negation. Therefore, but because parentheses take precedence
22
3=−9, (3) =9
over exponentiation.
GRE Quantitative Reasoning
This ebook was issued to HUAN-LIN CHANG, order #14803770223. Unlawful distribution of this ebook is prohibited.
Customer name HUAN-LIN CHANG , Order Id: 14803770223
GRE Quantitative Reasoning
135
Here are examples of other standard symbols with their meanings:
xyxis less than or equal to y
xyxand yare not equal
xyxand yare approximately equal
xthe absolute value of x
x
the nonnegative square root of x, where x0
x
the nonpositive square root of x, where x0
n! the product of all positive integers less than or equal to n, where nis any
positive integer and, as a special definition, 0! =1.
mlines and mare parallel
mlines and mare perpendicular
Because all numbers are assumed to be real, some expressions are not defined.
For example, for every number x, the expression is not defined; if then
xx<0, x
0
is not defined; and is not defined.
0
0
Sometimes special symbols or notation are introduced in a question. Here are two
examples:
The operation is defined for all integers rand sby rs
r
s=.
2
1+r
The operation is defined for all nonzero numbers xby 1
⬃⬃x=− .
x
Sometimes juxtaposition of letters does not denote multiplication, as in “consider
a three-digit integer denoted by XYZ, where X,Y, and Zare digits.” The meaning is
taken from the context.
Standard function notation is used in the test. For example, “the function gis
defined for all by ” If the domain of a function fis not given
x0g(x)=2x+x.
explicitly, it is assumed to be the set of all real numbers xfor which is a realf(x)
number. If fand gare two functions, then the composition of gwith fis denoted
by g(f(x)).
Geometry
In questions involving geometry, the conventions of plane (or Euclidean) geometry
are followed, including the assumption that the sum of the measures of the
interior angles of a triangle is 180 degrees.
Lines are assumed to be “straight” lines that extend in both directions without
end.
Angle measures are in degrees and are assumed to be positive and less than or
equal to 360 degrees.
When a square, circle, polygon, or other closed geometric figure is described in
words but not shown, the figure is assumed to enclose a convex region. It is also
assumed that such a closed geometric figure is not just a single point or a line
segment. For example, a quadrilateral cannot be any of the following:
Not closed Not convex
This ebook was issued to HUAN-LIN CHANG, order #14803770223. Unlawful distribution of this ebook is prohibited.
Customer name HUAN-LIN CHANG , Order Id: 14803770223
GRE Quantitative Reasoning
136
The phrase area of a rectangle means the area of the region enclosed by the
rectangle. The same terminology applies to circles, triangles, and other closed
figures.
The distance between a point and a line is the length of the perpendicular line
segment from the point to the line, which is the shortest distance between the
point and the line. Similarly, the distance between two parallel lines is the distance
between a point on one line and the other line.
In a geometric context, the phrase similar triangles (or other figures) means that
the figures have the same shape. See the Geometry section of the Math Review for
further explanation of the terms similar and congruent.
Geometric Figures
Geometric figures consist of points, lines, line segments, curves (such as circles),
angles, and regions; also included are labels, and markings or shadings that
identify these objects or their sizes. A point is indicated by a dot, a label, or the
intersection of two or more lines or curves. Points, lines, angles, etc., that are
shown as distinct are indeed distinct. All figures are assumed to lie in a plane
unless otherwise indicated.
If points A,B, and Cdo not lie on the same line, then line segments AB and BC
form two angles with vertex Bone angle with measure less than and the180
other with measure greater than as shown below. Unless otherwise180,
indicated, angle ABC, also denoted by ABC or B, refers to the smaller of the
two angles.
A
B
C
Less than
180°
Greater than
180°
The notation AB may mean the line segment with endpoints Aand B,oritmay
mean the length of the line segment. The meaning can be determined from the
context.
Geometric figures are not necessarily drawn to scale. That is, you should not
assume that quantities such as lengths and angle measures are as they appear in a
figure. However, you should assume that lines shown as straight are actually
straight, and when curves are shown, you should assume they are not straight.
Also, assume that points on a line or a curve are in the order shown, points shown
to be on opposite sides of a line or curve are so oriented, and more generally,
assume all geometric objects are in the relative positions shown. For questions
with geometric figures, you should base your answers on geometric reasoning, not
on estimating or comparing quantities by sight or by measurement.
To illustrate some of these conventions, consider the geometric figure below.
CA
E
T
B
S
R
m
D
9
6
35°
This ebook was issued to HUAN-LIN CHANG, order #14803770223. Unlawful distribution of this ebook is prohibited.
Customer name HUAN-LIN CHANG , Order Id: 14803770223
GRE Quantitative Reasoning
137
The following can be determined from the figure.
ABD and DBC are triangles, and points R,S, and Tlie on the closed curve.
Points A,D, and Clie on a straight line, so ABC is a triangle with sides AB,
BC, and AC.
Point Dis a distinct point between points Aand C.
Points Aand Sare on opposite sides of line m.
Point Eis on BD.
AD <AC
and the measure of angle Cis 35 degrees.
ST =9, DC =6,
Angle ABC is a right angle, as indicated by the small square symbol at
point B.
The measure of angle ABD is xdegrees, and x<90.
Line intersects the closed curve at points Sand T, and the curve is
tangent to AB at T.
The area of the region enclosed by the curve is greater than the area of
triangle RST.
The following cannot be determined from the figure.
AD >DC
The measures of angles BAD and BDA are equal.
The measure of angle DBC is less than xdegrees.
The area of triangle ABD is greater than the area of triangle DBC.
Angle SRT is a right angle.
Line mis parallel to line AC.
Coordinate Systems
Coordinate systems, such as xy-planes and number lines, are drawn to scale.
Therefore, you can read, estimate, or compare quantities in such figures by sight
or by measurement, including geometric figures that appear in coordinate
systems.
The positive direction of a number line is to the right.
As in geometry, distances in a coordinate system are nonnegative.
The rectangular coordinate plane, or rectangular coordinate system, commonly
known as the xy-plane, is shown below. The x-axis and y-axis intersect at the
origin O, and they partition the plane into four quadrants. Each point in the xy-
plane has coordinates that give its location with respect to the axes; for(x,y)
example, the point is located 2 units to the right of the y-axis and 8 unitsP(2,8)
below the x-axis. The units on the x-axis have the same length as the units on the
y-axis, unless otherwise noted.
5
Quadrant IQuadrant II
Quadrant IV
P(2, –8)
Quadrant III
10–5–10
–5
–10
5
10
Ox
y
This ebook was issued to HUAN-LIN CHANG, order #14803770223. Unlawful distribution of this ebook is prohibited.
Customer name HUAN-LIN CHANG , Order Id: 14803770223
GRE Quantitative Reasoning
138
Intermediate grid lines or tick marks in a coordinate system are evenly spaced
unless otherwise noted.
The term x-intercept refers to the x-coordinate of the point at which a graph in the
xy-plane intersects the x-axis. The term y-intercept is used analogously. Sometimes
the terms x-intercept and y-intercept refer to the actual intersection points.
Sets, Lists, and Sequences
Sets of numbers or other elements appear in some questions. Some sets are
infinite, such as the set of integers; other sets are finite and may have all of their
elements listed within curly brackets, such as the set When the2, 4, 6, 8 .
{}
elements of a set are given, repetitions are not counted as additional elements and
the order of the elements is not relevant. Elements are also called members. A set
with one or more members is called nonempty; there is a set with no members,
called the empty set and denoted by If Aand Bare sets, then the intersection of.
Aand B, denoted by is the set of elements that are in both Aand B, and theAB,
union of Aand B, denoted by is the set of elements that are in Aor B,orAB,
both. If all of the elements in Aare also in B, then Ais a subset of B.By
convention, the empty set is a subset of every set. If Aand Bhave no elements in
common, they are called disjoint sets or mutually exclusive sets.
Lists of numbers or other elements are also used in the test. When the elements of
a list are given, repetitions are counted as additional elements and the order of the
elements is relevant. For example, the list 3, 1, 2, 3, 3 contains five numbers, and
the first, fourth, and last numbers in the list are each 3.
The terms data set and set of data are not sets in the mathematical sense given
above. Rather they refer to a list of data because there may be repetitions in the
data, and if there are repetitions, they would be relevant.
Sequences are lists that often have an infinite number of elements, or terms. The
terms of a sequence are often represented by a fixed letter along with a subscript
that indicates the order of a term in the sequence. For example, a,a,a,...,a,...
123 n
represents an infinite sequence in which the first term is the second term isa,a,
12
and more generally, the nth term is for every positive integer n. Sometimes thea
n
nth term of a sequence is given by a formula, such as Sometimes the
n
b=2+1.
n
first few terms of a sequence are given explicitly, as in the following sequence of
consecutive even negative integers: 2,4,6,8,10,....
Sets of consecutive integers are sometimes described by indicating the first and
last integer, as in “the integers from 0 to 9, inclusive.” This phrase refers to 10
integers, with or without “inclusive” at the end. Thus, the phrase “during the years
from 1985 to 2005” refers to 21 years.
Data and Statistics
Numerical data are sometimes given in lists and sometimes displayed in other
ways, such as in tables, bar graphs, or circle graphs. Various statistics, or
measures of data, appear in questions: measures of central tendencymean,
median, and mode; measures of positionquartiles and percentiles; and
measures of dispersionstandard deviation, range, and interquartile range.
The term average is used in two ways, with and without the qualification
“(arithmetic mean).” For a list of data, the average (arithmetic mean) of the data is
This ebook was issued to HUAN-LIN CHANG, order #14803770223. Unlawful distribution of this ebook is prohibited.
Customer name HUAN-LIN CHANG , Order Id: 14803770223
GRE Quantitative Reasoning
139
the sum of the data divided by the number of data. The term average does not
refer to either median or mode in the test. Without the qualification of “arithmetic
mean,” average can refer to a rate or the ratio of one quantity to another, as in
“average number of miles per hour” or “average weight per truckload.”
When mean is used in the context of data, it means arithmetic mean.
The median of an odd number of data is the middle number when the data are
listed in increasing order; the median of an even number of data is the arithmetic
mean of the two middle numbers when the data are listed in increasing order.
For a list of data, the mode of the data is the most frequently occurring number in
the list. Thus, there may be more than one mode for a list of data.
For data listed in increasing order, the first quartile,second quartile, and third
quartile of the data are three numbers that divide the data into four groups that
are roughly equal in size. The first group of numbers is from the least number up
to the first quartile. The second group is from the first quartile up to the second
quartile, which is also the median of the data. The third group is from the second
quartile up to the third quartile, and the fourth group is from the third quartile up
to the greatest number. Note that the four groups themselves are sometimes
referred to as quartilesfirst quartile,second quartile,third quartile, and fourth
quartile. The latter usage is clarified by the word “in” as in the phrase “the cow’s
weight is in the third quartile of the weights of the herd.”
For data listed in increasing order, the percentiles of the data are 99 numbers that
divide the data into 100 groups that are roughly equal in size. The 25th percentile
equals the first quartile; the 50th percentile equals the second quartile, or median;
and the 75th percentile equals the third quartile.
For a list of data, where the arithmetic mean is denoted by m, the standard
deviation of the data refers to the nonnegative square root of the mean of the
squared differences between mand each of the data. This statistic is also known
as the population standard deviation and is not to be confused with the sample
standard deviation.
For a list of data, the range of the data is the greatest number in the list minus the
least number. The interquartile range of the data is the third quartile minus the
first quartile.
Data Distributions and Probability Distributions
Some questions display data in frequency distributions, where discrete data values
are repeated with various frequencies, or where preestablished intervals of
possible values are assigned frequencies corresponding to the numbers of data in
the intervals. For example, the lifetimes, rounded to the nearest hour, of 300
lightbulbs could be in the following 10 intervals: 501–550 hours, 551–600 hours,
601–650 hours, . . . , 951–1,000 hours; consequently, each of the intervals would
have a number, or frequency, of lifetimes, and the sum of the 10 frequencies is
300.
Questions may involve relative frequency distributions, where each frequency of a
frequency distribution is divided by the total number of data in the distribution,
resulting in a relative frequency. In the example above, the 10 frequencies of the
10 intervals would each be divided by 300, yielding 10 relative frequencies.
When a question refers to a random selection or a random sample, all possible
samples of equal size have the same probability of being selected unless there is
information to the contrary.
This ebook was issued to HUAN-LIN CHANG, order #14803770223. Unlawful distribution of this ebook is prohibited.
Customer name HUAN-LIN CHANG , Order Id: 14803770223
GRE Quantitative Reasoning
140
Some questions describe probability experiments,orrandom experiments, that have
a finite number of possible outcomes. In a random experiment, any particular set
of outcomes is called an event, and every event Ehas a probability, denoted by
where If each outcome of an experiment is equally likely, thenP(E), 0 P(E)1.
the probability of an event Eis defined as the following ratio:
the number of outcomes in the event E
P(E)=the number of possible outcomes in the experiment
If Eand Fare two events in an experiment, then “Eand F” is an event, which is
the set of outcomes that are in the intersection of events Eand F. Another event is
Eor F, ” which is the set of outcomes that are in the union of events Eand F.
If Eand Fare two events and Eand Fare mutually exclusive, then P(Eand F)=0.
If Eand Fare two events such that the occurrence of either event does not affect
the occurrence of the other, then Eand Fare said to be independent events. Events
Eand Fare independent if and only if P(Eand F)=P(E)P(F).
Arandom variable is a variable that represents values resulting from a random
experiment. The values of the random variable may be the actual outcomes of the
experiment if the outcomes are numerical, or the random variable may be related
to the outcomes more indirectly. In either case, random variables can be used to
describe events in terms of numbers.
A random variable from an experiment with only a finite number of possible
outcomes also has only a finite number of values and is called a discrete random
variable. When the values of a random variable form a continuous interval of real
numbers, such as all of the numbers between 0 and 2, the random variable is
called a continuous random variable.
Every value of a discrete random variable X, say has a probability denotedX=a,
by A histogram (or a table) showing all of the values of Xand theirP(a).
probabilities is called the probability distribution of X. The mean of theP(X)
random variable X is the sum of the products for all values of X.XP(X)
The mean of a random variable Xis also called the expected value of Xor the mean
of the probability distribution of X.
For a continuous random variable X, every interval of values, say has aaXb,
probability, which is denoted by The probability distribution of XcanP(aXb).
be described by a curve in the xy–plane that mimics the tops of the bars of a
histogram, only smoother. The curve is the graph of a function fwhose values are
nonnegative and whose graph is therefore above the x-axis. The curve isy=f(x)
related to the probability of each interval in the following way:aXb
is equal to the area of the region that is below the curve, above theP(aXb)
x-axis, and between the vertical lines and The area of the entire regionx=ax=b.
under the curve is 1.
The mean of a continuous random variable X is the point mon the x-axis at which
region under the distribution curve would perfectly balance if a fulcrum were
placed at The median of Xis the point Mon the x-axis at which the linex=m.
divides the region under the distribution curve into two regions of equalx=M
area.
The standard deviation of a random variable X is a measure of dispersion, which
indicates how spread out the probability distribution of Xis from its mean. It is
also called the standard deviation of the probability distribution of X.
The most important probability distribution is the normal distribution, whose
distribution curve is shaped like a bell. A random variable Xwith this distribution
This ebook was issued to HUAN-LIN CHANG, order #14803770223. Unlawful distribution of this ebook is prohibited.
Customer name HUAN-LIN CHANG , Order Id: 14803770223
GRE Quantitative Reasoning
141
is called normally distributed. The curve is symmetric about the line wherex=m,
mis the mean as well as the median. The right and left tails of the distribution
become ever closer to the x-axis but never touch it.
The standard normal distribution has mean 0 and standard deviation 1. The
following figure shows the distribution, including approximate probabilities
corresponding to the six intervals shown.
0.02
210–1–2
0.02 0.14 0.14
0.34 0.34
Graphical Representations of Data
Graphical data presentations, such as bar graphs, circle graphs, and line graphs,
are drawn to scale; therefore, you can read, estimate, or compare data values by
sight or by measurement.
Standard conventions apply to graphs of data unless otherwise indicated. For
example, a circle graph represents 100 percent of the data indicated in the graph’s
title, and the areas of the individual sectors are proportional to the percents they
represent. Scales, grid lines, dots, bars, shadings, solid and dashed lines, legends,
etc., are used on graphs to indicate the data. Sometimes, scales that do not begin
at 0 are used, and sometimes broken scales are used.
In Venn diagrams, various sets of objects are represented by circular regions and
by regions formed by intersections of the circles. In some Venn diagrams, all of
the circles are inside a rectangular region that represents a universal set. A
number placed in a region is the number of elements in the subset represented by
the smallest region containing the number, unless otherwise noted. Sometimes a
number is placed above a circle to indicate the number of elements in the entire
circle.
Miscellaneous Guidelines for Interpreting and Using Information in Test
Questions
Numbers given in a question are to be used as exact numbers, even though in
some real-life settings they are likely to have been rounded. For example, if a
question states that “30 percent of the company’s profit was from health
products,” then 30 is to be used as an exact number; it is not to be treated as
though it were a nearby number, say 29 or 30.1, that has been rounded up or
down.
An integer that is given as the number of certain objects, whether in a real-life or
pure-math setting, is to be taken as the total number of such objects. For example,
if a question states that “a bag contains 50 marbles, and 23 of the marbles are
red,” then 50 is to be taken as the total number of marbles in the bag and 23 is to
be taken as the total number of red marbles in the bag, so that the other 27
marbles are not red. Fractions and percents are understood in a similar way, so
“one-fifth, or 20 percent, of the 50 marbles in the bag are green” means 10
marbles in the bag are green and 40 marbles are not green.
This ebook was issued to HUAN-LIN CHANG, order #14803770223. Unlawful distribution of this ebook is prohibited.
Customer name HUAN-LIN CHANG , Order Id: 14803770223
GRE Quantitative Reasoning
142
When a multiple-choice question asks for an approximate quantity without
stipulating a degree of approximation, the correct answer is the choice that is
closest in value to the quantity that can be computed from the information given.
Unless otherwise indicated, the phrase “difference between two quantities” is
assumed to mean “positive difference,” that is, the greater quantity minus the
lesser quantity. For example, “for which two consecutive years was the difference
in annual rainfall least?” means “for which two consecutive years was the absolute
value of the difference in annual rainfall least?”
When the term profit is used in a question, it refers to gross profit, which is the
sales revenue minus the cost of production. The profit does not involve any other
amounts unless they are explicitly given.
The common meaning of terms such as months and years and other everyday
terms are assumed in questions where the terms appear.
In questions involving real-life scenarios in which a variable is given to represent
a number of existing objects or another nonnegative amount, the context implies
that the variable is greater than 0. For example, “Jane sold xrugs and deposited
her profit of ydollars into her savings account” implies that xand yare greater
than 0.
Some quantities may involve units, such as inches, pounds, and Celsius degrees,
while other quantities are pure numbers. Any units of measurement, such as
English units or metric units, may be used. However, if an answer to a question
requires converting one unit of measurement to another, then the relationship
between the units is given in the question, unless the relationship is a common
one, such as the relationships between minutes and hours, dollars and cents, and
metric units like centimeters and meters.
In any question, there may be some information that is not needed for obtaining
the correct answer.
When reading questions, do not introduce unwarranted assumptions. For
example, if a question describes a trip that begins and ends at certain times, the
intended answer will assume that the times are unaffected by crossing time zones
or by changes to the local time for daylight savings, unless those matters are
explicitly mentioned. As another example, do not consider sales taxes on
purchases unless explicitly mentioned.
The display of data in a Data Interpretation set of questions is the same for each
question in the set. Also, the display may contain more than one graph or table.
Each question will refer to some of the data, but it may happen that some part of
the data will have no question that refers to it.
In a Data Interpretation set of questions, each question should be considered
separately from the others. No information except what is given in the display of
data should be carried over from one question to another.
In many questions, mathematical expressions and words appear together in a
phrase. In such a phrase, each mathematical expression should be interpreted
separately from the words before it is interpreted along with the words. For
example, if nis an integer, then the phrase “the sum of the first two consecutive
integers greater than ” means ; it does not mean “the sum ofn+6(n+7) +(n+8)
the first two consecutive integers greater than n” plus 6, or
That is, the expression should be interpreted first,(n+1) +(n+2) +6. n+6
separately from the words. However, in a phrase like “the function gis defined for
all the phrase “for all ” is a mathematical shorthand for “for allx0, x0
numbers xsuch that x0.
This ebook was issued to HUAN-LIN CHANG, order #14803770223. Unlawful distribution of this ebook is prohibited.
Customer name HUAN-LIN CHANG , Order Id: 14803770223
gPractice answering
GRE
®Quantitative Reasoning
questions on your own
gStudy answers and explanations, particularly for
questions you answered incorrectly
Your goals
for this
chapter
143
6
GRE
®
Quantitative Reasoning
Practice Questions
This chapter contains four sets of GRE Quantitative Reasoning practice ques-
tions. Each of the first three practice sets consists of Quantitative Comparison
questions, both types of Multiple-choice questions, and Numeric Entry ques-
tions. These three sets are arranged in order of increasing difficulty. The first is easy,
the second is medium, and the third is hard. The fourth practice set consists of Data
Interpretation questions of varying levels of difficulty.
Following the last set is an answer key for quick reference. Then, at the end of the
chapter, you will find complete explanations for every question. Each explanation is
presented with the corresponding question, so that you can easily see what was asked
and what the various answer choices or Numeric Entry answer boxes were.
Sharpen your GRE Quantitative Reasoning skills by working your way through
these question sets. For the Discrete question sets, begin with the easy sets and then
move on to the medium and hard sets. Review the answers and explanations carefully,
paying particular attention to explanations for questions that you answered incorrectly.
For the practice questions in this chapter, use the directions that begin on the
following page.
This ebook was issued to HUAN-LIN CHANG, order #14803770223. Unlawful distribution of this ebook is prohibited.
Customer name HUAN-LIN CHANG , Order Id: 14803770223
GRE Quantitative Reasoning Practice Questions
144
General Directions
For each question, indicate the best answer, using the directions given.
Notes: All numbers used are real numbers.
All figures are assumed to lie in a plane unless otherwise indicated.
Geometric figures, such as lines, circles, triangles, and quadrilaterals, are not
necessarily drawn to scale. That is, you should not assume that quantities
such as lengths and angle measures are as they appear in a figure. You
should assume, however, that lines shown as straight are actually straight,
points on a line are in the order shown, and more generally, all geometric
objects are in the relative positions shown. For questions with geometric fig-
ures, you should base your answers on geometric reasoning, not on estimat-
ing or comparing quantities by sight or by measurement.
Coordinate systems, such as xy-planes and number lines, are drawn to scale;
therefore, you can read, estimate, or compare quantities in such figures by
sight or by measurement.
Graphical data presentations, such as bar graphs, circle graphs, and line
graphs, are drawn to scale; therefore, you can read, estimate, or compare
data values by sight or by measurement.
Directions for Quantitative Comparison questions
Compare Quantity A and Quantity B, using additional information centered
above the two quantities if such information is given. Select one of the
following four answer choices and fill in the corresponding oval to the right of
the question.
AQuantity A is greater.
BQuantity B is greater.
CThe two quantities are equal.
DThe relationship cannot be determined from the information given.
A symbol that appears more than once in a question has the same meaning
throughout the question.
Quantity A Quantity B Correct Answer
Example 1: (2)(6) 2 +6ABCD
––––––––––––––––––––––––––––––––––––
Q
RP S
Quantity A Quantity B Correct Answer
Example 2: PS SR ABCD
(since equal lengths cannot
be assumed, even though
PS and SR appear equal)
This ebook was issued to HUAN-LIN CHANG, order #14803770223. Unlawful distribution of this ebook is prohibited.
Customer name HUAN-LIN CHANG , Order Id: 14803770223
GRE Quantitative Reasoning Practice Questions
145
Directions for Numeric Entry questions
Enter your answer in the answer box(es) below the question.
Your answer may be an integer, a decimal, or a fraction, and it may be
negative.
If a question asks for a fraction, there will be two boxesone for the
numerator and one for the denominator.
Equivalent forms of the correct answer, such as 2.5 and 2.50, are all correct.
Fractions do not need to be reduced to lowest terms.
Enter the exact answer unless the question asks you to round your answer.
SET 1. Discrete Questions: Easy
Quantitative Comparison
For Questions 1 to 6, compare Quantity A and Quantity B, using additional
information centered above the two quantities if such information is given.
Select one of the following four answer choices and fill in the corresponding
oval to the right of the question.
AQuantity A is greater.
BQuantity B is greater.
CThe two quantities are equal.
DThe relationship cannot be determined from the information given.
A symbol that appears more than once in a question has the same meaning
throughout the question.
Emma spent $75 buying a used bicycle and $27
repairing it. Then she sold the bicycle for 40 percent
more than the total amount she spent buying and
repairing it.
Quantity A Quantity B
1. The price at which Emma
sold the bicycle
$140 ABCD
This ebook was issued to HUAN-LIN CHANG, order #14803770223. Unlawful distribution of this ebook is prohibited.
Customer name HUAN-LIN CHANG , Order Id: 14803770223
GRE Quantitative Reasoning Practice Questions
146
QS
PT
R
V
In the figure above, squares PQRV and VRST have
sides of length 6.
Quantity A Quantity B
2. The area of the shaded
region
36 ABCD
In 2009 the property tax on each home in Town X
was ppercent of the assessed value of the home,
where pis a constant. The property tax in 2009 on a
home in Town Xthat had an assessed value of
$125,000 was $2,500.
Quantity A Quantity B
3. The property tax in 2009 on
a home in Town Xthat had
an assessed value of
$160,000
$3,000 ABCD
x+y=−1
Quantity A Quantity B
4. xy
ABCD
r,s, and tare three consecutive odd integers such
that r<s<t.
Quantity A Quantity B
5. r+s+1s+t1ABCD
This ebook was issued to HUAN-LIN CHANG, order #14803770223. Unlawful distribution of this ebook is prohibited.
Customer name HUAN-LIN CHANG , Order Id: 14803770223
GRE Quantitative Reasoning Practice Questions
147
Ox
P
k
y
Quantity A Quantity B
6. The slope of line kThe slope of line ABCD
Multiple-choice QuestionsSelect One Answer Choice
For Questions 7 to 11, select a single answer choice.
x° z°
7. In the figure above, what is the value of x+y+z?
45
A2
B3
C4
D5
E6
8. A certain store sells two types of pens: one type for $2 per pen and the
other type for $3 per pen. If a customer can spend up to $25 to buy pens at
the store and there is no sales tax, what is the greatest number of pens the
customer can buy?
A9
B10
C11
D12
E20
This ebook was issued to HUAN-LIN CHANG, order #14803770223. Unlawful distribution of this ebook is prohibited.
Customer name HUAN-LIN CHANG , Order Id: 14803770223
GRE Quantitative Reasoning Practice Questions
148
9. If and what is in terms of x?y=3xz=2y,x+y+z
A10x
B9x
C8x
D6x
E5x
10. A certain shipping service charges an insurance fee of $0.75 when shipping
any package with contents worth $25.00 or less and an insurance fee of
$1.00 when shipping any package with contents worth over $25.00. If Dan
uses the shipping company to ship three packages with contents worth
$18.25, $25.00, and $127.50, respectively, what is the total insurance fee
that the company charges Dan to ship the three packages?
A$1.75
B$2.25
C$2.50
D$2.75
E$3.00
11. If 55 percent of the people who purchase a certain product are female,
what is the ratio of the number of females who purchase the product to the
number of males who purchase the product?
A11 to 9
B10 to 9
C9to10
D9to11
E5to9
Numeric Entry
For Questions 12 and 13, enter your answer in the answer box(es) below the
question.
Your answer may be an integer, a decimal, or a fraction, and it may be
negative.
If a question asks for a fraction, there will be two boxesone for the
numerator and one for the denominator.
Equivalent forms of the correct answer, such as 2.5 and 2.50, are all correct.
Fractions do not need to be reduced to lowest terms.
Enter the exact answer unless the question asks you to round your answer.
This ebook was issued to HUAN-LIN CHANG, order #14803770223. Unlawful distribution of this ebook is prohibited.
Customer name HUAN-LIN CHANG , Order Id: 14803770223
GRE Quantitative Reasoning Practice Questions
149
S
T
U
V
R
12. In the rectangular solid above, and What is the areaTU =3, UV =4, VR =2.
of the shaded rectangular region?
13. A list of numbers has a mean of 8 and a standard deviation of 2.5. If xis a
number in the list that is 2 standard deviations above the mean, what is the
value of x?
x=
Multiple-choice QuestionsSelect One or More Answer Choices
For Question 14, select all the answer choices that apply.
Pediatrics
21%
Internal
Medicine
25%
Surgery
24%
Anesthesiology
3%
Psychiatry
6%
Pathology
3%
Radiology
3% Other
15%
14. The circle graph above shows the distribution of 200,000 physicians by
specialty. Which of the following sectors of the circle graph represent more
than 40,000 physicians?
Indicate all such sectors.
APediatrics
BInternal Medicine
CSurgery
DAnesthesiology
EPsychiatry
This ebook was issued to HUAN-LIN CHANG, order #14803770223. Unlawful distribution of this ebook is prohibited.
Customer name HUAN-LIN CHANG , Order Id: 14803770223
GRE Quantitative Reasoning Practice Questions
150
SET 2. Discrete Questions: Medium
Quantitative Comparison
For Questions 1 to 5, compare Quantity A and Quantity B, using additional
information centered above the two quantities if such information is given.
Select one of the following four answer choices and fill in the corresponding
oval to the right of the question.
AQuantity A is greater.
BQuantity B is greater.
CThe two quantities are equal.
DThe relationship cannot be determined from the information given.
A symbol that appears more than once in a question has the same meaning
throughout the question.
Machine R, working alone at a constant rate,
produces xunits of a product in 30 minutes, and
machine S, working alone at a constant rate,
produces xunits of the product in 48 minutes, where
xis a positive integer.
Quantity A Quantity B
1. The number of units of
the product that machine
R, working alone at its
constant rate, produces
in 3 hours
The number of units of
the product that machine
S, working alone at its
constant rate, produces
in 4 hours
ABCD
Frequency Distribution for List X
Number 1 2 3 5
Frequency 10 20 18 12
Frequency Distribution for List Y
Number 6 7 8 9
Frequency 24 17 10 9
List Xand list Yeach contain 60 numbers. Frequency
distributions for each list are given above. The
average (arithmetic mean) of the numbers in list Xis
2.7, and the average of the numbers in list Yis 7.1.
List Zcontains 120 numbers: the 60 numbers in list
Xand the 60 numbers in list Y.
Quantity A Quantity B
2. The average of the 120
numbers in list Z
The median of the 120
numbers in list Z
ABCD
This ebook was issued to HUAN-LIN CHANG, order #14803770223. Unlawful distribution of this ebook is prohibited.
Customer name HUAN-LIN CHANG , Order Id: 14803770223
GRE Quantitative Reasoning Practice Questions
151
BD
C
A
In the figure above, the diameter of the circle is 10.
Quantity A Quantity B
3. The area of quadrilateral
ABCD
40 ABCD
x
2
y0
xy
2
0
Quantity A Quantity B
4. xy
ABCD
Among the 9,000 people attending a football game at
College C, there were xstudents from College Cand y
students who were not from College C.
Quantity A Quantity B
5. The number of people
attending the game who
were not students
9,000 xyABCD
Multiple-choice QuestionsSelect One Answer Choice
For Questions 6 to 10, select a single answer choice.
6. If which of the following is equivalent to
23
x(x)
x0, ?
2
x
A
2
x
B
3
x
C
4
x
D
5
x
E
6
x
This ebook was issued to HUAN-LIN CHANG, order #14803770223. Unlawful distribution of this ebook is prohibited.
Customer name HUAN-LIN CHANG , Order Id: 14803770223
GRE Quantitative Reasoning Practice Questions
152
Ox
y
1
1
7. The figure above shows the graph of the function fin the xy-plane. What is
the value of f(f(1)) ?
A2
B1
C0
D1
E2
8. If which of the following statements describes din terms of n?
d3n=1,
7nd
Adis 4 less than n.
Bdis 4 more than n.
Cdis of n.
3
7
Ddis 2 times n.
Edis 5 times n.
9. By weight, liquid Amakes up 8 percent of solution Rand 18 percent of
solution S. If 3 grams of solution Rare mixed with 7 grams of solution S,
then liquid Aaccounts for what percent of the weight of the resulting
solution?
A10%
B13%
C15%
D19%
E26%
10. Of the 700 members of a certain organization, 120 are lawyers. Two
members of the organization will be selected at random. Which of the
following is closest to the probability that neither of the members selected
will be a lawyer?
A0.5
B0.6
C0.7
D0.8
E0.9
This ebook was issued to HUAN-LIN CHANG, order #14803770223. Unlawful distribution of this ebook is prohibited.
Customer name HUAN-LIN CHANG , Order Id: 14803770223
GRE Quantitative Reasoning Practice Questions
153
Numeric Entry
For Questions 11 and 12, enter your answer in the answer box(es) below the
question.
Your answer may be an integer, a decimal, or a fraction, and it may be
negative.
If a question asks for a fraction, there will be two boxesone for the
numerator and one for the denominator.
Equivalent forms of the correct answer, such as 2.5 and 2.50, are all correct.
Fractions do not need to be reduced to lowest terms.
Enter the exact answer unless the question asks you to round your answer.
Garden Walkway
11. The figure above represents a rectangular garden with a walkway around it.
The garden is 18 feet long and 12 feet wide. The walkway is uniformly 3
feet wide, and its edges meet at right angles. What is the area of the
walkway?
square feet
12. Line klies in the xy-plane. The x-intercept of line kis and line kpasses4,
through the midpoint of the line segment whose endpoints are and(2, 9)
What is the slope of line k?(2, 0).
Give your answer as a fraction.
This ebook was issued to HUAN-LIN CHANG, order #14803770223. Unlawful distribution of this ebook is prohibited.
Customer name HUAN-LIN CHANG , Order Id: 14803770223
GRE Quantitative Reasoning Practice Questions
154
Multiple-choice QuestionsSelect One or More Answer Choices
For Questions 13 and 14, select all the answer choices that apply.
13. If the lengths of two sides of a triangle are 5 and 9, respectively, which of
the following could be the length of the third side of the triangle?
Indicate all such lengths.
A3
B5
C8
D15
0x
y
z
14. On the number line shown above, the tick marks are equally spaced. Which
of the following statements about the numbers x,y, and zmust be true?
Indicate all such statements.
Axyz <0
Bx+z=y
Cz(yx)>0
This ebook was issued to HUAN-LIN CHANG, order #14803770223. Unlawful distribution of this ebook is prohibited.
Customer name HUAN-LIN CHANG , Order Id: 14803770223
GRE Quantitative Reasoning Practice Questions
155
SET 3. Discrete Questions: Hard
Quantitative Comparison
For Questions 1 to 6, compare Quantity A and Quantity B, using additional
information centered above the two quantities if such information is given.
Select one of the following four answer choices and fill in the corresponding
oval to the right of the question.
AQuantity A is greater.
BQuantity B is greater.
CThe two quantities are equal.
DThe relationship cannot be determined from the information given.
A symbol that appears more than once in a question has the same meaning
throughout the question.
125°
6
4
BC
A
D
In the figure above, ABCD is a parallelogram.
Quantity A Quantity B
1. The area of ABCD 24 ABCD
1–5 6–10 11–15 16–20
Measurement Intervals
Frequency
21–25 2630
30
40
10
0
20
31–35
In the course of an experiment, 95 measurements
were recorded, and all of the measurements were
integers. The 95 measurements were then grouped
into 7 measurement intervals. The graph above
shows the frequency distribution of the 95
measurements by measurement interval.
Quantity A Quantity B
2. The average (arithmetic
mean) of the 95
measurements
The median of the 95
measurements
ABCD
This ebook was issued to HUAN-LIN CHANG, order #14803770223. Unlawful distribution of this ebook is prohibited.
Customer name HUAN-LIN CHANG , Order Id: 14803770223
GRE Quantitative Reasoning Practice Questions
156
xis an integer greater than 1.
Quantity A Quantity B
3.
x+1
3
x
4ABCD
A,B, and Care three rectangles. The length and width
of rectangle Aare 10 percent greater and 10 percent
less, respectively, than the length and width of
rectangle C. The length and width of rectangle Bare
20 percent greater and 20 percent less, respectively,
than the length and width of rectangle C.
Quantity A Quantity B
4. The area of rectangle AThe area of rectangle BABCD
The random variable Xis normally distributed. The
values 650 and 850 are at the 60th and 90th
percentiles of the distribution of X, respectively.
Quantity A Quantity B
5. The value at the 75th
percentile of the
distribution of X
750 ABCD
Set Sconsists of all positive integers less than 81 that
are not equal to the square of an integer.
Quantity A Quantity B
6. The number of integers in
set S
72 ABCD
This ebook was issued to HUAN-LIN CHANG, order #14803770223. Unlawful distribution of this ebook is prohibited.
Customer name HUAN-LIN CHANG , Order Id: 14803770223
GRE Quantitative Reasoning Practice Questions
157
Multiple-choice QuestionsSelect One Answer Choice
For Questions 7 to 12, select a single answer choice.
7. A manager is forming a 6-person team to work on a certain project. From
the 11 candidates available for the team, the manager has already chosen 3
to be on the team. In selecting the other 3 team members, how many
different combinations of 3 of the remaining candidates does the manager
have to choose from?
A6
B24
C56
D120
E462
8. Which of the following could be the graph of all values of xthat satisfy the
inequality 6x5
25x?
3
A0
B0
C0
D0
E0
9. If then the average (arithmetic mean) of x, and
23 23 4
1+x+x+x=60, x,x,x
is equal to which of the following?
A12x
B15x
C20x
D30x
E60x
This ebook was issued to HUAN-LIN CHANG, order #14803770223. Unlawful distribution of this ebook is prohibited.
Customer name HUAN-LIN CHANG , Order Id: 14803770223
GRE Quantitative Reasoning Practice Questions
158
Ox
y
P
R
Q
10. Parallelogram OPQR lies in the xy-plane, as shown in the figure above. The
coordinates of point Pare and the coordinates of point Qare(2, 4) (8, 6).
What are the coordinates of point R?
A(3, 2)
B(3, 3)
C(4, 4)
D(5, 2)
E(6, 2)
11. The relationship between the area Aof a circle and its circumference Cis
given by the formula where kis a constant. What is the value
2
A=kC ,
of k?
A1
4p
B1
2p
C1
4
D2p
E
2
4p
12. The sequence of numbers is defined by for
11
a,a,a,...,a,... a=−
123 nn
nn+2
each integer What is the sum of the first 20 terms of this sequence?n1.
A11
1+−
冢冣
220
B111
1+− +
冢冣冢 冣
22122
C11
1−+
冢冣
20 22
D1
122
E11
20 22
This ebook was issued to HUAN-LIN CHANG, order #14803770223. Unlawful distribution of this ebook is prohibited.
Customer name HUAN-LIN CHANG , Order Id: 14803770223
GRE Quantitative Reasoning Practice Questions
159
Numeric Entry
For Question 13, enter your answer in the answer box(es) below the question.
Your answer may be an integer, a decimal, or a fraction, and it may be
negative.
If a question asks for a fraction, there will be two boxesone for the
numerator and one for the denominator.
Equivalent forms of the correct answer, such as 2.5 and 2.50, are all correct.
Fractions do not need to be reduced to lowest terms.
Enter the exact answer unless the question asks you to round your answer.
YFrequency
1
22
3
47
5
48
3
28
7
49
13. The table above shows the frequency distribution of the values of a variable
Y. What is the mean of the distribution?
Give your answer to the nearest 0.01.
Multiple-choice QuestionsSelect One or More Answer Choices
For Questions 14 and 15, select all the answer choices that apply.
14. Let Sbe the set of all positive integers nsuch that is a multiple of both
2
n
24 and 108. Which of the following integers are divisors of every integer n
in S?
Indicate all such integers.
A12
B24
C36
D72
This ebook was issued to HUAN-LIN CHANG, order #14803770223. Unlawful distribution of this ebook is prohibited.
Customer name HUAN-LIN CHANG , Order Id: 14803770223
GRE Quantitative Reasoning Practice Questions
160
15. The range of the heights of the female students in a certain class is 13.2
inches, and the range of the heights of the male students in the class is 15.4
inches.
Which of the following statements individually provide(s) sufficient
additional information to determine the range of the heights of all the
students in the class?
Indicate all such statements.
AThe tallest male student in the class is 5.8 inches taller than the tallest
female student in the class.
BThe median height of the male students in the class is 1.1 inches
greater than the median height of the female students in the class.
CThe average (arithmetic mean) height of the male students in the class
is 4.6 inches greater than the average height of the female students in
the class.
This ebook was issued to HUAN-LIN CHANG, order #14803770223. Unlawful distribution of this ebook is prohibited.
Customer name HUAN-LIN CHANG , Order Id: 14803770223
GRE Quantitative Reasoning Practice Questions
161
SET 4. Data Interpretation Sets
For Questions 1 to 7, select a single answer choice unless otherwise directed.
Questions 1 to 3 are based on the following data.
Biological Sciences
Business
Education
Engineering
Fine Arts
Health Sciences
Humanities
Physical Sciences
Social Sciences
Other
0510
Percent
PERCENT OF FEMALE FACULTY AND PERCENT OF MALE FACULTY
AT UNIVERSITY X, BY FIELD
Total female faculty: 200
Total male faculty: 250
Field
15 20 25
Female
Male
Medium Question
1. There are 275 students in the field of engineering at University X.
Approximately what is the ratio of the number of students in engineering to
the number of faculty in engineering?
A8to1
B10 to 1
C12 to 1
D14 to 1
E20 to 1
Medium Question
2. Approximately what percent of the faculty in humanities are male?
A35%
B38%
C41%
D45%
E51%
This ebook was issued to HUAN-LIN CHANG, order #14803770223. Unlawful distribution of this ebook is prohibited.
Customer name HUAN-LIN CHANG , Order Id: 14803770223
GRE Quantitative Reasoning Practice Questions
162
For Question 3, use the directions for Numeric Entry questions.
Hard Question
3. For the biological sciences and health sciences faculty combined, of the
1
3
female and of the male faculty members are tenured professors. What
2
9
fraction of all the faculty members in those two fields combined are tenured
professors?
Questions 4 to 7 are based on the following data.
2000 2001 2002 2003 2004
Year
VALUE OF IMPORTS TO AND EXPORTS FROM COUNTRY T, 20002009
(in United States dollars)
2005 2006 2007 2008 2009
16
14
12
10
8
6
4
2
0
Exports
Imports
Billions* of Dollars
*1 billion = 1,000,000,000
For Question 4, select all the answer choices that apply.
Easy Question
4. For which of the eight years from 2001 to 2008 did exports exceed imports
by more than $5 billion?
Indicate all such years.
A2001
B2002
C2003
D2004
E2005
F2006
G2007
H2008
This ebook was issued to HUAN-LIN CHANG, order #14803770223. Unlawful distribution of this ebook is prohibited.
Customer name HUAN-LIN CHANG , Order Id: 14803770223
GRE Quantitative Reasoning Practice Questions
163
Medium Question
5. Which of the following is closest to the average (arithmetic mean) of the 9
changes in the value of imports between consecutive years from 2000 to
2009 ?
A$260 million
B$320 million
C$400 million
D$480 million
E$640 million
Medium Question
6. In 2008 the value of exports was approximately what percent greater than
the value of imports?
A40%
B60%
C70%
D120%
E140%
Hard Question
7. If it were discovered that the value of imports shown for 2007 was incorrect
and should have been $5 billion instead, then the average (arithmetic mean)
value of imports per year for the 10 years shown would have been
approximately how much less?
A$200 million
B$50 million
C$20 million
D$7 million
E$5 million
This ebook was issued to HUAN-LIN CHANG, order #14803770223. Unlawful distribution of this ebook is prohibited.
Customer name HUAN-LIN CHANG , Order Id: 14803770223
GRE Quantitative Reasoning Practice Questions
164
ANSWER KEY
SET 1. Discrete Questions: Easy
1. Choice A: Quantity A is greater.
2. Choice C: The two quantities are equal.
3. Choice A: Quantity A is greater.
4. Choice D: The relationship cannot be determined from the information
given.
5. Choice B: Quantity B is greater.
6. Choice A: Quantity A is greater.
7. Choice C:4
8. Choice D:12
9. Choice A:10x
10. Choice C: $2.50
11. Choice A:11to9
12. 10
13. 13
14. Choice A: Pediatrics
AND
Choice B: Internal Medicine
AND
Choice C: Surgery
SET 2. Discrete Questions: Medium
1. Choice A: Quantity A is greater.
2. Choice B: Quantity B is greater.
3. Choice D: The relationship cannot be determined from the information
given.
4. Choice B: Quantity B is greater.
5. Choice C: The two quantities are equal.
6. Choice D:
5
x
7. Choice D:1
8. Choice E:dis 5 times n.
9. Choice C: 15%
10. Choice C: 0.7
11. 216
12. (or any equivalent fraction)
3
4
13. Choice B:5
AND
Choice C: 8
14. Choice A:xyz <0
AND
Choice B:x+z=y
AND
Choice C:z(yx)>0
This ebook was issued to HUAN-LIN CHANG, order #14803770223. Unlawful distribution of this ebook is prohibited.
Customer name HUAN-LIN CHANG , Order Id: 14803770223
GRE Quantitative Reasoning Practice Questions
165
SET 3. Discrete Questions: Hard
1. Choice B: Quantity B is greater.
2. Choice A: Quantity A is greater.
3. Choice D: The relationship cannot be determined from the information
given.
4. Choice A: Quantity A is greater.
5. Choice B: Quantity B is greater.
6. Choice C: The two quantities are equal.
7. Choice C:56
8. Choice C:0
9. Choice B:15x
10. Choice E:(6,2)
11. Choice A:1
4p
12. Choice B:111
1+− +
冢冣冢 冣
22122
13. 1.29
14. Choice A:12
AND
Choice C:36
15. Choice A: The tallest male student in the class is 5.8 inches taller than the
tallest female student in the class.
SET 4. Data Interpretation Sets
1. Choice A: 8 to 1
2. Choice E: 51%
3. (or any equivalent fraction)
24
87
4. Choice A: 2001
AND
Choice B: 2002
AND
Choice C: 2003
AND
Choice F: 2006
AND
Choice G: 2007
AND
Choice H: 2008
5. Choice E: $640 million
6. Choice E: 140%
7. Choice A: $200 million
This ebook was issued to HUAN-LIN CHANG, order #14803770223. Unlawful distribution of this ebook is prohibited.
Customer name HUAN-LIN CHANG , Order Id: 14803770223
GRE Quantitative Reasoning Practice Questions
166
Answers and Explanations
For the practice questions in this chapter, use the following directions.
General Directions
For each question, indicate the best answer, using the directions given.
Notes: All numbers used are real numbers.
All figures are assumed to lie in a plane unless otherwise indicated.
Geometric figures, such as lines, circles, triangles, and quadrilaterals, are not
necessarily drawn to scale. That is, you should not assume that quantities
such as lengths and angle measures are as they appear in a figure. You
should assume, however, that lines shown as straight are actually straight,
points on a line are in the order shown, and more generally, all geometric
objects are in the relative positions shown. For questions with geometric fig-
ures, you should base your answers on geometric reasoning, not on estimat-
ing or comparing quantities by sight or by measurement.
Coordinate systems, such as xy-planes and number lines, are drawn to scale;
therefore, you can read, estimate, or compare quantities in such figures by
sight or by measurement.
Graphical data presentations, such as bar graphs, circle graphs, and line
graphs, are drawn to scale; therefore, you can read, estimate, or compare
data values by sight or by measurement.
Directions for Quantitative Comparison questions
Compare Quantity A and Quantity B, using additional information centered
above the two quantities if such information is given. Select one of the
following four answer choices and fill in the corresponding oval to the right of
the question.
AQuantity A is greater.
BQuantity B is greater.
CThe two quantities are equal.
DThe relationship cannot be determined from the information given.
A symbol that appears more than once in a question has the same meaning
throughout the question.
This ebook was issued to HUAN-LIN CHANG, order #14803770223. Unlawful distribution of this ebook is prohibited.
Customer name HUAN-LIN CHANG , Order Id: 14803770223
GRE Quantitative Reasoning Practice Questions
167
Directions for Numeric Entry questions
Enter your answer in the answer box(es) below the question.
Your answer may be an integer, a decimal, or a fraction, and it may be
negative.
If a question asks for a fraction, there will be two boxesone for the
numerator and one for the denominator.
Equivalent forms of the correct answer, such as 2.5 and 2.50, are all correct.
Fractions do not need to be reduced to lowest terms.
Enter the exact answer unless the question asks you to round your answer.
SET 1. Discrete Questions: Easy
Quantitative Comparison
For Questions 1 to 6, use the directions for Quantitative Comparison questions.
Emma spent $75 buying a used bicycle and $27
repairing it. Then she sold the bicycle for 40 percent
more than the total amount she spent buying and
repairing it.
Quantity A Quantity B
1. The price at which Emma
sold the bicycle
$140 ABCD
Explanation
In this question you are asked to compare the price at which Emma sold the
bicycle with $140. From the information given, you can conclude that Emma
spent a total of dollars buying and repairing the bicycle and that75 +27 =102
she sold it for 40 percent more than the $102 she spent buying and repairing it.
If you notice that 140 is 40 percent more than 100, you can conclude that 40
percent more than 102 is greater than 40 percent more than 100, and therefore,
Quantity A is greater than Quantity B. The correct answer is Choice A. (If you
solve the problem in this way, you do not have to calculate the value of Quantity A.)
Another way to solve the problem is by explicitly calculating the value of
Quantity A and comparing the result with $140 directly. Since 40 percent of 102
is it follows that Quantity A, the price at which Emma sold the(0.4)(102) =40.8,
bicycle, is dollars. Thus Quantity A, $142.80, is greater102.00 +40.80 =142.80
than Quantity B, $140, and the correct answer is Choice A.
This ebook was issued to HUAN-LIN CHANG, order #14803770223. Unlawful distribution of this ebook is prohibited.
Customer name HUAN-LIN CHANG , Order Id: 14803770223
GRE Quantitative Reasoning Practice Questions
168
QS
PT
R
V
In the figure above, squares PQRV and VRST have
sides of length 6.
Quantity A Quantity B
2. The area of the shaded
region
36 ABCD
Explanation
In this question you are asked to compare the area of the shaded region with 36.
You are given that both PQRV and VRST are squares with sides of length 6.
Therefore, you can conclude that the length of QS is 12, and the area of the
shaded right triangle PQS is or 36. Thus Quantity A is equal to Quantity
1(12)(6),
2
B, and the correct answer is Choice C.
In 2009 the property tax on each home in Town X
was ppercent of the assessed value of the home,
where pis a constant. The property tax in 2009 on a
home in Town Xthat had an assessed value of
$125,000 was $2,500.
Quantity A Quantity B
3. The property tax in 2009 on
a home in Town Xthat had
an assessed value of
$160,000
$3,000 ABCD
Explanation
Before making the comparison in this problem, you need to analyze the
information given to see what it tells you about the value of Quantity A, the
property tax in 2009 on a home in Town Xthat had an assessed value of
$160,000. One way of doing this is to determine the value of the constant pand
then use that value to calculate the tax on the home that had an assessed value
of $160,000.
Since it is given that a home that had an assessed value of $125,000 had a
property tax of $2,500, you can conclude that pis equal to or 2%. Once
2,500 ,
125,000
you know that the property tax is 2% of the assessed value, you can determine
that tax on the home that had an assessed value of $160,000 was 2% of 160,000,
or 3,200. The correct answer is Choice A.
This ebook was issued to HUAN-LIN CHANG, order #14803770223. Unlawful distribution of this ebook is prohibited.
Customer name HUAN-LIN CHANG , Order Id: 14803770223
GRE Quantitative Reasoning Practice Questions
169
Another way to calculate the property tax on a home with an assessed value
of $160,000 is by setting up a proportion. Because the tax rate is the same for
each home in Town X, you can let the variable xrepresent the tax for the home
assessed at $160,000 and solve for xas follows.
x2,500
=
160,000 125,000
125,000x=(160,000)(2,500)
(160,000)(2,500)
x=125,000
x=3,200
The correct answer is Choice A.
x+y=−1
Quantity A Quantity B
4. xy
ABCD
Explanation
One way to approach this question is to plug in values for one of the variables
and determine the corresponding value for the other variable.
One way to plug in: Plug in easy values. For example, you can plug in x=0
and find that the corresponding value of yis then you can plug in and
1; y=0
find that the corresponding value of xis Since in the first case xis greater
1.
than yand in the second case yis greater than x, the correct answer is Choice D,
the relationship cannot be determined from the information given.
A second way to plug in: If you prefer to always plug in values of xto
determine corresponding values of y, you can begin by writing the equation
as Writing it in this form makes it easier to find the
x+y=−1y=−x1.
corresponding values of y.
You can start by plugging in the value For this value of x, the
x=0.
corresponding value of yis and therefore, xis greater than y.Ifyou
y=−1,
continue plugging in a variety of values of x, some negative and some positive,
you will see that sometimes xis greater than yand sometimes yis greater than x.
If you inspect the equation you can conclude that since there is a
y=−x1,
negative sign in front of the xbut not in front of the y, for each value of xthat is
greater than 0, the corresponding value of yis less than 0; therefore, for each
xis greater than y.
x>0,
What about negative values of x? A quick inspection of the equation
y=−x1 allows you to conclude that if then so yis greaterx<−1, y>0,
than x.
So for some values of xand ythat satisfy the equation, xis greater than y;
and for other values, yis greater than x. Therefore, the relationship between the
two quantities xand ycannot be determined from the information given, and the
correct answer is Choice D.
r,s, and tare three consecutive odd integers such
that r<s<t.
Quantity A Quantity B
5. r+s+1s+t1ABCD
This ebook was issued to HUAN-LIN CHANG, order #14803770223. Unlawful distribution of this ebook is prohibited.
Customer name HUAN-LIN CHANG , Order Id: 14803770223
GRE Quantitative Reasoning Practice Questions
170
Explanation
You are given that three numbers, r,s, and t, are consecutive odd integers and
that This means that if you express the three consecutive odd integersr<s<t.
in terms of r, they are r, andr+2, r+4.
One way to approach this problem is to set up a placeholder relationship
between the two quantities and simplify it to see what conclusions you can draw.
Simplification 1: Begin simplifying by expressing sand tin terms of r. The
steps in this simplification can be done as follows.
r+s+1?s+t1
| |
r+(r+2) +1?(r+2) +(r+4) 1
| |
2r+3?2r+5
| |
3?5
| |
In the last step of the simplification, you can easily see that If you follow3 <5.
the simplification steps in reverse, you can see that the placeholder in each step
remains unchanged, so you can conclude that Quantity B is greater than
Quantity A, and the correct answer is Choice B.
Simplification 2: Since the number sappears in both quantities, you can
begin the simplification by subtracting sfrom both sides of the relationship and
then express tin terms of r. The steps in this simplification can be done as
follows.
r+s+1?s+t1
| |
r+1?t1
| |
r+1?(r+4) 1
| |
r+1?r+3
| |
1?3
| |
In the last step of the simplification, you can easily see that If you follow1 <3.
the simplification steps in reverse, you can see that the placeholder in each step
remains unchanged, so you can conclude that Quantity B is greater than
Quantity A, and the correct answer is Choice B.
Note that in this solution, the fact that ris odd is not used; what is used is
the fact that the consecutive odd integers differ by 2.
Ox
P
k
y
Quantity A Quantity B
6. The slope of line kThe slope of line ABCD
This ebook was issued to HUAN-LIN CHANG, order #14803770223. Unlawful distribution of this ebook is prohibited.
Customer name HUAN-LIN CHANG , Order Id: 14803770223
GRE Quantitative Reasoning Practice Questions
171
Explanation
Note that the slope of each of the lines is positive, since each line rises as it goes
to the right. Since the slopes of both lines are positive and line krises faster (or
is steeper) than line , line khas the greater slope, and the correct answer is
Choice A.
You can also use the definition of the slope to arrive at the correct answer.
Slope can be defined as the ratio of “rise” to “run” between any two points on a
line, where the rise is the vertical distance between the points and the run is the
horizontal distance, and the slope is respectively positive or negative depending
on whether the line rises or falls when viewed from left to right. Because both
lines pass through point Pon the y-axis, they have the same rise from Pto the
x-axis. However, line intersects the x-axis at a greater value than line k. Thus,
the run of line from the y-axis to the x-intercept is greater than the run of line
k. When the slope is expressed as a ratio, both lines have the same numerator
(rise), but line has a greater denominator (run). The greater denominator
results in a lesser fraction and a lesser slope for line Therefore, the correct.
answer is Choice A.
Multiple-choice QuestionsSelect One Answer Choice
For Questions 7 to 11, select a single answer choice.
x° z°
7. In the figure above, what is the value of x+y+z?
45
A2
B3
C4
D5
E6
Explanation
The sum of the measures, in degrees, of the three interior angles of any triangle
is As shown in the figure, the three angles of the triangle have measures of180.
and so Therefore, and the correct
x+y+z180
x,y,z,x+y+z=180. ==4,
45 45
answer is Choice C.
This ebook was issued to HUAN-LIN CHANG, order #14803770223. Unlawful distribution of this ebook is prohibited.
Customer name HUAN-LIN CHANG , Order Id: 14803770223
GRE Quantitative Reasoning Practice Questions
172
8. A certain store sells two types of pens: one type for $2 per pen and the
other type for $3 per pen. If a customer can spend up to $25 to buy pens at
the store and there is no sales tax, what is the greatest number of pens the
customer can buy?
A9
B10
C11
D12
E20
Explanation
It is fairly clear that the greatest number of pens that can be bought for $25 will
consist mostly, if not entirely, of $2 pens. In fact, it is reasonable to begin by
looking at how many of the $2 pens the customer can buy if the customer does
not buy any $3 pens. It is easy to see that the customer could buy 12 of the $2
pens, with $1 left over.
If the customer bought 11 of the $2 pens, there would be $3 left over with
which to buy a $3 pen. In this case, the customer could still buy 12 pens.
If the customer bought 10 of the $2 pens, there would be $5 left over. Only 1
of the $3 pens could be bought with the $5, so in this case, the customer could
buy only 11 pens.
As the number of $2 pens decreases, the total number of pens that the
customer can buy with $25 decreases as well. Thus the greatest number of pens
the customer can buy with $25 is 12. The correct answer is Choice D.
9. If and what is in terms of x?y=3xz=2y,x+y+z
A10x
B9x
C8x
D6x
E5x
Explanation
It is not necessary to find the individual values of x,y, and zto answer the
question. You are asked to rewrite the expression as an equivalentx+y+z
expression in terms of x. This means that you need to use the information
provided about yand zto express them in terms of the variable x. The variable y
is already given in terms of x; that is, and because it follows thaty=3x;z=2y,
Using substitution, you can rewrite the expression as follows.z=(2)(3x)=6x.
x+y+z=x+(3x)+(6x)
=(1 +3+6)x
=10x
The correct answer is Choice A.
This ebook was issued to HUAN-LIN CHANG, order #14803770223. Unlawful distribution of this ebook is prohibited.
Customer name HUAN-LIN CHANG , Order Id: 14803770223
GRE Quantitative Reasoning Practice Questions
173
10. A certain shipping service charges an insurance fee of $0.75 when shipping
any package with contents worth $25.00 or less and an insurance fee of
$1.00 when shipping any package with contents worth over $25.00. If Dan
uses the shipping company to ship three packages with contents worth
$18.25, $25.00, and $127.50, respectively, what is the total insurance fee
that the company charges Dan to ship the three packages?
A$1.75
B$2.25
C$2.50
D$2.75
E$3.00
Explanation
Note that two of the packages being shipped have contents that are worth $25.00
or less. Therefore, each of them has an insurance fee of $0.75, for a total of
$1.50. The third package has contents worth over $25.00, and it has an insurance
fee of $1.00. Therefore, the total insurance fee for the three packages is
and the correct answer is Choice C.$1.50 +$1.00 =$2.50,
11. If 55 percent of the people who purchase a certain product are female,
what is the ratio of the number of females who purchase the product to the
number of males who purchase the product?
A11 to 9
B10 to 9
C9to10
D9to11
E5to9
Explanation
Note that because 55 percent of the people who purchase the product are
females, it follows that 45 percent of the people who purchase the product are
males. Therefore, the ratio of the number of females who purchase the product
to the number of males who purchase the product is 55 to 45, or 11 to 9, and
the correct answer is Choice A.
This ebook was issued to HUAN-LIN CHANG, order #14803770223. Unlawful distribution of this ebook is prohibited.
Customer name HUAN-LIN CHANG , Order Id: 14803770223
GRE Quantitative Reasoning Practice Questions
174
Numeric Entry
For Questions 12 and 13, use the directions for Numeric Entry questions.
S
T
U
V
R
12. In the rectangular solid above, and What is the areaTU =3, UV =4, VR =2.
of the shaded rectangular region?
Explanation
To find the area of the shaded rectangular region, you need to multiply the
length of the rectangular region by its width. In this question you are given
the lengths of three edges: and Note that VR is theTU =3, UV =4, VR =2.
length of the shaded rectangle. To find the width of the shaded rectangle, you
need to find either RS or VT. Note that VT lies on the front face of the
rectangular solid. It is the hypotenuse of right triangle VUT. You know that
and so by the Pythagorean theorem you can conclude thatUV =4TU =3,
Therefore, the area of the shaded
22
VT =3+4=9+16 =25 =5.
冪冪
rectangular region is The correct answer is 10.(5)(2) =10.
13. A list of numbers has a mean of 8 and a standard deviation of 2.5. If xis a
number in the list that is 2 standard deviations above the mean, what is the
value of x?
x=
Explanation
You are given that xis 2 standard deviations above the mean, 8. Because the
standard deviation of the numbers in the list is 2.5, it follows that xis (2)(2.5),
or 5 units above the mean 8. Therefore, and the correct answer isx=8+5=13,
13.
This ebook was issued to HUAN-LIN CHANG, order #14803770223. Unlawful distribution of this ebook is prohibited.
Customer name HUAN-LIN CHANG , Order Id: 14803770223
GRE Quantitative Reasoning Practice Questions
175
Multiple-choice QuestionsSelect One or More Answer Choices
For Question 14, select all the answer choices that apply.
Pediatrics
21%
Internal
Medicine
25%
Surgery
24%
Anesthesiology
3%
Psychiatry
6%
Pathology
3%
Radiology
3% Other
15%
14. The circle graph above shows the distribution of 200,000 physicians by
specialty. Which of the following sectors of the circle graph represent more
than 40,000 physicians?
Indicate all such sectors.
APediatrics
BInternal Medicine
CSurgery
DAnesthesiology
EPsychiatry
Explanation
One approach to solve this problem is to find out what percent of 200,000 is
40,000 and then compare this percent with the percents given in the circle
graph. Because it follows that 40,000 is 20% of 200,000, and any
40,000 =0.2,
200,000
specialty that has more than 20% of the distribution has more than 40,000
physicians. This is true for the specialties of pediatrics, internal medicine, and
surgery. The correct answer consists of Choices A, B, and C.
This ebook was issued to HUAN-LIN CHANG, order #14803770223. Unlawful distribution of this ebook is prohibited.
Customer name HUAN-LIN CHANG , Order Id: 14803770223
GRE Quantitative Reasoning Practice Questions
176
SET 2. Discrete Questions: Medium
Quantitative Comparison
For Questions 1 to 5, use the directions for Quantitative Comparison questions.
Machine R, working alone at a constant rate,
produces xunits of a product in 30 minutes, and
machine S, working alone at a constant rate,
produces xunits of the product in 48 minutes, where
xis a positive integer.
Quantity A Quantity B
1. The number of units of
the product that machine
R, working alone at its
constant rate, produces
in 3 hours
The number of units of
the product that machine
S, working alone at its
constant rate, produces
in 4 hours
ABCD
Explanation
In this question you are given that machine R, working alone at its constant rate,
produces xunits of a product in 30 minutes. Since it is easy to see that 3 hours
is 6 times 30 minutes, you can conclude that Quantity A is 6x.
You can compare 6xwith Quantity B in two ways.
One: In the additional information centered above the quantities, you are
given that machine S, working alone at its constant rate, produces xunits of the
product in 48 minutes, so you can conclude that machine Scan produce 6xunits
of the product in minutes, or 4.8 hours. So in 4 hours, machine S(6)(48)
produces less than 6xunits, and Quantity B is less than 6x.
Two: First, convert 48 minutes to hour, then find the number of 48-minute
4
5
periods there are in 4 hours by computing Thus, Quantity B
45
=(4) =5.
冢冣
4
4
冢冣
5
is 5x.
Either way, Quantity A is greater than Quantity B, and the correct answer is
Choice A.
This ebook was issued to HUAN-LIN CHANG, order #14803770223. Unlawful distribution of this ebook is prohibited.
Customer name HUAN-LIN CHANG , Order Id: 14803770223
GRE Quantitative Reasoning Practice Questions
177
Frequency Distribution for List X
Number 1 2 3 5
Frequency 10 20 18 12
Frequency Distribution for List Y
Number 6 7 8 9
Frequency 24 17 10 9
List Xand list Yeach contain 60 numbers. Frequency
distributions for each list are given above. The
average (arithmetic mean) of the numbers in list Xis
2.7, and the average of the numbers in list Yis 7.1.
List Zcontains 120 numbers: the 60 numbers in list
Xand the 60 numbers in list Y.
Quantity A Quantity B
2. The average of the 120
numbers in list Z
The median of the 120
numbers in list Z
ABCD
Explanation
In this problem you are asked to compare the average with the median of the
120 numbers in list Z. Since list Zconsists of the numbers in lists Xand Y
combined, it is reasonable to try to use the information about lists Xand Yto
calculate the average and the median of the numbers in list Z.
To determine the average of the 120 numbers in list Z, you can use the
information given about the individual averages of the numbers in lists Xand Y.
Because lists Xand Yeach contain 60 numbers, the average of the numbers in
list Zis the average of the individual averages of the numbers in lists Xand Y.
Thus, the average of the numbers in list Zis or 4.9.
2.7 +7.1,
2
To determine the median of the 120 numbers in list Z, first note that list Z
contains an even number of numbers, so the median of the numbers in list Zis
the average of the middle two numbers when the numbers are listed in
increasing order. If you look at the numbers in the two lists, you will see that the
60 numbers in list Xare all less than or equal to 5, and the 60 numbers in list Y
are all greater than or equal to 6. Thus, the two middle numbers in list Zare 5
and 6, and the average of these numbers is or 5.5. Therefore, the median
5+6,
2
of the numbers in list Zis 5.5, and this is greater than the average of 4.9. The
correct answer is Choice B.
This ebook was issued to HUAN-LIN CHANG, order #14803770223. Unlawful distribution of this ebook is prohibited.
Customer name HUAN-LIN CHANG , Order Id: 14803770223
GRE Quantitative Reasoning Practice Questions
178
BD
C
A
In the figure above, the diameter of the circle is 10.
Quantity A Quantity B
3. The area of quadrilateral
ABCD
40 ABCD
Explanation
You are given that the circle has a diameter of 10, and from the figure you can
assume that points A,B,C, and Dlie on the circle in the order shown. However,
because figures are not necessarily drawn to scale, you cannot assume anything
else about the positions of points A,B,C, and Don the circle. Therefore, to get
an idea of how various possible positions of these four points could affect the
area of quadrilateral ABCD, it is a good idea to see how the figure can vary but
still have points A,B,C, and Din the same order as in the figure above.
One way that you might vary the figure is to evenly space the four points
along the circle, as shown below.
BD
A
C
Another way is to draw points Aand Copposite each other, with points B
and Dclose to point C, as shown below.
A
BCD
From these figures you can draw some basic conclusions about the area of
ABCD.
If points Aand Care opposite each other, with points Band Dvery close to
point C, the area of quadrilateral ABCD is very close to 0. Clearly, the area can
be less than 40 (Quantity B).
If points A,B,C, and Dare evenly spaced, the area is not close to 0. How
does the area compare with 40? To calculate the area of ABCD, draw the
This ebook was issued to HUAN-LIN CHANG, order #14803770223. Unlawful distribution of this ebook is prohibited.
Customer name HUAN-LIN CHANG , Order Id: 14803770223
GRE Quantitative Reasoning Practice Questions
179
diameters AC and BD in the figure. The two diameters are perpendicular
bisectors of each other, so they divide ABCD into four right triangles, as shown.
BD
A
C
55
5
5
The area of each of the right triangles is or 12.5. Thus, the area of
1(5)(5),
冢冣
2
ABCD is or 50.(4)(12.5),
Since the area of the quadrilateral in the first figure is less than 40 and the
area of the quadrilateral in the second figure is greater than 40, the relationship
cannot be determined from the information given. The correct answer is Choice D.
x
2
y0
xy
2
0
Quantity A Quantity B
4. xy
ABCD
Explanation
You are given that which means that the product of the two numbers
2 2
xy>0, x
and yis positive. Recall that the product of two numbers is positive only if both
numbers are positive or both numbers are negative. The square of a number is
always greater than or equal to 0. In this case, cannot equal 0 because the
2
x
product is not 0. Thus, is positive and it follows that yis also positive.
22
xy x
You are also given that which means that the product of the two
2
xy <0,
numbers xand is negative. The product of two numbers is negative only if one
2
y
of the numbers is negative and the other number is positive. In this case,
2
y
cannot be negative because it is the square of a number, and it cannot be 0
because the product is not 0. Thus, is positive and so xmust be negative.
22
xy y
Because xis negative and yis positive, ymust be greater than x, and the
correct answer is Choice B.
Among the 9,000 people attending a football game at
College C, there were xstudents from College Cand y
students who were not from College C.
Quantity A Quantity B
5. The number of people
attending the game who
were not students
9,000 xyABCD
This ebook was issued to HUAN-LIN CHANG, order #14803770223. Unlawful distribution of this ebook is prohibited.
Customer name HUAN-LIN CHANG , Order Id: 14803770223
GRE Quantitative Reasoning Practice Questions
180
Explanation
In this question you are not told whether all of the 9,000 people attending the
game were students. Let zbe the number of people attending the game who
were not students. The people attending the game can be broken down into three
groups: students from College C, students not from College C, and people who
were not students. This can be expressed algebraically as where9,000 =x+y+z,
xrepresents the number of students from College Cattending the game and y
represents the number of students attending the game who were not from
College C. Therefore, is the number of people attending the9,000 xy=z
game who were not students. The correct answer is Choice C.
Multiple-choice QuestionsSelect One Answer Choice
For Questions 6 to 10, select a single answer choice.
6. If which of the following is equivalent to
23
x(x)
x0, ?
2
x
A
2
x
B
3
x
C
4
x
D
5
x
E
6
x
Explanation
To simplify it can be helpful to write as in the given
23
x(x)
23 2 2 2
,(x)(x)(x)(x)
2
x
expression; that is, Because both numerator and
23 2 2 2
x(x)x(x)(x)(x)
=.x0,
22
xx
denominator can be divided by and the expression simplifies to
222
x,x(x)(x),
which, by the rules of exponents, is equal to
5
x.
Another way to simplify the expression using the rules of exponents directly
is as follows.
23 6 7
x(x)x(x)x
5
===x
222
xxx
The correct answer is Choice D.
This ebook was issued to HUAN-LIN CHANG, order #14803770223. Unlawful distribution of this ebook is prohibited.
Customer name HUAN-LIN CHANG , Order Id: 14803770223
GRE Quantitative Reasoning Practice Questions
181
Ox
y
1
1
7. The figure above shows the graph of the function fin the xy-plane. What is
the value of f(f(1)) ?
A2
B1
C0
D1
E2
Explanation
Note that to find you must apply the function ftwice, first to find thef(f(1)),
value of and then to find the value of To find the value off(1) f(f(1)).
find the point on the graph of the function fwhose x-coordinate isf(1),
This point has y-coordinate Therefore, the value of is 2, andx=−1. y=2. f(1)
Next you need to find the value of To find the value off(f(1)) =f(2). f(2). f(2),
find the point on the graph whose x-coordinate is This point hasx=2.
y-coordinate Therefore, and because you cany=1. f(2) =1, f(f(1)) =f(2),
conclude that The correct answer is Choice D.f(f(1)) =1.
8. If which of the following statements describes din terms of n?
d3n=1,
7nd
Adis 4 less than n.
Bdis 4 more than n.
Cdis of n.
3
7
Ddis 2 times n.
Edis 5 times n.
Explanation
To describe din terms of n, you need to solve the equation for d.To
d3n=1
7nd
simplify the equation, you can begin by multiplying both sides by and7nd
then proceed as follows.
d3n
(7nd)=(7nd)(1)
冢冣
7nd
d3n=7nd
d=10nd
2d=10n
d=5n
The correct answer is Choice E.
This ebook was issued to HUAN-LIN CHANG, order #14803770223. Unlawful distribution of this ebook is prohibited.
Customer name HUAN-LIN CHANG , Order Id: 14803770223
GRE Quantitative Reasoning Practice Questions
182
9. By weight, liquid Amakes up 8 percent of solution Rand 18 percent of
solution S. If 3 grams of solution Rare mixed with 7 grams of solution S,
then liquid Aaccounts for what percent of the weight of the resulting
solution?
A10%
B13%
C15%
D19%
E26%
Explanation
Liquid Amakes up 8 percent of the weight of solution Rand 18 percent of the
weight of solution S. Therefore, 3 grams of solution Rcontain or 0.24(0.08)(3),
gram of liquid A, and 7 grams of solution Scontain or 1.26 grams of(0.18)(7),
liquid A. When the two solutions are mixed, the resulting solution weighs 3 +7,
or 10 grams and contains or 1.5 grams of liquid A. This means that0.24 +1.26,
liquid Amakes up or or 15 percent of the weight of the resulting
1.5 15
,,
10 100
solution. The correct answer is Choice C.
10. Of the 700 members of a certain organization, 120 are lawyers. Two
members of the organization will be selected at random. Which of the
following is closest to the probability that neither of the members selected
will be a lawyer?
A0.5
B0.6
C0.7
D0.8
E0.9
Explanation
The probability that neither of the members selected will be a lawyer is equal to
the fraction
the number of ways 2 members who are not lawyers can be selected
the number of ways 2 members can be selected
where the order of selection does not matter.
Since there are 120 members who are lawyers, there must be or700 120,
580 members who are not lawyers. There are 580 ways of selecting a first
member who is not a lawyer and 579 ways of selecting a second member who
is not a lawyer. Multiplying these two numbers gives the number of ways to
select 2 members who are not lawyers. However, in the ways, each(580)(579)
group of 2 members who are not lawyers is counted twice. You can see this by
considering 2 members, Aand B. The 2 members can be chosen in 2 ways: A
first, followed by B, and Bfirst, followed by A. To adjust for double counting, you
need to divide by 2.(580)(579)
This ebook was issued to HUAN-LIN CHANG, order #14803770223. Unlawful distribution of this ebook is prohibited.
Customer name HUAN-LIN CHANG , Order Id: 14803770223
GRE Quantitative Reasoning Practice Questions
183
Similarly, the number of ways 2 members can be selected from among the
700 members is divided by 2. Thus, the desired probability is(700)(699)
(580)(579)
2 (580)(579)
=
(700)(699) (700)(699)
2
Since the answer choices are all tenths, you need to approximate the value of
this fraction to the nearest tenth. There are several ways to do this approximation.
One way is to use your calculator to convert the fraction to a decimal and round
the decimal to the nearest tenth.
Another way is to approximate the value of the fraction as follows.
2
6
(580)(579) (600)(600) 36 36
===0.72
冢冣
(700)(699) (700)(700) 49 50
7
Either way, the answer choice that is closest to the value of the fraction is 0.7.
The correct answer is Choice C.
Another approach to this problem is to consider the random selections as
two separate but successive events. The probability of selecting a first member
who is not a lawyer is because there are 580 members out of the 700
580,
700
members who are not lawyers. For the second selection, there are only 699
members left to select from, because one member has already been selected. If
the first member selected is not a lawyer, then there are only 579 members left
who are not lawyers. So the probability of selecting a second member who is not
a lawyer, given the condition that the first member selected was not a lawyer, is
The probability that both members selected will not be lawyers is the
579.
699
product of the two probabilities, or which is approximated above as
580 579 ,
冢冣冢冣
700 699
0.72. The correct answer is Choice C.
Numeric Entry
For Questions 11 and 12, use the directions for Numeric Entry questions.
Garden Walkway
11. The figure above represents a rectangular garden with a walkway around it.
The garden is 18 feet long and 12 feet wide. The walkway is uniformly 3
feet wide, and its edges meet at right angles. What is the area of the
walkway?
square feet
This ebook was issued to HUAN-LIN CHANG, order #14803770223. Unlawful distribution of this ebook is prohibited.
Customer name HUAN-LIN CHANG , Order Id: 14803770223
GRE Quantitative Reasoning Practice Questions
184
Explanation
You can see from the figure that the shaded region is the region between the two
rectangles. Looking at the shaded region in this way suggests that the area of the
walkway can be calculated as the difference between the area of the larger
rectangle and the area of the smaller rectangle.
The region represented by the smaller rectangle is the garden. Since the
garden is 18 feet long and 12 feet wide, its area is or 216 square feet.(18)(12),
The region represented by the larger rectangle is the garden and the walkway
combined. The length of the region is the length of the garden plus twice the
width of the walkway, or feet. The width of the region is the18 +(2)(3) =24
width of the garden plus twice the width of the walkway, or feet.12 +(2)(3) =18
Therefore, the area of the region represented by the larger rectangle is (24)(18),
or 432 square feet, and the area of the walkway is or 216 square feet.432 216,
Another way to approach this problem is to think of the walkway as being
composed of four rectangles and four squares, as shown in the figure below.
Garden Walkway
Each of the four squares is 3 feet long and 3 feet wide. The two rectangles
running along the length of the garden are 18 feet long and 3 feet wide, and the
two rectangles running along the width of the garden are 12 feet long and 3 feet
wide. Thus, the area of the walkway is
4(3)(3) +2(18)(3) +2(12)(3) =36 +108 +72 =216 square feet
The correct answer is 216.
12. Line klies in the xy-plane. The x-intercept of line kis and line kpasses4,
through the midpoint of the line segment whose endpoints are and(2, 9)
What is the slope of line k?(2, 0).
Give your answer as a fraction.
Explanation
You can calculate the slope of a line if you know the coordinates of two points
on the line. In this question you are given information about two points on line
k, namely,
the point at which line kcrosses the x-axis has x-coordinate 4;
the midpoint of the line segment with endpoints at and is on(2, 9) (2, 0)
line k.
This ebook was issued to HUAN-LIN CHANG, order #14803770223. Unlawful distribution of this ebook is prohibited.
Customer name HUAN-LIN CHANG , Order Id: 14803770223
GRE Quantitative Reasoning Practice Questions
185
The coordinates of the first point are since the x-coordinate is and(4, 0), 4
the y-coordinate of every point on the x-axis is 0. For the second point, the mid-
point of the line segment is halfway between the endpoints and(2, 9) (2, 0).
Thus, the midpoint has x-coordinate 2 and y-coordinate the number halfway
9,
2
between 9 and 0. Based on the coordinates and the slope of line k
9
(4, 0) 2, ,
冢冣
2
is
99
0
223
==
2(4)64
The correct answer is (or any equivalent fraction).
3
4
Multiple-choice QuestionsSelect One or More Answer Choices
For Questions 13 and 14, select all the answer choices that apply.
13. If the lengths of two sides of a triangle are 5 and 9, respectively, which of the
following could be the length of the third side of the triangle?
Indicate all such lengths.
A3
B5
C8
D15
Explanation
A good way to approach this problem is to think about how much the length of
the third side of a triangle with two fixed side lengths can vary. If you think about
it a bit, you will see that the smaller the interior angle between the two sides of
the triangle is, the smaller the length of the third side is; and the larger the
interior angle between the two sides of the triangle is, the larger the length of the
third side is. This suggests drawing two triangles, one in which the angle between
the two sides is close to 0 degrees and one in which the angle between the two
sides is close to 180 degrees, like the triangles below.
5
5
99
In the triangle in which the angle between the sides of length 5 and 9 is small,
you can see that the length of the third side is a bit greater than or 4. If it9 5,
were equal to 4, the triangle would degenerate into a line segment.
In the triangle in which the angle between the sides of length 5 and 9 is large,
you can see that the length of the third side is a bit less than or 14. If it9 +5,
were equal to 14, the triangle would degenerate into a line segment.
This ebook was issued to HUAN-LIN CHANG, order #14803770223. Unlawful distribution of this ebook is prohibited.
Customer name HUAN-LIN CHANG , Order Id: 14803770223
GRE Quantitative Reasoning Practice Questions
186
Therefore, the length of the third side of the triangle must be greater than 4
and less than 14. Furthermore, it is intuitive that any length between these two
numbers can be achieved by some triangle. The correct answer consists of
Choices B and C.
0
x
y
z
14. On the number line shown above, the tick marks are equally spaced. Which
of the following statements about the numbers x,y, and zmust be true?
Indicate all such statements.
Axyz <0
Bx+z=y
Cz(yx)>0
Explanation
You can see from their positions on the number line that xis less than 0 and both
yand zare greater than 0. Because the tick marks are equally spaced, you can also
see that and You need to evaluate each answer choice separately tox=−yz=2y.
determine whether it must be true.
Choice A says that the product of the three numbers x,y, and zis less than 0.
Recall that the product of three numbers is negative under either of the following
two conditions.
All three numbers are negative.
One of the numbers is negative and the other two numbers are positive.
Choice A must be true, since xis negative and yand zare positive.
Choice B is the equation To see whether the equation must be true,x+z=y.
it is a good idea to express two of the variables in terms of the third (that is, to
“get rid of” two of the variables). The equations and give xand zinx=−yz=2y
terms of y, so the equation can be rewritten, substituting for xandx+z=yy2y
for z, as In this form you can quickly conclude that the equation musty+2y=y.
be true.
Choice C says that the product of the two numbers zand is greater thanyx
0. Recall that the product of two numbers is positive under either of the following
two conditions.
Both numbers are positive.
Both numbers are negative.
Since you already know that zis positive, you can conclude that the product
will be positive if is positive. By adding xto both sides of the in-z(yx)yx
equality you can see that it is equivalent to the inequality which isyx>0, y>x,
clearly true from the number line. Since is positive, the productyxz(yx)
must be positive.
Therefore, the correct answer consists of Choices A, B, and C.
This ebook was issued to HUAN-LIN CHANG, order #14803770223. Unlawful distribution of this ebook is prohibited.
Customer name HUAN-LIN CHANG , Order Id: 14803770223
GRE Quantitative Reasoning Practice Questions
187
SET 3. Discrete Questions: Hard
Quantitative Comparison
For Questions 1 to 6, use the directions for Quantitative Comparison questions.
125°
6
4
BC
A
D
In the figure above, ABCD is a parallelogram.
Quantity A Quantity B
1. The area of ABCD 24 ABCD
Explanation
In this question you are asked to compare the area of a parallelogram with an
area of 24, given two side lengths and the measure of one interior angle of the
parallelogram. Since the measure of the interior angle given is you can125,
conclude that the parallelogram is not a rectangle.
Recall that the area of a parallelogram is found by multiplying the length of a
base by the height corresponding to the base. It is helpful to draw the vertical
height from vertex Cto base AD of the parallelogram, as shown in the figure
below.
125°
6height
4
BC
AD
Note that the newly drawn height is a leg in a newly formed right triangle. The
hypotenuse of the triangle is a side of the parallelogram and has length 6. Thus,
the leg of the triangle, which is the height of the parallelogram, must be less than
the hypotenuse 6. The area of the parallelogram is equal to the length of base AD,
which is 4, times the height, which is less than 6. Since the product of 4 and a
number less than 6 must be less than 24, the area of the parallelogram must be
less than 24. Quantity B is greater than Quantity A, and the correct answer is
Choice B.
This ebook was issued to HUAN-LIN CHANG, order #14803770223. Unlawful distribution of this ebook is prohibited.
Customer name HUAN-LIN CHANG , Order Id: 14803770223
GRE Quantitative Reasoning Practice Questions
188
1–5 6–10 11–15 16–20
Measurement Intervals
Frequency
21–25 2630
30
40
10
0
20
31–35
In the course of an experiment, 95 measurements
were recorded, and all of the measurements were
integers. The 95 measurements were then grouped
into 7 measurement intervals. The graph above shows
the frequency distribution of the 95 measurements by
measurement interval.
Quantity A Quantity B
2. The average (arithmetic
mean) of the 95
measurements
The median of the 95
measurements
ABCD
Explanation
From the histogram, you can observe that
all of the measurement intervals are the same size,
the distribution has a peak at the measurement interval 6–10, and
more of the measurement intervals are to the right of the peak than are to
the left of the peak.
Since in the histogram the 95 measurements have been grouped into intervals,
you cannot calculate the exact value of either the average or the median; you must
compare them without being able to determine the exact value of either one.
The median of the 95 measurements is the middle measurement when the
measurements are listed in increasing order. The middle measurement is the 48th
measurement. From the histogram, you can see that the measurement interval 1–5
contains the first 15 measurements, and the measurement interval 6–10 contains
the next 35 measurements (that is, measurements 16 through 50). Therefore, the
median is in the measurement interval 6–10 and could be 6, 7, 8, 9, or 10.
Estimating the average of the 95 measurements is more complicated.
Since you are asked to compare the average and the median, not necessarily
to calculate them, you may ask yourself if you can tell whether the average is
greater than or less than the median. Note that visually the measurements in the
first three measurement intervals are symmetric around the measurement interval
6–10, so you would expect the average of the measurements in just these three
measurement intervals to lie in the 6–10 measurement interval. The 30 measure-
ments in the remaining four measurement intervals are all greater than 10, some
significantly greater than 10. Therefore, the average of the 95 measurements is
greater than the average of the measurements in the first three measurement in-
This ebook was issued to HUAN-LIN CHANG, order #14803770223. Unlawful distribution of this ebook is prohibited.
Customer name HUAN-LIN CHANG , Order Id: 14803770223
GRE Quantitative Reasoning Practice Questions
189
tervals, probably greater than 10. At this point it seems likely that the average of
the 95 measurements is greater than the median of the 95 measurements. It turns
out that this is true.
To actually show that the average must be greater than 10, you can make the
average as small as possible and see if the smallest possible average is greater than
10. To make the average as small as possible, assume that all of the measurements
in each interval are as small as possible. That is to say, all 15 measurements in the
measurement interval 1–5 are equal to 1, all 35 measurements in the measure-
ment interval 6–10 are equal to 6, etc. Under this assumption, the average of the
95 measurements is
(1)(15) +(6)(35) +(11)(15) +(16)(12) +(21)(10) +(26)(5) +(31)(3) 1,015
=
95 95
The value of the smallest possible average, , is greater than 10.
1,015
95
Therefore, since the average of the 95 measurements is greater than 10 and
the median is in the measurement interval 6–10, it follows that the average is
greater than the median, and the correct answer is Choice A.
xis an integer greater than 1.
Quantity A Quantity B
3.
x+1
3
x
4ABCD
Explanation
One way to approach this question is to plug in numbers for the variables and see
what the relationship between the two quantities is for each of the numbers you
plug in.
If you plug in you see that Quantity A is or 27, and Quantity B
x+13
x=2, 3 =3,
is or 16. In this case, Quantity A is greater than Quantity B.
x2
4=4,
If you plug in you see that Quantity A is or 81, and Quantity B
x+14
x=3, 3 =3,
is or 64. In this case, Quantity A is greater than Quantity B.
x3
4=4,
If you plug in you see that Quantity A is or 243, and Quantity
x+15
x=4, 3 =3,
B is or 256. In this case, Quantity B is greater than Quantity A. Since for
x4
4=4,
and for Quantity A is greater than Quantity B, and for Quantityx=2x=3, x=4,
B is greater than Quantity A, it follows that the relationship between the two
quantities cannot be determined. The correct answer is Choice D.
Since both quantities are algebraic expressions, another way to approach this
problem is to set up a placeholder relationship between the two quantities and
simplify it to see what conclusions you can draw.
x+1 x
3?4
| |
xx
3(3)?4
| |
xx
3(3 ) 4
?
x
| |
x
33
x
4
3?
| |
冢冣
3
For any value of x, the value of is positive, so dividing by does not change
xx
33
any inequality that could be put in the placeholder. Since each step in this
simplification is reversible, this reduces the problem to comparing 3 with
x
4.
冢冣
3
This ebook was issued to HUAN-LIN CHANG, order #14803770223. Unlawful distribution of this ebook is prohibited.
Customer name HUAN-LIN CHANG , Order Id: 14803770223
GRE Quantitative Reasoning Practice Questions
190
You can see that because is greater than 1, the value of becomes greater
x
44
冢冣
33
as xbecomes larger. In particular, it is greater than 3 for large enough values of x.
For the smallest value of x, the relationship is
2
416
x=2, =<3.
冢冣
39
Since for Quantity A is greater than Quantity B and for large values of x,x=2,
Quantity B is greater than Quantity A, it follows that the relationship between the
two quantities cannot be determined. The correct answer is Choice D.
A,B, and Care three rectangles. The length and width
of rectangle Aare 10 percent greater and 10 percent
less, respectively, than the length and width of
rectangle C. The length and width of rectangle Bare
20 percent greater and 20 percent less, respectively,
than the length and width of rectangle C.
Quantity A Quantity B
4. The area of rectangle AThe area of rectangle BABCD
Explanation
In this question you are asked to compare the area of rectangle Aand the area of
rectangle B. Since the information given relates the dimensions of both rectangle
Aand rectangle Bto the corresponding dimensions of rectangle C, you can try to
use the relationships to make the desired comparison.
If represents the length of rectangle Cand wrepresents its width, then the
length and width of rectangles Aand Bcan be translated into algebraic
expressions as follows.
The length of rectangle Ais 10 percent greater than the length of rectangle
C,or1.1.
The width of rectangle Ais 10 percent less than the width of rectangle C,
or 0.9w.
The length of rectangle Bis 20 percent greater than the length of rectangle
C,or1.2.
The width of rectangle Bis 20 percent less than the width of rectangle C,
or 0.8w.
In terms of and w, the area of rectangle Ais or(1.1)(0.9w), 0.99w.
In terms of and w, the area of rectangle Bis or(1.2)(0.8w), 0.96w.
Since is greater than Quantity A is greater than Quantity B,0.99w0.96w,
and the correct answer is Choice A.
The random variable Xis normally distributed. The
values 650 and 850 are at the 60th and 90th
percentiles of the distribution of X, respectively.
Quantity A Quantity B
5. The value at the 75th
percentile of the
distribution of X
750 ABCD
This ebook was issued to HUAN-LIN CHANG, order #14803770223. Unlawful distribution of this ebook is prohibited.
Customer name HUAN-LIN CHANG , Order Id: 14803770223
GRE Quantitative Reasoning Practice Questions
191
Explanation
You are given that the distribution of random variable Xis normal and that the
values 650 and 850 are at the 60th and 90th percentiles of the distribution,
respectively.
Both of the values 650 and 850 are greater than the mean of the distribution.
If you draw a rough sketch of the graph of the normal distribution, the sketch
could look something like the one below. Note that it is not necessary to know the
exact location of 650 and 850, just that both values are above the mean.
650 850
Mean
To say that the value 650 is at the 60th percentile of the distribution means,
graphically, that 60 percent of the area between the normal curve and the
horizontal axis lies to the left of the vertical line segment at 650. To say that 850 is
at the 90th percentile of the distribution means that 90 percent of the area
between the normal curve and the horizontal axis lies to the left of the vertical
line segment at 850.
The value 750 is halfway between 650 and 850. However, because the curve is
decreasing in that interval, the area between 650 and 750 is greater than the area
between 750 and 850. Since the value at the 75th percentile should divide in half
the area between the value at the 60th percentile (650) and the value at the 90th
percentile (850), this value is closer to 650 than to 850. Thus you can conclude
that Quantity A, the value at the 75th percentile of the distribution of X, is less
than Quantity B. The correct answer is Choice B.
Set Sconsists of all positive integers less than 81 that are
not equal to the square of an integer.
Quantity A Quantity B
6. The number of integers in
set S
72 ABCD
Explanation
Set Sconsists of all integers from 1 to 80, except those that are equal to the
square of an integer. So, Quantity A, the number of integers in set S, is equal to
the number of positive integers that are less than 81 minus the number of positive
integers less than 81 that are equal to the square of an integer.
Clearly, there are 80 positive integers that are less than 81.
One way to determine the number of positive integers less than 81 that are
squares of integers is by noticing that 81 is equal to and concluding that the
2
9
squares of the integers from 1 to 8 are all positive integers that are less than 81.
You can also draw this conclusion by squaring each of the positive integers,
beginning with 1, until you get to an integer nsuch that is greater than or
2
n
equal to 81. Either way, there are 8 positive integers less than 81 that are squares
of integers.
This ebook was issued to HUAN-LIN CHANG, order #14803770223. Unlawful distribution of this ebook is prohibited.
Customer name HUAN-LIN CHANG , Order Id: 14803770223
GRE Quantitative Reasoning Practice Questions
192
Therefore, the number of integers in set Sis or 72, which is equal to80 8,
Quantity B. So Quantity A is equal to Quantity B, and the correct answer is
Choice C.
Multiple-choice QuestionsSelect One Answer Choice
For Questions 7 to 12, select a single answer choice.
7. A manager is forming a 6-person team to work on a certain project. From
the 11 candidates available for the team, the manager has already chosen 3
to be on the team. In selecting the other 3 team members, how many
different combinations of 3 of the remaining candidates does the manager
have to choose from?
A6
B24
C56
D120
E462
Explanation
To determine the number of different combinations of 3 of the remaining
candidates that the manager has to choose from, you first have to know the
number of remaining candidates. Since you know that the manager has already
chosen 3 of the 11 candidates to be on the team, it is easy to see that there are 8
remaining candidates. Now you need to count how many different combinations
of 3 objects can be chosen from a group of 8 objects.
If you remember the combinations formula, you know that the number of
combinations is (which is denoted symbolically as or ). You can
8
8! C
83
冢冣
3
3!(8 3)!
then calculate the number of different combinations of 3 of the remaining
candidates as follows.
8! (8)(7)(6)(5!) (8)(7)(6)
===56
3!(8 3)! (3!)(5!) 6
The correct answer is Choice C.
8. Which of the following could be the graph of all values of xthat satisfy the
inequality 6x5
25x?
3
A0
B0
C0
D0
E0
This ebook was issued to HUAN-LIN CHANG, order #14803770223. Unlawful distribution of this ebook is prohibited.
Customer name HUAN-LIN CHANG , Order Id: 14803770223
GRE Quantitative Reasoning Practice Questions
193
Explanation
To determine which of the graphs is the correct answer, you first need to
determine all values of xthat satisfy the inequality. To do that you need to simplify
the inequality until you isolate x.
You can begin by multiplying both sides of the inequality by 3 to obtain
Note that when you multiply by 3, the right-hand side of(3)(2 5x)(6x5).
the inequality becomes not
(6x5), 6x5.
The rest of the simplification is as follows.
(3)(2 5x)6x+5
615x6x+5
15x6x1
9x1
1
x9
Note that when an inequality is multiplied (or divided) by a negative number, the
direction of the inequality reverses.
The graphs in the answer choices are number lines on which only the number
0 is indicated. Therefore, you do not need to locate on the number line; it is
1
9
enough to know that is a positive number. Choice C is the only choice in which
1
9
the shaded part of the line is equal to or greater than a positive number.
Therefore, the correct answer is Choice C.
9. If then the average (arithmetic mean) of x, and
23 23 4
1+x+x+x=60, x,x,x
is equal to which of the following?
A12x
B15x
C20x
D30x
E60x
Explanation
A quick inspection of the answer choices shows that it is not necessary to solve
the equation for xto answer this question. You are being
23
1+x+x+x=60
asked to express the average of the four quantities x, and in terms of x.
23 4
x,x,x
To express this average in terms of x, you need to add the 4 quantities and divide
the result by 4; that is,
234
x+x+x+x.
4
The only information given in the question is that the sum of the 4 quantities,
is 60, so you need to think of a way to use this information to
23
1+x+x+x,
simplify the expression
234
x+x+x+x.
4
Note that the numerator of the fraction is a sum of 4 quantities, each of which
has an xterm raised to a power. Thus, the expression in the numerator can be
This ebook was issued to HUAN-LIN CHANG, order #14803770223. Unlawful distribution of this ebook is prohibited.
Customer name HUAN-LIN CHANG , Order Id: 14803770223
GRE Quantitative Reasoning Practice Questions
194
factored as By using the information in the
234 23
x+x+x+x=x(1 +x+x+x).
question, you can make the following simplification.
234 23
x+x+x+xx(1 +x+x+x)x(60)
===15x
444
Therefore, the correct answer is Choice B.
Ox
y
P
R
Q
10. Parallelogram OPQR lies in the xy-plane, as shown in the figure above. The
coordinates of point Pare and the coordinates of point Qare(2, 4) (8, 6).
What are the coordinates of point R?
A(3, 2)
B(3, 3)
C(4, 4)
D(5, 2)
E(6, 2)
Explanation
Ox
y
R
N
M
2
2
6
6
P (2, 4)
Q (8, 6)
Since OPQR is a parallelogram, line segments PQ and OR have the same
length and the same slope. Therefore, in the figure above, PQM and ORN are
congruent right triangles. From the coordinates of Pand Q, the lengths of the legs
This ebook was issued to HUAN-LIN CHANG, order #14803770223. Unlawful distribution of this ebook is prohibited.
Customer name HUAN-LIN CHANG , Order Id: 14803770223
GRE Quantitative Reasoning Practice Questions
195
of triangle PQM are and Thus, the lengths of thePM =82=6QM =64=2.
legs ON and RN of triangle ORN are also 6 and 2, respectively. So the coordinates
of point Rare The correct answer is Choice E.(6, 2).
11. The relationship between the area Aof a circle and its circumference Cis
given by the formula where kis a constant. What is the value of k?
2
A=kC ,
A1
4p
B1
2p
C1
4
D2p
E
2
4p
Explanation
One way to approach this problem is to realize that the value of the constant kis
the same for all circles. Therefore, you can pick a specific circle and substitute the
circumference and the area of that particular circle into the formula and calculate
the value of k.
Say, for example, that you pick a circle with radius 1. The area of the circle is
and the circumference of the circle is . Inserting these values into the formulap2p
gives Solving this equation for kgives , and the correct answer is
1
2
p=k(2p). k=4p
Choice A.
Another way to approach the problem is to express Aand Cin terms of a
common variable and then solve the resulting equation for k. Recall the
commonly used formulas for the area and the circumference of a circle:
2
A=pr
and Note that in these formulas, both Aand Care expressed in terms ofC=2pr.
the radius r. So, in the formula you can substitute expressions for Aand
2
A=kC ,
Cin terms of r.
Substituting for Aand for Cgives
222
pr2prpr=k(2pr).
Now you can determine the value of kby solving the equation for kas follows.
22
pr=k(2pr)
222
pr=k(4pr)
2
p=k(4p)
1=k
4p
The correct answer is Choice A.
This ebook was issued to HUAN-LIN CHANG, order #14803770223. Unlawful distribution of this ebook is prohibited.
Customer name HUAN-LIN CHANG , Order Id: 14803770223
GRE Quantitative Reasoning Practice Questions
196
12. The sequence of numbers is defined by for
11
a,a,a,...,a,... a=−
123 nn
nn+2
each integer What is the sum of the first 20 terms of this sequence?n1.
A11
1+−
冢冣
220
B111
1+− +
冢冣冢 冣
22122
C11
1−+
冢冣
20 22
D1
122
E11
20 22
Explanation
This question asks for the sum of the first 20 terms of the sequence. Obviously, it
would be very time-consuming to write out the first 20 terms of the sequence and
add them together, so it is reasonable to try to find a more efficient way to
calculate the sum. Questions involving sequences can often be answered by
looking for a pattern. Scanning the answer choices and noting that they contain
fractions with denominators 2, 20, 21, and 22, and nothing in between, seems to
confirm that looking for a pattern is a good approach to try.
To look for a pattern, begin by adding the first two terms of the sequence.
11 11 11 11
−+−=+−+
冢冣冢冣冢冣冢冣
13 24 12 34
Now, if you add the first three terms of the sequence, you get
11 11 11
−+−+−
冢冣冢冣冢冣
13 24 35
Note that you can simplify the sum by canceling the fraction that is, the sum of
1;
3
positive and negative is 0.
11
33
11 11 11 11 11
+−+−=+−+
冢冣冢冣冢冣冢冣冢冣
13 24 35 12 45
If you add the first four terms, you get
11 11 11 11
−+−+−+−
冢冣冢冣冢冣冢冣
13 24 35 46
Again, you can simplify the sum by canceling. This time, you can cancel the
fractions and
11
.
34
11 11 11 11 11 11
+−+−+−=+−+
冢冣冢冣冢冣冢冣冢冣冢冣
13 24 35 46 12 56
This ebook was issued to HUAN-LIN CHANG, order #14803770223. Unlawful distribution of this ebook is prohibited.
Customer name HUAN-LIN CHANG , Order Id: 14803770223
GRE Quantitative Reasoning Practice Questions
197
If you write out the next two sums and simplify them, you will see that they are
11 11 11 11
+−+ and +−+
冢冣冢冣冢冣冢冣
12 67 12 78
Working with the sums makes it clear that this pattern continues to hold as you
add more and more terms of the sequence together and that a formula for the
sum of the first kterms of the sequence is
11 1 1
+− +
冢冣冢 冣
12 k+1k+2
Therefore, the sum of the first 20 terms of the sequence is equal to
11 1 1 1 1 1
+− + =1+− +
冢冣 冢冣冢 冣
12 20+120+2 2 21 22
The correct answer is Choice B.
Numeric Entry
For Question 13, use the directions for Numeric Entry questions.
YFrequency
1
22
3
47
5
48
3
28
7
49
13. The table above shows the frequency distribution of the values of a variable
Y. What is the mean of the distribution?
Give your answer to the nearest 0.01.
Explanation
The mean of distribution of the variable Yis the sum of all the values of Ydivided
by the number of values of Y. However, before you begin the summing process,
you need to understand how the information is presented in the question.
Information about the variable is given in a table, where any repetitions of values
have been summarized in the column labeled “Frequency.” Reading from the
This ebook was issued to HUAN-LIN CHANG, order #14803770223. Unlawful distribution of this ebook is prohibited.
Customer name HUAN-LIN CHANG , Order Id: 14803770223
GRE Quantitative Reasoning Practice Questions
198
table, you can see that the value occurs twice, the value occurs seven times,
13
24
and so on. To sum all the values of Y, you could add the value twice, add the
1
2
value seven times, and continue the addition process in this manner. It is easier,
3
4
however, to multiply the values by their corresponding frequencies and then sum
the individual products, as shown below.
13537421404863
(2) +(7) +(8) +(8) +(9) =++++
冢冣 冢冣 冢冣 冢冣 冢冣
2442444444
176
=4
=44
To find the average, you need to divide the sum, 44, by the number of values of Y.
The number of values can be found by looking at the column of frequencies in the
table. The sum of the numbers in this column, or 34, is the2 +7+8+8+9,
number of values of Y. Thus, the mean of the distribution is which, as a
44,
34
decimal, equals 1.2941.. . . Rounded to the nearest 0.01, the correct answer is 1.29.
Multiple-choice QuestionsSelect One or More Answer Choices
For Questions 14 and 15, select all the answer choices that apply.
14. Let Sbe the set of all positive integers nsuch that is a multiple of both
2
n
24 and 108. Which of the following integers are divisors of every integer n
in S?
Indicate all such integers.
A12
B24
C36
D72
Explanation
To determine which of the integers in the answer choices is a divisor of every
positive integer nin S, you must first understand the integers that are in S. Note
that in this question you are given information about not about nitself.
2
n,
Therefore, you must use the information about to derive information about n.
2
n
The fact that is a multiple of both 24 and 108 implies that is a multiple
2 2
nn
of the least common multiple of 24 and 108. To determine the least common
multiple of 24 and 108, factor 24 and 108 into prime factors as and
3
(2 )(3)
respectively. Because these are prime factorizations, you can conclude
23
(2 )(3 ),
that the least common multiple of 24 and 108 is
33
(2 )(3 ).
Knowing that must be a multiple of does not mean that every
233
n(2 )(3 )
multiple of is a possible value of because must be the square of an
33 2 2
(2 )(3 ) n,n
This ebook was issued to HUAN-LIN CHANG, order #14803770223. Unlawful distribution of this ebook is prohibited.
Customer name HUAN-LIN CHANG , Order Id: 14803770223
GRE Quantitative Reasoning Practice Questions
199
integer. The prime factorization of a square number must contain only even
exponents. Thus, the least multiple of that is a square is This is
33 44
(2 )(3 ) (2 )(3 ).
the least possible value of and so the least possible value of nis or
222
n, (2 )(3 ),
36. Furthermore, since every value of is a multiple of the values of n
244
n(2 )(3 ),
are the positive multiples of 36; that is, S=36, 72, 108, 144, 180,. .. .
{}
The question asks for integers that are divisors of every integer nin S, that
is, divisors of every positive multiple of 36. Since Choice A, 12, is a divisor of 36,
it is also a divisor of every multiple of 36. The same is true for Choice C, 36.
Choices B and D, 24 and 72, are not divisors of 36, so they are not divisors of
every integer in S. The correct answer consists of Choices A and C.
15. The range of the heights of the female students in a certain class is 13.2
inches, and the range of the heights of the male students in the class is 15.4
inches.
Which of the following statements individually provide(s) sufficient
additional information to determine the range of the heights of all the
students in the class?
Indicate all such statements.
AThe tallest male student in the class is 5.8 inches taller than the tallest
female student in the class.
BThe median height of the male students in the class is 1.1 inches
greater than the median height of the female students in the class.
CThe average (arithmetic mean) height of the male students in the class
is 4.6 inches greater than the average height of the female students in
the class.
Explanation
Choice A tells you that the tallest male student is 5.8 inches taller than the tallest
female student. You can combine this information with the given information
about the male and female height ranges to place four studentsthe shortest
male, the shortest female, the tallest male, and the tallest femalein relative
order according to height, as shown in the figure below.
Shortest
female
Shortest
male
Tallest
female
Tallest
male
5.8
13.2
15.4
You can see from the figure that the tallest student must be a male and the
shortest student must be a female. You can also see the difference in height
between those two students, which is the range of the heights of the entire class.
Therefore, Choice A provides sufficient additional information to determine the
range.
Choice B provides information about one of the centers of the datathe
median; it does not say anything about how spread out the data are around that
center. You are given that the median height of the males is 1.1 inches greater
than that of the females. First note that it is possible for two different sets of
This ebook was issued to HUAN-LIN CHANG, order #14803770223. Unlawful distribution of this ebook is prohibited.
Customer name HUAN-LIN CHANG , Order Id: 14803770223
GRE Quantitative Reasoning Practice Questions
200
data to have the same median but have very different ranges. Choice B gives the
difference between the medians of the male heights and the female heights,
without giving the actual medians. However, even if you knew the medians, the
fact that the ranges can vary widely indicates that the range of the heights of the
entire class can also vary widely.
It is possible to construct examples of heights of students that satisfy all of
the information in the question and in Choice B but have different ranges for the
heights of the entire class. Here are two such examples, each of which has only
three females and three males. Although the examples are small, they illustrate
the fact that the range of the heights of the entire class can vary. In both
examples, the range of female heights is 13.2, the range of male heights is 15.4,
and the difference between the median heights is 1.1 inches.
Example 1
Female heights: 50.0 56.6 63.2 which have a median of 56.6
Male heights: 50.0 57.7 65.4 which have a median of 57.7
Range of heights of entire class: 15.4
Example 2
Female heights: 50.0 56.6 63.2 which have a median of 56.6
Male heights: 51.0 57.7 66.4 which have a median of 57.7
Range of heights of entire class: 16.4
Therefore, Choice B does not provide sufficient additional information to
determine the range of the heights of the entire class.
Choice C provides information about another center of the datathe
average. You are given that the average height of the males is 4.6 inches greater
than that of the females. However, like Choice B, the statement gives no
information about how spread out the data are around that center. Again, it is
possible for two different sets of data to have the same average but have very
different ranges. Examples similar to the two examples above can be constructed
that satisfy all of the information in the question and in Choice C but have
different ranges for the heights of the entire class. Therefore, Choice C does not
provide sufficient additional information to determine the range of the heights of
the entire class.
The correct answer consists of Choice A.
This ebook was issued to HUAN-LIN CHANG, order #14803770223. Unlawful distribution of this ebook is prohibited.
Customer name HUAN-LIN CHANG , Order Id: 14803770223
GRE Quantitative Reasoning Practice Questions
201
SET 4. Data Interpretation Sets
For Questions 1 to 7, select a single answer choice unless otherwise directed.
Questions 1 to 3 are based on the following data.
Biological Sciences
Business
Education
Engineering
Fine Arts
Health Sciences
Humanities
Physical Sciences
Social Sciences
Other
0510
Percent
PERCENT OF FEMALE FACULTY AND PERCENT OF MALE FACULTY
AT UNIVERSITY X, BY FIELD
Total female faculty: 200
Total male faculty: 250
Field
15 20 25
Female
Male
Medium Question
1. There are 275 students in the field of engineering at University X.
Approximately what is the ratio of the number of students in engineering to
the number of faculty in engineering?
A8to1
B10 to 1
C12 to 1
D14 to 1
E20 to 1
Explanation
According to the graph, 2 percent of the female faculty and 12 percent of the
male faculty are in the engineering field. To determine the total number of
faculty members in engineering, you need to add 2 percent of 200, which is 4, to
12 percent of 250, which is 30, to get 34. Thus, the ratio of the numbers of
students to faculty in engineering is 275 to 34, which is approximately equal to
280 to 35, or 8 to 1. The correct answer is Choice A.
This ebook was issued to HUAN-LIN CHANG, order #14803770223. Unlawful distribution of this ebook is prohibited.
Customer name HUAN-LIN CHANG , Order Id: 14803770223
GRE Quantitative Reasoning Practice Questions
202
Medium Question
2. Approximately what percent of the faculty in humanities are male?
A35%
B38%
C41%
D45%
E51%
Explanation
You need to determine the numbers of female and male faculty in the
humanities field. According to the graph, 17 percent of the 200 females, or 34,
and 14 percent of the 250 males, or 35, are in humanities. Thus, the fraction of
humanities faculty who are male is or approximately 0.507. As a
35 35
=,
34 +35 69
percent, the answer choice that is closest to 0.507 is 51 percent. The correct
answer is Choice E.
For Question 3, use the directions for Numeric Entry questions.
Hard Question
3. For the biological sciences and health sciences faculty combined, of the
1
3
female and of the male faculty members are tenured professors. What
2
9
fraction of all the faculty members in those two fields combined are tenured
professors?
Explanation
You need to determine the number of female faculty and the number of male
faculty in the combined group. According to the graph, 5 percent of the female
faculty, or 10, and 10 percent of the male faculty, or 25, are in the biological
sciences. Similarly, 16 percent of the female faculty, or 32, and 8 percent of the
male faculty, or 20, are in the health sciences. When you combine the groups,
you get a total of 42 females ( ) and 45 males ( ), which is a total of10 +32 25 +20
87 faculty. The tenured faculty are of the 42 females, or 14 females, and of
12
39
the 45 males, or 10 males. Thus, there are 24 tenured faculty, and the fraction
that are tenured professors is . The correct answer is (or any equivalent
24 24
87 87
fraction).
This ebook was issued to HUAN-LIN CHANG, order #14803770223. Unlawful distribution of this ebook is prohibited.
Customer name HUAN-LIN CHANG , Order Id: 14803770223
GRE Quantitative Reasoning Practice Questions
203
Questions 4 to 7 are based on the following data.
2000 2001 2002 2003 2004
Year
VALUE OF IMPORTS TO AND EXPORTS FROM COUNTRY T, 20002009
(in United States dollars)
2005 2006 2007 2008 2009
16
14
12
10
8
6
4
2
0
Exports
Imports
Billions* of Dollars
*1 billion = 1,000,000,000
For Question 4, select all the answer choices that apply.
Easy Question
4. For which of the eight years from 2001 to 2008 did exports exceed imports
by more than $5 billion?
Indicate all such years.
A2001
B2002
C2003
D2004
E2005
F2006
G2007
H2008
Explanation
Note that for all years shown, the dollar value of exports is greater than the
dollar value of imports. For each year, the difference between the dollar value of
exports and the dollar value of imports can be read directly from the graph. The
difference was more than $5 billion for each of the years 2001, 2002, 2003, 2006,
2007, and 2008. The correct answer consists of Choices A, B, C, F, G, and H.
This ebook was issued to HUAN-LIN CHANG, order #14803770223. Unlawful distribution of this ebook is prohibited.
Customer name HUAN-LIN CHANG , Order Id: 14803770223
GRE Quantitative Reasoning Practice Questions
204
Medium Question
5. Which of the following is closest to the average (arithmetic mean) of the 9
changes in the value of imports between consecutive years from 2000 to
2009 ?
A$260 million
B$320 million
C$400 million
D$480 million
E$640 million
Explanation
The average of the 9 changes in the value of imports between consecutive years
can be represented as follows, where the function v(year) represents the value of
imports for the indicated year.
...
(v(2001) v(2000)) +(v(2002) v(2001)) +(v(2003) v(2002)) ++(v(2009) v(2008))
9
Note that in the numerator of the fraction, each term, with the exception of
v(2000) and v(2009), appears first as positive and then again as negative. The
positive and negative pairs sum to 0, and the fraction simplifies to
.
v(2009) v(2000)
9
Reading the values from the graph, you can approximate the value of the
simplified fraction as billion dollars. The answer choice
9.0 3.2 5.8
=0.644
99
that is closest to $0.644 billion is $640 million. The correct answer is Choice E.
Medium Question
6. In 2008 the value of exports was approximately what percent greater than
the value of imports?
A40%
B60%
C70%
D120%
E140%
Explanation
The difference between the value of exports and the value of imports expressed
as a percent of the value of imports is
(value of exports) (value of imports) (100%)
冢冣
value of imports
In 2008 the value of imports was approximately $5 billion and the value of
exports was approximately $12 billion, so the value of the fraction is
approximately , or .
12 57
55
This ebook was issued to HUAN-LIN CHANG, order #14803770223. Unlawful distribution of this ebook is prohibited.
Customer name HUAN-LIN CHANG , Order Id: 14803770223
GRE Quantitative Reasoning Practice Questions
205
Since the fraction is greater than 1, expressing it as a percent will give a
percent greater than 100. The fraction is equal to 1.4, or 140 percent. The correct
answer is Choice E.
Hard Question
7. If it were discovered that the value of imports shown for 2007 was incorrect
and should have been $5 billion instead, then the average (arithmetic mean)
value of imports per year for the 10 years shown would have been
approximately how much less?
A$200 million
B$50 million
C$20 million
D$7 million
E$5 million
Explanation
To answer this question, you do not need to compute either of the two 10-year
averages referred to in the question; you just need to calculate the difference
between the two averages.
The average value of imports for the 10 years shown in the graph is found by
adding the 10 values and then dividing the sum by 10. The value of imports in
2007 is $7 billion. If that amount were $5 billion instead, then the sum of
the values would be $2 billion less. If the sum were $2 billion less than what
it was, then the average would decrease by 2 billion divided by 10, or
. The average would therefore be $200 million less,
2,000,000,000 =200,000,000
10
and the correct answer is Choice A.
A more algebraic approach to the problem is to let Srepresent the sum, in
billions, of the 10 values of imports in the graph. The average of the 10 values is
. Note that S2 represents the sum, in billions, of the 10 values adjusted for
S
10
the $2 billion correction for 2007. The average of the adjusted sum is . The
S2
10
difference between the two averages is
SS2S(S2)
−=
10 10 10
SS+2
=10
2
=10
The difference is 0.2 billion dollars, or $200 million. The correct answer is
Choice A.
This ebook was issued to HUAN-LIN CHANG, order #14803770223. Unlawful distribution of this ebook is prohibited.
Customer name HUAN-LIN CHANG , Order Id: 14803770223
This page intentionally left blank
This ebook was issued to HUAN-LIN CHANG, order #14803770223. Unlawful distribution of this ebook is prohibited.
Customer name HUAN-LIN CHANG , Order Id: 14803770223
GRE
®
Math Review
gReview the math topics likely to appear on the
GRE
®revised General Test
gStudy examples with worked-out solutions
gTest your skills with practice exercises
Your goal
for this
chapter
207
7
This Math Review will familiarize you with the mathematical skills and con-
cepts that are important to understand in order to solve problems and to rea-
son quantitatively on the Quantitative Reasoning measure of the GRE revised
General Test. The following material includes many definitions, properties, and exam-
ples, as well as a set of exercises (with answers) at the end of each review section.
Note, however, that this review is not intended to be all-inclusive there may be some
concepts on the test that are not explicitly presented in this review. Also, if any topics
in this review seem especially unfamiliar or are covered too briefly, we encourage you
to consult appropriate mathematics texts for a more detailed treatment.
The Math Review covers the following topics:
1. Arithmetic 3. Geometry
1.1 Integers 3.1 Lines and Angles
1.2 Fractions 3.2 Polygons
1.3 Exponents and Roots 3.3 Triangles
1.4 Decimals 3.4 Quadrilaterals
1.5 Real Numbers 3.5 Circles
1.6 Ratio 3.6 Three-dimensional Figures
1.7 Percent
2. Algebra 4. Data Analysis
2.1 Operations with Algebraic 4.1 Graphical Methods for
Expressions Describing Data
2.2 Rules of Exponents 4.2 Numerical Methods for
2.3 Solving Linear Equations Describing Data
2.4 Solving Quadratic Equations 4.3 Counting Methods
2.5 Solving Linear Inequalities 4.4 Probability
2.6 Functions 4.5 Distributions of Data, Random
2.7 Applications Variables, and Probability
2.8 Coordinate Geometry Distributions
2.9 Graphs of Functions 4.6 Data Interpretation Examples
This ebook was issued to HUAN-LIN CHANG, order #14803770223. Unlawful distribution of this ebook is prohibited.
Customer name HUAN-LIN CHANG , Order Id: 14803770223
GRE Math Review
208
1. ARITHMETIC
The review of arithmetic begins with integers, fractions, and decimals and progresses
to the set of real numbers. The basic arithmetic operations of addition, subtraction,
multiplication, and division are discussed, along with exponents and roots. The sec-
tion ends with the concepts of ratio and percent.
1.1 Integers
The integers are the numbers 1, 2, 3, and so on, together with their negatives, 1,
2, 3,. . ., and 0. Thus, the set of integers is {...., 3, 2, 1, 0, 1, 2, 3,. ..}.
The positive integers are greater than 0, the negative integers are less than 0, and
0 is neither positive nor negative. When integers are added, subtracted, or multiplied,
the result is always an integer; division of integers is addressed below. The many ele-
mentary number facts for these operations, such as 7 +8=15, 78 87 =−9,
and should be familiar to you; they are not reviewed7 (18) =25, (7)(8) =56,
here. Here are some general facts regarding multiplication of integers.
The product of two positive integers is a positive integer.
The product of two negative integers is a positive integer.
The product of a positive integer and a negative integer is a negative integer.
When integers are multiplied, each of the multiplied integers is called a factor or
divisor of the resulting product. For example, so 2, 3, and 10 are(2)(3)(10) =60,
factors of 60. The integers 4, 15, 5, and 12 are also factors of 60, since (4)(15) =60
and The positive factors of 60 are 1, 2, 3, 4, 5, 6, 10, 12, 15, 20, 30,(5)(12) =60.
and 60. The negatives of these integers are also factors of 60, since, for example,
(2)(30) =60. There are no other factors of 60. We say that 60 is a multiple of
each of its factors and that 60 is divisible by each of its divisors. Here are some
more examples of factors and multiples.
The positive factors of 100 are 1, 2, 4, 5, 10, 20, 25, 50, and 100.
25 is a multiple of only six integers: 1, 5, 25, and their negatives.
The list of positive multiples of 25 has no end: 25, 50, 75, 100, 125, 150, etc.;
likewise, every nonzero integer has infinitely many multiples.
1 is a factor of every integer; 1 is not a multiple of any integer except 1 and 1.
0 is a multiple of every integer; 0 is not a factor of any integer except 0.
The least common multiple of two nonzero integers aand bis the least positive
integer that is a multiple of both aand b. For example, the least common multiple
of 30 and 75 is 150. This is because the positive multiples of 30 are 30, 60, 90, 120,
150, 180, 210, 240, 270, 300, etc., and the positive multiples of 75 are 75, 150, 225,
300, 375, 450, etc. Thus, the common positive multiples of 30 and 75 are 150, 300,
450, etc., and the least of these is 150.
The greatest common divisor (or greatest common factor) of two nonzero in-
tegers aand bis the greatest positive integer that is a divisor of both aand b. For
example, the greatest common divisor of 30 and 75 is 15. This is because the posi-
tive divisors of 30 are 1, 2, 3, 5, 6, 10, 15, and 30, and the positive divisors of 75 are
1, 3, 5, 15, 25, and 75. Thus, the common positive divisors of 30 and 75 are 1, 3, 5,
and 15, and the greatest of these is 15.
When an integer ais divided by an integer b, where bis a divisor of a, the result
is always a divisor of a. For example, when 60 is divided by 6 (one of its divisors), the
result is 10, which is another divisor of 60. If bis not a divisor of a, then the result
This ebook was issued to HUAN-LIN CHANG, order #14803770223. Unlawful distribution of this ebook is prohibited.
Customer name HUAN-LIN CHANG , Order Id: 14803770223
GRE Math Review
209
can be viewed in three different ways. The result can be viewed as a fraction or as a
decimal, both of which are discussed later, or the result can be viewed as a quotient
with a remainder, where both are integers. Each view is useful, depending on the
context. Fractions and decimals are useful when the result must be viewed as a sin-
gle number, while quotients with remainders are useful for describing the result in
terms of integers only.
Regarding quotients with remainders, consider two positive integers aand bfor
which bis not a divisor of a; for example, the integers 19 and 7. When 19 is divided
by 7, the result is greater than 2, since (2)(7) <19, but less than 3, since 19 <(3)(7).
Because 19 is 5 more than we say that the result of 19 divided by 7 is the(2)(7),
quotient 2 with remainder 5, or simply “2 remainder 5.” In general, when a positive
integer ais divided by a positive integer b, you first find the greatest multiple of b
that is less than or equal to a. That multiple of bcan be expressed as the product qb,
where qis the quotient. Then the remainder is equal to aminus that multiple of b,or
where ris the remainder. The remainder is always greater than or equalr=aqb,
to 0 and less than b.
Here are examples that illustrate a few different cases of division resulting in a
quotient and remainder.
100 divided by 45 is 2 remainder 10, since the greatest multiple of 45 that’s less
than or equal to 100 is or 90, which is 10 less than 100.(2)(45),
24 divided by 4 is 6 remainder 0, since the greatest multiple of 4 that’s less than
or equal to 24 is 24 itself, which is 0 less than 24. In general, the remainder is 0
if and only if ais divisible by b.
6 divided by 24 is 0 remainder 6, since the greatest multiple of 24 that’s less than
or equal to 6 is or 0, which is 6 less than 6.(0)(24),
Here are some other examples.
100 divided by 3 is 33 remainder 1, since 100 =(33)(3) +1.
100 divided by 25 is 4 remainder 0, since 100 =(4)(25) +0.
80 divided by 100 is 0 remainder 80, since 80 =(0)(100) +80.
When you divide 100 by 2, the remainder is 0.
When you divide 99 by 2, the remainder is 1.
If an integer is divisible by 2, it is called an even integer; otherwise it is an odd
integer. Note that when a positive odd integer is divided by 2, the remainder is al-
ways 1. The set of even integers is and the set of odd...,6, 4, 2, 0, 2, 4, 6, .. . ,
{}
integers is There are several useful facts regarding the...,5, 3, 1, 1, 3, 5, .. . .
{}
sum and product of even and odd integers.
The sum of two even integers is an even integer.
The sum of two odd integers is an even integer.
The sum of an even integer and an odd integer is an odd integer.
The product of two even integers is an even integer.
The product of two odd integers is an odd integer.
The product of an even integer and an odd integer is an even integer.
Aprime number is an integer greater than 1 that has only two positive divisors:
1 and itself. The first ten prime numbers are 2, 3, 5, 7, 11, 13, 17, 19, 23, and 29.
The integer 14 is not a prime number, since it has four positive divisors: 1, 2, 7, and
14. The integer 1 is not a prime number, and the integer 2 is the only prime number
that is even.
This ebook was issued to HUAN-LIN CHANG, order #14803770223. Unlawful distribution of this ebook is prohibited.
Customer name HUAN-LIN CHANG , Order Id: 14803770223
GRE Math Review
210
Every integer greater than 1 either is a prime number or can be uniquely ex-
pressed as a product of factors that are prime numbers, or prime divisors. Such an
expression is called a prime factorization. Here are several examples of prime fac-
torizations.
2
12 =(2)(2)(3) =(2 )(3)
14 =(2)(7)
4
81 =(3)(3)(3)(3) =3
2
338 =(2)(13)(13) =(2)(13 )
52
800 =(2)(2)(2)(2)(2)(5)(5) =(2 )(5 )
1,155 =(3)(5)(7)(11)
An integer greater than 1 that is not a prime number is called a composite
number. The first ten composite numbers are 4, 6, 8, 9, 10, 12, 14, 15, 16, and 18.
1.2 Fractions
Afraction is a number of the form where aand bare integers and The
a,b0.
b
integer ais called the numerator of the fraction, and bis called the denominator.
For example, is a fraction in which is the numerator and 5 is the denomina-
77
5
tor. Such numbers are also called rational numbers.
If both the numerator aand denominator bare multiplied by the same nonzero
integer, the resulting fraction will be equivalent to For example,
a.
b
7(7)(4) 28
==
5 (5)(4) 20
7(7)(1) 7
==
5 (5)(1) 5
A fraction with a negative sign in either the numerator or denominator can be
written with the negative sign in front of the fraction; for example, 77 7
==.
555
If both the numerator and denominator have a common factor, then the numera-
tor and denominator can be factored and reduced to an equivalent fraction. For ex-
ample,
40 (8)(5) 5
==
72 (8)(9) 9
To add two fractions with the same denominator, you add the numerators and
keep the same denominator. For example,
858+533
−+ = = =
11 11 11 11 11
To add two fractions with different denominators, first find a common denomi-
nator, which is a common multiple of the two denominators. Then convert both
fractions to equivalent fractions with the same denominator. Finally, add the numer-
ators and keep the common denominator. For example, to add the fractions and
1
3
use the common denominator 15:
2
,
5
1215 23 5 65+(6) 1
+= + =+= =
冢冣冢冣 冢 冢冣
3 5 3 5 5 3 15 15 15 15
This ebook was issued to HUAN-LIN CHANG, order #14803770223. Unlawful distribution of this ebook is prohibited.
Customer name HUAN-LIN CHANG , Order Id: 14803770223
GRE Math Review
211
The same method applies to subtraction of fractions.
To multiply two fractions, multiply the two numerators and multiply the two de-
nominators. For example,
10 1 (10)(1) 10 10
===
冢冣
7 3 (7)(3) 21 21
87 56
=
冢冣冢冣
33 9
To divide one fraction by another, first invert the second fractionthat is, find
its reciprocalthen multiply the first fraction by the inverted fraction. For example,
17 3 17 4 4 17 1 17 17
====
冢冣冢冣冢冣
8 4 83 83 23 6
3
10 3 13 39
==
冢冣冢冣
710770
13
An expression such as is called a mixed number. It consists of an integer
3
48
part and a fraction part; the mixed number means To convert a mixed
33
44+.
88
number to an ordinary fraction, convert the integer part to an equivalent fraction
and add it to the fraction part. For example,
3 3 48 332335
4=4+= += +=
冢冣冢冣
8 8 18 8888
Note that numbers of the form where either aor bis not an integer and
a,
b
are fractional expressions that can be manipulated just like fractions. For ex-
b0,
ample, the numbers and can be added together as follows.
pp
23
pp p3p23p2p5p
+=+=+=
冢冣冢冣 冢冣冢冣
2 3 23 32 6 6 6
And the number can be simplified as follows.
1
2
3
5
=
1
55
冪冪
1
2
=
33
冢冣冢冣
232
冪冪
5
1.3 Exponents and Roots
Exponents are used to denote the repeated multiplication of a number by itself; for
example, and In the expression 3 is
43 4
3=(3)(3)(3)(3) =81 5 =(5)(5)(5) =125. 3 ,
called the base, 4 is called the exponent, and we read the expression as “3 to the
fourth power.” So 5 to the third power is 125. When the exponent is 2, we call the
process squaring. Thus, 6 squared is 36, and 7 squared is 49,
2
6=(6)(6) =36,
2
7=(7)(7) =49.
This ebook was issued to HUAN-LIN CHANG, order #14803770223. Unlawful distribution of this ebook is prohibited.
Customer name HUAN-LIN CHANG , Order Id: 14803770223
GRE Math Review
212
When negative numbers are raised to powers, the result may be positive or nega-
tive. For example, (3)
2
=(3)(3) =9, while (3)
5
=(3)(3)(3)(3)(3) =−243.
A negative number raised to an even power is always positive, and a negative num-
ber raised to an odd power is always negative. Note that without the parentheses,
the expression means “the negative of ‘3 squared’ ”; that is, the exponent is ap-
2
3
plied before the negative sign. So but
22
(3) =9, 3=−9.
Exponents can also be negative or zero; such exponents are defined as follows.
For all nonzero numbers a, The expression is undefined.
00
a=1. 0
For all nonzero numbers a, etc. Note that
111
123
a=,a=,a=,
23
aa a
1
1
(a)(a)=(a)=1.
冢冣
a
Asquare root of a nonnegative number nis a number rsuch that For
2
r=n.
example, 4 is a square root of 16 because Another square root of 16 is
2
4=16. 4,
since All positive numbers have two square roots, one positive and one
2
(4) =16.
negative. The only square root of 0 is 0. The symbol is used to denote the nonneg-
n
ative square root of the nonnegative number n. Therefore, 100 =10, 100 =−10,
冪冪
and Square roots of negative numbers are not defined in the real number0 =0.
system.
Here are some important rules regarding operations with square roots, where
anda>0b>0.
Rule Examples
2
a=a
冢冣
2
a=a
ab=ab
冪冪 冪
a
a
=b
b
2
3=3
冢冣
4=2
310= 30
冪冪 冪
5
51
==
冪冪
15 3
15
2
p=p
冢冣
2
p=p
24 = 4 6 =26
冪冪冪 冪
18
18
==9=3
2
2
A square root is a root of order 2. Higher-order roots of a positive number nare
defined similarly. For orders 3 and 4, the cube root and fourth root
34
nn
冪冪
represent numbers such that when they are raised to the powers 3 and 4,
respectively, the result is n. These roots obey rules similar to those above (but with
the exponent 2 replaced by 3 or 4 in the first two rules). There are some notable
differences between odd-order roots and even-order roots (in the real number
system):
For odd-order roots, there is exactly one root for every number n, even when nis
negative.
For even-order roots, there are exactly two roots for every positive number nand
no roots for any negative number n.
For example, 8 has exactly one cube root, but 8 has two fourth roots: and
34
8=2, 8
冪冪
; and has exactly one cube root, but has no fourth root, since
43
888=−2, 8
冪冪
it is negative.
This ebook was issued to HUAN-LIN CHANG, order #14803770223. Unlawful distribution of this ebook is prohibited.
Customer name HUAN-LIN CHANG , Order Id: 14803770223
GRE Math Review
213
1.4 Decimals
The decimal number system is based on representing numbers using powers of 10.
The place value of each digit corresponds to a power of 10. For example, the digits
of the number 7,532.418 have the following place values.
Thousands
Hundreds
Tens
Ones or Units
Tenths
Hundredths
Thousandths
That is,
11 1
7,532.418 =7(1,000) +5(100) +3(10) +2(1) +4+1+8
冢冣 冢 冣
10 100 1,000
3210123
=7(10 ) +5(10 ) +3(10 ) +2(10 ) +4(10 ) +1(10 ) +8(10 )
If there are a finite number of digits to the right of the decimal point, converting
a decimal to an equivalent fraction with integers in the numerator and denominator
is a straightforward process. Since each place value is a power of 10, every decimal
can be converted to an integer divided by a power of 10. For example,
323
2.3 =2+=
10 10
17 9,000 +17 9,017
90.17 =90 += =
100 100 100
612 153
0.612 ==
1,000 250
Conversely, every fraction with integers in the numerator and denominator can
be converted to an equivalent decimal by dividing the numerator by the denominator
using long division (which is not in this review). The decimal that results from the
long division will either terminate, as in and or the decimal will
152
=0.25 =2.08,
425
repeat without end, as in and One
11 25
=0.111..., =0.0454545..., =2.08333....
922 12
way to indicate the repeating part of a decimal that repeats without end is to use a
bar over the digits that repeat. Here are some examples of fractions converted to
decimals.
3=0.375
8
259 19
=6+=6.475
40 40
1
−=0.3
3
15 =1.0714285
14
Every fraction with integers in the numerator and denominator is equivalent to a
decimal that terminates or repeats. That is, every rational number can be expressed
This ebook was issued to HUAN-LIN CHANG, order #14803770223. Unlawful distribution of this ebook is prohibited.
Customer name HUAN-LIN CHANG , Order Id: 14803770223
GRE Math Review
214
as a terminating or repeating decimal. The converse is also true; that is, every termi-
nating or repeating decimal represents a rational number.
Not all decimals are terminating or repeating; for instance, the decimal that is
equivalent to is and it can be shown that this decimal does not
2 1.41421356237.. .,
terminate or repeat. Another example is which has0.010110111011110111110... ,
groups of consecutive 1’s separated by a 0, where the number of 1’s in each succes-
sive group increases by one. Since these two decimals do not terminate or repeat,
they are not rational numbers. Such numbers are called irrational numbers.
1.5 Real Numbers
The set of real numbers consists of all rational numbers and all irrational numbers.
The real numbers include all integers, fractions, and decimals. The set of real num-
bers can be represented by a number line called the real number line.
123
0
–1–2–3
53
2
1
222.6
0.4
Every real number corresponds to a point on the number line, and every point
on the number line corresponds to a real number. On the number line, all numbers
to the left of 0 are negative and all numbers to the right of 0 are positive. Only the
number 0 is neither negative nor positive.
A real number xis less than a real number yif xis to the left of yon the num-
ber line, which is written as A real number yis greater than xif yis to thex<y.
right of xon the number line, which is written as For example,y>x.
52
10
2
122
To say that a real number xis between 2 and 3 on the number line means that
and which can also be written as the double inequality The setx>2x<3, 2 <x<3.
of all real numbers that are between 2 and 3 is called an interval, and the double
inequality is often used to represent that interval. Note that the endpoints2 <x<3
of the interval, 2 and 3, are not included in the interval. Sometimes one or both of
the endpoints are to be included in an interval. The following inequalities represent
four types of intervals, depending on whether the endpoints are included.
2<x<3
2x<3
2<x3
2x3
There are also four types of intervals with only one endpoint, each of which con-
sists of all real numbers to the right or to the left of the endpoint, perhaps including
the endpoint. The following inequalities represent these types of intervals.
x<4
x4
x>4
x4
This ebook was issued to HUAN-LIN CHANG, order #14803770223. Unlawful distribution of this ebook is prohibited.
Customer name HUAN-LIN CHANG , Order Id: 14803770223
GRE Math Review
215
The entire real number line is also considered to be an interval.
The distance between a number xand 0 on the number line is called the abso-
lute value of x, written as Therefore, and because each of thex.3=33=3
numbers 3 and is a distance of 3 from 0. Note that if xis positive, then ;3x=x
if xis negative, then ; and lastly, It follows that the absolute valuex=−x0=0.
of any nonzero number is positive. Here are some examples.
5=5
冪冪
23=−(23) =23
10.2=10.2
There are several general properties of real numbers that are used frequently. If
a,b, and care real numbers, then
anda+b=b+aab=ba.
For example, and8 +2=2+8=10 (3)(17) =(17)(3) =−51.
and(a+b)+c=a+(b+c)(ab)c=a(bc).
For example, and
(7 +3) +8=7+(3 +8) =18 (72)2=7(22)=(7)(2) =14.
冪冪 冪
a(b+c)=ab +ac
For example, 5(3 +16) =(5)(3) +(5)(16) =95.
anda+0=a,(a)(0) =0, (a)(1) =a.
If then or , or both.ab =0, a=0b=0
For example, if then2b=0, b=0.
Division by 0 is not defined; for example, and are undefined.
70
50, ,
00
If both aand bare positive, then both and ab are positive.a+b
If both aand bare negative, then is negative and ab is positive.a+b
If ais positive and bis negative, then ab is negative.
This is known as the triangle inequality.a+b⎪≤⎪a+b.
For example, if and then anda=5b=−2, 5+(2)=52=3=3
Therefore,5+2=5+2=7. 5+(2)⎪≤⎪5+2.
For example,a⎪⎪ b=ab.5⎪⎪ 2=(5)(2)=10=10.
If then If then
22
a>1, a>a.0<b<1, b<b.
For example, but
2
111
2
5=25 >5, =<.
冢冣
5255
1.6 Ratio
The ratio of one quantity to another is a way to express their relative sizes, often in
the form of a fraction, where the first quantity is the numerator and the second
quantity is the denominator. Thus, if sand tare positive quantities, then the ratio of
sto tcan be written as the fraction The notation “sto t” or “ ” is also used to
s.s:t
t
express this ratio. For example, if there are 2 apples and 3 oranges in a basket, we
can say that the ratio of the number of apples to the number of oranges is or that
2
3
it is 2 to 3 or that it is Like fractions, ratios can be reduced to lowest terms. For2:3.
example, if there are 8 apples and 12 oranges in a basket, then the ratio of the num-
bers of apples to oranges is still 2 to 3. Similarly, the ratio 9 to 12 is equivalent to
the ratio 3 to 4.
If three or more positive quantities are being considered, say r,s, and t, then
their relative sizes can also be expressed as a ratio with the notation “rto sto t.” For
This ebook was issued to HUAN-LIN CHANG, order #14803770223. Unlawful distribution of this ebook is prohibited.
Customer name HUAN-LIN CHANG , Order Id: 14803770223
GRE Math Review
216
example, if there are 5 apples, 30 pears, and 20 oranges in a basket, then the ratio of
the numbers of apples to pears to oranges is 5 to 30 to 20. This ratio can be reduced
to 1 to 6 to 4 by dividing each number by the greatest common divisor of 5, 30, and
20, which is 5.
Aproportion is an equation relating two ratios; for example, To solve a
93
=.
12 4
problem involving ratios, you can often write a proportion and solve it by cross
multiplication.
Example 1.6.1: To find a number xso that the ratio of xto 49 is the same as the
ratio of 3 to 21, you can write
x3
=
49 21
Then cross multiply to get and solve for xto get (3)(49)
21x=(3)(49), x==7.
21
1.7 Percent
The term percent means per hundred,orhundredths. Percents are ratios that are of-
ten used to represent parts of a whole, where the whole is considered as having 100
parts.
1 percent means 1 part out of 100 parts, or 1.
100
32 percent means 32 parts out of 100 parts, or 32 .
100
50 percent means 50 parts out of 100 parts, or 1.
2
Note that the part is the numerator of the ratio and the whole is the denominator.
Percents are often written with the %symbol; fractional and decimal equivalents are
often used as well but without the %symbol, as follows.
1
1%==0.01
100
100
100%==1
100
32
32%==0.32
100
50
50%==0.5
100
0.3
0.3%==0.003
100
Be careful not to confuse 0.01 with 0.01%. The percent symbol matters. For exam-
ple, but 0.01
0.01 =1%0.01%==0.0001.
100
To compute a percent, given the part and the whole, divide the part by the whole.
The result will be the decimal equivalent, so multiply the result by 100 to convert
to percent.
This ebook was issued to HUAN-LIN CHANG, order #14803770223. Unlawful distribution of this ebook is prohibited.
Customer name HUAN-LIN CHANG , Order Id: 14803770223
GRE Math Review
217
Example 1.7.1: If the whole is 20 and the part is 13, you can find the percent as
follows.
part 13
==0.65 =65%
whole 20
Example 1.7.2: What percent of 150 is 12.9?
Solution: Here the whole is 150 and the part is 12.9.
part 12.9
==0.086 =8.6%
whole 150
To find the part that is a certain percent of a whole, you can either multiply the
whole by the decimal equivalent of the percent or set up a proportion to find the
part.
Example 1.7.3: To find 30%of 350, multiply 350 by the decimal equivalent of 30%,
or 0.3, as follows.
x=(350)(0.3) =105
To use a proportion, you need to find the number of parts of 350 that yields the
same ratio as 30 out of 100 parts. You want a number xthat satisfies the propor-
tion
part 30
=
whole 100
x30
=
350 100
Solving for xyields so 30%of 350 is 105.
(30)(350)
x==105,
100
Given the percent and the part, you can calculate the whole. To do this you can
either use the decimal equivalent of the percent or you can set up a proportion
and solve it.
Example 1.7.4:15is60%of what number?
Solution: Use the decimal equivalent of 60%. Because 60%of some number zis 15,
multiply zby the decimal equivalent of 60%, or 0.6.
0.6z=15
Now solve for zby dividing both sides of the equation by 0.6 as follows.
15
z==25
0.6
Using a proportion, look for a number zsuch that
part 60
=
whole 100
15 60
=
z100
Hence, and therefore, That is, 15 is
(15)(100) 1, 500
60z=(15)(100), z===25.
60 60
60%of 25.
This ebook was issued to HUAN-LIN CHANG, order #14803770223. Unlawful distribution of this ebook is prohibited.
Customer name HUAN-LIN CHANG , Order Id: 14803770223
GRE Math Review
218
Although the discussion about percent so far assumes a context of a part and a
whole, it is not necessary that the part be less than the whole. In general, the whole
is called the base of the percent. When the numerator of a percent is greater than
the base, the percent is greater than 100%. For example, 15 is 300%of 5, since
15 300
=
5 100
and 250%of 16 is Note that the decimal equivalent of
250 (16) =(2.5)(16) =40.
冢冣
100
250%is 2.5.
It is also not necessary for the part to be related to the whole at all, as in the
question, “a teacher’s salary is what percent of a banker’s salary?”
When a quantity changes from an initial positive amount to another positive
amount, for example, an employee’s salary that is raised, you can compute the
amount of change as a percent of the initial amount. This is called percent change.
If a quantity increases from 600 to 750, then the percent increase is found by divid-
ing the amount of increase, 150, by the base, 600, which is the initial number given:
amount of increase 750 600 150 25
====0.25 =25%
base 600 600 100
We say the percent increase is 25%. Sometimes this computation is written as
750 600 150
(100%)=(100%)=25%
冢冣冢
600 600
If a quantity doubles in size, then the percent increase is 100%. For example, if a
quantity changes from 150 to 300, then the percent increase is
change 300 150 150
===100%
base 150 150
If a quantity decreases from 500 to 400, calculate the percent decrease as follows.
change 500 400 100 20
====0.20 =20%
base 500 500 100
The quantity decreased by 20%.
When computing a percent increase, the base is the smaller number. When com-
puting a percent decrease, the base is the larger number. In either case, the base is
the initial number, before the change.
Example 1.7.5: An investment in a mutual fund increased by 12%in a single day. If
the value of the investment before the increase was $1,300, what was the value af-
ter the increase?
Solution: The percent increase is 12%. Therefore, the value of the increase is 12%
of $1,300, or, using the decimal equivalent, the increase is (0.12)($1,300) =$156.
Thus, the value of the investment after the change is
$1,300 +$156 =$1,456
Because the final result is the sum of the initial investment100%of $1,300and
the increase12%of $1,300the final result is of $1,300.100%+12%=112%
This ebook was issued to HUAN-LIN CHANG, order #14803770223. Unlawful distribution of this ebook is prohibited.
Customer name HUAN-LIN CHANG , Order Id: 14803770223
GRE Math Review
219
Thus, another way to get the final result is to multiply the value of the investment
by the decimal equivalent of 112%, which is 1.12:
($1,300)(1.12) =$1,456
A quantity may have several successive percent changes. The base of each successive
percent change is the result of the preceding percent change.
Example 1.7.6: The monthly enrollment at a preschool decreased by 8%during one
month and increased by 6%during the next month. What was the cumulative per-
cent change for the two months?
Solution:IfEis the enrollment before the first month, then the enrollment as a
result of the 8%decrease can be found by multiplying the base Eby the decimal
equivalent of which is 0.92:100%8%=92%,
0.92E
The enrollment as a result of the second percent changethe 6%increasecan be
found by multiplying the new base 0.92Eby the decimal equivalent of
which is 1.06:100%+6%=106%,
(1.06)(0.92)E=0.9752E
The percent equivalent of 0.9752 is 97.52%, which is 2.48%less than 100%. Thus,
the cumulative percent change in the enrollment for the two months is a 2.48%de-
crease.
ARITHMETIC EXERCISES
1. Evaluate the following.
(a) 15 (6 4)(2)
(b) (2 17) 5
(c) (60 12) (7+4)
(d)
43
(3) (2)
(e) (5)(3) 15
(f)
44
(2) (15 18)
(g)
23
(20 5) (2+6)
(h) (85)(0) (17)(3)
2. Evaluate the following.
(a) 11 1
−+
2312
(b) 312
+
冢冣
475
(c)
2
74
冢冣
85
(d) 327
冢冣冢
832
3. Which of the integers 312, 98, 112, and 144 are divisible by 8?
4. (a) What is the prime factorization of 372?
(b) What are the positive divisors of 372?
5. (a) What are the prime divisors of 100?
(b) What are the prime divisors of 144?
6. Which of the integers 2, 9, 19, 29, 30, 37, 45, 49, 51, 83, 90, and 91 are
prime numbers?
7. What is the prime factorization of 585?
This ebook was issued to HUAN-LIN CHANG, order #14803770223. Unlawful distribution of this ebook is prohibited.
Customer name HUAN-LIN CHANG , Order Id: 14803770223
GRE Math Review
220
8. Which of the following statements are true?
(a) 5<3.1
(b) 16 =4
(c) 7 0=0
(d) 1
0<−
⎪⎪
7
(e) 1
0.3 <3
(f)
87
(1) =−1
(g)
2
(3) <0
(h) 21 3
=
28 4
(i) 23=23
(j) 11
>
217
(k)
32 6
(59) (59) =59
(l) 25 <−4
9. Find the following.
(a) 40%of 15
(b) 150%of 48
(c) 0.6%of 800
(d) 15 is 30%of which number?
(e) 11 is what percent of 55?
10. If a person’s salary increased from $200 per week to $234 per week, what
was the percent increase in the person’s salary?
11. If an athlete’s weight decreased from 160 pounds to 152 pounds, what was
the percent decrease in the athlete’s weight?
12. A particular stock is valued at $40 per share. If the value increases by 20
percent and then decreases by 25 percent, what will be the value of the
stock per share after the decrease?
13. If the ratio of the number of men to the number of women on a committee
of 20 members is 3 to 2, how many members of the committee are women?
14. The integer ais even and the integer bis odd. For each of the following
integers, indicate whether the integer is even or odd.
Integer Even Odd
a+2b
2a+b
ab
b
a
2
(a+b)
22
ab
15. When the positive integer nis divided by 3, the remainder is 2 and when n
is divided by 5, the remainder is 1. What is the least possible value of n?
ANSWERS TO ARITHMETIC EXERCISES
1. (a) 19
(b) 3
(c) 8
(d) 89
(e) 0
(f) 1,296
(g) 1,024
(h) 51
This ebook was issued to HUAN-LIN CHANG, order #14803770223. Unlawful distribution of this ebook is prohibited.
Customer name HUAN-LIN CHANG , Order Id: 14803770223
GRE Math Review
221
2. (a) 1
4
(b) 5
14
(c) 9
1, 600
(d) 4
9
3. 312, 112, and 144
4. (a)
2
372 =(2 )(3)(31)
(b) The positive divisors of 372 are 1, 2, 3, 4, 6, 12, 31, 62, 93, 124, 186, and
372.
5. (a) so the prime divisors are 2 and 5.
22
100 =(2 )(5 ),
(b) so the prime divisors are 2 and 3.
42
144 =(2 )(3 ),
6. 2, 19, 29, 37, and 83
7.
2
585 =(3 )(5)(13)
8. (a) True
(b) True
(c) False; division by 0 is undefined
(d) True
(e) True
(f) True
(g) False;
2
(3) =9=3>0
冪冪
(h) True
(i) False; 23=−23
(j) True
(k) False;
32 3+25
(59) (59) =59 =59
(l) True
9. (a) 6
(b) 72
(c) 4.8
(d) 50
(e) 20%
10. 17%
11. 5%
12. $36 per share
13. 8 women
14.
Integer Even Odd
a+2b
2a+b
ab
b
a
2
(a+b)
22
ab
15. 11
This ebook was issued to HUAN-LIN CHANG, order #14803770223. Unlawful distribution of this ebook is prohibited.
Customer name HUAN-LIN CHANG , Order Id: 14803770223
GRE Math Review
222
2. ALGEBRA
Basic algebra can be viewed as an extension of arithmetic. The main concept that
distinguishes algebra from arithmetic is that of a variable, which is a letter that rep-
resents a quantity whose value is unknown. The letters xand yare often used as
variables, although any letter can be used. Variables enable you to present a word
problem in terms of unknown quantities by using algebraic expressions, equations,
inequalities, and functions. This section reviews these algebraic tools and then pro-
gresses to several examples of applying them to solve real-life word problems. The
section ends with coordinate geometry and graphs of functions as other important
algebraic tools for solving problems.
2.1 Operations with Algebraic Expressions
An algebraic expression has one or more variables and can be written as a single
term or as a sum of terms. Here are some examples of algebraic expressions.
18
322
2xywz+5zz+6
4n+p
In the examples above, is a single term, has two terms,
1
2xy4
has four terms, and has one term. In the expression
8
322
wz+5zz+6n+p
the terms and are called like terms because they have
322 2 2
wz+5zz+6, 5zz
the same variables, and the corresponding variables have the same exponents. A
term that has no variable is called a constant term. A number that is multiplied by
variables is called the coefficient of a term. For example, in the expression
2 is the coefficient of the term 7 is the coefficient of the term
22
2x+7x5, 2x,7x,
and is a constant term.5
The same rules that govern operations with numbers apply to operations with
algebraic expressions. One additional rule, which helps in simplifying algebraic ex-
pressions, is that like terms can be combined by simply adding their coefficients, as
the following examples show.
2x+5x=7x
322 32
wz+5zz+6=wz+4z+6
3xy +2xxy 3x=2xy x
A number or variable that is a factor of each term in an algebraic expression can
be factored out, as the following examples show.
4x+12 =4(x+3)
2
15y9y=3y(5y3)
2
7x+14x7x(x+2) 7x
==(where x2, since division by 0 is not defined)
2x+42(x+2) 2
To multiply two algebraic expressions, each term of the first expression is multi-
plied by each term of the second expression, and the results are added, as the follow-
ing examples show.
This ebook was issued to HUAN-LIN CHANG, order #14803770223. Unlawful distribution of this ebook is prohibited.
Customer name HUAN-LIN CHANG , Order Id: 14803770223
GRE Math Review
223
(x+2)(3x7) =x(3x)+x(7) +2(3x)+2(7)
2
=3x7x+6x14
2
=3xx14
A statement of equality between two algebraic expressions that is true for all
possible values of the variables involved is called an identity. All of the preceding
equality statements in this section are identities. Here are some standard identities
that are useful.
22 2
(a+b)=a+2ab +b
33 2 23
(ab)=a3ab+3ab b
22
ab=(a+b)(ab)
All of the identities above can be used to modify and simplify algebraic expres-
sions. For example, the identity can be used to simplify the
22
ab=(a+b)(ab)
following algebraic expression.
2
x9(x+3)(x3) x+3
==(where x3)
4x12 4(x3) 4
A statement of equality between two algebraic expressions that is true for only
certain values of the variables involved is called an equation. The values are called
the solutions of the equation.
The following are examples of some basic types of equations.
3x+5=−2Alinear equation in one variable, x
x3y=10 A linear equation in two variables, xand y
2
20y+6y17 =0Aquadratic equation in one variable, y
2.2 Rules of Exponents
In the algebraic expression where xis raised to the power a,xis called the base
a
x,
and ais called the exponent. For some equations involving bases and exponents, the
following property is very useful: if x
a
=x
b
, then a=b. This is true for all positive
numbers x, except x=1, and for all integers aand b. For example, if 2
y
=64, then
since 64 is 2
6
, you have 2
y
=2
6
, and you can conclude that y=6.
Here are the basic rules of exponents, where the bases xand yare nonzero real
numbers and the exponents aand bare integers.
1. 1
a
x=
a
x
Examples: and
11 1 1
310 a
4== ,x=,=2
310
a
464 x2
2.
ab a+b
(x)(x)=x
Examples: and
24 2+46 312
(3 )(3 ) =3=3=729 ( y)( y)=y
3.
a
x1
ab
=x=
bb
a
xx
Examples: and
73
5t1
743 5
=5=5=125 =t=
485
5tt
4.
0
x=1
Examples: and Note that is not defined.
00 0
7=1(3) =1. 0
5.
aa a
(x)( y)=(xy)
Examples: and
33 3 3 33 3
(2 )(3 ) =6=216 (10z)=10 z=1,000z
This ebook was issued to HUAN-LIN CHANG, order #14803770223. Unlawful distribution of this ebook is prohibited.
Customer name HUAN-LIN CHANG , Order Id: 14803770223
GRE Math Review
224
6.
aa
xx
=
a
冢冣
yy
Examples: and
23
23
339 rr
== =
23
冢冣 冢 冣
4 4 16 4t64t
7.
ab ab
(x)=x
Examples: and
52 10 62 2 62 12
(2 ) =2=1,024 (3y)=(3 )( y)=9y
The rules above are identities that are used to simplify expressions. Sometimes
algebraic expressions look like they can be simplified in similar ways, but in fact
they cannot. Here are several pairs of expressions that are commonly mistaken to be
identities.
ab a
+
b
xy (xy)
Note that the bases are not the same.
ab a b
(x)xx
Instead, and for example, and
ab ab a b a
+
b23 6 2 3 5
(x)=xxx=x;(4)=444=4.
aa a
(x+y)x+y
Recall that ; that is, the correct expansion contains
22 2
(x+y)=x+2xy +y
terms such as 2xy.
22
(x)x
Instead, Note carefully where each minus sign appears.
22
(x)=x.
22
x+yx+y
aaa
+
x+yx y
But it is true that x+yxy
=+.
aaa
2.3 Solving Linear Equations
To solve an equation means to find the values of the variables that make the equation
true, that is, the values that satisfy the equation. Two equations that have the same
solutions are called equivalent equations. For example, andx+1=22x+2=4
are equivalent equations; both are true when and are false otherwise. The gen-x=1
eral method for solving an equation is to find successively simpler equivalent equa-
tions so that the simplest equivalent equation makes the solutions obvious.
The following rules are important for producing equivalent equations.
When the same constant is added to or subtracted from both sides of an
equation, the equality is preserved and the new equation is equivalent to the
original equation.
When both sides of an equation are multiplied or divided by the same nonzero
constant, the equality is preserved and the new equation is equivalent to the
original equation.
Alinear equation is an equation involving one or more variables in which each
term in the equation is either a constant term or a variable multiplied by a coefficient.
None of the variables are multiplied together or raised to a power greater than 1. For
example, and are linear equations, but and
2
2x+1=7x10x9yz=3x+y=0
are not.xz =3
This ebook was issued to HUAN-LIN CHANG, order #14803770223. Unlawful distribution of this ebook is prohibited.
Customer name HUAN-LIN CHANG , Order Id: 14803770223
GRE Math Review
225
Linear Equations in One Variable
To solve a linear equation in one variable, simplify each side of the equation by com-
bining like terms. Then use the rules for producing simpler equivalent equations.
Example 2.3.1:
11x48x=2(x+4) 2x
3x4=2x+82x(like terms combined)
3x4=8 (simplified)
3x4+4=8+4 (4 added to both sides)
3x=12
3x12
=(both sides divided by 3)
33
x=4
You can always check your solution by substituting it into the original equation.
Note that it is possible for a linear equation to have no solutions. For example, the
equation has no solution, since it is equivalent to the equation2x+3=2(7 +x)
which is false. Also, it is possible that what looks to be a linear equation turns3 =14,
out to be an identity when you try to solve it. For example, is true3x6=−3(2 x)
for all values of x, so it is an identity.
Linear Equations in Two Variables
A linear equation in two variables, xand y, can be written in the form
ax +by =c
where a,b, and care real numbers and aand bare not both zero. For example,
is a linear equation in two variables.3x+2y=8
A solution of such an equation is an ordered pair of numbers that makes(x,y)
the equation true when the values of xand yare substituted into the equation. For
example, both and are solutions of the equation but is
2
(2, 1) (,5) 3x+2y=8, (1, 2)
3
not a solution. A linear equation in two variables has infinitely many solutions. If an-
other linear equation in the same variables is given, it is usually possible to find a
unique solution of both equations. Two equations with the same variables are called a
system of equations, and the equations in the system are called simultaneous equa-
tions. To solve a system of two equations means to find an ordered pair of numbers
that satisfies both equations in the system.
There are two basic methods for solving systems of linear equations, by substitu-
tion or by elimination. In the substitution method, one equation is manipulated to
express one variable in terms of the other. Then the expression is substituted in the
other equation. For example, to solve the system of equations
4x+3y=13
x+2y=2
you can express xin the second equation in terms of yas Then substitutex=22y.
for xin the first equation to find the value of y.2 2y
This ebook was issued to HUAN-LIN CHANG, order #14803770223. Unlawful distribution of this ebook is prohibited.
Customer name HUAN-LIN CHANG , Order Id: 14803770223
GRE Math Review
226
4(2 2y)+3y=13
88y+3y=13
8y+3y=5 (8 subtracted from both sides)
5y=5 (like terms combined)
y=−1 (both sides divided by 5)
Then can be substituted for yin either equation to find the value of x. We use the1
second equation:
x+2y=2
x+2(1) =2
x2=2
x=4 (2 added to both sides)
In the elimination method, the object is to make the coefficients of one variable
the same in both equations so that one variable can be eliminated either by adding
the equations together or by subtracting one from the other. In the example above,
multiplying both sides of the second equation by 4 yields or4(x+2y)=4(2),
Now you have two equations with the same coefficient of x.4x+8y=8.
4x+3y=13
4x+8y=8
If you subtract the second equation from the first, the result is Thus,5y=5.
and substituting for yin either of the original equations yieldsy=−1, 1x=4.
By either method, the solution of the system is and orx=4y=−1,
(x,y)=(4, 1).
2.4 Solving Quadratic Equations
Aquadratic equation in the variable xis an equation that can be written in the form
2
ax +bx +c=0
where a,b, and care real numbers and When such an equation has solutions,a0.
they can be found using the quadratic formula:
2
bb4ac
x=2a
where the notation is shorthand for indicating two solutionsone that uses the
plus sign and the other that uses the minus sign.
Example 2.4.1: In the quadratic equation , we have and
2
2xx6=0a=2, b=−1,
Therefore, the quadratic formula yieldsc=−6.
2
(1) (1) 4(2)(6)
x=2(2)
149
=4
17
=4
Hence the two solutions are and
1+7173
x==2x==.
442
This ebook was issued to HUAN-LIN CHANG, order #14803770223. Unlawful distribution of this ebook is prohibited.
Customer name HUAN-LIN CHANG , Order Id: 14803770223
GRE Math Review
227
Quadratic equations have at most two real solutions, as in the example above.
However, some quadratic equations have only one real solution. For example, the qua-
dratic equation has only one solution, which is In this case, the
2
x+4x+4=0x=−2.
expression under the square root symbol in the quadratic formula is equal to 0, and
so adding or subtracting 0 yields the same result. Other quadratic equations have no
real solutions; for example, In this case, the expression under the
2
x+x+5=0.
square root symbol is negative, so the entire expression is not a real number.
Some quadratic equations can be solved more quickly by factoring. For example,
the quadratic equation in example 2.4.1 can be factored as
2
2xx6=0
When a product is equal to 0, at least one of the factors must be(2x+3)(x2) =0.
equal to 0, which leads to two cases: either or Therefore,2x+3=0x2=0.
2x+3=0
x2=0
2x=−3OR
x=2
3
x=−
2
and the solutions are and 2.
3
2
Example 2.4.2: Here is another example of a quadratic equation that can be easily
factored.
2
5x+3x2=0
(5x2)(x+1) =0
Therefore,
5x2=0x+1=0
OR
2x=−1
x=5
2.5 Solving Linear Inequalities
A mathematical statement that uses one of the following inequality signs is called an
inequality.
less than
greater than
less than or equal to
greater than or equal to
Inequalities can involve variables and are similar to equations, except that the two
sides are related by one of the inequality signs instead of the equality sign used in
equations. For example, the inequality is a linear inequality in one variable,4x17
which states that “ is less than or equal to 7.” To solve an inequality means to4x1
find the set of all values of the variable that make the inequality true. This set of val-
ues is also known as the solution set of an inequality. Two inequalities that have the
same solution set are called equivalent inequalities.
The procedure used to solve a linear inequality is similar to that used to solve a
linear equation, which is to simplify the inequality by isolating the variable on one
side of the inequality, using the following two rules.
This ebook was issued to HUAN-LIN CHANG, order #14803770223. Unlawful distribution of this ebook is prohibited.
Customer name HUAN-LIN CHANG , Order Id: 14803770223
GRE Math Review
228
When the same constant is added to or subtracted from both sides of an
inequality, the direction of the inequality is preserved and the new inequality
is equivalent to the original.
When both sides of the inequality are multiplied or divided by the same
nonzero constant, the direction of the inequality is preserved if the constant is
positive but the direction is reversed if the constant is negative. In either case,
the new inequality is equivalent to the original.
Example 2.5.1: The inequality can be solved as follows.3x+517
3x+517
3x12 (5 subtracted from both sides)
3x12
33(both sides divided by 3, which reverses
the direction of the inequality)
x4
Therefore, the solution set of consists of all real numbers greater than3x+517
or equal to 4.
Example 2.5.2:
4x+95
11
4x+955 (both sides multiplied by 11)
4x46 (9 subtracted from both sides)
46
x(both sides divided by 4)
4
x11.5
Therefore, the solution set of consists of all real numbers less than 11.5.
4x+9<5
11
2.6 Functions
An algebraic expression in one variable can be used to define a function of that vari-
able. Functions are usually denoted by letters such as f,g, and h. For example, the
algebraic expression can be used to define a function fby3x+5
f(x)=3x+5
where is called the value of fat xand is obtained by substituting the value of xinf(x)
the expression above. For example, if is substituted in the expression above, thex=1
result is f(1) =8.
It might be helpful to think of a function fas a machine that takes an input,
which is a value of the variable x, and produces the corresponding output, Forf(x).
any function, each input xgives exactly one output However, more than onef(x).
value of xcan give the same output For example, if gis the function defined byf(x).
then and
2
g(x)=x2x+3, g(0) =3g(2) =3.
The domain of a function is the set of all permissible inputs, that is, all permissi-
ble values of the variable x. For the functions fand gdefined above, the domain is the
set of all real numbers. Sometimes the domain of the function is given explicitly and
This ebook was issued to HUAN-LIN CHANG, order #14803770223. Unlawful distribution of this ebook is prohibited.
Customer name HUAN-LIN CHANG , Order Id: 14803770223
GRE Math Review
229
is restricted to a specific set of values of x. For example, we can define the function h
by for Without an explicit restriction, the domain is assumed
2
h(x)=x42x2.
to be the set of all values of xfor which is a real number.f(x)
Example 2.6.1: Let fbe the function defined by In this case, fis not de-
2x
f(x)=.
x6
fined at because is not defined. Hence, the domain of fconsists of all real
12
x=60
numbers except for 6.
Example 2.6.2: Let gbe the function defined by In this case,
3
g(x)=x+x+210.
is not a real number if Hence, the domain of gconsists of all real num-g(x)x<−2.
bers xsuch that x2.
Example 2.6.3: Let hbe the function defined by the absolute value of x,h(x)=x,
which is the distance between xand 0 on the number line (see section 1.5). The do-
main of his the set of all real numbers. Also, for all real numbers x,h(x)=h(x)
which reflects the property that on the number line the distance between xand 0 is
the same as the distance between and 0.x
2.7 Applications
Translating verbal descriptions into algebraic expressions is an essential initial step in
solving word problems. Some examples are given below.
If the square of the number xis multiplied by 3, and then 10 is added to that
product, the result can be represented by
2
3x+10.
If John’s present salary sis increased by 14 percent, then his new salary is
1.14s.
If ygallons of syrup are to be distributed among 5 people so that one
particular person gets 1 gallon and the rest of the syrup is divided equally
among the remaining 4, then each of those 4 people will get gallons of
y1
4
syrup.
Here are several examples of using algebraic techniques to solve word problems.
Example 2.7.1: Ellen has received the following scores on 3 exams: 82, 74, and 90.
What score will Ellen need to receive on the next exam so that the average (arithme-
tic mean) score for the 4 exams will be 85?
Solution: Let xrepresent the score on Ellen’s next exam. This initial step of assigning
a variable to the quantity that is sought is an important beginning to solving the
problem. Then in terms of x, the average of the 4 exams is
82 +74 +90 +x
4
which is supposed to equal 85. Now simplify the expression and set it equal to 85:
82 +74 +90 +x246 +x
==85
44
This ebook was issued to HUAN-LIN CHANG, order #14803770223. Unlawful distribution of this ebook is prohibited.
Customer name HUAN-LIN CHANG , Order Id: 14803770223
GRE Math Review
230
Solving the resulting linear equation for x, you get
246 +x=340
x=94
Therefore, Ellen will need to attain a score of 94 on the next exam.
Example 2.7.2: A mixture of 12 ounces of vinegar and oil is 40 percent vinegar,
where all of the measurements are by weight. How many ounces of oil must be
added to the mixture to produce a new mixture that is only 25 percent vinegar?
Solution: Let xrepresent the number of ounces of oil to be added. Then the total
number of ounces of the new mixture will be and the total number of ounces12 +x,
of vinegar in the new mixture will be Since the new mixture must be 25(0.40)(12).
percent vinegar,
(0.40)(12) =0.25
12 +x
Therefore,
(0.40)(12) =(12 +x)(0.25)
4.8 =3+0.25x
1.8 =0.25x
7.2 =x
Thus, 7.2 ounces of oil must be added to produce a new mixture that is 25 percent
vinegar.
Example 2.7.3: In a driving competition, Jeff and Dennis drove the same course at
average speeds of 51 miles per hour and 54 miles per hour, respectively. If it took Jeff
40 minutes to drive the course, how long did it take Dennis?
Solution: Let xbe the time, in minutes, that it took Dennis to drive the course. The
distance d, in miles, is equal to the product of the rate r, in miles per hour, and the
time t, in hours; that is,
d=rt
Note that since the rates are given in miles per hour, it is necessary to express the
times in hours; for example, 40 minutes equals of an hour. Thus, the distance
40
60
traveled by Jeff is the product of his speed and his time, miles, and the dis-
40
(51)冢冣
60
tance traveled by Dennis is similarly represented by miles. Since the dis-
x
(54)冢冣
60
tances are equal,
40 x
(51) =(54)
冢冣 冢冣
60 60
(51)(40) =54x
(51)(40)
x=37.8
54
Thus, it took Dennis approximately 37.8 minutes to drive the course.
This ebook was issued to HUAN-LIN CHANG, order #14803770223. Unlawful distribution of this ebook is prohibited.
Customer name HUAN-LIN CHANG , Order Id: 14803770223
GRE Math Review
231
Example 2.7.4: Working alone at its constant rate, machine Atakes 3 hours to pro-
duce a batch of identical computer parts. Working alone at its constant rate, ma-
chine Btakes 2 hours to produce an identical batch of parts. How long will it take
the two machines, working simultaneously at their respective constant rates, to pro-
duce an identical batch of parts?
Solution: Since machine Atakes 3 hours to produce a batch, machine Acan produce
of the batch in 1 hour. Similarly, machine Bcan produce of the batch in 1 hour.
11
32
If we let xrepresent the number of hours it takes both machines, working simulta-
neously, to produce the batch, then the two machines will produce of the job in 1
1
x
hour. When the two machines work together, adding their individual production
rates, and gives their combined production rate Therefore,
11 1
,.
32 x
111
+=
32x
231
+=
66x
51
=
6x
6=x
5
Thus, working together, the machines will take hours, or 1 hour 12 minutes, to
6
5
produce a batch of parts.
Example 2.7.5: At a fruit stand, apples can be purchased for $0.15 each and pears for
$0.20 each. At these rates, a bag of apples and pears was purchased for $3.80. If the
bag contained 21 pieces of fruit, how many of the pieces were pears?
Solution:Ifarepresents the number of apples purchased and prepresents the num-
ber of pears purchased, the information can be translated into the following system
of equations.
0.15a+0.20p=3.80 (total cost)
a+p=21 (total number of fruit)
From the second equation, Substituting into the first equation fora=21 p.21p
agives
0.15(21 p)+0.20p=3.80
(0.15)(21) 0.15p+0.20p=3.80
3.15 0.15p+0.20p=3.80
0.05p=0.65
p=13
Thus, of the 21 pieces of fruit, 13 were pears.
This ebook was issued to HUAN-LIN CHANG, order #14803770223. Unlawful distribution of this ebook is prohibited.
Customer name HUAN-LIN CHANG , Order Id: 14803770223
GRE Math Review
232
Example 2.7.6: To produce a particular radio model, it costs a manufacturer $30 per
radio, and it is assumed that if 500 radios are produced, all of them will be sold.
What must be the selling price per radio to ensure that the profit (revenue from the
sales minus the total production cost) on the 500 radios is greater than $8,200 ?
Solution:Ifyrepresents the selling price per radio, then the profit is 500(y30).
Therefore, we set
500(y30) >8,200
Solving the inequality, we get
500y15,000 8,200
500y23,200
y46.4
Thus, the selling price must be greater than $46.40 to ensure that the profit is greater
than $8,200.
Some applications involve computing interest earned on an investment during a
specified time period. The interest can be computed as simple interest or compound
interest.
Simple interest is based only on the initial deposit, which serves as the amount
on which interest is computed, called the principal, for the entire time period. If the
amount Pis invested at a simple annual interest rate of r percent, then the value Vof
the investment at the end of tyears is given by the formula
rt
V=P1+
冢冣
100
where Pand Vare in dollars.
In the case of compound interest, interest is added to the principal at regular
time intervals, such as annually, quarterly, and monthly. Each time interest is added to
the principal, the interest is said to be compounded. After each compounding, interest
is earned on the new principal, which is the sum of the preceding principal and the
interest just added. If the amount Pis invested at an annual interest rate of r percent,
compounded annually, then the value Vof the investment at the end of tyears is given
by the formula
t
r
V=P1+
冢冣
100
If the amount Pis invested at an annual interest rate of r percent, compounded n
times per year, then the value Vof the investment at the end of tyears is given by the
formula
nt
r
V=P1+
冢冣
100n
Example 2.7.7: If $10,000 is invested at a simple annual interest rate of 6 percent,
what is the value of the investment after half a year?
Solution: According to the formula for simple interest, the value of the investment
after year is
1
2
1
$10,000 1 +0.06 =$10,000(1.03) =$10,300
冢冢
2
This ebook was issued to HUAN-LIN CHANG, order #14803770223. Unlawful distribution of this ebook is prohibited.
Customer name HUAN-LIN CHANG , Order Id: 14803770223
GRE Math Review
233
Example 2.7.8: If an amount Pis to be invested at an annual interest rate of 3.5 per-
cent, compounded annually, what should be the value of Pso that the value of the
investment is $1,000 at the end of 3 years?
Solution: According to the formula for 3.5 percent annual interest, compounded an-
nually, the value of the investment after 3 years is
3
P(1 +0.035)
and we set it to be equal to $1,000
3
P(1 +0.035) =$1,000
To find the value of P, we divide both sides of the equation by (1 +0.035)
3
.
$1,000
P=$901.94
3
(1 +0.035)
Thus, an amount of approximately $901.94 should be invested.
Example 2.7.9: A college student expects to earn at least $1,000 in interest on an ini-
tial investment of $20,000. If the money is invested for one year at interest com-
pounded quarterly, what is the least annual interest rate that would achieve the goal?
Solution: According to the formula for rpercent annual interest, compounded quar-
terly, the value of the investment after 1 year is
4
r
$20,000 1 +
冢冣
400
By setting this value greater than or equal to $21,000 and solving for r, we get
4
r
$20,000 1 +$21,000
冢冣
400
4
r
1+1.05
冢冣
400
We can use the fact that taking the positive fourth root of each side of an inequality
preserves the direction of the inequality. This is also true for the positive square root
or any other positive root.
r
4
1+1.05
400
4
r400 1.05 1
冢冣
To compute the fourth root, we can use the fact that for that is, we
4
x=xx0;
冪冪
can compute a fourth root by taking a square root twice:
4
r400 1.05 1=400 1.05 14.91
冪冪
冢冣冢 冣
So the least annual interest rate is approximately 4.91 percent.
2.8 Coordinate Geometry
Two real number lines that are perpendicular to each other and that intersect at their
respective zero points define a rectangular coordinate system, often called the
xy-coordinate system or xy-plane. The horizontal number line is called the x-axis
This ebook was issued to HUAN-LIN CHANG, order #14803770223. Unlawful distribution of this ebook is prohibited.
Customer name HUAN-LIN CHANG , Order Id: 14803770223
GRE Math Review
234
and the vertical number line is called the y-axis. The point where the two axes inter-
sect is called the origin, denoted by O. The positive half of the x-axis is to the right of
the origin, and the positive half of the y-axis is above the origin. The two axes divide
the plane into four regions called quadrants I, II, III, and IV, as shown in the figure
below.
O11232
III
IVIII
34
1
1
2
3
2
3
4
x
y
P(4, 2)
P(4, 2)
P(4, 2)
P′′′(4, 2)
Each point Pin the xy-plane can be identified with an ordered pair of real(x,y)
numbers and is denoted by The first number is called the x-coordinate, andP(x,y).
the second number is called the y-coordinate. A point with coordinates is lo-(x,y)
cated units to the right of the y-axis if xis positive or to the left of the y-axis if xisx
negative. Also, the point is located units above the x-axis if yis positive or belowy
the x-axis if yis negative. If the point lies on the y-axis, and if the pointx=0, y=0,
lies on the x-axis. The origin has coordinates Unless otherwise noted, the units(0, 0).
used on the x-axis and the y-axis are the same.
In the figure above, the point is 4 units to the right of the y-axis and 2 unitsP(4, 2)
above the x-axis, and the point P′′′(4, 2) is 4 units to the left of the y-axis and 2
units below the x-axis.
Note that the three points and P′′′(4, 2) have the same co-P(4, 2), P(4, 2),
ordinates as Pexcept for the sign. These points are geometrically related to Pas fol-
lows.
Pis the reflection of Pabout the x-axis,orPand Pare symmetric about
the x-axis.
Pis the reflection of Pabout the y-axis,orPand Pare symmetric about
the y-axis.
P′′′ is the reflection of Pabout the origin,orP′′′ and Pare symmetric about
the origin.
The distance between two points in the xy-plane can be found by using the Py-
thagorean theorem. For example, the distance between the two points andQ(2, 3)
in the figure at the top of the following page is the length of line segment QR.R(4, 1.5)
To find this distance, construct a right triangle (indicated by the dashed lines) and
then note that the two shorter sides of the triangle have lengths andQS =4(2) =6
(3) =4.5.RS =1.5
This ebook was issued to HUAN-LIN CHANG, order #14803770223. Unlawful distribution of this ebook is prohibited.
Customer name HUAN-LIN CHANG , Order Id: 14803770223
GRE Math Review
235
Since line segment QR is the hypotenuse of the triangle, you can apply the Pythag-
orean theorem:
22
QR =6+4.5 =56.25 =7.5
冪冪
(For a discussion of right triangles and the Pythagorean theorem, see section 3.3.)
Equations in two variables can be represented as graphs in the coordinate plane.
In the xy-plane, the graph of an equation in the variables xand yis the set of all
points whose ordered pairs satisfy the equation.(x,y)
The graph of a linear equation of the form is a straight line in they=mx +b
xy-plane, where mis called the slope of the line and bis called the y-intercept.
The x-intercepts of a graph are the x-coordinates of the points at which the graph
intersects the x-axis. Similarly, the y-intercepts of a graph are the y-coordinates of the
points at which the graph intersects the y-axis. Sometimes the terms x-intercept and
y-intercept refer to the actual intersection points.
The slope of a line passing through two points and whereQ(x,y)R(x,y),
11 22
is defined asxx,
12
yy
21
xx
21
This ratio is often called “rise over run,” where rise is the change in ywhen mov-
ing from Qto Rand run is the change in xwhen moving from Qto R. A horizontal
line has a slope of 0, since the rise is 0 for any two points on the line. So the equation
of every horizontal line has the form where bis the y-intercept. The slope of ay=b,
vertical line is not defined, since the run is 0. The equation of every vertical line has
the form where ais the x-intercept.x=a,
Two lines are parallel if their slopes are equal. Two lines are perpendicular if
their slopes are negative reciprocals of each other. For example, the line with equation
is perpendicular to the line with equation 1
y=2x+5y=− x+9.
2
Example 2.8.1: In the xy-plane above, the slope of the line passing through the points
and isQ(2, 3) R(4, 1.5)
1.5 (3) 4.5
==0.75
4(2) 6
This ebook was issued to HUAN-LIN CHANG, order #14803770223. Unlawful distribution of this ebook is prohibited.
Customer name HUAN-LIN CHANG , Order Id: 14803770223
GRE Math Review
236
Line QR appears to intersect the y-axis close to the point so the y-intercept(0, 1.5),
of the line must be close to To get the exact value of the y-intercept, substitute1.5.
the coordinates of any point on the line, say into the equationQ(2, 3), y=0.75x+b,
and solve it for bas follows.
y=0.75x
+
b
3=(0.75)(2) +b
b=−3+(0.75)(2)
b=−1.5
Therefore, the equation of line QR is y=0.75x1.5.
You can see from the graph that the x-intercept of line QR is 2, since QR passes
through the point More generally, you can find the x-intercept by setting(2, 0). y=0
in an equation of the line and solving it for xas follows.
0=0.75x1.5
1.5 =0.75x
1.5
x==2
0.75
Graphs of linear equations can be used to illustrate solutions of systems of linear
equations and inequalities.
Example 2.8.2: Consider the system of linear equations in two variables in section
2.3:
4x+3y=13
x+2y=2
Solving each equation for yin terms of xyields
413
y=− x+
33
1
y=− x+1
2
The graphs of the two equations are below, and the solution of the system of equa-
tions is the point at which the two graphs intersect, which is (4, 1).
O
–2 2 4
–2
2
4
x
y
(4, –1)
x
+ 2y = 2 4x + 3y = 13
This ebook was issued to HUAN-LIN CHANG, order #14803770223. Unlawful distribution of this ebook is prohibited.
Customer name HUAN-LIN CHANG , Order Id: 14803770223
GRE Math Review
237
Example 2.8.3: Consider the following system of linear inequalities.
x3y6
2x+y1
Solving each inequality for yin terms of xyields
1
yx+2
3
y2x1
Each point that satisfies the first inequality is either on the line
1
(x,y)yx+2
3
or below the line because the y-coordinate is either equal to or less than
1
y=x+2
3
Therefore, the graph of consists of the line and the en-
111
x+2. yx+2y=x+2
333
tire region below it. Similarly, the graph of consists of the liney2x1
and the entire region above it. Thus, the solution set of the system ofy=−2x1
inequalities consists of all of the points that lie in the shaded region shown in the
figure below, which is the intersection of the two regions described.
–2–4624
–2
–4
2
4
x
y
1
3
y = –2x – 1 y = x + 2
Symmetry with respect to the x-axis, the y-axis, and the origin is mentioned above.
Another important symmetry is symmetry with respect to the line with equation
The line passes through the origin, has a slope of 1, and makes a 45-degreey=x.y=x
angle with each axis. For any point with coordinates the point with inter-(a,b),
changed coordinates is the reflection of about the line that is,(b,a)(a,b)y=x;(a,b)
and are symmetric about the line It follows that interchanging xand yin(b,a)y=x.
the equation of any graph yields another graph that is the reflection of the original
graph about the line y=x.
This ebook was issued to HUAN-LIN CHANG, order #14803770223. Unlawful distribution of this ebook is prohibited.
Customer name HUAN-LIN CHANG , Order Id: 14803770223
GRE Math Review
238
Example 2.8.4: Consider the line whose equation is Interchanging xand yy =2x+5.
in the equation yields Solving this equation for yyields The
15
x=2y+5. y=x.
22
line and its reflection are graphed below.
15
y=2x+5y=x
22
10
y = x
y = 2x + 5
y = x
5
10 5
5
10
5
10
x
y
1
2
5
2
45°
The line is a line of symmetry for the graphs of and 15
y=xy=2x+5y=x.
22
The graph of a quadratic equation of the form where a,b, and c
2
y=ax +bx +c,
are constants and is a parabola. The x-intercepts of the parabola are the solu-a0,
tions of the equation If ais positive, the parabola opens upward and
2
ax +bx +c=0.
the vertex is its lowest point. If ais negative, the parabola opens downward and the
vertex is the highest point. Every parabola is symmetric with itself about the vertical
line that passes through its vertex. In particular, the two x-intercepts are equidistant
from this line of symmetry.
This ebook was issued to HUAN-LIN CHANG, order #14803770223. Unlawful distribution of this ebook is prohibited.
Customer name HUAN-LIN CHANG , Order Id: 14803770223
GRE Math Review
239
Example 2.8.5: The equation has the following graph.
2
y=x2x3
O
224
2
4(1, 4)
2
4
6
x
y = x2 – 2x – 3
y
The graph indicates that the x-intercepts of the parabola are and 3. The values of1
the x-intercepts can be confirmed by solving the quadratic equation
2
x2x3=0
to get and The point is the vertex of the parabola, and the linex=−1x=3. (1, 4)
is its line of symmetry. The y-intercept is the y-coordinate of the point on thex=1
parabola at which which is
2
x=0, y=02(0) 3=−3.
The graph of an equation of the form is a circle with its
222
(xa)+(yb)=r
center at the point and with radius r.(a,b)
Example 2.8.6: The graph of is a circle with its center at the origin and
22
x+y=100
with radius 10, as shown in the figure below. The smaller circle has center (6, 5)
and radius 3, so its equation is
22
(x6) +(y+5) =9.
10
5
–10 –5
–10
–5
10
5
x
y
O
x2 + y2 = 100
(x – 6)2 + ( y + 5)2 = 9
This ebook was issued to HUAN-LIN CHANG, order #14803770223. Unlawful distribution of this ebook is prohibited.
Customer name HUAN-LIN CHANG , Order Id: 14803770223
GRE Math Review
240
2.9 Graphs of Functions
The coordinate plane can be used for graphing functions. To graph a function in the
xy-plane, you represent each input xand its corresponding output as a pointf(x)
where In other words, you use the x-axis for the input and the y-axis for(x,y), y=f(x).
the output.
Below are several examples of graphs of elementary functions.
Example 2.9.1: Consider the linear function defined by Its graph in
1
f(x)=− x+1.
2
the xy-plane is the line with the linear equation as shown in the figure
1
y=− x+1,
2
below.
Example 2.9.2: Consider the quadratic function defined by The graph of g
2
g(x)=x.
is the parabola with the quadratic equation as shown in the figure below.
2
y=x,
O
–2 –1–3 21
y = x + 1
y = x2
2
3
1
4
x
y
1
2
Note that the graphs of fand gfrom the two examples above intersect at two points.
These are the points at which We can find these points algebraically by set-g(x)=f(x).
ting
g(x)=f(x)
1
2
x=− x+1
2
and solving for x, using the quadratic formula, as follows.
1
2
x=− x+1
2
1
2
x+x1=0
2
2
2x+x2=0
We get which represent the x-coordinates of the two solutions
11+16
x=,
4
1+17 117
冪冪
x=0.78 and x=1.28
44
With these input values, the corresponding y-coordinates can be found using either
for g:
This ebook was issued to HUAN-LIN CHANG, order #14803770223. Unlawful distribution of this ebook is prohibited.
Customer name HUAN-LIN CHANG , Order Id: 14803770223
GRE Math Review
241
22
1+17 1+17 117 117
冪冪 冪冪
g=0.61 and g=1.64
44 44
冢冣冢冣 冢冣冢冣
Thus, the two intersection points can be approximated by and(0.78, 0.61)
(1.28, 1.64).
Example 2.9.3: Consider the absolute value function defined by By usingh(x)=x.
the definition of absolute value (see section 1.5), hcan be expressed as a piecewise-
defined function:
x,x0
h(x)=x,x<0
The graph of this function is V-shaped and consists of two linear pieces, andy=x
joined at the origin, as shown in the figure below.y=−x,
O
–2 –1–3 2 3 41
y =
2
3
1
–1
–2
4
x
y
x
Example 2.9.4: Consider the positive square-root function defined by forj(x)=x
whose graph is half of a parabola lying on its side. Also consider the negativex0,
square-root function defined by for whose graph is the other half
k(x)=− xx0,
of the parabola lying on its sidethe dashed curve below the x-axis. Both graphs are
shown in the figure below, along with the parabola (with its left half dashed).
2
y=x
O
–2 –1–3 2 3 41
y =
y = x2
2
3
1
–1
–2
4
x
y
y = x
x
This ebook was issued to HUAN-LIN CHANG, order #14803770223. Unlawful distribution of this ebook is prohibited.
Customer name HUAN-LIN CHANG , Order Id: 14803770223
GRE Math Review
242
The graphs of and are halves of a parabola because they are reflec-
y=xy=− x
冪冪
tions of the right and left halves, respectively, of the parabola about the line
2
y=x
This follows from squaring both sides of the two square root equations to gety=x.
and then interchanging xand yto get
22
y=xy=x.
Also note that is the reflection of about the x-axis. In general,
y=− xy=x
冪冪
for any function h, the graph of is the reflection of the graph ofy=−h(x)y=h(x)
about the x-axis.
Example 2.9.5: Consider the functions defined by and
2
f(x)=x+2g(x)=(x+1) .
These functions are related to the absolute value function and the quadraticx
function respectively, in simple ways.
2
x,
The graph of fis the graph of shifted upward by 2 units, as shown in thex
figure below. Similarly, the graph of the function is the graph of shiftedx5x
downward by 5 units (not shown).
The graph of gis the graph of shifted to the left by 1 unit, as shown in the
2
x
figure below. Similarly, the graph of the function is the graph of shifted to
22
(x4) x
the right by 4 units (not shown). To double-check the direction of the shift, you can
plot some corresponding values of the original function and the shifted function.
O
213231
y = | x| + 2
y = ( x + 1)2
2
3
1
4
x
y
In general, for any function and any positive number c, the following areh(x)
true.
The graph of is the graph of shifted upward by cunits.h(x)+ch(x)
The graph of is the graph of shifted downward by cunits.h(x)ch(x)
The graph of is the graph of shifted to the left by cunits.h(x+c)h(x)
The graph of is the graph of shifted to the right by cunits.h(xc)h(x)
Example 2.9.6: Consider the functions defined by and
2
x
f(x)=2x1g(x)=− .
4
These functions are related to the absolute value function and the quadraticx
function respectively, in more complicated ways than in the preceding example.
2
x,
The graph of fis the graph of shifted to the right by 1 unit and then stretchedx
vertically away from the x-axis by a factor of 2, as shown in the figure on the next
page. Similarly, the graph of the function is the graph of shifted to the
1x1⎪⎪x
2
right by 1 unit and then shrunk vertically toward the x-axis by a factor of (not
1
2
shown).
This ebook was issued to HUAN-LIN CHANG, order #14803770223. Unlawful distribution of this ebook is prohibited.
Customer name HUAN-LIN CHANG , Order Id: 14803770223
GRE Math Review
243
The graph of gis the graph of shrunk vertically by a factor of and then reflected
1
2
x4
in the x-axis, as shown in the figure below.
O
21
323
1
y = 2| x 1|
y = 2
2
3
1
1
2
4
x
y
x
4
In general, for any function and any positive number c, the following areh(x)
true.
The graph of is the graph of stretched vertically by a factor of cifch(x)h(x)
c>1.
The graph of is the graph of shrunk vertically by a factor of cifch(x)h(x)
0<c<1.
ALGEBRA EXERCISES
1. Find an algebraic expression to represent each of the following.
(a) The square of yis subtracted from 5, and the result is multiplied by 37.
(b) Three times xis squared, and the result is divided by 7.
(c) The product of and yis added to 18.(x+4)
2. Simplify each of the following algebraic expressions.
(a)
22
3x6+x+11 x+5x
(b) 3(5x1)x+4
(c) where
2
x16,x4
x4
(d) (2x+5)(3x1)
3. (a) What is the value of when
2
f(x)=3x7x+23 x=−2?
(b) What is the value of when
32
h(x)=x2x+x2x=2?
(c) What is the value of when
5
k(x)=x7x=0?
3
4. If the function gis defined for all nonzero numbers yby find the
y
g(y)=,
y
value of each of the following.
(a) g(2)
(b) g(2)
(c) g(2) g(2)
This ebook was issued to HUAN-LIN CHANG, order #14803770223. Unlawful distribution of this ebook is prohibited.
Customer name HUAN-LIN CHANG , Order Id: 14803770223
GRE Math Review
244
5. Use the rules of exponents to simplify the following.
(a)
53
(n)(n) (e)
53
(w)
(b)
77
(s)(t)(f)
03
(5 )(d)
(c)
12
r
4
r(g)
10 1
(x)( y)
55
(x)( y)
(d)
5
2a
冢冣
b(h)
25
3x1
冢冣 冢
yy
6. Solve each of the following equations for x.
(a) 5x7=28
(b) 12 5x=x+30
(c) 5(x+2) =13x
(d) (x+6)(2x1) =0
(e)
2
x+5x14 =0
(f )
2
xx1=0
7. Solve each of the following systems of equations for xand y.
(a) x+y=24
x
y=18 (b) 3x
y=−5
x+2y=3(c) 15x18 2y=−3x+y
10x+7y+20 =4x+2
8. Solve each of the following inequalities for x.
(a) 3x>7+x
(b) 25x+16 10 x
(c) 16 +x>8x12
9. For a given two-digit positive integer, the tens digit is 5 more than the units
digit. The sum of the digits is 11. Find the integer.
10. If the ratio of 2xto 5yis 3 to 4, what is the ratio of xto y?
11. Kathleen’s weekly salary was increased by 8 percent to $237.60. What was
her weekly salary before the increase?
12. A theater sells children’s tickets for half the adult ticket price. If 5 adult
tickets and 8 children’s tickets cost a total of $27, what is the cost of an adult
ticket?
13. Pat invested a total of $3,000. Part of the money was invested in a money
market account that paid 10 percent simple annual interest, and the
remainder of the money was invested in a fund that paid 8 percent simple
annual interest. If the interest earned at the end of the first year from these
investments was $256, how much did Pat invest at 10 percent and how much
at 8 percent?
14. Two cars started from the same point and traveled on a straight course in
opposite directions for exactly 2 hours, at which time they were 208 miles
apart. If one car traveled, on average, 8 miles per hour faster than the other
car, what was the average speed of each car for the 2-hour trip?
This ebook was issued to HUAN-LIN CHANG, order #14803770223. Unlawful distribution of this ebook is prohibited.
Customer name HUAN-LIN CHANG , Order Id: 14803770223
GRE Math Review
245
15. A group can charter a particular aircraft at a fixed total cost. If 36 people
charter the aircraft rather than 40 people, then the cost per person is greater
by $12.
(a) What is the fixed total cost to charter the aircraft?
(b) What is the cost per person if 40 people charter the aircraft?
16. An antiques dealer bought cantique chairs for a total of xdollars. The dealer
sold each chair for ydollars.
(a) Write an algebraic expression for the profit, P, earned from buying and
selling the chairs.
(b) Write an algebraic expression for the profit per chair.
17. In the coordinate system below, find the following.
(a) Coordinates of point Q
(b) Lengths of PQ,QR, and PR
(c) Perimeter of PQR
(d) Area of PQR
(e) Slope, y-intercept, and equation of the line passing through points
Pand R
x
y
R(5, 0)
P(–2, 6)
O
Q
18. In the xy-plane, find the following.
(a) Slope and y-intercept of the line with equation 2y+x=6
(b) Equation of the line passing through the point with y-intercept 1(3, 2)
(c) The y-intercept of a line with slope 3 that passes through the point
(2, 1)
(d) The x-intercepts of the graphs in (a), (b), and (c)
19. For the parabola in the xy-plane, find the following.
2
y=x4x12
(a) The x-intercepts
(b) The y-intercept
(c) Coordinates of the vertex
20. For the circle in the xy-plane, find the following.
22
(x1) +(y+1) =20
(a) Coordinates of the center
(b) Radius
(c) Area
This ebook was issued to HUAN-LIN CHANG, order #14803770223. Unlawful distribution of this ebook is prohibited.
Customer name HUAN-LIN CHANG , Order Id: 14803770223
GRE Math Review
246
21. For each of the following functions, give the domain and a description of the
graph in the xy-plane, including its shape, and the x- and y-intercepts.y=f(x)
(a) f(x)=−4
(b) f(x)=100 900x
(c)
2
f(x)=5(x+20)
(d) f(x)=x+2
(e) f(x)=x+x
ANSWERS TO ALGEBRA EXERCISES
1. (a)
22
37(5 y), or 185 37y
(b)
22
(3x)9x
,or
77
(c) or18 +(x+4)( y), 18 +xy +4y
2. (a)
2
2x+6x+5
(b) 14x+1
(c) x+4
(d)
2
6x+13x5
3. (a) 49 (b) 0 (c) 7
4. (a) 1 (b) 1 (c) 2
5. (a)
2
n
(b)
7
(st)
(c)
8
r
(d)
5
32a
5
b
(e) 1
15
w
(f )
3
d
(g)
15
x
6
y
(h)
23
9xy
6. (a) 7
(b) 3
(c) 9
8
(d) 1
6, 2
(e) 27,
(f ) 1+515
冪冪
,
22
7. (a) x=21
y=3(b) x=−1
y=2(c) x=1
2
y=−3
8. (a) 7
x<−
4
(b) 3
x13
(c) x<4
9. 83
10. 15 to 8
11. $220
12. $3
13. $800 at 10%and $2,200 at 8%
14. 48 mph and 56 mph
This ebook was issued to HUAN-LIN CHANG, order #14803770223. Unlawful distribution of this ebook is prohibited.
Customer name HUAN-LIN CHANG , Order Id: 14803770223
GRE Math Review
247
15. (a) $4,320 (b) $108
16. (a) P=cy x(b) Profit per chair: Pcyxx
==y
cc c
17. (a) (2, 0)
(b) PQ =6, QR =7, PR =85
(c) 13 +85
(d) 21
(e) Slope: ; y-intercept: ; equation of line: or
630 630
y=− x+,
77 77
7y+6x=30
18. (a) Slope: ; y-intercept: 3
1
2
(b) x
y=+1
3
(c) 7
(d) 6, 7
3, 3
19. (a) andx=−2x=6
(b) y=−12
(c) (2, 16)
20. (a) (1, 1)
(b) 20
(c) 20p
21. (a) Domain: the set of all real numbers. The graph is a horizontal line with y-
intercept and no x-intercept.4
(b) Domain: the set of all real numbers. The graph is a line with slope 900,
y-intercept 100, and x-intercept 1.
9
(c) Domain: the set of all real numbers. The graph is a parabola opening
downward with vertex at line of symmetry y-intercept(20, 5), x=−20,
and x-intercepts
395, 20 5.
(d) Domain: the set of numbers greater than or equal to The graph is2.
half a parabola opening to the right with vertex at x-intercept(2, 0), 2,
and y-intercept 2.
(e) Domain: the set of all real numbers. The graph is two half-lines joined at
the origin: one half-line is the negative x-axis and the other is a line
starting at the origin with slope 2. Every nonpositive number is an
x-intercept, and the y-intercept is 0. The function is equal to the following
piecewise-defined function
2x,x0
f(x)=0, x0
This ebook was issued to HUAN-LIN CHANG, order #14803770223. Unlawful distribution of this ebook is prohibited.
Customer name HUAN-LIN CHANG , Order Id: 14803770223
GRE Math Review
248
3. GEOMETRY
The review of geometry begins with lines and angles and progresses to other plane
figures, such as polygons, triangles, quadrilaterals, and circles. The section ends with
some basic three-dimensional figures. Coordinate geometry is covered in the Algebra
section.
3.1 Lines and Angles
Plane geometry is devoted primarily to the properties and relations of plane figures,
such as angles, triangles, other polygons, and circles. The terms “point,” “line,” and
“plane” are familiar intuitive concepts. A point has no size and is the simplest geo-
metric figure. All geometric figures consist of points. A line is understood to be a
straight line that extends in both directions without ending. A plane can be thought
of as a floor or a tabletop, except that a plane extends in all directions without end-
ing and has no thickness.
Given any two points on a line, a line segment is the part of the line that con-
tains the two points and all the points between them. The two points are called end-
points. Line segments that have equal lengths are called congruent line segments.
The point that divides a line segment into two congruent line segments is called the
midpoint of the line segment.
In the figure below, A,B,C, and Dare points on line .
866
A
BC D
Line segment AB consists of points Aand Band all the points on the line be-
tween Aand B. Sometimes the notation AB denotes line segment AB, and sometimes
it denotes the length of line segment AB. The meaning of the notation can be deter-
mined from the context. According to the figure above, the lengths of line segments
AB,BC, and CD are 8, 6, and 6, respectively. Hence, line segments BC and CD are
congruent. Since Cis halfway between Band D, point Cis the midpoint of line seg-
ment BD.
When two lines intersect at a point, they form four angles, as indicated below.
Each angle has a vertex at point P, which is the point of intersection of the two
lines.
C
P
B
AD
1
2
In the figure, angles APC and BPD are called opposite angles, also known as
vertical angles. Angles APD and CPB are also opposite angles. Opposite angles have
equal measures, and angles that have equal measures are called congruent angles.
Hence, opposite angles are congruent. The sum of the measures of the four angles is
360.
Sometimes the angle symbol is used instead of the word “angle.” For example,
angle APC can be written as APC.
This ebook was issued to HUAN-LIN CHANG, order #14803770223. Unlawful distribution of this ebook is prohibited.
Customer name HUAN-LIN CHANG , Order Id: 14803770223
GRE Math Review
249
Two lines that intersect to form four congruent angles are called perpendicular
lines. Each of the four angles has a measure of An angle with a measure of90.90
is called a right angle. The figure below shows two lines, and , that are perpen-
12
dicular, denoted by .
12
1
2
90°
90°
90°
90°
12
A common way to indicate that an angle is a right angle is to draw a small
square at the vertex of the angle, as shown below, where PON is a right angle.
O
N
P
ON OP
An angle with a measure less than is called an acute angle, and an angle90
with a measure between and is called an obtuse angle.90180
Two lines in the same plane that do not intersect are called parallel lines. The
figure below shows two lines, and that are parallel, denoted by The two,.
12 12
lines are intersected by a third line, forming eight angles. Note that four of the,
3
angles have the measure and the remaining four angles have the measurex,y,
where x+y=180.
3
2
1
y°
y°
x°
x°
y°
y°
x°
x°
12
This ebook was issued to HUAN-LIN CHANG, order #14803770223. Unlawful distribution of this ebook is prohibited.
Customer name HUAN-LIN CHANG , Order Id: 14803770223
GRE Math Review
250
3.2 Polygons
Apolygon is a closed figure formed by three or more line segments, called sides.
Each side is joined to two other sides at its endpoints, and the endpoints are called
vertices. In this discussion, the term “polygon” means “convex polygon,” that is, a
polygon in which the measure of each interior angle is less than The figures180.
below are examples of such polygons.
Triangle
(3 sides)
Pentagon
(5 sides)
Quadrilateral
(4 sides)
The simplest polygon is a triangle. Note that a quadrilateral can be divided into
2 triangles, and a pentagon can be divided into 3 triangles, as shown below.
If a polygon has nsides, it can be divided into triangles. Since the sum ofn2
the measures of the interior angles of a triangle is it follows that the sum of180,
the measures of the interior angles of an n-sided polygon is For exam-(n2)(180).
ple, the sum for a quadrilateral is and the sum for a(n=4) (4 2)(180)=360,
hexagon is(n=6) (6 2)(180)=720.
A polygon in which all sides are congruent and all interior angles are congruent
is called a regular polygon. For example, in a regular octagon (8 sides), the sum of
the measures of the interior angles is Therefore, the measure(8 2)(180)=1,080.
of each angle is 1,080=135.
8
The perimeter of a polygon is the sum of the lengths of its sides. The area of a
polygon refers to the area of the region enclosed by the polygon.
In the next two sections, we will look at some basic properties of triangles and
quadrilaterals.
3.3 Triangles
Every triangle has three sides and three interior angles. The measures of the interior
angles add up to The length of each side must be less than the sum of the180.
lengths of the other two sides. For example, the sides of a triangle could not have
the lengths 4, 7, and 12 because 12 is greater than 4 +7.
The following are special triangles.
A triangle with three congruent sides is called an equilateral triangle. The
measures of the three interior angles of such a triangle are also equal, and
each measure is .60
A triangle with at least two congruent sides is called an isosceles triangle.If
a triangle has two congruent sides, then the angles opposite the two sides are
This ebook was issued to HUAN-LIN CHANG, order #14803770223. Unlawful distribution of this ebook is prohibited.
Customer name HUAN-LIN CHANG , Order Id: 14803770223
GRE Math Review
251
congruent. The converse is also true. For example, in below, sinceABC
both and have measure it follows that Also, sinceAC50,AB =BC.
the measure of is50 +50 +x=180, B80.
B
AC
50°50°
x°
A triangle with an interior right angle is called a right triangle. The side
opposite the right angle is called the hypotenuse; the other two sides are
called legs.
D
F
E
In right triangle DEF above, EF is the hypotenuse and DE and DF are legs. The
Pythagorean theorem states that in a right triangle, the square of the length of the
hypotenuse is equal to the sum of the squares of the lengths of the legs. Thus, for
triangle DEF above,
22 2
(EF)=(DE)+(DF)
This relationship can be used to find the length of one side of a right triangle if
the lengths of the other two sides are known. For example, if one leg of a right trian-
gle has length 5 and the hypotenuse has length 8, then the length of the other side
can be determined as follows.
58
x
8
2
=5
2
+x
2
64 =25 +x
2
39 =x
2
Since and xmust be positive, it follows that or approximately
2
x=39 x=39,
6.2.
The Pythagorean theorem can be used to determine the ratios of the lengths of
the sides of two special right triangles. One special right triangle is an isosceles right
This ebook was issued to HUAN-LIN CHANG, order #14803770223. Unlawful distribution of this ebook is prohibited.
Customer name HUAN-LIN CHANG , Order Id: 14803770223
GRE Math Review
252
triangle. Applying the Pythagorean theorem to such a triangle shows that the lengths
of its sides are in the ratio 1 to 1 to as indicated below.
2,
xx
y
45° 45°
y
2
=x
2
+x
2
y
2
=2x
2
y=x2
The other special right triangle is a 30-60-90right triangle, which is half of
an equilateral triangle, as indicated below.
x
y
30°
60° 60°
30°
2x
Note that the length of the shortest side, x, is one-half the length of the longest
side, 2x. By the Pythagorean theorem, the ratio of xto yis 1 to because
3
22 2
x+y=(2x)
22 2
x+y=4x
222
y=4xx
22
y=3x
y=3x
Hence, the ratio of the lengths of the three sides of such a triangle is 1 to to 2.3
The area Aof a triangle equals one-half the product of the length of a base and
the height corresponding to the base. In the figure below, the base is denoted by b
and the corresponding height is denoted by h.
h
b
bh
A=2
Any side of a triangle can be used as a base; the height that corresponds to the
base is the perpendicular line segment from the opposite vertex to the base (or to an
extension of the base). The examples below show three different configurations of a
base and the corresponding height.
6
15
6
15
6
15
In all three triangles above, the area is , or 45.
(15)(6)
2
Two triangles that have the same shape and size are called congruent triangles.
More precisely, two triangles are congruent if their vertices can be matched up so
that the corresponding angles and the corresponding sides are congruent.
The following three propositions can be used to determine whether two triangles
are congruent by comparing only some of their sides and angles.
This ebook was issued to HUAN-LIN CHANG, order #14803770223. Unlawful distribution of this ebook is prohibited.
Customer name HUAN-LIN CHANG , Order Id: 14803770223
GRE Math Review
253
If the three sides of one triangle are congruent to the three sides of another
triangle, then the triangles are congruent.
If two sides and the included angle of one triangle are congruent to two sides
and the included angle of another triangle, then the triangles are congruent.
If two angles and the included side of one triangle are congruent to two
angles and the included side of another triangle, then the triangles are
congruent.
Two triangles that have the same shape but not necessarily the same size are
called similar triangles. More precisely, two triangles are similar if their vertices can
be matched up so that the corresponding angles are congruent or, equivalently, the
lengths of corresponding sides have the same ratio, called the scale factor of similar-
ity. For example, all 30-60-90right triangles, discussed above, are similar trian-
gles, though they may differ in size.
When we say that triangles ABC and DEF are similar, it is assumed that angles A
and Dare congruent, angles Band Eare congruent, and angles Cand Fare congru-
ent, as shown in the figure below. In other words, the order of the letters indicates
the correspondences.
A
C
B
DF
E
Since triangles ABC and DEF are similar, we have By cross mul-
AB BC AC
==.
DE EF DF
tiplication, we can obtain other proportions, such as .
AB DE
=
BC EF
3.4 Quadrilaterals
Every quadrilateral has four sides and four interior angles. The measures of the inte-
rior angles add up to The following are special quadrilaterals.360.
A quadrilateral with four right angles is called a rectangle. Opposite sides of
a rectangle are parallel and congruent, and the two diagonals are also
congruent.
BC
A
D
ABCD and AD BC
AB =CD and AD =BC
AC =BD
A rectangle with four congruent sides is called a square.
This ebook was issued to HUAN-LIN CHANG, order #14803770223. Unlawful distribution of this ebook is prohibited.
Customer name HUAN-LIN CHANG , Order Id: 14803770223
GRE Math Review
254
A quadrilateral in which both pairs of opposite sides are parallel is called a
parallelogram. In a parallelogram, opposite sides are congruent and
opposite angles are congruent.
P
QR
S
x° y°
PQSR and PS QR
PQ =SR and PS =QR
A quadrilateral in which two opposite sides are parallel is called a trapezoid.
K
LM
N
KN LM
For all parallelograms, including rectangles and squares, the area Aequals the
product of the length of a base band the corresponding height h; that is,
A=bh
Any side can be used as a base. The height corresponding to the base is the
perpendicular line segment from any point of a base to the opposite side (or to an
extension of that side). Below are examples of finding the areas of a rectangle and a
parallelogram.
10
20
6
8
A=(6)(10) =60 A=(20)(8) =160
The area Aof a trapezoid equals half the product of the sum of the lengths of
the two parallel sides and and the corresponding height h; that is,bb
12
1
A=(b+b)(h)
12
2
This ebook was issued to HUAN-LIN CHANG, order #14803770223. Unlawful distribution of this ebook is prohibited.
Customer name HUAN-LIN CHANG , Order Id: 14803770223
GRE Math Review
255
For example, for the trapezoid below with bases of length 10 and 18 and a
height of 7.5, the area is
1
A=(10 +18)(7.5) =105
2
10
18
7.5
3.5 Circles
Given a point Oin a plane and a positive number r, the set of points in the plane
that are a distance of runits from Ois called a circle. The point Ois called the
center of the circle and the distance ris called the radius of the circle. The diame-
ter of the circle is twice the radius. Two circles with equal radii are called congru-
ent circles.
Any line segment joining two points on the circle is called a chord. The terms
“radius” and “diameter” can also refer to line segments: A radius is any line segment
joining a point on the circle and the center of the circle, and a diameter is any
chord that passes through the center of the circle. In the figure below, Ois the cen-
ter of the circle, ris the radius, PQ is a chord, and ST is a diameter.
P
S
T
O
r
Q
The distance around a circle is called the circumference of the circle, which is
analogous to the perimeter of a polygon. The ratio of the circumference Cto the di-
ameter dis the same for all circles and is denoted by the Greek letter ; that is,p
C=p
d
The value of is approximately 3.14 and can also be approximated by the fractionp
22.
7
If ris the radius of a circle, then and so the circumference is related
CC
==p,
d2r
to the radius by the equation
C=2pr
This ebook was issued to HUAN-LIN CHANG, order #14803770223. Unlawful distribution of this ebook is prohibited.
Customer name HUAN-LIN CHANG , Order Id: 14803770223
GRE Math Review
256
For example, if a circle has a radius of 5.2, then its circumference is
(2)(p)(5.2) =(10.4)(p)(10.4)(3.14)
which is approximately 32.7.
Given any two points on a circle, an arc is the part of the circle containing the
two points and all the points between them. Two points on a circle are always the
endpoints of two arcs. It is customary to identify an arc by three points to avoid am-
biguity. In the figure below, arc ABC is the shorter arc between Aand C, and arc
ADC is the longer arc between Aand C.
50°
5
A
B
C
D
Acentral angle of a circle is an angle with its vertex at the center of the circle.
The measure of an arc is the measure of its central angle, which is the angle
formed by two radii that connect the center of the circle to the two endpoints of the
arc. An entire circle is considered to be an arc with measure In the figure360.
above, the measure of arc ABC is and the measure of arc ADC is50310.
To find the length of an arc of a circle, note that the ratio of the length of an
arc to the circumference is equal to the ratio of the degree measure of the arc to
The circumference of the circle above is Therefore,360.10p.
length of arc ABC 50
=
10p360
50 25p(25)(3.14)
length of arc ABC =(10p)=⬇⬇4.4
冢冣
360 18 18
The area of a circle with radius ris equal to For example, the area of the circle
2
pr.
above with radius 5 is
2
p(5) =25p.
Asector of a circle is a region bounded by an arc of the circle and two radii. In
the circle above, the region bounded by arc ABC and the two dashed radii is a sector
with central angle Just as in the case of the length of an arc, the ratio of the50.
area of a sector of a circle to the area of the entire circle is equal to the ratio of the
degree measure of its arc to Therefore, if Srepresents the area of the sector360.
with central angle then50,
S50
=
25p360
50 125p(125)(3.14)
S=(25p)=⬇⬇10.9
冢冣
360 36 36
Atangent to a circle is a line that intersects the circle at exactly one point,
called the point of tangency, denoted by Pin the figure at the top of the following
page. If a line is tangent to a circle, then a radius drawn to the point of tangency is
perpendicular to the tangent line. The converse is also true; that is, if a line is per-
pendicular to a radius at its endpoint on the circle, then the line is a tangent to the
circle at that endpoint.
This ebook was issued to HUAN-LIN CHANG, order #14803770223. Unlawful distribution of this ebook is prohibited.
Customer name HUAN-LIN CHANG , Order Id: 14803770223
GRE Math Review
257
Tangent line
P
A polygon is inscribed in a circle if all its vertices lie on the circle, or equiva-
lently, the circle is circumscribed about the polygon. Triangles RST and XYZ below
are inscribed in the circles with centers Oand W, respectively.
W
X
Y
Z
O
R
S
T
If one side of an inscribed triangle is a diameter of the circle, as in triangle XYZ
above, then the triangle is a right triangle. Conversely, if an inscribed triangle is a
right triangle, then one of its sides is a diameter of the circle.
A polygon is circumscribed about a circle if each side of the polygon is tangent
to the circle, or equivalently, the circle is inscribed in the polygon. In the figure be-
low, quadrilateral ABCD is circumscribed about the circle with center O.
O
A
B
D
C
Two or more circles with the same center are called concentric circles,as
shown in the figure below.
This ebook was issued to HUAN-LIN CHANG, order #14803770223. Unlawful distribution of this ebook is prohibited.
Customer name HUAN-LIN CHANG , Order Id: 14803770223
GRE Math Review
258
3.6 Three-dimensional Figures
Basic three-dimensional figures include rectangular solids, cubes, cylinders, spheres,
pyramids, and cones. In this section, we look at some properties of rectangular sol-
ids and right circular cylinders.
Arectangular solid has six rectangular surfaces called faces, as shown in the
figure below. Adjacent faces are perpendicular to each other. Each line segment that
is the intersection of two faces is called an edge, and each point at which the edges
intersect is called a vertex. There are 12 edges and 8 vertices. The dimensions of a
rectangular solid are the length the width w, and the height h.,
w
h
A rectangular solid with six square faces is called a cube, in which case =w=h.
The volume Vof a rectangular solid is the product of its three dimensions, or
V=wh
The surface area Aof a rectangular solid is the sum of the areas of the six faces,
or
A=2(w+h+wh)
For example, if a rectangular solid has length 8.5, width 5, and height 10, then
its volume is
V=(8.5)(5)(10) =425
and its surface area is
A=2((8.5)(5) +(8.5)(10) +(5)(10)) =355
Acircular cylinder consists of two bases that are congruent circles and a lateral
surface made of all line segments that join points on the two circles and that are
parallel to the line segment joining the centers of the two circles. The latter line seg-
ment is called the axis of the cylinder. A right circular cylinder is a circular cylin-
der whose axis is perpendicular to its bases.
The right circular cylinder shown in the figure below has circular bases with
centers Pand Q. Line segment PQ is the axis of the cylinder and is perpendicular to
both bases. The length of PQ is called the height of the cylinder.
P
Q
This ebook was issued to HUAN-LIN CHANG, order #14803770223. Unlawful distribution of this ebook is prohibited.
Customer name HUAN-LIN CHANG , Order Id: 14803770223
GRE Math Review
259
The volume Vof a right circular cylinder that has height hand a base with ra-
dius ris the product of the height and the area of the base, or
2
V=prh
The surface area Aof a right circular cylinder is the sum of the areas of the two
bases and the lateral area, or
2
A=2(pr)+2prh
For example, if a right circular cylinder has height 6.5 and a base with radius 3,
then its volume is
2
V=p(3) (6.5) =58.5p
and its surface area is
2
A=(2)(p)(3) +(2)(p)(3)(6.5) =57p
GEOMETRY EXERCISES
1. Lines and mbelow are parallel. Find the values of xand y.
42°
57°
x°y°
m
2. In the figure below, Find the values of xand y.AC =BC.
125° x°
y°
B
AC
3. In the figure below, what is the relationship between x,y, and z?
x°
y°z°
This ebook was issued to HUAN-LIN CHANG, order #14803770223. Unlawful distribution of this ebook is prohibited.
Customer name HUAN-LIN CHANG , Order Id: 14803770223
GRE Math Review
260
4. What is the sum of the measures of the interior angles of a decagon (10-
sided polygon)?
5. If the decagon in exercise 4 is regular, what is the measure of each interior
angle?
6. The lengths of two sides of an isosceles triangle are 15 and 22, respectively.
What are the possible values of the perimeter?
7. Triangles PQR and XYZ are similar. If and what isPQ =6, PR =4, XY =9,
the length of side XZ?
8. What are the lengths of sides NO and OP in triangle NOP below?
N
P
40
24
10
O
9. In the figure below, If the area of triangle CDE is 42, what isAB =BC =CD.
the area of triangle ADG?
A
B
C
D
E
F
G
10. In rectangle ABCD below, and Find the following.AB =5, AF =7, FD =3.
(a) Area of ABCD
(b) Area of triangle AEF
(c) Length of BD
(d) Perimeter of ABCD
B
A
C
D
F
E
This ebook was issued to HUAN-LIN CHANG, order #14803770223. Unlawful distribution of this ebook is prohibited.
Customer name HUAN-LIN CHANG , Order Id: 14803770223
GRE Math Review
261
11. In parallelogram ABCD below, find the following.
(a) Area of ABCD
(b) Perimeter of ABCD
(c) Length of diagonal BD
12 2
4
B
A
C
D
12. The circle with center Obelow has radius 4. Find the following.
(a) Circumference of the circle
(b) Length of arc ABC
(c) Area of the shaded region
40°
B
A
C
O
13. The figure below shows two concentric circles, each with center O. Given
that the larger circle has radius 12 and the smaller circle has radius 7, find
the following.
(a) Circumference of the larger circle
(b) Area of the smaller circle
(c) Area of the shaded region
O
This ebook was issued to HUAN-LIN CHANG, order #14803770223. Unlawful distribution of this ebook is prohibited.
Customer name HUAN-LIN CHANG , Order Id: 14803770223
GRE Math Review
262
14. For the rectangular solid below, find the following.
(a) Surface area of the solid
(b) Length of diagonal AB
10
2
7B
A
ANSWERS TO GEOMETRY EXERCISES
1. x=57 and y=138
2. x=70 and y=125
3. z=x+y
4. 1,440
5. 144
6. 52 and 59
7. 6
8. NO =30 and OP =10 34
9. 378
10. (a) 50
(b) 17.5
(c) 5 5
(d) 30
11. (a) 48
(b) 24 +45
(c) 2 29
12. (a) 8p
(b) 8p
9
(c) 16p
9
13. (a) 24p
(b) 49p
(c) 95p
14. (a) 208
(b) 3 17
This ebook was issued to HUAN-LIN CHANG, order #14803770223. Unlawful distribution of this ebook is prohibited.
Customer name HUAN-LIN CHANG , Order Id: 14803770223
GRE Math Review
263
4. DATA ANALYSIS
The goal of data analysis is to understand data well enough to describe past and
present trends, predict future events with some certainty, and thereby make better
decisions. In this limited review of data analysis, we begin with tools for describing
data; follow with tools for understanding counting and probability; review the con-
cepts of distributions of data, random variables, and probability distributions; and
end with examples of interpreting data.
4.1 Graphical Methods for Describing Data
Data can be organized and summarized using a variety of methods. Tables are
commonly used, and there are many graphical and numerical methods as well. The
appropriate type of representation for a collection of data depends in part on the na-
ture of the data, such as whether the data are numerical or nonnumerical. In this
section, we review some common graphical methods for describing and summariz-
ing data.
Variables play a major role in algebra because a variable serves as a convenient
name for many values at once, and it also can represent a particular value in a given
problem to solve. In data analysis, variables also play an important role but with a
somewhat different meaning. In data analysis, a variable is any characteristic that
can vary for the population of individuals or objects being analyzed. For example,
both gender and age represent variables among people.
Data are collected from a population after observing either a single variable or
observing more than one variable simultaneously. The distribution of a variable, or
distribution of data, indicates the values of the variable and how frequently the val-
ues are observed in the data.
Frequency Distributions
The frequency,orcount, of a particular category or numerical value is the number
of times that the category or value appears in the data. A frequency distribution is
a table or graph that presents the categories or numerical values along with their
associated frequencies. The relative frequency of a category or a numerical value is
the associated frequency divided by the total number of data. Relative frequencies
may be expressed in terms of percents, fractions, or decimals. A relative frequency
distribution is a table or graph that presents the relative frequencies of the catego-
ries or numerical values.
Example 4.1.1: A survey was taken to find the number of children in each of 25
families. A list of the values collected in the survey follows.
1204133120452323241230231
This ebook was issued to HUAN-LIN CHANG, order #14803770223. Unlawful distribution of this ebook is prohibited.
Customer name HUAN-LIN CHANG , Order Id: 14803770223
GRE Math Review
264
Here are the resulting frequency and relative frequency distributions of the data.
Frequency Distribution
Number of
Children Frequency
0
1
2
3
4
5
Total
3
5
7
6
3
1
25
Relative Frequency Distribution
Number of
Children
Relative
Frequency
0
1
2
3
4
5
Total
12%
20%
28%
24%
12%
4%
100%
Note that the total for the relative frequencies is 100%. If decimals were used instead
of percents, the total would be 1. The sum of the relative frequencies in a relative
frequency distribution is always 1.
Bar Graphs
A commonly used graphical display for representing frequencies, or counts, is a bar
graph, or bar chart. In a bar graph, rectangular bars are used to represent the catego-
ries of the data, and the height of each bar is proportional to the corresponding fre-
quency or relative frequency. All of the bars are drawn with the same width, and the
bars can be presented either vertically or horizontally. Bar graphs enable comparisons
across several categories, making it easy to identify frequently and infrequently occur-
ring categories.
This ebook was issued to HUAN-LIN CHANG, order #14803770223. Unlawful distribution of this ebook is prohibited.
Customer name HUAN-LIN CHANG , Order Id: 14803770223
GRE Math Review
265
Example 4.1.2:
2,000
1,000
0
4,000
3,000
6,000
5,000
8,000
7,000
Enrollment
Colle
g
e AColle
g
e BColle
g
e CColle
g
e DColle
g
e E
FALL 2009 ENROLLMENT AT FIVE COLLEGES
From the graph, we can conclude that the college with the greatest fall 2009 enroll-
ment was College E, and the college with the least enrollment was College A. Also,
we can estimate that the enrollment for College Dwas about 6,400.
Asegmented bar graph is used to show how different subgroups or subcatego-
ries contribute to an entire group or category. In a segmented bar graph, each bar rep-
resents a category that consists of more than one subcategory. Each bar is divided
into segments that represent the different subcategories. The height of each segment
is proportional to the frequency or relative frequency of the subcategory that the seg-
ment represents.
Example 4.1.3:
2,000
1,000
0
4,000
3,000
6,000
5,000
8,000
7,000
Enrollment
Colle
g
e AColle
g
e BColle
g
e CColle
g
e DColle
g
e E
Full-Time
Part-Time
FALL 2009 ENROLLMENT AT FIVE COLLEGES
Different values can be estimated from the segmented bar graph above. For example,
for College D, the total enrollment was approximately 6,400 students, the part-time
enrollment was approximately 2,200, and the full-time enrollment was approxi-
mately or 4,200 students.6,400 2,200,
This ebook was issued to HUAN-LIN CHANG, order #14803770223. Unlawful distribution of this ebook is prohibited.
Customer name HUAN-LIN CHANG , Order Id: 14803770223
GRE Math Review
266
Bar graphs can also be used to compare different groups using the same categories.
Example 4.1.4:
2,000
1,000
0
4,000
3,000
6,000
5,000
Enrollment
Colle
g
e AColle
g
e BColle
g
e C
FALL 2009 AND SPRING 2010 ENROLLMENT
AT THREE COLLEGES
Fall 2009
Spring 2010
Observe that for all three colleges, the fall 2009 enrollment was greater than the
spring 2010 enrollment. Also, the greatest decrease in the enrollment from fall 2009
to spring 2010 occurred at College B.
Although bar graphs are commonly used to compare frequencies, as in the exam-
ples above, they are sometimes used to compare numerical data that could be dis-
played in a table, such as temperatures, dollar amounts, percents, heights, and
weights. Also, the categories sometimes are numerical in nature, such as years or
other time intervals.
Circle Graphs
Circle graphs, often called pie charts, are used to represent data with a relatively
small number of categories. They illustrate how a whole is separated into parts. The
area of the circle graph representing each category is proportional to the part of the
whole that the category represents.
This ebook was issued to HUAN-LIN CHANG, order #14803770223. Unlawful distribution of this ebook is prohibited.
Customer name HUAN-LIN CHANG , Order Id: 14803770223
GRE Math Review
267
Example 4.1.5:
UNITED STATES PRODUCTION OF PHOTOGRAPHIC
EQUIPMENT AND SUPPLIES IN 1971
Total: $3,980 million
Sensitized Goods
47%
Office
Copiers
25%
4%
7%
12%
5% Motion-Picture
Equipment
Still-Picture
Equipment
Prepared Photochemicals
Microfilm E
q
ui
p
ment
The graph shows that of all United States production of photographic equipment
and supplies in 1971, Sensitized Goods was the category with the greatest dollar
value.
Each part of a circle graph is called a sector. Because the area of each sector is
proportional to the percent of the whole that the sector represents, the measure of the
central angle of a sector is proportional to the percent of 360 degrees that the sector
represents. For example, the measure of the central angle of the sector representing
the category Prepared Photochemicals is 7 percent of 360 degrees, or 25.2 degrees.
Histograms
When a list of data is large and contains many different values of a numerical vari-
able, it is useful to organize it by grouping the values into intervals, often called
classes. To do this, divide the entire interval of values into smaller intervals of equal
length and then count the values that fall into each interval. In this way, each interval
has a frequency and a relative frequency. The intervals and their frequencies (or rela-
tive frequencies) are often displayed in a histogram. Histograms are graphs of fre-
quency distributions that are similar to bar graphs, but they have a number line for
the horizontal axis. Also, in a histogram, there are no regular spaces between the bars.
Any spaces between bars in a histogram indicate that there are no data in the inter-
vals represented by the spaces.
Example 4.5.1 in section 4.5 illustrates a histogram with 50 bars. Numerical vari-
ables with just a few values can also be displayed using histograms, where the fre-
quency or relative frequency of each value is represented by a bar centered over the
value, as in the histogram in the following example.
This ebook was issued to HUAN-LIN CHANG, order #14803770223. Unlawful distribution of this ebook is prohibited.
Customer name HUAN-LIN CHANG , Order Id: 14803770223
GRE Math Review
268
Example 4.1.6: The relative frequency distribution in example 4.1.1 can be displayed
as a histogram.
Relative Frequency Distribution
0%
5%
10%
15%
20%
25%
30%
012345
Number of Children
Relative Frequency
Histograms are useful for identifying the general shape of a distribution of data.
Also evident are the “center” and degree of “spread” of the distribution, as well as
high-frequency and low-frequency intervals. From the histogram above, you can see
that the distribution is shaped like a mound with one peak; that is, the data are fre-
quent in the middle and sparse at both ends. The central values are 2 and 3, and the
distribution is close to being symmetric about those values. Because the bars all have
the same width, the area of each bar is proportional to the amount of data that the
bar represents. Thus, the areas of the bars indicate where the data are concentrated
and where they are not.
Finally, note that because each bar has a width of 1, the sum of the areas of the
bars equals the sum of the relative frequencies, which is 100% or 1, depending on
whether percents or decimals are used. This fact is central to the discussion of proba-
bility distributions in section 4.5.
Scatterplots
All examples used thus far have involved data resulting from a single characteristic or
variable. These types of data are referred to as univariate, that is, data observed for
one variable. Sometimes data are collected to study two different variables in the
same population of individuals or objects. Such data are called bivariate data. We
might want to study the variables separately or investigate a relationship between the
two variables. If the variables were to be analyzed separately, each of the graphical
methods for univariate numerical data presented above could be applied.
To show the relationship between two numerical variables, the most useful type of
graph is a scatterplot. In a scatterplot, the values of one variable appear on the hori-
zontal axis of a rectangular coordinate system and the values of the other variable ap-
pear on the vertical axis. For each individual or object in the data, an ordered pair of
numbers is collected, one number for each variable, and the pair is represented by a
point in the coordinate system.
A scatterplot makes it possible to observe an overall pattern, or trend, in the rela-
tionship between the two variables. Also, the strength of the trend as well as striking
deviations from the trend are evident. In many cases, a line or a curve that best repre-
sents the trend is also displayed in the graph and is used to make predictions about
the population.
This ebook was issued to HUAN-LIN CHANG, order #14803770223. Unlawful distribution of this ebook is prohibited.
Customer name HUAN-LIN CHANG , Order Id: 14803770223
GRE Math Review
269
Example 4.1.7: A bicycle trainer studied 50 bicyclists to examine how the finishing
time for a certain bicycle race was related to the amount of physical training in the
three months before the race. To measure the amount of training, the trainer de-
veloped a training index, measured in “units” and based on the intensity of each
bicyclist’s training. The data and the trend of the data, represented by a line, are
displayed in the scatterplot below.
FINISHING TIMES AND TRAINING INDICES
FOR 50 BICYCLISTS IN A RACE
010 20 30 40 50 60 70 80 90 100
Training Index
(units)
0.0
3.0
3.5
4.0
4.5
5.0
5.5
6.0
Finishing Time
(hours)
Trend line
In addition to the given trend line, you can see how scattered or close the data are to
the trend line; or to put it another way, you can see how well the trend line fits the
data. You can also see that the finishing times generally decrease as the training indi-
ces increase and that three or four data are relatively far from the trend.
Several types of predictions can be based on the trend line. For example, it can
be predicted, based on the trend line, that a bicyclist with a training index of 70
units would finish the race in approximately 4 hours. This value is obtained by not-
ing that the vertical line at the training index of 70 units intersects the trend line
very close to 4 hours.
Another prediction based on the trend line is the number of minutes that a bicy-
clist can expect to lower his or her finishing time for each increase of 10 training
index units. This prediction is basically the ratio of the change in finishing time to
the change in training index, or the slope of the trend line. Note that the slope is
negative. To estimate the slope, estimate the coordinates of any two points on the
linefor instance, the points at the extreme left and right ends of the line: (0, 5.8)
and The slope is(100, 3.2).
3.2 5.8 2.6
==0.026
100 0 100
which is measured in hours per unit. The slope can be interpreted as follows: the
finishing time is predicted to decrease 0.026 hours for every unit by which the train-
This ebook was issued to HUAN-LIN CHANG, order #14803770223. Unlawful distribution of this ebook is prohibited.
Customer name HUAN-LIN CHANG , Order Id: 14803770223
GRE Math Review
270
ing index increases. Since we want to know how much the finishing time decreases
for an increase of 10 units, we multiply the rate by 10 to get 0.26 hour per 10 units.
To compute the decrease in minutes per 10 units, we multiply 0.26 by 60 to get ap-
proximately 16 minutes. Based on the trend line, the bicyclist can expect to decrease
the finishing time by 16 minutes for every increase of 10 training index units.
Time Plots
Sometimes data are collected in order to observe changes in a variable over time. For
example, sales for a department store may be collected monthly or yearly. A time plot
(sometimes called a time series) is a graphical display useful for showing changes in
data collected at regular intervals of time. A time plot of a variable plots each observa-
tion corresponding to the time at which it was measured. A time plot uses a coordi-
nate plane similar to a scatterplot, but the time is always on the horizontal axis, and
the variable measured is always on the vertical axis. Additionally, consecutive observa-
tions are connected by a line segment to emphasize increases and decreases over
time.
Example 4.1.8:
FALL ENROLLMENT FOR COLLEGE A
2001–2009
0
1,000
2,000
3,000
4,000
5,000
2001 2002 2003 2004 2005 2006 2007 2008 2009
Year
Enrollment
You can observe from the graph that the greatest increase in fall enrollment between
consecutive years occurred from 2008 to 2009. One way to determine this is by not-
ing that the slope of the line segment joining the values for 2008 and 2009 is greater
than the slopes of the line segments joining all other consecutive years, because the
time intervals are regular.
Although time plots are commonly used to compare frequencies, as in the exam-
ple above, they can be used to compare any numerical data as the data change over
time, such as temperatures, dollar amounts, percents, heights, and weights.
4.2 Numerical Methods for Describing Data
Data can be described numerically by various statistics,orstatistical measures.
These statistical measures are often grouped in three categories: measures of central
tendency, measures of position, and measures of dispersion.
Measures of Central Tendency
Measures of central tendency indicate the “center” of the data along the number line
and are usually reported as values that represent the data. There are three common
measures of central tendency: (i) the arithmetic meanusually called the average or
simply the mean, (ii) the median, and (iii) the mode.
This ebook was issued to HUAN-LIN CHANG, order #14803770223. Unlawful distribution of this ebook is prohibited.
Customer name HUAN-LIN CHANG , Order Id: 14803770223
GRE Math Review
271
To calculate the mean of nnumbers, take the sum of the nnumbers and divide it
by n.
Example 4.2.1: For the five numbers 6, 4, 7, 10, and 4, the mean is
6+4+7+10 +431
==6.2
55
When several values are repeated in a list, it is helpful to think of the mean of the
numbers as a weighted mean of only those values in the list that are different.
Example 4.2.2: Consider the following list of 16 numbers.
2, 4, 4, 5, 7, 7, 7, 7, 7, 7, 8, 8, 9, 9, 9, 9
There are only 6 different values in the list: 2, 4, 5, 7, 8, and 9. The mean of the num-
bers in the list can be computed as
1(2) +2(4) +1(5) +6(7) +2(8) +4(9) 109
==6.8125
1+2+1+6+2+416
The number of times a value appears in the list, or the frequency, is called the
weight of that value. So the mean of the 16 numbers is the weighted mean of the val-
ues 2, 4, 5, 7, 8, and 9, where the respective weights are 1, 2, 1, 6, 2, and 4. Note that
the sum of the weights is the number of numbers in the list, 16.
The mean can be affected by just a few values that lie far above or below the rest
of the data, because these values contribute directly to the sum of the data and there-
fore to the mean. By contrast, the median is a measure of central tendency that is
fairly unaffected by unusually high or low values relative to the rest of the data.
To calculate the median of nnumbers, first order the numbers from least to great-
est. If nis odd, then the median is the middle number in the ordered list of numbers.
If nis even, then there are two middle numbers, and the median is the average of
these two numbers.
Example 4.2.3: The five numbers in example 4.2.1 listed in increasing order are 4, 4,
6, 7, 10, so the median is 6, the middle number. Note that if the number 10 in the list
is replaced by the number 24, the mean increases from 6.2 to
4+4+6+7+24 45
==9
55
but the median remains equal to 6. This example shows how the median is relatively
unaffected by an unusually large value.
The median, as the “middle value” of an ordered list of numbers, divides the list
into roughly two equal parts. However, if the median is equal to one of the data values
and it is repeated in the list, then the numbers of data above and below the median
may be rather different. See example 4.2.2, where the median is 7, but four of the data
are less than 7 and six of the data are greater than 7.
The mode of a list of numbers is the number that occurs most frequently in the
list.
This ebook was issued to HUAN-LIN CHANG, order #14803770223. Unlawful distribution of this ebook is prohibited.
Customer name HUAN-LIN CHANG , Order Id: 14803770223
GRE Math Review
272
Example 4.2.4: The mode of the numbers in the list 1, 3, 6, 4, 3, 5 is 3. A list of num-
bers may have more than one mode. For example, the list 1, 2, 3, 3, 3, 5, 7, 10, 10,
10, 20 has two modes, 3 and 10.
Measures of Position
The three most basic positions, or locations, in a list of data ordered from least to
greatest are the beginning, the end, and the middle. It is useful here to label these as L
for the least, Gfor the greatest, and Mfor the median. Aside from these, the most
common measures of position are quartiles and percentiles. Like the median M,
quartiles and percentiles are numbers that divide the data into roughly equal groups
after the data have been ordered from the least value Lto the greatest value G. There
are three quartile numbers that divide the data into four roughly equal groups, and
there are 99 percentile numbers that divide the data into 100 roughly equal groups. As
with the mean and median, the quartiles and percentiles may or may not themselves
be values in the data.
The first quartile the second quartile (which is simply the median M),Q,Q
12
and the third quartile divide a group of data into four roughly equal groups asQ
3
follows. After the data are listed in increasing order, the first group consists of the
data from Lto the second group is from to M, the third group is from MtoQ,QQ,
11 3
and the fourth group is from to G. Because the number of data in a list may not beQ
3
divisible by 4, there are various rules to determine the exact values of and andQQ
13
some statisticians use different rules, but in all cases We use perhaps theQ=M.
2
most common rule, in which divides the data into two equal partsthe lesserQ=M
2
numbers and the greater numbersand then is the median of the lesser numbersQ
1
and is the median of the greater numbers.Q
3
Example 4.2.5: To find the quartiles for the list of 16 numbers 2, 4, 4, 5, 7, 7, 7, 7, 7,
7, 8, 8, 9, 9, 9, 9 (already listed in order), first divide the data into two groups of 8
numbers each. The first group is 2, 4, 4, 5, 7, 7, 7, 7 and the second group is 7, 7, 8,
8, 9, 9, 9, 9, so that the second quartile, or median, is To find the otherQ=M=7.
2
quartiles, you can take each of the two smaller groups and find its median: the first
quartile is (the average of 5 and 7) and the third quartile is (the aver-Q=6Q=8.5
1 3
age of 8 and 9).
In example 4.2.5, note that the number 4 is in the lowest 25 percent of the distri-
bution of data. There are different ways to describe this. We can say that 4 is below
the first quartile, that is, below we can also say that 4 is in the first quartile. TheQ;
1
phrase “in a quartile” refers to being in one of the four groups determined by Q,Q,
12
and Q.
3
Percentiles are mostly used for very large lists of numerical data ordered from
least to greatest. Instead of dividing the data into four groups, the 99 percentiles P
1
,
divide the data into 100 groups. Consequently,P,P,...,PQ=P,M=Q=P,
23 99 125 250
and Because the number of data in a list may not be divisible by 100, statis-Q=P.
375
ticians apply various rules to determine values of percentiles.
This ebook was issued to HUAN-LIN CHANG, order #14803770223. Unlawful distribution of this ebook is prohibited.
Customer name HUAN-LIN CHANG , Order Id: 14803770223
GRE Math Review
273
Measures of Dispersion
Measures of dispersion indicate the degree of “spread” of the data. The most com-
mon statistics used as measures of dispersion are the range, the interquartile range,
and the standard deviation. These statistics measure the spread of the data in differ-
ent ways.
The range of the numbers in a group of data is the difference between the great-
est number Gin the data and the least number Lin the data, that is, For exam-GL.
ple, given the list 11, 10, 5, 13, 21, the range of the numbers is .21 5=16
The simplicity of the range is useful in that it reflects that maximum spread of the
data. However, sometimes a data value is so unusually small or so unusually large in
comparison with the rest of the data that it is viewed with suspicion when the data
are analyzedthe value could be erroneous or accidental in nature. Such data are
called outliers because they lie so far out that in most cases, they are ignored when
analyzing the data. Unfortunately, the range is directly affected by outliers.
A measure of dispersion that is not affected by outliers is the interquartile range.
It is defined as the difference between the third quartile and the first quartile, that is,
Thus, the interquartile range measures the spread of the middle half of theQQ.
31
data.
Example 4.2.6: In the list of 16 numbers 2, 4, 4, 5, 7, 7, 7, 7, 7, 7, 8, 8, 9, 9, 9, 9, the
range is the first quartile is and the third quartile is So9 2=7, Q=6, Q=8.5.
13
the interquartile range for the numbers in this list is 8.5 6=2.5.
One way to summarize a group of numerical data and to illustrate its center and
spread is to use the five numbers L, and G. These five numbers can be plot-Q,Q,Q,
123
ted along a number line to show where the four quartile groups lie. Such plots are
called boxplots or box-and-whisker plots, because a box is used to identify each of
the two middle quartile groups of data, and “whiskers” extend outward from the
boxes to the least and greatest values. The following graph shows the boxplot for the
list of 16 numbers in example 4.2.6.
109876543
LMG
210
1
Q
3
Q
There are a few variations in the way boxplots are drawnthe position of the
ends of the boxes can vary slightly, and some boxplots identify outliers with certain
symbolsbut all boxplots show the center of the data at the median and illustrate the
spread of the data in each of the four quartile groups. As such, boxplots are useful for
comparing sets of data side by side.
Example 4.2.7: Two large lists of numerical data, list I and list II, are summarized by
the following boxplots.
900800700600500
List I
List II
400300200100
This ebook was issued to HUAN-LIN CHANG, order #14803770223. Unlawful distribution of this ebook is prohibited.
Customer name HUAN-LIN CHANG , Order Id: 14803770223
GRE Math Review
274
Based on the boxplots, several different comparisons of the two lists can be made.
First, the median of list II, which is approximately 550, is greater than the median of
list I, which is approximately 450. Second, the two measures of spread, range and
interquartile range, are greater for list I than for list II. For list I, these measures are
approximately 520 and 430, respectively; and for list II, they are approximately 500
and 220, respectively.
Unlike the range and the interquartile range, the standard deviation is a measure
of spread that depends on each number in the list. Using the mean as the center of the
data, the standard deviation takes into account how much each value differs from the
mean and then takes a type of average of these differences. As a result, the more the
data are spread away from the mean, the greater the standard deviation; and the more
the data are clustered around the mean, the lesser the standard deviation.
The standard deviation of a group of nnumerical data is computed by (1) calcu-
lating the mean of the nvalues, (2) finding the difference between the mean and each
of the nvalues, (3) squaring each of the differences, (4) finding the average of the n
squared differences, and (5) taking the nonnegative square root of the average
squared difference.
Example 4.2.8: For the five data 0, 7, 8, 10, and 10, the standard deviation can be
computed as follows. First, the mean of the data is 7, and the squared differences
from the mean are
222 2 2
(7 0) , (7 7) , (7 8) , (7 10) , (7 10)
or 49, 0, 1, 9, 9. The average of the five squared differences is or 13.6, and the
68 ,
5
positive square root of 13.6 is approximately 3.7.
Note on terminology: The term “standard deviation” defined above is slightly dif-
ferent from another measure of dispersion, the sample standard deviation. The lat-
ter term is qualified with the word “sample” and is computed by dividing the sum of
the squared differences by instead of n. The sample standard deviation is only
n1
slightly different from the standard deviation but is preferred for technical reasons for
a sample of data that is taken from a larger population of data. Sometimes the stan-
dard deviation is called the population standard deviation to help distinguish it
from the sample standard deviation.
Example 4.2.9: Six hundred applicants for several post office jobs were rated on a
scale from 1 to 50 points. The ratings had a mean of 32.5 points and a standard devi-
ation of 7.1 points. How many standard deviations above or below the mean is a rat-
ing of 48 points? A rating of 30 points? A rating of 20 points?
Solution: Let dbe the standard deviation, so points. Note that 1 standard de-
d=7.1
viation above the mean is
32.5 +d=32.5 +7.1 =39.6
and 2 standard deviations above the mean is
32.5 +2d=32.5 +2(7.1) =46.7
So 48 is a little more than 2 standard deviations above the mean. Since 48 is actually
15.5 points above the mean, the number of standard deviations that 48 is above the
mean is Thus, to answer the question, we first found the difference from
15.5 2.2.
7.1
This ebook was issued to HUAN-LIN CHANG, order #14803770223. Unlawful distribution of this ebook is prohibited.
Customer name HUAN-LIN CHANG , Order Id: 14803770223
GRE Math Review
275
the mean and then we divided by the standard deviation. The number of standard
deviations that a rating of 30 is away from the mean is
30 32.5 2.5
=0.4
7.1 7.1
where the negative sign indicates that the rating is 0.4 standard deviation below the
mean.
The number of standard deviations that a rating of 20 is away from the mean is
20 32.5 12.5
=1.8
7.1 7.1
where the negative sign indicates that the rating is 1.8 standard deviations below the
mean.
To summarize:
48 points is 15.5 points above the mean, or approximately 2.2 standard
deviations above the mean.
30 points is 2.5 points below the mean, or approximately 0.4 standard
deviation below the mean.
20 points is 12.5 points below the mean, or approximately 1.8 standard
deviations below the mean.
One more instance, which may seem trivial, is important to note:
32.5 points is 0 points from the mean, or 0 standard deviations from the
mean.
Example 4.2.9 shows that for a group of data, each value can be located with re-
spect to the mean by using the standard deviation as a ruler. The process of subtract-
ing the mean from each value and then dividing the result by the standard deviation is
called standardization. Standardization is a useful tool because for each data value,
it provides a measure of position relative to the rest of the data independently of the
variable for which the data was collected and the units of the variable.
Note that the standardized values 2.2, and from example 4.2.9 are all0.4, 1.8
between and 3; that is, the corresponding ratings 48, 30, and 20 are all within 33
standard deviations above or below the mean. This is not surprising, based on the fol-
lowing fact about the standard deviation.
In any group of data, most of the data are within about 3standard deviations
above or below the mean.
Thus, when any group of data are standardized, most of the data are transformed
to an interval on the number line centered about 0 and extending from about to 3.3
The mean is always transformed to 0.
4.3 Counting Methods
Uncertainty is part of the process of making decisions and predicting outcomes. Un-
certainty is addressed with the ideas and methods of probability theory. Since elemen-
tary probability requires an understanding of counting methods, we now turn to a
discussion of counting objects in a systematic way before reviewing probability.
When a set of objects is small, it is easy to list the objects and count them one by
one. When the set is too large to count that way, and when the objects are related in a
This ebook was issued to HUAN-LIN CHANG, order #14803770223. Unlawful distribution of this ebook is prohibited.
Customer name HUAN-LIN CHANG , Order Id: 14803770223
GRE Math Review
276
patterned or systematic way, there are some useful techniques for counting the objects
without actually listing them.
Sets and Lists
The term set has been used informally in this review to mean a collection of objects
that have some property, whether it is the collection of all positive integers, all points
in a circular region, or all students in a school that have studied French. The objects
of a set are called members or elements. Some sets are finite, which means that
their members can be completely counted. Finite sets can, in principle, have all of
their members listed, using curly brackets, such as the set of even digits 0, 2, 4, 6, 8 .
{}
Sets that are not finite are called infinite sets, such as the set of all integers. A set that
has no members is called the empty set and is denoted by the symbol A set with
.
one or more members is called nonempty.IfAand Bare sets and all of the members
of Aare also members of B, then Ais a subset of B. For example, is a subset of
2, 8
{}
Also, by convention, is a subset of every set.
0, 2, 4, 6, 8 .
{}
Alist is like a finite set, having members that can all be listed, but with two differ-
ences. In a list, the members are ordered; that is, rearranging the members of a list
makes it a different list. Thus, the terms “first element,” “second element,” etc., make
sense in a list. Also, elements can be repeated in a list and the repetitions matter. For
example, the lists 1, 2, 3, 2 and 1, 2, 2, 3 are different lists, each with four elements,
and they are both different from the list 1, 2, 3, which has three elements.
In contrast to a list, when the elements of a set are given, repetitions are not
counted as additional elements and the order of the elements does not matter. For ex-
ample, the sets and are the same set, which has three elements. For
1, 2, 3, 2 3, 1, 2
{}{}
any finite set S, the number of elements of Sis denoted by Thus, if
S.
then Also,
S=6.2, 9, p, 0.01, 0 , S=5. =0.
{}
Sets can be formed from other sets. If Sand Tare sets, then the intersection of S
and Tis the set of all elements that are in both Sand Tand is denoted by The
ST.
union of Sand Tis the set of all elements that are in Sor T, or both, and is denoted
by If sets Sand Thave no elements in common, they are called disjoint or mu-
ST.
tually exclusive.
A useful way to represent two or three sets and their possible intersections and
unions is a Venn diagram. In a Venn diagram, sets are represented by circular re-
gions that overlap if they have elements in common but do not overlap if they are dis-
joint. Sometimes the circular regions are drawn inside a rectangular region, which
represents a universal set, of which all other sets involved are subsets.
Example 4.3.1: The sets A,B, and Care represented in the Venn diagram below,
where Urepresents a universal set.
U
A
C
B
This ebook was issued to HUAN-LIN CHANG, order #14803770223. Unlawful distribution of this ebook is prohibited.
Customer name HUAN-LIN CHANG , Order Id: 14803770223
GRE Math Review
277
The regions with vertical stripes represent the set The regions with horizontal
AC.
stripes represent the set B. The region with both kinds of stripes represents the set
The sets Band Care mutually exclusive, often written
AB.BC=.
The example above can be used to illustrate an elementary counting principle in-
volving intersecting sets, called the inclusion-exclusion principle for two sets. This
principle relates the numbers of elements in the union and intersection of two finite
sets: The number of elements in the union of two sets equals the sum of their indi-
vidual numbers of elements minus the number of elements in their intersection. If
the sets in example 4.3.1 are finite, then we have for the union of Aand B,
AB=A+BAB
Because is a subset of both Aand B, the subtraction is necessary to avoid
AB
counting the elements in twice. For the union of Band C, we have
AB
BC=B+C
because BC=.
Multiplication Principle
Suppose there are two choices to be made sequentially and that the second choice is
independent of the first choice. Suppose also that there are kdifferent possibilities for
the first choice and mdifferent possibilities for the second choice. The multiplication
principle states that under those conditions, there are km different possibilities for
the pair of choices.
For example, suppose that a meal is to be ordered from a restaurant menu and
that the meal consists of one entre´e and one dessert. If there are 5 entre´es and 3 des-
serts on the menu, then there are different meals that can be ordered from
(5)(3) =15
the menu.
The multiplication principle applies in more complicated situations as well. If
there are more than two independent choices to be made, then the number of differ-
ent possible outcomes of all of the choices is the product of the numbers of possibili-
ties for each choice.
Example 4.3.2: Suppose that a computer password consists of four characters such
that the first character is one of the 10 digits from 0 to 9 and each of the next 3 char-
acters is any one of the uppercase letters from the 26 letters of the English alphabet.
How many different passwords are possible?
Solution: The description of the password allows repetitions of letters. Thus, there are
10 possible choices for the first character in the password and 26 possible choices
for each of the next 3 characters in the password. Therefore, applying the multiplica-
tion principle, the number of possible passwords is (10)(26)(26)(26) =175,760.
Note that if repetitions of letters are not allowed in the password, then the
choices are not all independent, but a modification of the multiplication principle
can still be applied. There are 10 possible choices for the first character in the
password, 26 possible choices for the second character, 25 for the third character
because the first letter cannot be repeated, and 24 for the fourth character be-
cause the first two letters cannot be repeated. Therefore, the number of possible
passwords is (10)(26)(25)(24) =156,000.
Example 4.3.3: Each time a coin is tossed, there are 2 possible outcomeseither it lands
heads up or it lands tails up. Using this fact and the multiplication principle, you can
conclude that if a coin is tossed 8 times, there are
8
(2)(2)(2)(2)(2)(2)(2)(2) =2=256
possible outcomes.
This ebook was issued to HUAN-LIN CHANG, order #14803770223. Unlawful distribution of this ebook is prohibited.
Customer name HUAN-LIN CHANG , Order Id: 14803770223
GRE Math Review
278
Permutations and Factorials
Suppose you want to determine the number of different ways the 3 letters A, B, and C
can be placed in order from 1st to 3rd. The following is a list of all the possible orders
in which the letters can be placed.
ABC ACB BAC BCA CAB CBA
There are 6 possible orders for the 3 letters.
Now suppose you want to determine the number of different ways the 4 letters A,
B, C, and D can be placed in order from 1st to 4th. Listing all of the orders for 4 let-
ters is time-consuming, so it would be useful to be able to count the possible orders
without listing them.
To order the 4 letters, one of the 4 letters must be placed first, one of the remain-
ing 3 letters must be placed second, one of the remaining 2 letters must be placed
third, and the last remaining letter must be placed fourth. Therefore, applying the
multiplication principle, there are or 24, ways to order the 4 letters.(4)(3)(2)(1),
More generally, suppose nobjects are to be ordered from 1st to nth, and we want
to count the number of ways the objects can be ordered. There are nchoices for the
first object, choices for the second object, choices for the third object, andn1n2
so on, until there is only 1 choice for the nth object. Thus, applying the multiplication
principle, the number of ways to order the nobjects is equal to the product
n(n1)(n2)⋅⋅⋅(3)(2)(1)
Each order is called a permutation, and the product above is called the number of
permutations of nobjects.
Because products of the form occur frequently whenn(n1)(n2)⋅⋅⋅(3)(2)(1)
counting objects, a special symbol called nfactorial, is used to denote this product.n!,
For example,
1!=1
2! = (2)(1) = 2
3! = (3)(2)(1) = 6
4! = (4)(3)(2)(1) = 24
As a special definition, 0! =1.
Note that and so on.n!=n(n1)! =n(n1)(n2)! =n(n1)(n2)(n3)!
Example 4.3.4: Suppose that 10 students are going on a bus trip, and each of the
students will be assigned to one of the 10 available seats. Then the number of possi-
ble different seating arrangements of the students on the bus is
10! =(10)(9)(8)(7)(6)(5)(4)(3)(2)(1) =3,628,800
Now suppose you want to determine the number of ways in which you can select 3
of the 5 letters A, B, C, D, and E and place them in order from 1st to 3rd. Reasoning
as in the preceding examples, you find that there are or 60, ways to select(5)(4)(3),
and order them.
More generally, suppose that kobjects will be selected from a set of nobjects,
where and the kobjects will be placed in order from 1st to kth. Then there are nk n,
choices for the first object, choices for the second object, choices for then1n2
third object, and so on, until there are choices for the kth object. Thus, ap-nk+1
This ebook was issued to HUAN-LIN CHANG, order #14803770223. Unlawful distribution of this ebook is prohibited.
Customer name HUAN-LIN CHANG , Order Id: 14803770223
GRE Math Review
279
plying the multiplication principle, the number of ways to select and order kobjects
from a set of nobjects is It is useful to note that
n(n1)(n2)⋅⋅⋅(nk+1).
(nk)!
n(n1)(n2)⋅⋅⋅(nk+1) =n(n1)(n2)⋅⋅⋅(nk+1)(nk)!
n!
=(nk)!
This expression represents the number of permutations of nobjects taken kat a
time, that is, the number of ways to select and order kobjects out of nobjects.
Example 4.3.5: How many different five-digit positive integers can be formed using
the digits 1, 2, 3, 4, 5, 6, and 7 if none of the digits can occur more than once in the
integer?
Solution: This example asks how many ways there are to order 5 integers chosen
from a set of 7 integers. According to the counting principle above, there
are ways to do this. Note that this is equal to
(7)(6)(5)(4)(3) =2,520
7! (7)(6)(5)(4)(3)(2!)
==(7)(6)(5)(4)(3).
(7 5)! 2!
Combinations
Given the five letters A, B, C, D, and E, suppose that you want to determine the num-
ber of ways in which you can select 3 of the 5 letters, but unlike before, you do not
want to count different orders for the 3 letters. The following is a list of all of the ways
in which 3 of the 5 letters can be selected without regard to the order of the letters.
ABC ABD ABE ACD ACE ADE BCD BCE BDE CDE
There are 10 ways of selecting the 3 letters without order. There is a relationship be-
tween selecting with order and selecting without order.
The number of ways to select 3 of the 5 letters without order, which is 10, multi-
plied by the number of ways to order the 3 letters, which is or 6, is equal to the
3!,
number of ways to select 3 of the 5 letters and order them, which is In short,
5! =60.
2!
(number of ways to select without order) (number of ways to order)
=(number of ways to select with order)
This relationship can also be described as follows.
(number of ways to select with order)
(number of ways to select without order) =(number of ways to order)
5!
2! 5!
===10
3! 3!2!
More generally, suppose that kobjects will be chosen from a set of nobjects, where
but that the kobjects will not be put in order. The number of ways in which thiskn,
can be done is called the number of combinations of nobjects taken kat a time
and is given by the formula n!.
k!(nk)!
Another way to refer to the number of combinations of nobjects taken kat a time
is nchoose k, and two notations commonly used to denote this number are
n
C
k
and n.
冢冣
k
This ebook was issued to HUAN-LIN CHANG, order #14803770223. Unlawful distribution of this ebook is prohibited.
Customer name HUAN-LIN CHANG , Order Id: 14803770223
GRE Math Review
280
Example 4.3.6: Suppose you want to select a 3-person committee from a group of 9
students. How many ways are there to do this?
Solution: Since the 3 students on the committee are not ordered, you can use the
formula for the combination of 9 objects taken 3 at a time, or “9 choose 3”:
9! 9! (9)(8)(7)
== =84
3!(9 3)! 3!6! (3)(2)(1)
Using the terminology of sets, given a set Sconsisting of nelements, nchoose kis
simply the number of subsets of Sthat consist of kelements. The formula n!
k!(nk)!
also holds when andk=0k=n.
nchoose 0 is which corresponds to the fact that there is only one
n!=1,
0!n!
subset of Swith 0 elements, namely the empty set.
nchoose nis since there is only one subset of Swith nelements,
n!=1,
n!0!
namely the set Sitself.
Finally, note that nchoose kis always equal to nchoose becausenk,
n!n!n!
==
(nk)!(n(nk))! (nk)!k!k!(nk)!
4.4 Probability
Probability is a way of describing uncertainty in numerical terms. In this section we
review some of the terminology used in elementary probability theory.
Aprobability experiment, also called a random experiment, is an experiment
for which the result, or outcome, is uncertain. We assume that all of the possible out-
comes of an experiment are known before the experiment is performed, but which
outcome will actually occur is unknown. The set of all possible outcomes of a random
experiment is called the sample space, and any particular set of outcomes is called
an event. For example, consider a cube with faces numbered 1 to 6, called a 6-sided
die. Rolling the die once is an experiment in which there are 6 possible outcomes
either 1, 2, 3, 4, 5, or 6 will appear on the top face. The sample space for this experi-
ment is the set of numbers 1, 2, 3, 4, 5, and 6. Two examples of events for this experi-
ment are (i) rolling the number 4, which has only one outcome, and (ii) rolling an odd
number, which has three outcomes.
The probability of an event is a number from 0 to 1, inclusive, that indicates the
likelihood that the event occurs when the experiment is performed. The greater the
number, the more likely the event.
Example 4.4.1: Consider the following experiment. A box contains 15 pieces of paper,
each of which has the name of one of the 15 students in a class consisting of 7 male
and 8 female students, all with different names. The instructor will shake the box for
a while and then, without looking, choose a piece of paper at random and read the
name. Here the sample space is the set of 15 names. The assumption of random
selection means that each of the names is equally likely to be selected. If this as-
sumption is made, then the probability that any one particular name is selected is
This ebook was issued to HUAN-LIN CHANG, order #14803770223. Unlawful distribution of this ebook is prohibited.
Customer name HUAN-LIN CHANG , Order Id: 14803770223
GRE Math Review
281
equal to For any event E, the probability that Eoccurs, denoted by is de-
1.P(E),
15
fined by the ratio
the number of names in the event E
P(E)=15
If Mis the event that the student selected is male, then 7
P(M)=.
15
In general, for a random experiment with a finite number of possible outcomes, if
each outcome is equally likely to occur, then the probability that an event Eoccurs is
defined by the ratio
the number of outcomes in the event E
P(E)=the number of possible outcomes in the experiment
In the case of rolling a 6-sided die, if the die is “fair,” then the 6 outcomes are equally
likely. So the probability of rolling a 4 is , and the probability of rolling an odd
1
6
numberrolling a 1, 3, or 5can be calculated as 31
=.
62
The following are general facts about probability.
If an event Eis certain to occur, then P(E)=1.
If an event Eis certain not to occur, then P(E)=0.
If an event Eis possible but not certain to occur, then 0 <P(E)<1.
The probability that an event Ewill not occur is equal to 1 P(E).
If Eis an event, then the probability of Eis the sum of the probabilities of the
outcomes in E.
The sum of the probabilities of all possible outcomes of an experiment is 1.
If Eand Fare two events of an experiment, we consider two other events related
to Eand F.
The event that both Eand Foccur, that is, all outcomes in the set EF.
The event that Eor F, or both, occur, that is, all outcomes in the set EF.
Events that cannot occur at the same time are said to be mutually exclusive. For
example, if a 6-sided die is rolled once, the event of rolling an odd number and the
event of rolling an even number are mutually exclusive. But rolling a 4 and rolling an
even number are not mutually exclusive, since 4 is an outcome that is common to
both events.
For events Eand F, we have the following rules.
which is theP(Eor F, or both, occur) =P(E)+P(F)P(both Eand Foccur),
inclusion-exclusion principle applied to probability.
If Eand Fare mutually exclusive, then andP(both Eand Foccur) =0,
therefore, P(Eor F, or both, occur) =P(E)+P(F).
Eand Fare said to be independent if the occurrence of either event does not
affect the occurrence of the other. If two events Eand Fare independent, then
For example, if a fair 6-sided die is rolledP(both Eand Foccur) =P(E)P(F).
twice, the event Eof rolling a 3 on the first roll and the event Fof rolling a 3
on the second roll are independent, and the probability of rolling a 3 on both
rolls is In this example, the experiment is actually
11 1
P(E)P(F)==.
冢冣冢冣
66 36
This ebook was issued to HUAN-LIN CHANG, order #14803770223. Unlawful distribution of this ebook is prohibited.
Customer name HUAN-LIN CHANG , Order Id: 14803770223
GRE Math Review
282
“rolling the die twice,” and each outcome is an ordered pair of results like “4
on the first roll and 1 on the second roll.” But event Erestricts only the first
rollto a 3having no effect on the second roll; similarly, event Frestricts
only the second rollto a 3having no effect on the first roll.
Note that if and then events Eand Fcannot be bothP(E)0P(F)0,
mutually exclusive and independent. For if Eand Fare independent, then
P(both Eand Foccur) =P(E)P(F)0; but if Eand Fare mutually exclusive, then
P(both Eand Foccur) =0.
It is common to use the shorter notation “Eand F” instead of “both Eand Foc-
cur” and use “Eor F” instead of “Eor F, or both, occur.” With this notation, we have
the following rules.
P(Eor F)=P(E)+P(F)P(Eand F)
if Eand Fare mutually exclusive.P(Eor F)=P(E)+P(F)
if Eand Fare independent.P(Eand F)=P(E)P(F)
Example 4.4.2: If a fair 6-sided die is rolled once, let Ebe the event of rolling a 3 and
let Fbe the event of rolling an odd number. These events are not independent. This
is because rolling a 3 makes certain that the event of rolling an odd number occurs.
Note that sinceP(Eand F)P(E)P(F),
1111
P(Eand F)=P(E)=and P(E)P(F)==
冢冣冢冣
66212
Example 4.4.3: A 12-sided die, with faces numbered 1 to 12, is to be rolled once, and
each of the 12 possible outcomes is equally likely to occur. The probability of rolling
a 4 is so the probability of rolling a number that is not a 4 is The
1111
,1−=.
12 12 12
probability of rolling a number that is either a multiple of 5a 5 or a 10or an odd
numbera 1, 3, 5, 7, 9, or 11is equal to
26 1
P(multiple of 5) +P(odd) P(multiple of 5 and odd) =+ −
12 12 12
7
=12
Another way to calculate this probability is to notice that rolling a number that
is either a multiple of 5 (5 or 10) or an odd number (1, 3, 5, 7, 9, or 11) is the same
as rolling one of 1, 3, 5, 7, 9, 10, and 11, which are 7 equally likely outcomes. So by
using the ratio formula to calculate the probability, the required probability is 7.
12
Example 4.4.4: Consider an experiment with events A,B, and Cfor which P(A)=0.23,
and Suppose that events Aand Bare mutually exclusive andP(B)=0.40, P(C)=0.85.
events Band Care independent. What are the probabilities andP(Aor B)P(Bor C)?
Solution: Since Aand Bare mutually exclusive,
P(Aor B)=P(A)+P(B)=0.23 +0.40 =0.63
This ebook was issued to HUAN-LIN CHANG, order #14803770223. Unlawful distribution of this ebook is prohibited.
Customer name HUAN-LIN CHANG , Order Id: 14803770223
GRE Math Review
283
Since Band Care independent, So,P(Band C)=P(B)P(C).
P(Bor C)=P(B)+P(C)P(Band C)=P(B)+P(C)P(B)P(C)
Therefore,
P(Bor C)=0.40 +0.85 (0.40)(0.85) =1.25 0.34
=0.91
Example 4.4.5: Suppose that there is a 6-sided die that is weighted in such a way
that each time the die is rolled, the probabilities of rolling any of the numbers from
1 to 5 are all equal, but the probability of rolling a 6 is twice the probability of roll-
ing a 1. When you roll the die once, the 6 outcomes are not equally likely. What are
the probabilities of the 6 outcomes?
Solution: Using the notation for the probability of rolling a 1, let ThenP(1) p=P(1).
each of the probabilities of rolling a 2, 3, 4, or 5 is equal to p, and the probability of
rolling a 6 is 2p. Therefore, since the sum of the probabilities of all possible out-
comes is 1, it follows that
1=P(1) +P(2) +P(3) +P(4) +P(5) +P(6) =p+p+p+p+p+2p
=7p
So the probability of rolling each of the numbers from 1 to 5 is and the proba-
1
p=,
7
bility of rolling a 6 is 2.
7
Example 4.4.6: Suppose that you roll the weighted 6-sided die from example 4.4.5
twice. What is the probability that the first roll will be an odd number and the sec-
ond roll will be an even number?
Solution: To calculate the probability that the first roll will be odd and the second
roll will be even, note that these two events are independent. To calculate the proba-
bility that both occur, you must multiply the probabilities of the two independent
events. First compute the individual probabilities.
3
P(odd) =P(1) +P(3) +P(5) =7
4
P(even) =P(2) +P(4) +P(6) =7
Then, 34 12
P(first roll is odd and second roll is even) =P(odd)P(even) ==.
冢冣冢冣
77 49
Two events that happen sequentially are not always independent. The occurrence
of one event may affect the occurrence of a following event. In that case, the probabil-
ity that both events happen is equal to the probability that the first event happens
multiplied by the probability that given that the first event has already happened, the
second event happens as well.
Example 4.4.7: A box contains 5 orange disks, 4 red disks, and 1 blue disk. You are
to select two disks at random and without replacement from the box. What is the
probability that the first disk you select will be red and the second disk you select
will be orange?
This ebook was issued to HUAN-LIN CHANG, order #14803770223. Unlawful distribution of this ebook is prohibited.
Customer name HUAN-LIN CHANG , Order Id: 14803770223
GRE Math Review
284
Solution: To solve, you need to calculate the following two probabilities and then
multiply them
The probability that the first disk selected from the box will be red
The probability that the second disk selected from the box will be orange,
given that the first disk selected from the box is red
The probability that the first disk you select will be red is If the first disk
42
=.
10 5
you select is red, there will be 5 orange disks, 3 red disks, and 1 blue disk left in the
box, for a total of 9 disks. Therefore, the probability that the second disk you select
will be orange, given that the first disk you selected is red, is Multiply the two
5.
9
probabilities to get 25 2
=.
冢冣冢冣
59 9
4.5 Distributions of Data, Random Variables,
and Probability Distributions
In data analysis, variables whose values depend on chance play an important role in
linking distributions of data to probability distributions. Such variables are called ran-
dom variables. We begin with a review of distributions of data.
Distributions of Data
Recall that relative frequency distributions given in a table or histogram are a com-
mon way to show how numerical data are distributed. In a histogram, the areas of the
bars indicate where the data are concentrated. The histogram in example 4.1.6 illus-
trates a small group of data, with only 6 possible values and only 25 data altogether.
Many groups of data are much larger than 25 and have many more than 6 possible
values, which are often measurements of quantities like length, money, and time.
Example 4.5.1: The lifetimes of 800 electric devices were measured. Because the life-
times had many different values, the measurements were grouped into 50 intervals,
or classes, of 10 hours each: 601–610 hours, 611–620 hours, . . . , 1,091–1,100 hours.
The resulting relative frequency distribution, as a histogram, has 50 thin bars and
many different bar heights, as shown below.
m d
600 700 800 900 1,000 1,100
m + dm + 2dm + 3dMm
Relative Frequency Distribution for Lifetimes of 800 Electric Devices
This ebook was issued to HUAN-LIN CHANG, order #14803770223. Unlawful distribution of this ebook is prohibited.
Customer name HUAN-LIN CHANG , Order Id: 14803770223
GRE Math Review
285
Note that the tops of the bars of the histogram have a relatively smooth appear-
ance and begin to look like a curve. In general, histograms that represent very large
data sets with many classes appear to have smooth shapes. Consequently, the distribu-
tion can be modeled by a smooth curve that is close to the tops of the bars. Such a
model retains the shape of the distribution but is independent of classes.
Recall from example 4.1.6 that the sum of the areas of the bars of a relative fre-
quency histogram is 1. Although the units on the horizontal axis of a histogram vary
from one data set to another, the vertical scale can be adjusted (stretched or shrunk)
so that the sum of the areas of the bars is 1. With this vertical scale adjustment, the
area under the curve that models the distribution is also 1. This model curve is called
adistribution curve, but it has other names as well, including density curve and fre-
quency curve.
The purpose of the distribution curve is to give a good illustration of a large distri-
bution of numerical data that doesn’t depend on specific classes. To achieve this, the
main property of a distribution curve is that the area under the curve in any vertical
slice, just like a histogram bar, represents the proportion of the data that lies in the
corresponding interval on the horizontal axis, which is at the base of the slice.
Before leaving this histogram, note that the mean mand the median Mof the data
are marked on the horizontal axis. Also, several standard deviations above and below
the mean are marked, where dis the standard deviation of the data. The standard de-
viation marks show how most of the data are within about 3 standard deviations
above or below the mean (that is, between the numbers (not shown) andm3d
).m+3d
Finally, regarding the mean and the median, recall that the median splits the data
into a lower half and an upper half, so that the sum of the areas of the bars to the left
of Mis the same as the sum of the areas to the right. On the other hand, mtakes into
account the exact value of each of the data, not just whether a value is high or low.
The nature of the mean is such that if an imaginary fulcrum were placed somewhere
under the horizontal axis in order to balance the distribution perfectly, the balancing
position would be exactly at m. That is why mis somewhat to the right of M. The
balance point at mtakes into account how high the few very high values are (to the
far right), while Mjust counts them as “high.” To summarize, the median is the “halv-
ing point” and the mean is the “balance point.”
Random Variables
When analyzing data, it is common to choose a value of the data at random and con-
sider that choice as a random experiment, as introduced in section 4.4. Then, the
probabilities of events involving the randomly chosen value may be determined. Given
a distribution of data, a variable, say X, may be used to represent a randomly chosen
value from the distribution. Such a variable Xis an example of a random variable,
which is a variable whose value is a numerical outcome of a random experiment.
Example 4.5.2: In the data from example 4.1.1 consisting of numbers of children, let
Xrepresent the number of children in a randomly chosen family among the 25 fami-
lies. What is the probability that That That Xis less than the mean ofX=3? X>3?
the distribution?
This ebook was issued to HUAN-LIN CHANG, order #14803770223. Unlawful distribution of this ebook is prohibited.
Customer name HUAN-LIN CHANG , Order Id: 14803770223
GRE Math Review
286
Solution: For convenience, here is the frequency distribution of the data.
Number of
Children Frequency
0
1
2
3
4
5
Total
3
5
7
6
3
1
25
Since there are 6 families with 3 children and each of the 25 families is equally likely
to be chosen, the probability that a family with 3 children will be chosen is That
6.
25
is, is an event, and its probability is or 0.24. It is common to use
6
X=3P(X=3) =,
25
the shorter notation instead of so you could write NoteP(3) P(X=3), P(3) =0.24.
that in the histogram, shown below, the area of the bar corresponding to X=3asa
proportion of the combined areas of all of the bars is equal to this probability. This
indicates how probability is related to area in a histogram for a relative frequency
distribution.
Relative Frequency Distribution
0%
5%
10%
15%
20%
25%
30%
012345
Number of Children
Relative Frequency
As for the event it is the same as the event “ or ”. Because andX>3, X=4X=5X=4
are mutually exclusive events, we can use the rules of probability from sectionX=5
4.4.
31
P(X>3) =P(4) +P(5) =+=0.12 +0.04 =0.16
25 25
For the last question, first compute the mean of the distribution.
0(3) + 1(5) + 2(7) + 3(6) + 4(3) + 5(1) 54
= = 2.16
25 25
Then,
35715
P(X<2.16) =P(0) +P(1) +P(2) =++ = =0.6
25 25 25 25
This ebook was issued to HUAN-LIN CHANG, order #14803770223. Unlawful distribution of this ebook is prohibited.
Customer name HUAN-LIN CHANG , Order Id: 14803770223
GRE Math Review
287
A table showing all 6 possible values of Xand their probabilities is called the
probability distribution of the random variable X.
Probability Distribution of the Random Variable X
X
0
1
2
3
4
5
P(X)
0.12
0.20
0.28
0.24
0.12
0.04
Note that the probabilities are simply the relative frequencies of the 6 possible val-
ues expressed as decimals instead of percents. The following statement indicates a
fundamental link between data distributions and probability distributions.
For a random variable that represents a randomly chosen value from a distribution
of data, the probability distribution of the random variable is the same as the relative
frequency distribution of the data.
Because the probability distribution and the relative frequency distribution are es-
sentially the same, the probability distribution can be represented by a histogram.
Also, all of the descriptive statisticssuch as mean, median, and standard deviation
that apply to the distribution of data also apply to the probability distribution. For
example, we say that the probability distribution above has a mean of 2.16, a median
of 2, and a standard deviation of about 1.3, since the 25 data values have these statis-
tics, as you can check.
These statistics are similarly defined for the random variable Xabove. Thus, we
would say that the mean of the random variable Xis 2.16. Another name for the
mean of a random variable is expected value. So we would also say that the expected
value of Xis 2.16. Note that the mean of Xcan be expressed in terms of probabilities
as follows.
0(3) + 1(5) + 2(7) + 3(6) + 4(3) + 5(1) 357631
=0 +1 +2 +3 +4 +5
冢冣 冢冣 冢冣 冢冣 冢冣 冢冣
25 25 25 25 25 25 25
=0P(0)+1P(1)+2P(2)+3P(3)+4P(4)+5P(5)
which is the sum of the products that is, the sum of each value of XmultipliedXP(X),
by its corresponding probability P(X).
The preceding example involves a common type of random variableone that
represents a randomly chosen value from a distribution of data. However, the concept
of a random variable is more general. A random variable can be any quantity whose
value is the result of a random experiment. The possible values of the random variable
are the same as the outcomes of the experiment. So any random experiment with nu-
merical outcomes naturally has a random variable associated with it, as in the follow-
ing example.
This ebook was issued to HUAN-LIN CHANG, order #14803770223. Unlawful distribution of this ebook is prohibited.
Customer name HUAN-LIN CHANG , Order Id: 14803770223
GRE Math Review
288
Example 4.5.3: Let Yrepresent the outcome of the experiment in example 4.4.5 of
rolling a weighted 6-sided die. Then Yis a random variable with 6 possible values,
the numbers 1 through 6. Each value of Yhas a probability, which is listed in the
probability distribution of the random variable Yand is shown in a histogram for Y.
Probability Distribution of the Random Variable Y
Y123456
P(Y)1
7
1
7
1
7
1
7
1
7
2
7
Histogram for the Random Variable Y
0
3
7
612345
Value of Y
Probability
2
7
1
7
The mean, or expected value, of Ycan be computed as
111112
P(1) +2P(2) +3P(3) +4P(4) +5P(5) +6P(6) =+2+3+4+5+6
冢冣 冢冣 冢冣 冢冣 冢冣 冢冣
777777
1234512
=+++++
77777 7
27
=3.86
7
Both of the random variables Xand Yabove are examples of discrete random vari-
ables because their values consist of discrete points on a number line.
A basic fact about probability from section 4.4 is that the sum of the probabilities
of all possible outcomes of an experiment is 1, which can be confirmed by adding all
of the probabilities in each of the probability distributions for the random variables X
and Yabove. Also, the sum of the areas of the bars in a histogram for the probability
distribution of a random variable is 1. This fact is related to a fundamental link be-
tween the areas of the bars of a histogram and the probabilities of a discrete random
variable.
In the histogram for a random variable, the area of each bar is proportional to the
probability represented by the bar.
If the die in example 4.4.5 were a fair die instead of weighted, then the probability
of each of the outcomes would be and consequently, each of the bars in the histo-
1,
6
gram would have the same height. Such a flat histogram indicates a uniform distri-
bution, since the probability is distributed uniformly over all possible outcomes.
This ebook was issued to HUAN-LIN CHANG, order #14803770223. Unlawful distribution of this ebook is prohibited.
Customer name HUAN-LIN CHANG , Order Id: 14803770223
GRE Math Review
289
The Normal Distribution
Many natural processes yield data that have a relative frequency distribution shaped
somewhat like a bell, as in the distribution below with mean mand standard devia-
tion d.
m 3d m 2d
Approximately Normal Relative Frequency Distribution
m d mm + d m + 2dm + 3d
Such data are said to be approximately normally distributed and have the following
properties.
The mean, median, and mode are all nearly equal.
The data are grouped fairly symmetrically about the mean.
About two-thirds of the data are within 1 standard deviation of the mean.
Almost all of the data are within 2 standard deviations of the mean.
As stated above, you can always associate a random variable Xwith a distribution
of data by letting Xbe a randomly chosen value from the distribution. If Xis such a
random variable for the distribution above, we say that Xis approximately normally
distributed.
As described in example 4.5.1, relative frequency distributions are often approxi-
mated using a smooth curvea distribution curve or density curvefor the tops of
the bars in the histogram. The region below such a curve represents a distribution,
called a continuous probability distribution. There are many different continuous
probability distributions, but the most important one is the normal distribution,
which has a bell-shaped curve like the one shown in the figure below.
Normal Distribution
m 3d m 2dm d mm + d m + 2dm + 3d
This ebook was issued to HUAN-LIN CHANG, order #14803770223. Unlawful distribution of this ebook is prohibited.
Customer name HUAN-LIN CHANG , Order Id: 14803770223
GRE Math Review
290
Just as a data distribution has a mean and standard deviation, the normal probability
distribution has a mean and standard deviation. Also, the properties listed above for
the approximately normal distribution of data hold for the normal distribution, except
that the mean, median, and mode are exactly the same and the distribution is per-
fectly symmetric about the mean.
A normal distribution, though always shaped like a bell, can be centered around
any mean and can be spread out to a greater or lesser degree, depending on the stan-
dard deviation. Below are three normal distributions that have different centers and
spreads. From left to right, the means of the three distributions are 1, and 20;10,
and the standard deviations are 5, 10, and 2.
–30 –25 –20 –15 –10 –5 0 5 10 15 20 25 30
Three Normal Distributions
As mentioned earlier, areas of the bars in a histogram for a discrete random variable
correspond to probabilities for the values of the random variable; the sum of the areas
is 1 and the sum of the probabilities is 1. This is also true for a continuous probability
distribution: the area of the region under the curve is 1, and the areas of vertical slices
of the regionsimilar to the bars of a histogramare equal to probabilities of a ran-
dom variable associated with the distribution. Such a random variable is called a
continuous random variable, and it plays the same role as a random variable that
represents a randomly chosen value from a distribution of data. The main difference
is that we seldom consider the event in which a continuous random variable is equal
to a single value like ; rather, we consider events that are described by intervalsX=3
of values such as and Such events correspond to vertical slices under1 <X<3X>10.
a continuous probability distribution, and the areas of the vertical slices are the prob-
abilities of the corresponding events. (Consequently, the probability of an event such
as would correspond to the area of a line segment, which is 0.)X=3
Example 4.5.4:IfWis a random variable that is normally distributed with a mean of
5 and a standard deviation of 2, what is Approximately what isP(W>5) ?
Which of the four numbers 0.5, 0.1, 0.05, or 0.01 is the best estimateP(3 <W<7) ?
of P(W<−1) ?
This ebook was issued to HUAN-LIN CHANG, order #14803770223. Unlawful distribution of this ebook is prohibited.
Customer name HUAN-LIN CHANG , Order Id: 14803770223
GRE Math Review
291
–2 –1 0 1 2 3 4 5 6 7 8 9 10 11 12
Solution: Since the mean of the distribution is 5, and the distribution is symmetric
about the mean, the event corresponds to exactly half of the area under theW>5
normal distribution. So 1
P(W>5) =.
2
For the event note that since the standard deviation of the distribu-3 <W<7,
tion is 2, the values 3 and 7 are one standard deviation below and above the mean,
respectively. Since about two-thirds of the area is within one standard deviation of
the mean, is approximately 2
P(3 <W<7) .
3
For the event note that is 3 standard deviations below the mean.W<−1, 1
Since the graph makes it fairly clear that the area of the region under the normal
curve to the left of is much less than 5 percent of all of the area, the best of the1
four estimates given for is 0.01.P(W<−1)
The standard normal distribution is a normal distribution with a mean of 0 and
standard deviation equal to 1. To transform a normal distribution with a mean of m
and a standard deviation of dto a standard normal distribution, you standardize the
values (as explained below example 4.2.9); that is, you subtract mfrom any observed
value of the normal distribution and then divide the result by d.
Very precise values for probabilities associated with normal distributions can be
computed using calculators, computers, or statistical tables for the standard normal
distribution. For example, more precise values for and areP(3 <W<7) P(W<−1)
0.683 and 0.0013. Such calculations are beyond the scope of this review.
This ebook was issued to HUAN-LIN CHANG, order #14803770223. Unlawful distribution of this ebook is prohibited.
Customer name HUAN-LIN CHANG , Order Id: 14803770223
GRE Math Review
292
4.6 Data Interpretation Examples
Example 4.6.1:
DISTRIBUTION OF CUSTOMER COMPLAINTS
RECEIVED BY AIRLINE P, 2003 AND 2004
Category 2003 2004
Flight problem 20.0% 22.1%
Baggage 18.3 21.8
Customer service 13.1 11.3
Oversales of seats 10.5 11.8
Refund problem 10.1 8.1
Fare 6.4 6.0
Reservation and ticketing 5.8 5.6
Tours 3.3 2.3
Smoking 3.2 2.9
Advertising 1.2 1.1
Credit 1.0 0.8
Special passenger accommodation 0.9 0.9
Other 6.2 5.3
Total 100.0% 100.0%
Total number of complaints 22,998 13,278
(a) Approximately how many complaints concerning credit were received by Airline
Pin 2003?
(b) By approximately what percent did the total number of complaints decrease
from 2003 to 2004?
(c) Based on the information in the table, which of the following statements are
true?
I. In each of the years 2003 and 2004, complaints about flight problems,
baggage, and customer service together accounted for more than 50 percent
of all customer complaints received by Airline P.
II. The number of special passenger accommodation complaints was
unchanged from 2003 to 2004.
III. From 2003 to 2004, the number of flight problem complaints increased by
more than 2 percent.
This ebook was issued to HUAN-LIN CHANG, order #14803770223. Unlawful distribution of this ebook is prohibited.
Customer name HUAN-LIN CHANG , Order Id: 14803770223
GRE Math Review
293
Solutions:
(a) According to the table, in 2003, 1 percent of the total number of complaints
concerned credit. Therefore, the number of complaints concerning credit is
equal to 1 percent of 22,998. By converting 1 percent to its decimal equivalent,
you obtain that the number of complaints in 2003 is equal to (0.01)(22,998), or
about 230.
(b) The decrease in the total number of complaints from 2003 to 2004 was
22,998 13,278, or 9,720. Therefore, the percent decrease was 9,720 (100%),
冢冣
22,998
or approximately 42 percent.
(c) Since and are both greater than 50,
20.0 +18.3 +13.1 22.1 +21.8 +11.3
statement I is true. For statement II, the percent of special passenger
accommodation complaints did remain the same from 2003 to 2004, but the
number of such complaints decreased because the total number of complaints
decreased. Thus, statement II is false. For statement III, the percents shown in
the table for flight problems do in fact increase by more than 2 percentage
points, but the bases of the percents are different. The total number of
complaints in 2004 was much lower than the total number of complaints in
2003, and clearly 20 percent of 22,998 is greater than 22.1 percent of 13,278. So,
the number of flight problem complaints actually decreased from 2003 to 2004,
and statement III is false.
Example 4.6.2:
UNITED STATES PRODUCTION OF PHOTOGRAPHIC
EQUIPMENT AND SUPPLIES IN 1971
Total: $3,980 million
Sensitized Goods
47%
Office
Copiers
25%
4%
7%
12%
5% Motion-Picture
Equipment
Still-Picture
Equipment
Prepared Photochemicals
Microfilm E
q
ui
p
ment
(a) Approximately what was the ratio of the value of sensitized goods to the value of
still-picture equipment produced in 1971 in the United States?
(b) If the value of office copiers produced in 1971 was 30 percent greater than the
corresponding value in 1970, what was the value of office copiers produced in
1970 ?
This ebook was issued to HUAN-LIN CHANG, order #14803770223. Unlawful distribution of this ebook is prohibited.
Customer name HUAN-LIN CHANG , Order Id: 14803770223
GRE Math Review
294
Solutions:
(a) The ratio of the value of sensitized goods to the value of still-picture equipment
is equal to the ratio of the corresponding percents shown because the percents
have the same base, which is the total value. Therefore, the ratio is 47 to 12, or
approximately 4 to 1.
(b) The value of office copiers produced in 1971 was 0.25 times $3,980 million, or
$995 million. Therefore, if the corresponding value in 1970 was xmillion dollars,
then million. Solving for xyields so the value of office
995
1.3x=995 x=765,
1.3
copiers produced in 1970 was approximately $765 million.
Example 4.6.3:
FINISHING TIMES AND TRAINING INDICES
FOR 50 BICYCLISTS IN A RACE
010 20 30 40 50 60 70 80 90 100
Training Index
(units)
0.0
3.0
3.5
4.0
4.5
5.0
5.5
6.0
Finishing Time
(hours)
Trend line
A bicycle trainer studied 50 bicyclists to examine how the finishing time for a certain
bicycle race was related to the amount of physical training in the three months be-
fore the race. To measure the amount of training, the trainer developed a training
index, measured in “units” and based on the intensity of each bicyclist’s training. The
data and the trend of the data, represented by a line, are displayed in the scatterplot
above.
(a) How many of the 50 bicyclists had both a training index less than 50 units and a
finishing time less than 4.5 hours?
(b) What percent of the 10 fastest bicyclists in the race had a training index less
than 90 units?
This ebook was issued to HUAN-LIN CHANG, order #14803770223. Unlawful distribution of this ebook is prohibited.
Customer name HUAN-LIN CHANG , Order Id: 14803770223
GRE Math Review
295
Solutions:
(a) The number of bicyclists who had both a training index less than 50 units and a
finishing time less than 4.5 hours is equal to the number of points on the graph
to the left of 50 and below 4.5. Since there are five data points that are both to
the left of 50 units and below 4.5 hours, the correct answer is five.
(b) The 10 lowest data points represent the 10 fastest bicyclists. Of these 10 data
points, 3 points are to the right of 90 units, so the number of points to the left of
90 units is 7, which represents 70 percent of the 10 fastest bicyclists.
Example 4.6.4:
Africa Asia
TRAVELERS SURVEYED: 250
70
93 155
In a survey of 250 European travelers, 93 have traveled to Africa, 155 have traveled
to Asia, and of these two groups, 70 have traveled to both continents, as illustrated in
the Venn diagram above.
(a) How many of the travelers surveyed have traveled to Africa but not to Asia?
(b) How many of the travelers surveyed have traveled to at least one of the two
continents of Africa and Asia?
(c) How many of the travelers surveyed have traveled neither to Africa nor to Asia?
Solutions: In the Venn diagram, the rectangular region represents the set of all trav-
elers surveyed; the two circular regions represent the two sets of travelers to Africa
and Asia, respectively; and the shaded region represents the subset of those who
have traveled to both continents.
(a) The set described here is represented by the part of the left circle that is not
shaded. This description suggests that the answer can be found by taking the
shaded part away from the first circlein effect, subtracting the 70 from the 93,
to get 23 travelers who have traveled to Africa but not to Asia.
(b) The set described here is represented by that part of the rectangle that is in at
least one of the two circles. This description suggests adding the two numbers 93
and 155. But the 70 travelers who have traveled to both continents would be
counted twice in the sum To correct the double counting, subtract 7093 +155.
from the sum so that these 70 travelers are counted only once:
93 +155 70 =178
This ebook was issued to HUAN-LIN CHANG, order #14803770223. Unlawful distribution of this ebook is prohibited.
Customer name HUAN-LIN CHANG , Order Id: 14803770223
GRE Math Review
296
(c) The set described here is represented by the part of the rectangle that is not in
either circle. Let Nbe the number of these travelers. Note that the entire
rectangular region has two main nonoverlapping parts: the part outside the
circles and the part inside the circles. The first part represents Ntravelers and
the second part represents travelers (from question (b)).93 +155 70 =178
Therefore,
250 =N+178
and solving for Nyields
N=250 178 =72
DATA ANALYSIS EXERCISES
1. The daily temperatures, in degrees Fahrenheit, for 10 days in May were 61,
62, 65, 65, 65, 68, 74, 74, 75, and 77.
(a) Find the mean, median, mode, and range of the temperatures.
(b) If each day had been 7 degrees warmer, what would have been the mean,
median, mode, and range of those 10 temperatures?
2. The numbers of passengers on 9 airline flights were 22, 33, 21, 28, 22, 31, 44,
50, and 19. The standard deviation of these 9 numbers is approximately
equal to 10.2.
(a) Find the mean, median, mode, range, and interquartile range of the 9
numbers.
(b) If each flight had had 3 times as many passengers, what would have been
the mean, median, mode, range, interquartile range, and standard
deviation of the 9 numbers?
(c) If each flight had had 2 fewer passengers, what would have been the
interquartile range and standard deviation of the 9 numbers?
3. A group of 20 values has a mean of 85 and a median of 80. A different group
of 30 values has a mean of 75 and a median of 72.
(a) What is the mean of the 50 values?
(b) What is the median of the 50 values?
4. Find the mean and median of the values of the random variable X, whose
relative frequency distribution is given in the table below.
XRelative Frequency
0
1
2
3
4
0.18
0.33
0.10
0.06
0.33
This ebook was issued to HUAN-LIN CHANG, order #14803770223. Unlawful distribution of this ebook is prohibited.
Customer name HUAN-LIN CHANG , Order Id: 14803770223
GRE Math Review
297
5. Eight hundred insects were weighed, and the resulting measurements, in
milligrams, are summarized in the boxplot below.
150140130120110100
(a) What are the range, the three quartiles, and the interquartile range of the
measurements?
(b) If the 80th percentile of the measurements is 130 milligrams, about how
many measurements are between 126 milligrams and 130 milligrams?
6. In how many different ways can the letters in the word STUDY be ordered?
7. Martha invited 4 friends to go with her to the movies. There are 120 different
ways in which they can sit together in a row of 5 seats, one person per seat.
In how many of those ways is Martha sitting in the middle seat?
8. How many 3-digit positive integers are odd and do not contain the digit 5 ?
9. From a box of 10 lightbulbs, you are to remove 4. How many different sets of
4 lightbulbs could you remove?
10. A talent contest has 8 contestants. Judges must award prizes for first, second,
and third places, with no ties.
(a) In how many different ways can the judges award the 3 prizes?
(b) How many different groups of 3 people can get prizes?
11. If an integer is randomly selected from all positive 2-digit integers, what is
the probability that the integer chosen has
(a) a 4 in the tens place?
(b) at least one 4 in the tens place or the units place?
(c) no 4 in either place?
12. In a box of 10 electrical parts, 2 are defective.
(a) If you choose one part at random from the box, what is the probability
that it is not defective?
(b) If you choose two parts at random from the box, without replacement,
what is the probability that both are defective?
13. The table below shows the distribution of a group of 40 college students by
gender and class.
Sophomores Juniors Seniors
Males
Females
6
10
10
9
2
3
If one student is randomly selected from this group, find the probability that
the student chosen is
(a) not a junior
(b) a female or a sophomore
(c) a male sophomore or a female senior
This ebook was issued to HUAN-LIN CHANG, order #14803770223. Unlawful distribution of this ebook is prohibited.
Customer name HUAN-LIN CHANG , Order Id: 14803770223
GRE Math Review
298
14. Let A,B,C, and Dbe events for which P(Aor B)=0.6, P(A)=0.2,
and The events Aand Bare mutually exclusive,P(Cor D)=0.6, P(C)=0.5.
and the events Cand Dare independent.
(a) Find P(B)
(b) Find P(D)
15. Lin and Mark each attempt independently to decode a message. If the
probability that Lin will decode the message is 0.80 and the probability that
Mark will decode the message is 0.70, find the probability that
(a) both will decode the message
(b) at least one of them will decode the message
(c) neither of them will decode the message
16.
2% 2%
14% 14%
34% 34%
m 2dm d m m + dm + 2d
The figure above shows a normal distribution with mean mand standard
deviation d, including approximate percents of the distribution
corresponding to the six regions shown.
Suppose the heights of a population of 3,000 adult penguins are
approximately normally distributed with a mean of 65 centimeters and a
standard deviation of 5 centimeters.
(a) Approximately how many of the adult penguins are between 65
centimeters and 75 centimeters tall?
(b) If an adult penguin is chosen at random from the population,
approximately what is the probability that the penguin’s height will be
less than 60 centimeters? Give your answer to the nearest 0.05.
This ebook was issued to HUAN-LIN CHANG, order #14803770223. Unlawful distribution of this ebook is prohibited.
Customer name HUAN-LIN CHANG , Order Id: 14803770223
GRE Math Review
299
17.
1987 1989 1991 1993 1995
Year
PUBLIC AND PRIVATE SCHOOL EXPENDITURES
1987–2001
(in billions of dollars)
0
20
40
60
80
100
120
140
$160
0
20
40
60
80
100
120
140
$160
1997 1999 2001
Expenditures
Total
Public
Private
(a) For which year did total expenditures increase the most from the year
before?
(b) For 2001, private school expenditures were approximately what percent
of total expenditures?
18.
DISTRIBUTION OF WORKFORCE BY OCCUPATIONAL CATEGORY
FOR REGION Y IN 2001 AND PROJECTED FOR 2015
15% 20%
Service
Agricultural
Managerial
Sales
Clerical
Manufacturing 16%
24%
13%
7%
18%
18%
10%
8%
20%
28%
Professional 1% Professional 2%
Service
Agricultural
Managerial
2001
Total workforce: 150 million Total workforce: 175 million
2015 (Pro
j
ected)
Sales
Clerical
Manufacturing
(a) In 2001, how many categories each comprised more than 25 million
workers?
(b) What is the ratio of the number of workers in the Agricultural category in
2001 to the projected number of such workers in 2015 ?
(c) From 2001 to 2015, there is a projected increase in the number of
workers in which of the following categories?
I. Sales II. Service III. Clerical
This ebook was issued to HUAN-LIN CHANG, order #14803770223. Unlawful distribution of this ebook is prohibited.
Customer name HUAN-LIN CHANG , Order Id: 14803770223
GRE Math Review
300
19.
Mortgage, Insurance,
Property Taxes
Food
Automobile
Expenses
Savings
Miscellaneous
Income Taxes
*2003 Gross annual income: $50,000
2004 Gross annual income: $45,000
A FAMILY’S EXPENDITURES AND SAVINGS
AS A PERCENT OF ITS GROSS ANNUAL INCOME*
12
9
6
3
0
15
18
21
24
27
30
33
Percent
Utilities
2003
2004
(a) In 2003 the family used a total of 49 percent of its gross annual income
for two of the categories listed. What was the total amount of the family’s
income used for those same categories in 2004?
(b) Of the seven categories listed, which category of expenditure had the
greatest percent increase from 2003 to 2004 ?
ANSWERS TO DATA ANALYSIS EXERCISES
1. In degrees Fahrenheit, the statistics are
(a) mean =68.6, median =66.5, mode =65, range =16
(b) mean =75.6, median =73.5, mode =72, range =16
2. (a) mean =30, median =28, mode =22,
range =31, interquartile range =17
(b) mean =90, median =84, mode =66,
range =93, interquartile range =51,
940
standard deviation =330.7
9
(c) interquartile range =17, standard deviation 10.2
3. (a) mean =79
(b) The median cannot be determined from the information given.
4. mean =2.03, median =1
5. (a) range =41, Q=114, Q=118, Q=126, interquartile range =12
123
(b) 40 measurements
This ebook was issued to HUAN-LIN CHANG, order #14803770223. Unlawful distribution of this ebook is prohibited.
Customer name HUAN-LIN CHANG , Order Id: 14803770223
GRE Math Review
301
6. 5! =120
7. 24
8. 288
9. 210
10. (a) 336 (b) 56
11. (a) 1
9(b) 1
5(c) 4
5
12. (a) 4
5(b) 1
45
13. (a) 21
40 (b) 7
10 (c) 9
40
14. (a) 0.4 (b) 0.2
15. (a) 0.56 (b) 0.94 (c) 0.06
16. (a) 1,440 (b) 0.15
17. (a) 1998 (b) 19%
18. (a) Three (b) 9 to 14, or 9
14 (c) I, II, and III
19. (a) $17,550 (b) Miscellaneous
This ebook was issued to HUAN-LIN CHANG, order #14803770223. Unlawful distribution of this ebook is prohibited.
Customer name HUAN-LIN CHANG , Order Id: 14803770223
This page intentionally left blank
This ebook was issued to HUAN-LIN CHANG, order #14803770223. Unlawful distribution of this ebook is prohibited.
Customer name HUAN-LIN CHANG , Order Id: 14803770223
Taking Practice Test 1
Now that you have become familiar with the three measures of the revised
General Test, it is time to take the first practice revised General Test to see
how well you do. Practice Test 1 begins on the following pages. The total time
that you should allow for this practice test is 3 hours and 30 minutes. The time allotted
for each section appears at the beginning of that section.
Try to take Practice Test 1 under actual test conditions. Find a quiet place to work, and
set aside enough time to complete the test without being disturbed. Work on only one sec-
tion at a time. Use your watch or a timer to keep track of the time limit for each section.
For the Verbal Reasoning and Quantitative Reasoning portions of this practice test,
mark your answers directly in this book. However, when you take the real GRE revised
General Test on computer, you will click on your answer choices. For the Analytical
Writing sections of this test, how you respond will depend on how you plan to take the
actual test. If you plan to take the actual test on computer, you should type your re -
sponses. If you plan to take the paper-based test, you should handwrite your responses
on paper. You will need a supply of lined paper for this task.
It is important to note that this practice test is structured like the paper-based test.
As a result, the number of questions and time allowed per section are not the same as in
the computer-based test. The results of this practice test will give you an accurate guide
to your readiness to take the test, but for a more realistic experience of the computer-
based test, you should use the practice tests included in the POWERPREP II®software.
Following this practice test you will find an answer key. Check your answers
against the key, then follow the instructions for calculating your Verbal Reasoning and
Quantitative Reasoning scores and evaluating your Analytical Writing performance.
You will also find explanations for each test question. Review the explanations, paying
particular attention to the ones for the questions that you answered incorrectly.
Once you have worked your way through Practice Test 1, you will have a better idea
of how ready you are to take the actual GRE revised General Test. You will also have a
better sense of whether you are able to work fast enough to finish each section within
the time allowed, or whether you need to improve your test-taking speed. After you
have evaluated your test-taking performance, you can determine what type of addi-
tional preparation you might want to do for the test. For additional test-preparation
materials and suggestions, visit the GRE website at www.ets.org/gre/prepare. Once you
feel you have sufficiently prepared, you can take Practice Test 2 (Chapter 9).
gTake the first full-length authentic
GRE
®revised
General Test under actual test time limits.
gCheck your answers and read explanations for
every question.
gUse your results to identify your strengths and
weaknesses.
Your goals
for this
chapter
303
8
GRE
®
Practice Test 1
This ebook was issued to HUAN-LIN CHANG, order #14803770223. Unlawful distribution of this ebook is prohibited.
Customer name HUAN-LIN CHANG , Order Id: 14803770223
The Graduate Record
Examinations®
SECTION 1
Analytical Writing
ANALYZE AN ISSUE
30 minutes
You will be given a brief quotation that states or implies an issue of general interest and specific
instructions on how to respond to that issue. You will then have 30 minutes to plan and compose
a response according to the specific instructions. A response to any other issue will receive a
score of zero.
Make sure that you respond according to the specific instructions and support your position on the
issue with reasons and examples drawn from such areas as your reading, experience, observations,
and/or academic studies.
Trained GRE readers will evaluate your response for its overall quality, based on how well you:
bRespond to the specific task instructions
bConsider the complexities of the issue
bOrganize, develop, and express your ideas
bSupport your ideas with relevant reasons and/or examples
bControl the elements of standard written English
Before you begin writing, you may want to think for a few minutes about the issue and the specific
task instructions and then plan your response. Be sure to develop your position fully and organize
it coherently, but leave time to reread what you have written and make any revisions you think are
necessary.
304
GRE Practice Test 1
This ebook was issued to HUAN-LIN CHANG, order #14803770223. Unlawful distribution of this ebook is prohibited.
Customer name HUAN-LIN CHANG , Order Id: 14803770223
305
Section 1. Analytical Writing
Issue Topic
The best way to teach is to praise positive actions and ignore negative ones.
Write a response in which you discuss the extent to which you agree or disagree
with the recommendation and explain your reasoning for the position you take.
In developing and supporting your position, describe specific circumstances in
which adopting the recommendation would or would not be advantageous and
explain how these examples shape your position.
This ebook was issued to HUAN-LIN CHANG, order #14803770223. Unlawful distribution of this ebook is prohibited.
Customer name HUAN-LIN CHANG , Order Id: 14803770223
The Graduate Record
Examinations®
SECTION 2
Analytical Writing
ANALYZE AN ARGUMENT
30 minutes
You will be given a short passage that presents an argument and specific instructions on how to
respond to that passage. You will then have 30 minutes to plan and compose a response in which
you evaluate the passage according to the specific instructions. A response to any other argument
will receive a score of zero.
Note that you are NOT being asked to present your own views on the subject. Make sure that
you respond according to the specific instructions and support your evaluation with relevant
reasons and/or examples.
Trained GRE readers will evaluate your response for its overall quality, based on how well you:
bRespond to the specific task instructions
bIdentify and analyze features of the argument relevant to the assigned task
bOrganize, develop, and express your ideas
bSupport your analysis with relevant reasons and/or examples
bControl the elements of standard written English
Before you begin writing, you may want to think a few minutes about the argument passage and
the specific task instructions and then plan your response. Be sure to develop your response fully
and organize it coherently, but leave time to reread what you have written and make any revisions
that you think are necessary.
306
GRE Practice Test 1
This ebook was issued to HUAN-LIN CHANG, order #14803770223. Unlawful distribution of this ebook is prohibited.
Customer name HUAN-LIN CHANG , Order Id: 14803770223
307
Section 2. Analytical Writing
Argument Topic
The following appeared in an article written by Dr. Karp, an anthropologist.
“Twenty years ago, Dr. Field, a noted anthropologist, visited the island of Tertia and
concluded from his observations that children in Tertia were reared by an entire village
rather than by their own biological parents. However, my recent interviews with children
living in the group of islands that includes Tertia show that these children spend much more
time talking about their biological parents than about other adults in the village. This
research of mine proves that Dr. Field's conclusion about Tertian village culture is invalid
and thus that the observation-centered approach to studying cultures is invalid as well. The
interview-centered method that my team of graduate students is currently using in Tertia will
establish a much more accurate understanding of child-rearing traditions there and in other
island cultures.”
Write a response in which you discuss what specific evidence is needed to evaluate the
argument and explain how the evidence would weaken or strengthen the argument.
This ebook was issued to HUAN-LIN CHANG, order #14803770223. Unlawful distribution of this ebook is prohibited.
Customer name HUAN-LIN CHANG , Order Id: 14803770223
308
GRE Practice Test 1
NO TEST MATERIAL ON THIS PAGE
This ebook was issued to HUAN-LIN CHANG, order #14803770223. Unlawful distribution of this ebook is prohibited.
Customer name HUAN-LIN CHANG , Order Id: 14803770223
309
Section 3. Verbal Reasoning
GO ON TO NEXT PAGE g
SECTION 3
Verbal Reasoning
Time —35 minutes
25 Questions
For each of Questions 1 to 4, select one answer choice unless otherwise instructed.
Questions 1 to 3 are based on the following reading passage.
Whether the languages of the ancient American peoples were used for expressing
abstract universal concepts can be clearly answered in the case of Nahuatl. Nahuatl,
like Greek and German, is a language that allows the formation of extensive com-
pounds. By the combination of radicals or semantic elements, single compound words
can express complex conceptual relations, often of an abstract universal character.
The tlamatinime (those who know) were able to use this rich stock of abstract terms
to express the nuances of their thought. They also availed themselves of other forms of
expression with metaphorical meaning, some probably original, some derived from
Toltec coinages. Of these forms, the most characteristic in Nahuatl is the juxtaposition
of two words that, because they are synonyms, associated terms, or even contraries,
complement each other to evoke one single idea. Used metaphorically, the juxtaposed
terms connote specific or essential traits of the being they refer to, introducing a mode
of poetry as an almost habitual form of expression.
For the following question, consider each of the choices separately and select all that
apply.
1. Which of the following can be inferred from the passage regarding present-day
research relating to Nahuatl?
A
Some record or evidence of the thought of the tlamatinime is available.
B
For at least some Nahuatl expressions, researchers are able to trace their
derivation from another ancient American language.
C
Researchers believe that in Nahuatl, abstract universal concepts are
always expressed metaphorically.
2. Select the sentence in the passage in which the author introduces a specific
Nahuatl mode of expression that is not identified as being shared with certain
European languages.
3. In the context in which it appears, “coinages” (line 9) most nearly means
A
adaptations
B
creations
C
idiosyncrasies
D
pronunciations
E
currencies
line
5
10
This ebook was issued to HUAN-LIN CHANG, order #14803770223. Unlawful distribution of this ebook is prohibited.
Customer name HUAN-LIN CHANG , Order Id: 14803770223
Question 4 is based on the following reading passage.
At a certain period in Earth’s history, its atmosphere contained almost no oxygen,
although plants were producing vast quantities of oxygen. As a way of reconciling these
two facts, scientists have hypothesized that nearly all of the oxygen being produced was
taken up by iron on Earth’s surface. Clearly, however, this explanation is inadequate.
New studies show that the amount of iron on Earth’s surface was not sufficient to
absorb anywhere near as much oxygen as was being produced. Therefore, some-
thing in addition to the iron on Earth’s surface must have absorbed much of the oxy-
gen produced by plant life.
4. In the argument given, the two portions in boldface play which of the following
roles?
A
The first is a claim made by the argument in support of a certain position;
the second is that position.
B
The first is a judgment made by the argument about a certain explanation;
the second is that explanation.
C
The first expresses the argument’s dismissal of an objection to the position
it seeks to establish; the second is that position.
D
The first sums up the argument’s position with regard to a certain
hypothesis; the second provides grounds for that position.
E
The first is a concession by the argument that its initial formulation of the
position it seeks to establish requires modification; the second presents
that position in a modified form.
For Questions 5 to 8, select one entry for each blank from the corresponding column
of choices. Fill all blanks in the way that best completes the text.
5. In her later years, Bertha Pappenheim was an apostle of noble but already
(i)__________ notions, always respected for her integrity, her energy, and her
resolve but increasingly out of step and ultimately (ii)__________ even her own
organization.
Blank (i) Blank (ii)
A
anachronistic
D
emulated by
B
accepted
E
appreciated by
C
exotic
F
alienated from
6. The reception given to Kimura’s radical theory of molecular evolution shows that
when __________ fights orthodoxy to a draw, then novelty has seized a good
chunk of space from convention.
A
imitation
B
reaction
C
dogmatism
D
invention
E
caution
310
GRE Practice Test 1
This ebook was issued to HUAN-LIN CHANG, order #14803770223. Unlawful distribution of this ebook is prohibited.
Customer name HUAN-LIN CHANG , Order Id: 14803770223
7. The (i)__________ of Vladimir Nabokov as one of North America’s literary giants
has thrown the spotlight on his peripheral activities and has thus served to
(ii)__________ his efforts as an amateur entomologist.
Blank (i) Blank (ii)
A
stigmatization
D
foreground
B
lionization
E
transcend
C
marginalization
F
obscure
8. Mathematicians have a distinctive sense of beauty: they strive to present their
ideas and results in a clear and compelling fashion, dictated by __________ as
well as by logic.
A
caprice
B
aesthetics
C
obligation
D
methodologies
E
intellect
311
Section 3. Verbal Reasoning
GO ON TO NEXT PAGE g
This ebook was issued to HUAN-LIN CHANG, order #14803770223. Unlawful distribution of this ebook is prohibited.
Customer name HUAN-LIN CHANG , Order Id: 14803770223
312
GRE Practice Test 1
For each of Questions 9 to 13, select one answer choice unless otherwise instructed.
Questions 9 to 12 are based on the following reading passage.
Animal signals, such as the complex songs of birds, tend to be costly. A bird, by singing,
may forfeit time that could otherwise be spent on other important behaviors such as
foraging or resting. Singing may also advertise an individual’s location to rivals or pred-
ators and impair the ability to detect their approach. Although these types of cost may
be important, discussions of the cost of singing have generally focused on energy costs.
Overall the evidence is equivocal: for instance, while Eberhardt found increases in
energy consumption during singing for Carolina wrens, Chappell found no effect of
crowing on energy consumption in roosters.
To obtain empirical data regarding the energy costs of singing, Thomas examined
the relationship between song rate and overnight changes in body mass of male
nightingales. Birds store energy as subcutaneous fat deposits or “body reserves”;
changes in these reserves can be reliably estimated by measuring changes in body
mass. If singing has important energy costs, nightingales should lose more body mass
on nights when their song rate is high. Thomas found that nightingales reached a sig-
nificantly higher body mass at dusk and lost more mass overnight on nights when their
song rate was high.
These results suggest that there may be several costs of singing at night associated
with body reserves. The increased metabolic cost of possessing higher body mass con-
tributes to the increased overnight mass loss. The strategic regulation of evening body
reserves is also likely to incur additional costs, as nightingales must spend more time
foraging in order to build up larger body reserves. The metabolic cost of singing itself
may also contribute to increased loss of reserves. This metabolic cost may arise from
the muscular and neural activity involved in singing or from behaviors associated with
singing. For example, birds may expend more of their reserves on thermoregulation if
they spend the night exposed to the wind on a song post than if they are in a sheltered
roost site. Thomas’s data therefore show that whether or not singing per se has an
important metabolic cost, metabolic costs associated with singing can have an impor-
tant measurable effect on a bird’s daily energy budget, at least in birds with high song
rates such as nightingales.
9. The primary purpose of the passage is to
A
compare the different types of cost involved for certain birds in singing
B
question a hypothesis regarding the energy costs of singing for certain
birds
C
present evidence suggesting that singing has an important energy cost for
certain birds
D
discuss the benefits provided to an organism by a behavior that is costly in
energy
E
describe an experiment that supports an alternative model of how
birdsong functions
line
5
10
15
20
25
This ebook was issued to HUAN-LIN CHANG, order #14803770223. Unlawful distribution of this ebook is prohibited.
Customer name HUAN-LIN CHANG , Order Id: 14803770223
For the following question, consider each of the choices separately and select all that
apply.
10. The passage implies that during the day before a night on which a male
nightingale’s song rate is high, that nightingale probably does which of the
following?
A
Expends less of its reserves on thermoregulation than on other days
B
Stores more energy as body reserves than on other days
C
Hides to avoid predators
11. Select the sentence in the first or second paragraph that presents empirical
results in support of a hypothesis about the energy costs of singing.
For the following question, consider each of the choices separately and select all that
apply.
12. It can be inferred from the passage that compared with other costs of singing,
which of the following is true of the energy costs of singing?
A
They are the single greatest cost to an individual bird.
B
They have generally received more attention from scientists.
C
They vary less from one bird species to another.
Question 13 is based on the following reading passage.
In the past ten years, there have been several improvements in mountain-climbing
equipment. These improvements have made the sport both safer and more enjoyable
for experienced climbers. Despite these improvements, however, the rate of mountain-
climbing injuries has doubled in the past ten years.
13. Which of the following, if true, best reconciles the apparent discrepancy
presented in the passage?
A
Many climbers, lulled into a false sense of security, use the new equipment
to attempt climbing feats of which they are not capable.
B
Some mountain-climbing injuries are caused by unforeseeable weather
conditions.
C
Mountain climbing, although a dangerous sport, does not normally result
in injury to the experienced climber.
D
In the past ten years there have been improvements in mountain-climbing
techniques as well as in mountain-climbing equipment.
E
Although the rate of mountain-climbing injuries has increased, the rate of
mountain-climbing deaths has not changed.
313
Section 3. Verbal Reasoning
GO ON TO NEXT PAGE g
This ebook was issued to HUAN-LIN CHANG, order #14803770223. Unlawful distribution of this ebook is prohibited.
Customer name HUAN-LIN CHANG , Order Id: 14803770223
For Questions 14 to 17, select one entry for each blank from the corresponding
column of choices. Fill all blanks in the way that best completes the text.
14. Unenlightened authoritarian managers rarely recognize a crucial reason for the
low levels of serious conflict among members of democratically run work groups:
a modicum of tolerance for dissent often prevents __________.
A
demur
B
schism
C
cooperation
D
compliance
E
shortsightedness
15. The novelist devotes so much time to avid descriptions of his characters’ clothes
that the reader soon feels that such __________ concerns, although worthy of
attention, have superseded any more directly literary aims.
A
didactic
B
syntactical
C
irrelevant
D
sartorial
E
frivolous
16. Belanger dances with an (i)__________ that draws one’s attention as if by seeking
to (ii)__________ it; through finesse and understatement, he manages to seem at
once intensely present and curiously detached.
Blank (i) Blank (ii)
A
undemonstrative panache
D
focus
B
unrestrained enthusiasm
E
overwhelm
C
unattractive gawkiness
F
deflect
17. The most striking thing about the politician is how often his politics have been
(i)__________ rather than ideological, as he adapts his political positions at any
particular moment to the political realities that constrain him. He does not,
however, piously (ii)__________ political principles only to betray them in
practice. Rather, he attempts in subtle ways to balance his political self-interest
with a (iii) __________, viewing himself as an instrument of some unchanging
higher purpose.
Blank (i) Blank (ii) Blank (iii)
A
quixotic
D
brandish
G
profound
cynicism
B
self-righteous
E
flout
H
deeply felt moral
code
C
strategic
F
follow
I
thoroughgoing
pragmatism
314
GRE Practice Test 1
This ebook was issued to HUAN-LIN CHANG, order #14803770223. Unlawful distribution of this ebook is prohibited.
Customer name HUAN-LIN CHANG , Order Id: 14803770223
315
Section 3. Verbal Reasoning
GO ON TO NEXT PAGE g
For each of Questions 18 to 20, select one answer choice unless otherwise instructed.
Questions 18 to 20 are based on the following reading passage.
The condition of scholarship devoted to the history of women in photography is con-
founding. Recent years have witnessed the posthumous inflation of the role of the
hobby ist Alice Austen into that of a pioneering documentarian while dozens of notable
senior figures Marion Palfi, whose photographs of civil-rights activities in the South
served as early evidence of the need for protective legislation, to name one received
scant attention from scholars. And, while Naomi Rosenblum’s synoptic History of
Women Photographers covers the subject through 1920 in a generally useful fashion,
once she reaches the 1920s, when the venues, forms, applications, and movements of
the medium expanded exponentially, she resorts to an increasingly terse listing of un -
familiar names, with approaches and careers summarized in a sentence or two.
18. The author of the passage cites Rosenblum’s book most likely in order to
A
suggest that the works documented most thoroughly by historians of
women in photography often do not warrant that attention
B
offer an explanation for the observation that not all aspects of the history
of women in photography have received the same level of attention
C
provide an example of a way in which scholarship on the history of
women in photography has been unsatisfactory
D
suggest that employing a strictly chronological approach when studying
the history of women in photography may be unproductive
E
provide support for the notion that certain personalities in women’s
photography have attained undue prominence
For the following question, consider each of the choices separately and select all that
apply.
19. Which of the following statements about Marion Palfi is supported by the
passage?
A
Marion Palfi’s photographs would have received greater recognition from
historians had her work been done in an era when most aspects of
photography were static rather than in a state of transition.
B
Alice Austen has achieved greater notoriety than has Marion Palfi
primarily because the subjects that Austen photographed were more
familiar to her contemporaries.
C
In addition to providing a record of certain historical events, Marion
Palfi’s photographs played a role in subsequent events.
20. In the context in which it appears, “inflation” (line 2) most nearly means
A
exaggeration
B
acquisition
C
evaluation
D
distortion
E
attenuation
line
5
10
This ebook was issued to HUAN-LIN CHANG, order #14803770223. Unlawful distribution of this ebook is prohibited.
Customer name HUAN-LIN CHANG , Order Id: 14803770223
316
GRE Practice Test 1
For Questions 21 to 25, select the two answer choices that, when used to complete
the sentence, fit the meaning of the sentence as a whole and produce completed
sentences that are alike in meaning.
21. The plan, which the engineers said would save the aquifer by reducing pumping
to __________ levels, has passed a governmental environmental review but faces
opposition from outdoor and environmental groups.
A
innocuous
B
feasible
C
practicable
D
minimal
E
remedial
F
benign
22. Though feminist in its implications, Yvonne Rainer’s 1974 film __________ the
filmmaker’s active involvement in feminist politics.
A
antedated
B
cloaked
C
portrayed
D
preceded
E
renewed
F
represented
23. Congress is having great difficulty developing a consensus on energy policy,
primarily because the policy objectives of various members of Congress rest on
such __________ assumptions.
A
commonplace
B
disparate
C
divergent
D
fundamental
E
trite
F
trivial
This ebook was issued to HUAN-LIN CHANG, order #14803770223. Unlawful distribution of this ebook is prohibited.
Customer name HUAN-LIN CHANG , Order Id: 14803770223
317
Section 3. Verbal Reasoning
24. During the opera’s most famous aria, the tempo chosen by the orchestra’s
conductor seemed __________, without necessary relation to what had gone
before.
A
arbitrary
B
capricious
C
cautious
D
compelling
E
exacting
F
meticulous
25. Because they had expected the spacecraft Voyager 2 to be able to gather data only
about the planets Jupiter and Saturn, scientists were __________ the wealth of
information it sent back from Neptune twelve years after leaving Earth.
A
anxious for
B
confident in
C
thrilled about
D
keen on
E
elated by
F
eager for
STOP. This is the end of Section 3.
This ebook was issued to HUAN-LIN CHANG, order #14803770223. Unlawful distribution of this ebook is prohibited.
Customer name HUAN-LIN CHANG , Order Id: 14803770223
318
GRE Practice Test 1
SECTION 4
Verbal Reasoning
Time — 35 minutes
25 Questions
For Questions 1 to 4, select the two answer choices that, when used to complete
the sentence, fit the meaning of the sentence as a whole and produce completed
sentences that are alike in meaning.
1. Only by ignoring decades of mismanagement and inefficiency could investors
conclude that a fresh infusion of cash would provide anything other than a
__________ solution to the company’s financial woes.
A
complete
B
fleeting
C
momentary
D
premature
E
trivial
F
total
2. Some scientists argue that carbon compounds play such a central role in life on
Earth because of the possibility of __________ resulting from the carbon atom’s
ability to form an unending series of different molecules.
A
diversity
B
deviation
C
variety
D
reproduction
E
stability
F
invigoration
3. Given the flood of information presented by the mass media, the only way for
someone to keep abreast of the news is to rely on __________ accounts.
A
synoptic
B
abridged
C
sensational
D
copious
E
lurid
F
understated
This ebook was issued to HUAN-LIN CHANG, order #14803770223. Unlawful distribution of this ebook is prohibited.
Customer name HUAN-LIN CHANG , Order Id: 14803770223
319
Section 4. Verbal Reasoning
GO ON TO NEXT PAGE g
4. Always circumspect, she was reluctant to make judgments, but once arriving at a
conclusion, she was __________ in its defense.
A
deferential
B
intransigent
C
lax
D
negligent
E
obsequious
F
resolute
This ebook was issued to HUAN-LIN CHANG, order #14803770223. Unlawful distribution of this ebook is prohibited.
Customer name HUAN-LIN CHANG , Order Id: 14803770223
320
GRE Practice Test 1
For each of Questions 5 to 9, select one answer choice unless otherwise instructed.
Questions 5 and 6 are based on the following reading passage.
When marine organisms called phytoplankton photosynthesize, they absorb carbon
dioxide dissolved in seawater, potentially causing a reduction in the concentration of
atmospheric carbon dioxide, a gas that contributes to global warming. However, phy-
toplankton flourish only in surface waters where iron levels are sufficiently high. Mar-
tin therefore hypothesized that adding iron to iron-poor regions of the ocean could
help alleviate global warming. While experiments subsequently confirmed that such a
procedure increases phytoplankton growth, field tests have shown that such growth
does not significantly lower atmospheric carbon dioxide. When phytoplankton utilize
carbon dioxide for photosynthesis, the carbon becomes a building block for organic
matter, but the carbon leaks back into the atmosphere when predators consume the
phytoplankton and respire carbon dioxide.
For the following question, consider each of the choices separately and select all that
apply.
5. It can be inferred from the passage that Martin’s hypothesis includes which of the
following elements?
A
A correct understanding of how phytoplankton photosynthesis utilizes
carbon dioxide
B
A correct prediction about how the addition of iron to iron-poor waters
would affect phytoplankton growth
C
An incorrect prediction about how phytoplankton growth would affect the
concentration of atmospheric carbon dioxide
6. It can be inferred that the author of the passage mentions predators (line 10)
primarily in order to
A
help explain why Martin’s hypothesis is incorrect
B
identify one effect of adding iron to iron-poor waters
C
indicate how some carbon dioxide is converted to solid organic matter
D
help account for differences in the density of phytoplankton between
different regions of the ocean
E
point out a factor that was not anticipated by the scientists who
conducted the field tests mentioned in the passage
line
5
10
This ebook was issued to HUAN-LIN CHANG, order #14803770223. Unlawful distribution of this ebook is prohibited.
Customer name HUAN-LIN CHANG , Order Id: 14803770223
321
Section 4. Verbal Reasoning
GO ON TO NEXT PAGE g
Question 7 is based on the following reading passage.
Sparva, unlike Treland’s other provinces, requires automobile insurers to pay for any
medical treatment sought by someone who has been involved in an accident; in the
other provinces, insurers pay for nonemergency treatment only if they preapprove the
treatment. Clearly, Sparva’s less restrictive policy must be the explanation for the fact
that altogether insurers there pay for far more treatments after accidents than insurers
in other provinces, even though Sparva does not have the largest population.
7. Which of the following, if true, most strengthens the argument?
A
Car insurance costs more in Sparva than in any other province.
B
The cost of medical care in Sparva is higher than the national average.
C
Different insurance companies have different standards for determining
what constitutes emergency treatment.
D
Fewer insurance companies operate in Sparva than in any other province.
E
There are fewer traffic accidents annually in Sparva than in any of the
provinces of comparable or greater population.
This ebook was issued to HUAN-LIN CHANG, order #14803770223. Unlawful distribution of this ebook is prohibited.
Customer name HUAN-LIN CHANG , Order Id: 14803770223
Questions 8 and 9 are based on the following reading passage.
Elements of the Philosophy of Newton, published by Voltaire in 1738, was an early
attempt to popularize the scientific ideas of Isaac Newton. In the book’s frontispiece,
Voltaire is seen writing at his desk, and over him a shaft of light from heaven, the light
of truth, passes through Newton to Voltaire’s collaborator Madame du Châtelet; she
reflects that light onto the inspired Voltaire. Voltaire’s book commanded a wide audi-
ence, according to Feingold, because “he was neither a mathematician nor a physicist,
but a literary giant aloof from the academic disputes over Newtonian ideas.” In other
words, Voltaire’s amateurism in science “was a source of his contemporary appeal,
demonstrating for the first time the accessibility of Newton’s ideas to nonspecialists.”
For the following question, consider each of the choices separately and select all that
apply.
8. Which of the following statements about Voltaire’s Elements of the Philosophy of
Newton can be inferred from the passage?
A
Voltaire’s literary stature helped secure a large audience for this attempt to
popularize Newton’s ideas.
B
Voltaire’s status as a nonscientist was an advantage in this effort to bring
Newtonian science to the attention of the general public.
C
The frontispiece of the book implies that Voltaire’s understanding of
Newton’s ideas was not achieved without assistance.
9. Select the sentence that describes an allegory for Voltaire’s acquisition of
knowledge concerning Newton’s ideas.
For Questions 10 to 13, select one entry for each blank from the corresponding
column of choices. Fill all blanks in the way that best completes the text.
10. Ironically, the writer so wary of (i)__________ was (ii)__________ with ink and
paper, his novel running to 2,500 shagreen-bound folio pages a fortune in
stationery at the time.
Blank (i) Blank (ii)
A
probity
D
acquisitive
B
extravagance
E
illiberal
C
disapprobation
F
profligate
322
GRE Practice Test 1
This ebook was issued to HUAN-LIN CHANG, order #14803770223. Unlawful distribution of this ebook is prohibited.
Customer name HUAN-LIN CHANG , Order Id: 14803770223
323
Section 4. Verbal Reasoning
GO ON TO NEXT PAGE g
11. What readers most commonly remember about John Stuart Mill’s classic
exploration of the liberty of thought and discussion concerns the danger of
(i)__________: in the absence of challenge, one’s opinions, even when they are
correct, grow weak and flabby. Yet Mill had another reason for encouraging the
liberty of thought and discussion: the danger of partiality and incompleteness.
Since one’s opinions, even under the best circumstances, tend to (ii)__________,
and because opinions opposed to one’s own rarely turn out to be completely
(iii)__________, it is crucial to supplement one’s opinions with alternative points
of view.
Blank (i) Blank (ii) Blank (iii)
A
tendentiousness
D
embrace only a
G
erroneous
portion of the truth
B
complacency
E
change over
H
antithetical
time
C
fractiousness
F
focus on matters
I
immutable
close at hand
12. Just as the authors’ book on eels is often a key text for courses in marine
vertebrate zoology, their ideas on animal development and phylogeny __________
teaching in this area.
A
prevent
B
defy
C
replicate
D
inform
E
use
13. Mechanisms develop whereby every successful species can __________ its innate
capacity for population growth with the constraints that arise through its
interactions with the natural environment.
A
enhance
B
replace
C
produce
D
surpass
E
reconcile
This ebook was issued to HUAN-LIN CHANG, order #14803770223. Unlawful distribution of this ebook is prohibited.
Customer name HUAN-LIN CHANG , Order Id: 14803770223
324
GRE Practice Test 1
For each of Questions 14 to 16, select one answer choice unless otherwise instructed.
Questions 14 to 16 are based on the following reading passage.
It would be expected that a novel by a figure as prominent as W. E. B. DuBois would
attract the attention of literary critics. Additionally, when the novel subtly engages the
issue of race, as DuBois’ The Quest of the Silver Fleece (1911) does, it would be a sur-
prise not to encounter an abundance of scholarly work about that text. But though
valuable scholarship has examined DuBois’ political and historical thought, his novels
have received scant attention. Perhaps DuBois the novelist must wait his turn behind
DuBois the philosopher, historian, and editor. But what if the truth lies elsewhere: what
if his novels do not speak to current concerns?
14. Which of the following can be inferred from the passage regarding DuBois’ The
Quest of the Silver Fleece?
A
The lack of attention devoted to The Quest of the Silver Fleece can be
attributed to the fact that it was DuBois’ first novel.
B
Among DuBois’ novels, The Quest of the Silver Fleece is unusual in that it
has received scant attention from scholars.
C
The Quest of the Silver Fleece has at least one feature that typically would
attract the attention of literary scholars.
D
The Quest of the Silver Fleece, given its subtle exploration of race, is
probably the best novel written by DuBois.
E
Much of the scholarly work that has focused on The Quest of the Silver
Fleece has been surprisingly critical of it.
15. In the fourth sentence (“Perhaps DuBois . . . editor.”), the author of the passage is
most likely suggesting that
A
scholars will find that DuBois’ novels are more relevant to current
concerns than is his work as philosopher, historian, and editor
B
more scholarly attention will be paid to The Quest of the Silver Fleece than
to DuBois’ other novels
C
DuBois’ novels will come to overshadow his work as philosopher,
historian, and editor
D
DuBois’ novels may eventually attract greater scholarly interest than they
have to date
E
it will be shown that DuBois’ work as philosopher, historian, and editor
had an important influence on his work as novelist
This ebook was issued to HUAN-LIN CHANG, order #14803770223. Unlawful distribution of this ebook is prohibited.
Customer name HUAN-LIN CHANG , Order Id: 14803770223
325
Section 4. Verbal Reasoning
GO ON TO NEXT PAGE g
16. Which of the following best describes the central issue with which the passage is
concerned?
A
The perfunctoriness of much of the critical work devoted to DuBois’
novels
B
The nature of DuBois’ engagement with the issue of race in The Quest of
the Silver Fleece
C
Whether DuBois’ novels are of high quality and relevant to current
concerns
D
The relationship between DuBois the novelist and DuBois the philosopher,
historian, and editor
E
The degree of consideration that has been given to DuBois’ novels,
including The Quest of the Silver Fleece
For Questions 17 to 20, select one entry for each blank from the corresponding
column of choices. Fill all blanks in the way that best completes the text.
17. In the midst of so many evasive comments, this forthright statement, whatever its
intrinsic merit, plainly stands out as __________.
A
a paradigm
B
a misnomer
C
a profundity
D
an inaccuracy
E
an anomaly
18. The activists’ energetic work in the service of both woman suffrage and the
temperance movement in the late nineteenth century (i)__________ the assertion
that the two movements were (ii)__________.
Blank (i) Blank (ii)
A
undermines
D
diffuse
B
supports
E
inimical
C
underscores
F
predominant
19. There is nothing quite like this movie, and indeed I am not altogether sure there
is much more to it than its lovely (i) __________. At a moment when so many
films strive to be as (ii)__________ as possible, it is gratifying to find one that is so
subtle and puzzling.
Blank (i) Blank (ii)
A
peculiarity
D
indirect
B
pellucidity
E
assertive
C
conventionality
F
enigmatic
This ebook was issued to HUAN-LIN CHANG, order #14803770223. Unlawful distribution of this ebook is prohibited.
Customer name HUAN-LIN CHANG , Order Id: 14803770223
20. Wills argues that certain malarial parasites are especially (i)__________ because
they have more recently entered humans than other species and therefore have
had (ii)__________ time to evolve toward (iii)__________. Yet there is no reliable
evidence that the most harmful Plasmodium species has been in humans for a
shorter time than less harmful species.
Blank (i) Blank (ii) Blank (iii)
A
populous
D
ample
G
virulence
B
malignant
E
insufficient
H
benignity
C
threatened
F
adequate
I
variability
For each of Questions 21 to 25, select one answer choice unless otherwise instructed.
Question 21 is based on the following reading passage.
Saturn’s giant moon Titan is the only planetary satellite with a significant atmosphere
and the only body in the solar system other than Earth that has a thick atmosphere
dominated by molecular nitrogen. For a long time, the big question about Titan’s
atmosphere was how it could be so thick, given that Jupiter’s moons Ganymede and
Callisto, which are the same size as Titan, have none. The conditions for acquiring and
retaining a thick nitrogen atmosphere are now readily understood. The low tempera-
ture of the protosaturnian nebula enabled Titan to acquire the moderately volatile com-
pounds methane and ammonia (later converted to nitrogen) in addition to water. The
higher temperatures of Jupiter’s moons, which were closer to the Sun, prevented them
from acquiring such an atmosphere.
21. According to the passage, Titan differs atmospherically from Ganymede and
Callisto because of a difference in
A
rate of heat loss
B
proximity to the Sun
C
availability of methane and ammonia
D
distance from its planet
E
size
326
GRE Practice Test 1
This ebook was issued to HUAN-LIN CHANG, order #14803770223. Unlawful distribution of this ebook is prohibited.
Customer name HUAN-LIN CHANG , Order Id: 14803770223
Question 22 is based on the following reading passage.
Observations of the Arctic reveal that the Arctic Ocean is covered by less ice each sum-
mer than the previous summer. If this warming trend continues, within 50 years the
Arctic Ocean will be ice free during the summer months. This occurrence would in
itself have little or no effect on global sea levels, since the melting of ice floating in
water does not affect the water level. However, serious consequences to sea levels
would eventually result, because __________.
22. Which of the following most logically completes the passage?
A
large masses of floating sea ice would continue to form in the wintertime
B
significant changes in Arctic sea temperatures would be accompanied by
changes in sea temperatures in more temperate parts of the world
C
such a warm Arctic Ocean would trigger the melting of massive land-
based glaciers in the Arctic
D
an ice-free Arctic Ocean would support a very different ecosystem than it
does presently
E
in the spring, melting sea ice would cause more icebergs to be created and
to drift south into shipping routes
327
Section 4. Verbal Reasoning
GO ON TO NEXT PAGE g
This ebook was issued to HUAN-LIN CHANG, order #14803770223. Unlawful distribution of this ebook is prohibited.
Customer name HUAN-LIN CHANG , Order Id: 14803770223
Questions 23 to 25 are based on the following reading passage.
In a recent study, David Cressy examines two central questions concerning English
immigration to New England in the 1630s: what kinds of people immigrated and why?
Using contemporary literary evidence, shipping lists, and customs records, Cressy finds
that most adult immigrants were skilled in farming or crafts, were literate, and were
organized in families. Each of these characteristics sharply distinguishes the 21,000
people who left for New England in the 1630s from most of the approximately 377,000
English people who had immigrated to America by 1700.
With respect to their reasons for immigrating, Cressy does not deny the frequently
noted fact that some of the immigrants of the 1630s, most notably the organizers and
clergy, advanced religious explanations for departure, but he finds that such explana-
tions usually assumed primacy only in retrospect. When he moves beyond the princi-
pal actors, he finds that religious explanations were less frequently offered, and he
concludes that most people immigrated because they were recruited by promises of
material improvement.
For the following question, consider each of the choices separately and select all that
apply.
23. The passage indicates that Cressy would agree with which of the following
statements about the organizers among the English immigrants to New England
in the 1630s?
A
Some of them offered a religious explanation for their immigration.
B
They did not offer any reasons for their immigration until some time after
they had immigrated.
C
They were more likely than the average immigrant to be motivated by
material considerations.
24. Select the sentence that provides Cressy’s opinion about what motivated English
immigrants to go to New England in the 1630s.
25. In the passage, the author is primarily concerned with
A
summarizing the findings of an investigation
B
analyzing a method of argument
C
evaluating a point of view
D
hypothesizing about a set of circumstances
E
establishing categories
STOP. This is the end of Section 4.
328
GRE Practice Test 1
line
5
10
This ebook was issued to HUAN-LIN CHANG, order #14803770223. Unlawful distribution of this ebook is prohibited.
Customer name HUAN-LIN CHANG , Order Id: 14803770223
Section 5. Quantitative Reasoning
329
SECTION 5
Quantitative Reasoning
Time40 minutes
25 Questions
For each question, indicate the best answer, using the directions given.
Notes: All numbers used are real numbers.
All figures are assumed to lie in a plane unless otherwise indicated.
Geometric figures, such as lines, circles, triangles, and quadrilaterals, are not
necessarily drawn to scale. That is, you should not assume that quantities
such as lengths and angle measures are as they appear in a figure. You
should assume, however, that lines shown as straight are actually straight,
points on a line are in the order shown, and more generally, all geometric
objects are in the relative positions shown. For questions with geometric fig-
ures, you should base your answers on geometric reasoning, not on estimat-
ing or comparing quantities by sight or by measurement.
Coordinate systems, such as xy-planes and number lines, are drawn to scale;
therefore, you can read, estimate, or compare quantities in such figures by
sight or by measurement.
Graphical data presentations, such as bar graphs, circle graphs, and line
graphs, are drawn to scale; therefore, you can read, estimate, or compare
data values by sight or by measurement.
For each of Questions 1 to 9, compare Quantity A and Quantity B, using
additional information centered above the two quantities if such information is
given. Select one of the following four answer choices and fill in the
corresponding oval to the right of the question.
AQuantity A is greater.
BQuantity B is greater.
CThe two quantities are equal.
DThe relationship cannot be determined from the information given.
A symbol that appears more than once in a question has the same meaning
throughout the question.
Quantity A Quantity B Correct Answer
Example 1: (2)(6) 2 +6ABCD
––––––––––––––––––––––––––––––––––––
Q
RP S
Quantity A Quantity B Correct Answer
Example 2: PS SR ABCD
(since equal lengths cannot
be assumed, even though
PS and SR appear equal)
This ebook was issued to HUAN-LIN CHANG, order #14803770223. Unlawful distribution of this ebook is prohibited.
Customer name HUAN-LIN CHANG , Order Id: 14803770223
GRE Practice Test 1
330
AQuantity A is greater.
BQuantity B is greater.
CThe two quantities are equal.
DThe relationship cannot be determined from the information given.
O
3
3
4
x
Ois the center of the circle above.
Quantity A Quantity B
1. x5ABCD
Runner Aran kilometer and Runner Bran 800 meters.
4
5
Quantity A Quantity B
2. The distance that Aran The distance that Bran ABCD
x<y<z
Quantity A Quantity B
3. x+y+z
3
yABCD
x
y
50°
Quantity A Quantity B
4. x
y
1ABCD
This ebook was issued to HUAN-LIN CHANG, order #14803770223. Unlawful distribution of this ebook is prohibited.
Customer name HUAN-LIN CHANG , Order Id: 14803770223
Section 5. Quantitative Reasoning
331
AQuantity A is greater.
BQuantity B is greater.
CThe two quantities are equal.
DThe relationship cannot be determined from the information given.
0<x<y<1
Quantity A Quantity B
5. 1 yyxABCD
pis the probability that event Ewill occur, and sis the
probability that event Ewill not occur.
Quantity A Quantity B
6. p+sps
ABCD
Xis the set of all integers nthat satisfy the inequality
2≤⎪n⎪≤5.
Quantity A Quantity B
7. The absolute value of the
greatest integer in X
The absolute value of the
least integer in X
ABCD
xand mare positive numbers, and mis a multiple of 3.
Quantity A Quantity B
8.
m
x
3
x
m/3
xABCD
A random variable Yis normally distributed with a
mean of 200 and a standard deviation of 10.
Quantity A Quantity B
9. The probability of the event
that the value of Yis
greater than 220
1
6
ABCD
GO ON TO THE NEXT PAGE b
This ebook was issued to HUAN-LIN CHANG, order #14803770223. Unlawful distribution of this ebook is prohibited.
Customer name HUAN-LIN CHANG , Order Id: 14803770223
GRE Practice Test 1
332
Questions 10 to 25 have several different formats. Unless otherwise directed,
select a single answer choice. For Numeric Entry questions, follow the
instructions below.
Numeric Entry Questions
Enter your answer in the answer box(es) below the question.
Your answer may be an integer, a decimal, or a fraction, and it may be
negative.
If a question asks for a fraction, there will be two boxesone for the
numerator and one for the denominator.
Equivalent forms of the correct answer, such as 2.5 and 2.50, are all correct.
Fractions do not need to be reduced to lowest terms.
Enter the exact answer unless the question asks you to round your answer.
10. The ratio of is equal to the ratio of
13
to
38
A1to8
B8to1
C8to3
D8to9
E9to8
11. A reading list for a humanities course consists of 10 books, of which 4 are
biographies and the rest are novels. Each student is required to read a
selection of 4 books from the list, including 2 or more biographies. How
many selections of 4 books satisfy the requirements?
A90
B115
C130
D144
E195
For the following question, enter your answer in the box.
12. In a graduating class of 236 students, 142 took algebra and 121 took chemistry.
What is the greatest possible number of students that could have taken both
algebra and chemistry?
students
This ebook was issued to HUAN-LIN CHANG, order #14803770223. Unlawful distribution of this ebook is prohibited.
Customer name HUAN-LIN CHANG , Order Id: 14803770223
Section 5. Quantitative Reasoning
333
m
k
s°
t°
13. In the figure above, if mkand s=t+30, then t=
A30
B60
C75
D80
E105
14. If then2x=3y=4z=20, 12xyz =
A16,000
B8,000
C4,000
D800
E10
For the following question, select all the answer choices that apply.
15. The total amount that Mary paid for a book was equal to the price of the book
plus a sales tax that was 4 percent of the price of the book. Mary paid for the
book with a $10 bill and received the correct change, which was less than $3.00.
Which of the following statements must be true?
Indicate all such statements.
AThe price of the book was less than $9.50.
BThe price of the book was greater than $6.90.
CThe sales tax was less than $0.45.
16. If is expressed as a terminating decimal, how many nonzero digits
1
11 17
(2 )(5 )
will the decimal have?
AOne
BTw o
CFour
DSix
EEleven
GO ON TO THE NEXT PAGE b
This ebook was issued to HUAN-LIN CHANG, order #14803770223. Unlawful distribution of this ebook is prohibited.
Customer name HUAN-LIN CHANG , Order Id: 14803770223
GRE Practice Test 1
334
Questions 17 to 20 are based on the following data.
VARIATION IN THE AMOUNT OF CAFFEINE IN COMMON BEVERAGES AND DRUGS*
Amount of Caffeine (milligrams)
Decaffeinated coffee
Source: Food and Drug Administration
Coffee
Drugs
Other
beverages
Percolated coffee
Drip-brewed coffee
Instant coffee
Brewed tea
Instant tea
Cocoa
Caffeinated soft drinks
Weight-loss drugs,
diuretics, and stimulants
Pain relievers
Cold/allergy remedies
025 50 75 100 125 150 175 200
*Based on 5-ounce cups of coffee, tea, and cocoa; 12-ounce cups of soft drinks;
and sin
g
le doses of dru
g
s.
17. The least amount of caffeine in a 5-ounce cup of drip-brewed coffee exceeds
the greatest amount of caffeine in a 5-ounce cup of cocoa by approximately
how many milligrams?
A160
B80
C60
D40
E20
For the following question, enter your answer in the box.
18. For how many of the 11 categories of beverages and drugs listed in the graph
can the amount of caffeine in the given serving size be less than 50 milligrams?
categories
19. Approximately what is the minimum amount of caffeine, in milligrams,
consumed per day by a person who daily drinks two 10-ounce mugs of
percolated coffee and one 12-ounce cup of a caffeinated soft drink?
A230
B190
C140
D110
E70
This ebook was issued to HUAN-LIN CHANG, order #14803770223. Unlawful distribution of this ebook is prohibited.
Customer name HUAN-LIN CHANG , Order Id: 14803770223
Section 5. Quantitative Reasoning
335
20. Which of the following shows the four types of coffee listed in order
according to the range of the amounts of caffeine in a 5-ounce cup, from
the least range to the greatest range?
ADecaffeinated, instant, percolated, drip-brewed
BDecaffeinated, instant, drip-brewed, percolated
CInstant, decaffeinated, drip-brewed, percolated
DInstant, drip-brewed, decaffeinated, percolated
EInstant, percolated, drip-brewed, decaffeinated
300
200
100
0102
S
p
eed (miles
p
er hour)
ENERGY USED PER METER VERSUS SPEED
DURING RUNNING AND WALKING
Energy Used per Meter
(joules per meter)
34 56789
Running
Walking
21. If sis a speed, in miles per hour, at which the energy used per meter during
running is twice the energy used per meter during walking, then, according
to the graph above, sis between
A2.5 and 3.0
B3.0 and 3.5
C3.5 and 4.0
D4.0 and 4.5
E4.5 and 5.0
22. If then
3n
n=2, n=
A
6
2
B
11
2
C
18
2
D
24
2
E
27
2
GO ON TO THE NEXT PAGE b
This ebook was issued to HUAN-LIN CHANG, order #14803770223. Unlawful distribution of this ebook is prohibited.
Customer name HUAN-LIN CHANG , Order Id: 14803770223
GRE Practice Test 1
336
For the following question, select all the answer choices that apply.
A
C
B
D
The length of AB is 10 3.
23. Which of the following statements individually provide(s) sufficient additional
information to determine the area of triangle ABC above?
Indicate all such statements.
ADBC is an equilateral triangle.
BABD is an isosceles triangle.
CThe length of BC is equal to the length of AD.
DThe length of BC is 10.
EThe length of AD is 10.
For the following question, enter your answer in the box.
a,a,a,...,a,...
123 n
24. In the sequence above, each term after the first term is equal to the preceding
term plus the constant c. If what is the value ofa+a+a=27, a+a?
135 24
a+a=
24
25. A desert outpost has a water supply that is sufficient to last 21 days for 15
people. At the same average rate of water consumption per person, how many
days would the water supply last for 9 people?
A28.0
B32.5
C35.0
D37.5
E42.0
STOP. This is the end of Section 5.
This ebook was issued to HUAN-LIN CHANG, order #14803770223. Unlawful distribution of this ebook is prohibited.
Customer name HUAN-LIN CHANG , Order Id: 14803770223
Section 6. Quantitative Reasoning
337
SECTION 6
Quantitative Reasoning
Time40 minutes
25 Questions
For each question, indicate the best answer, using the directions given.
Notes: All numbers used are real numbers.
All figures are assumed to lie in a plane unless otherwise indicated.
Geometric figures, such as lines, circles, triangles, and quadrilaterals, are not
necessarily drawn to scale. That is, you should not assume that quantities
such as lengths and angle measures are as they appear in a figure. You
should assume, however, that lines shown as straight are actually straight,
points on a line are in the order shown, and more generally, all geometric
objects are in the relative positions shown. For questions with geometric fig-
ures, you should base your answers on geometric reasoning, not on estimat-
ing or comparing quantities by sight or by measurement.
Coordinate systems, such as xy-planes and number lines, are drawn to scale;
therefore, you can read, estimate, or compare quantities in such figures by
sight or by measurement.
Graphical data presentations, such as bar graphs, circle graphs, and line
graphs, are drawn to scale; therefore, you can read, estimate, or compare
data values by sight or by measurement.
For each of Questions 1 to 9, compare Quantity A and Quantity B, using
additional information centered above the two quantities if such information is
given. Select one of the following four answer choices and fill in the
corresponding oval to the right of the question.
AQuantity A is greater.
BQuantity B is greater.
CThe two quantities are equal.
DThe relationship cannot be determined from the information given.
A symbol that appears more than once in a question has the same meaning
throughout the question.
Quantity A Quantity B Correct Answer
Example 1: (2)(6) 2+6ABCD
––––––––––––––––––––––––––––––––––––
Q
RP S
Quantity A Quantity B Correct Answer
Example 2: PS SR ABCD
(since equal lengths cannot
be assumed, even though
PS and SR appear equal)
This ebook was issued to HUAN-LIN CHANG, order #14803770223. Unlawful distribution of this ebook is prohibited.
Customer name HUAN-LIN CHANG , Order Id: 14803770223
GRE Practice Test 1
338
AQuantity A is greater.
BQuantity B is greater.
CThe two quantities are equal.
DThe relationship cannot be determined from the information given.
Country Value of 1 United States Dollar
Argentina 0.93 peso
Kenya 32.08 shillings
Quantity A Quantity B
1. The dollar value of 1
Argentine peso according
to the table above
The dollar value of 1
Kenyan shilling according
to the table above
ABCD
kis a digit in the decimal 1.3k5, and 1.3k5 is less
than 1.33.
Quantity A Quantity B
2. k1ABCD
A
CBD
AB is a diameter of the circle above.
Quantity A Quantity B
3. The length of AB The average (arithmetic
mean) of the lengths of AC
and AD
ABCD
st=10
Quantity A Quantity B
4.
2
s10
2
t
ABCD
This ebook was issued to HUAN-LIN CHANG, order #14803770223. Unlawful distribution of this ebook is prohibited.
Customer name HUAN-LIN CHANG , Order Id: 14803770223
Section 6. Quantitative Reasoning
339
AQuantity A is greater.
BQuantity B is greater.
CThe two quantities are equal.
DThe relationship cannot be determined from the information given.
Three consecutive integers have a sum of 84.
Quantity A Quantity B
5. The least of the three
integers
28 ABCD
In the xy-plane, the equation of line kis 3x2y=0.
Quantity A Quantity B
6. The x-intercept of line kThe y-intercept of line kABCD
nis a positive integer that is divisible by 6.
Quantity A Quantity B
7. The remainder when nis
divided by 12
The remainder when nis
divided by 18
ABCD
1x1
=
x1x
Quantity A Quantity B
8. x1
2
ABCD
In a set of 24 positive integers, 12 of the integers are
less than 50. The rest are greater than 50.
Quantity A Quantity B
9. The median of the 24
integers
50 ABCD
GO ON TO THE NEXT PAGE b
This ebook was issued to HUAN-LIN CHANG, order #14803770223. Unlawful distribution of this ebook is prohibited.
Customer name HUAN-LIN CHANG , Order Id: 14803770223
GRE Practice Test 1
340
Questions 10 to 25 have several different formats. Unless otherwise directed,
select a single answer choice. For Numeric Entry questions, follow the
instructions below.
Numeric Entry Questions
Enter your answer in the answer box(es) below the question.
Your answer may be an integer, a decimal, or a fraction, and it may be
negative.
If a question asks for a fraction, there will be two boxesone for the
numerator and one for the denominator.
Equivalent forms of the correct answer, such as 2.5 and 2.50, are all correct.
Fractions do not need to be reduced to lowest terms.
Enter the exact answer unless the question asks you to round your answer.
10. The fabric needed to make 3 curtains sells for $8.00 per yard and can be
purchased only by the full yard. If the length of fabric required for each
curtain is 1.6 yards and all of the fabric is purchased as a single length,
what is the total cost of the fabric that needs to be purchased for the 3
curtains?
A$40.00
B$38.40
C$24.00
D$16.00
E$12.80
For the following question, select all the answer choices that apply.
11. In the xy-plane, line kis a line that does not pass through the origin.
Which of the following statements individually provide(s) sufficient additional
information to determine whether the slope of line kis negative?
Indicate all such statements.
AThe x-intercept of line kis twice the y-intercept of line k.
BThe product of the x-intercept and the y-intercept of line kis positive.
CLine kpasses through the points and where(a,b)(r,s),
(ar)(bs)<0.
This ebook was issued to HUAN-LIN CHANG, order #14803770223. Unlawful distribution of this ebook is prohibited.
Customer name HUAN-LIN CHANG , Order Id: 14803770223
Section 6. Quantitative Reasoning
341
Distance from Centerville
(miles)
Freight train 10t+ 115
Passenger train 20t+ 150
12. The expressions in the table above give the distance from Centerville to
each of two trains thours after 12:00 noon. At what time after 12:00 noon
will the trains be equidistant from Centerville?
A1:30
B3:30
C5:10
D8:50
E11:30
13. The company at which Mark is employed has 80 employees, each of whom
has a different salary. Mark’s salary of $43,700 is the second-highest salary
in the first quartile of the 80 salaries. If the company were to hire 8 new
employees at salaries that are less than the lowest of the 80 salaries, what
would Mark’s salary be with respect to the quartiles of the 88 salaries at the
company, assuming no other changes in the salaries?
AThe fourth-highest salary in the first quartile
BThe highest salary in the first quartile
CThe second-lowest salary in the second quartile
DThe third-lowest salary in the second quartile
EThe fifth-lowest salary in the second quartile
For the following question, enter your answer in the box.
14. In the xy-plane, the point with coordinates is the center of circle C.(6, 7)
The point with coordinates lies inside C, and the point with(6, 5)
coordinates lies outside C.Ifmis the radius of Cand mis an(8, 7)
integer, what is the value of m?
m=
15. If is an even integer, which of the following must be true?
m
19
Amis a negative number.
Bmis a positive number.
Cmis a prime number.
Dmis an odd integer.
Emis an even integer.
GO ON TO THE NEXT PAGE b
This ebook was issued to HUAN-LIN CHANG, order #14803770223. Unlawful distribution of this ebook is prohibited.
Customer name HUAN-LIN CHANG , Order Id: 14803770223
GRE Practice Test 1
342
For the following question, select all the answer choices that apply.
16. The integer vis greater than 1. If vis the square of an integer, which of the
following numbers must also be the square of an integer?
Indicate all such numbers.
A81v
B25v+10 v+1
C
2
4v+4v+1
Questions 17 to 20 are based on the following data.
110
100
90
80
70
60
50
40
30
20
10
0
Distance (feet)
Speed (miles per hour)
0 102030405060708090100
Speed (miles per hour)
0 102030405060708090100
Distance Traveled
During Reaction Time*
550
500
450
400
350
300
250
200
150
100
50
0
Distance (feet)
Distance Traveled After Brakes
Have Been Applied
922 49
88
137
198
550
445
352
269
DISTANCE TRAVELED BY A CAR ACCORDING TO THE CAR’S SPEED
WHEN THE DRIVER IS SIGNALED TO STOP
*Reaction time is the time period that begins when the driver is signaled to stop
and ends when the driver applies the brakes.
Note: Total stopping distance is the sum of the distance traveled during reaction time
and the distance traveled after brakes have been applied.
17. The speed, in miles per hour, at which the car travels a distance of 52 feet
during reaction time is closest to which of the following?
A43
B47
C51
D55
E59
This ebook was issued to HUAN-LIN CHANG, order #14803770223. Unlawful distribution of this ebook is prohibited.
Customer name HUAN-LIN CHANG , Order Id: 14803770223
Section 6. Quantitative Reasoning
343
18. Approximately what is the total stopping distance, in feet, if the car is
traveling at a speed of 40 miles per hour when the driver is signaled to
stop?
A130
B110
C90
D70
E40
19. Of the following, which is the greatest speed, in miles per hour, at which
the car can travel and stop with a total stopping distance of less than 200
feet?
A50
B55
C60
D65
E70
20. The total stopping distance for the car traveling at 60 miles per hour is
approximately what percent greater than the total stopping distance for the
car traveling at 50 miles per hour?
A22%
B30%
C38%
D45%
E52%
21. What is the least positive integer that is not a factor of and is not a
25!
prime number?
A26
B28
C36
D56
E58
22. If which of the following is true about the reciprocals of a
0<a<1<b,
and b?
A11
1<<
ab
B11
<1<
ab
C11
<<1
ab
D11
<1<
ba
E11
<<1
ba GO ON TO THE NEXT PAGE b
This ebook was issued to HUAN-LIN CHANG, order #14803770223. Unlawful distribution of this ebook is prohibited.
Customer name HUAN-LIN CHANG , Order Id: 14803770223
GRE Practice Test 1
344
OP
23. In the figure above, Oand Pare the centers of the two circles. If each circle
has radius r, what is the area of the shaded region?
A2
2
r
2
B3
2
r
2
C
2
2r
D
2
3r
E
2
23r
For the following question, enter your answer in the boxes.
24. Of the 20 lightbulbs in a box, 2 are defective. An inspector will select 2
lightbulbs simultaneously and at random from the box. What is the probability
that neither of the lightbulbs selected will be defective?
Give your answer as a fraction.
25. What is the perimeter, in meters, of a rectangular playground 24 meters wide
that has the same area as a rectangular playground 64 meters long and 48
meters wide?
A112
B152
C224
D256
E304
STOP. This is the end of Section 6.
This ebook was issued to HUAN-LIN CHANG, order #14803770223. Unlawful distribution of this ebook is prohibited.
Customer name HUAN-LIN CHANG , Order Id: 14803770223
Evaluating Your Performance
Now that you have completed Practice Test 1, it is time to evaluate your performance.
Analytical Writing Measure
One way to evaluate your performance on the Issue and Argument topics you answered
on this practice test is to compare your essay responses to the scored sample essay
responses for these topics and review the rater commentary. Scored sample essay
responses and rater commentary are presented starting on page 353 for the one Issue
topic and one Argument topic presented in the Analytical Writing sections of Practice
Test 1. The Issue and Argument scoring guides start on page 37.
To better understand the analytical writing abilities characteristic of particular
score levels, you should review the score level descriptions on page 41.
Verbal Reasoning and Quantitative Reasoning Measures
The tables that follow contain information to help you evaluate your performance on
the Verbal Reasoning and Quantitative Reasoning measures of Practice Test 1. An
answer key with the correct answers to the questions in the Verbal Reasoning and
Quantitative Reasoning sections in this practice test begins on page 347. Compare your
answers to the correct answers given in the table, crossing out questions you answered
incorrectly or omitted. Partially correct answers should be treated as incorrect. Know-
ing which questions you answered incorrectly or omitted can help you identify content
areas in which you need more practice or review.
The answer key contains additional information to help you evaluate your per-
formance. With each answer, the key provides a number, the P+. The P+ is the percent
of a group of actual GRE takers who were administered that same question at a previ-
ous test administration and who answered it correctly. P+ is used to gauge the relative
difficulty of a test question. The higher the P+, the easier the test question. You can use
the P+ to compare your performance on each test question to the performance of other
test takers on that same question. For example, if the P+ for a question is 89, that
means that 89 percent of GRE test takers who received this question answered it cor-
rectly. Alternatively, if the P+ for a question is 14, that means that 14 percent of GRE
test takers who received this question answered it correctly. A question with a P+ of 89
may be interpreted as a relatively easy question, and a question with a P+ of 14 may be
interpreted as a difficult question.
To calculate your scores on Practice Test 1:
bAdd the number of correct answers in Sections 3 and 4 to obtain your raw
Verbal Reasoning score.
bAdd the number of correct answers in Sections 5 and 6 to obtain your raw
Quantitative Reasoning score.
bOnce you have calculated your raw scores, refer to the Practice Test 1 score
conversion table on pages 351–352. Find the scores on the 130–170 score scales
that correspond to your Verbal Reasoning and Quantitative Reasoning raw
scores. Note the scaled scores provided.
345
Evaluating Your Performance
This ebook was issued to HUAN-LIN CHANG, order #14803770223. Unlawful distribution of this ebook is prohibited.
Customer name HUAN-LIN CHANG , Order Id: 14803770223
Once you determine your scaled scores, you will need to evaluate your perform-
ance. To get a sense of how test takers are scoring on the Verbal Reasoning and Quan-
titative Reasoning measures of the actual test, you can review Verbal Reasoning and
Quantitative Reasoning percentile ranks on the GRE website at www.ets.org/gre
/percentile (PDF). A percentile rank for a score indicates the percentage of examinees
who took that test and received a lower score. Updated annually in July, this table
includes the Verbal Reasoning and Quantitative Reasoning scores on the 130–170 scale
in one-point increments and the corresponding percentile ranks. For each score you
earned on Practice Test 1, note the percent of GRE test takers who earned lower scores.
This is a reasonable indication of your rank among GRE revised General Test exami-
nees if you took Practice Test 1 under standard timed conditions.
346
GRE Practice Test 1
This ebook was issued to HUAN-LIN CHANG, order #14803770223. Unlawful distribution of this ebook is prohibited.
Customer name HUAN-LIN CHANG , Order Id: 14803770223
Answer Key
Section 3. Verbal Reasoning
Question
Number P+ Correct Answer
139Choice A: Some record or evidence of the thought of the tlamatinime is
available.
AND
Choice B: For at least some Nahuatl expressions, researchers are able to
trace their derivation from another ancient American language.
254Sentence 6: Of these forms, the most characteristic in Nahuatl is the
juxtaposition of two words that, because they are synonyms, associated
terms, or even contraries, complement each other to evoke one single idea.
349Choice B: creations
448Choice D: The first sums up the argument’s position with regard to a certain
hypothesis; the second provides grounds for that position.
557Choice A: anachronistic; Choice F: alienated from
663Choice D: invention
720Choice B: lionization; Choice D: foreground
858Choice B: aesthetics
969Choice C: present evidence suggesting that singing has an important energy
cost for certain birds
10 66 Choice B: Stores more energy as body reserves than on other days
11 65 Sentence 9: Thomas found that nightingales reached a significantly higher
body mass at dusk and lost more mass overnight on nights when their song
rate was high.
12 14 Choice B: They have generally received more attention from scientists.
13 86 Choice A: Many climbers, lulled into a false sense of security, use the new
equipment to attempt climbing feats of which they are not capable.
14 46 Choice B: schism
15 10 Choice D: sartorial
16 36 Choice A: undemonstrative panache; Choice F: deflect
17 18 Choice C: strategic; Choice D: brandish; Choice H: deeply felt moral code
18 47 Choice C: provide an example of a way in which scholarship on the history
of women in photography has been unsatisfactory
19 37 Choice C: In addition to providing a record of certain historical events,
Marion Palfi’s photographs played a role in subsequent events.
20 72 Choice A: exaggeration
21 24 Choice A: innocuous AND Choice F: benign
22 39 Choice A: antedated AND Choice D: preceded
23 61 Choice B: disparate AND Choice C: divergent
24 67 Choice A: arbitrary AND Choice B: capricious
25 85 Choice C: thrilled about AND Choice E: elated by
347
Evaluating Your Performance
This ebook was issued to HUAN-LIN CHANG, order #14803770223. Unlawful distribution of this ebook is prohibited.
Customer name HUAN-LIN CHANG , Order Id: 14803770223
Answer Key
Section 4. Verbal Reasoning
Question
Number P+ Correct Answer
170Choice B: fleeting AND Choice C: momentary
280Choice A: diversity AND Choice C: variety
349Choice A: synoptic AND Choice B: abridged
468Choice B: intransigent AND Choice F: resolute
526Choice A: A correct understanding of how phytoplankton photosynthesis
utilizes carbon dioxide
AND
Choice B: A correct prediction about how the addition of iron to iron-poor
waters would affect phytoplankton growth
AND
Choice C: An incorrect prediction about how phytoplankton growth would
affect the concentration of atmospheric carbon dioxide
626Choice A: help explain why Martin’s hypothesis is incorrect
731Choice E: There are fewer traffic accidents annually in Sparva than in any of
the provinces of comparable or greater population.
823Choice A: Voltaire’s literary stature helped secure a large audience for this
attempt to popularize Newton’s ideas.
AND
Choice B: Voltaire’s status as a nonscientist was an advantage in this effort
to bring Newtonian science to the attention of the general public.
AND
Choice C: The frontispiece of the book implies that Voltaire’s understanding
of Newton’s ideas was not achieved without assistance.
982Sentence 2: In the book’s frontispiece, Voltaire is seen writing at his desk,
and over him a shaft of light from heaven, the light of truth, passes through
Newton to Voltaire’s collaborator Madame du Châtelet; she reflects that light
onto the inspired Voltaire.
10 60 Choice B: extravagance; Choice F: profligate
11 37 Choice B: complacency; Choice D: embrace only a portion of the truth;
Choice G: erroneous
12 35 Choice D: inform
13 43 Choice E: reconcile
14 54 Choice C: The Quest of the Silver Fleece has at least one feature that typically
would attract the attention of literary scholars.
15 68 Choice D: DuBois’ novels may eventually attract greater scholarly interest
than they have to date
16 49 Choice E: The degree of consideration that has been given to DuBois’ novels,
including The Quest of the Silver Fleece
17 50 Choice E: an anomaly
18 19 Choice A: undermines; Choice E: inimical
19 54 Choice A: peculiarity; Choice E: assertive
20 23 Choice B: malignant; Choice E: insufficient; Choice H: benignity
21 53 Choice B: proximity to the Sun
22 59 Choice C: such a warm Arctic Ocean would trigger the melting of massive
land-based glaciers in the Arctic
348
GRE Practice Test 1
This ebook was issued to HUAN-LIN CHANG, order #14803770223. Unlawful distribution of this ebook is prohibited.
Customer name HUAN-LIN CHANG , Order Id: 14803770223
Answer Key
Question
Number P+ Correct Answer
23 33 Choice A: Some of them offered a religious explanation for their immigration.
24 94 Sentence 5: When he moves beyond the principal actors, he finds that reli-
gious explanations were less frequently offered, and he concludes that most
people immigrated because they were recruited by promises of material
improvement.
25 64 Choice A: summarizing the findings of an investigation
Section 5. Quantitative Reasoning
Question
Number P+ Correct Answer
165Choice B: Quantity B is greater.
283Choice C: The two quantities are equal.
362Choice D: The relationship cannot be determined from the information
given.
448Choice A: Quantity A is greater.
547Choice D: The relationship cannot be determined from the information
given.
636Choice A: Quantity A is greater.
726Choice C: The two quantities are equal.
834Choice D: The relationship cannot be determined from the information
given.
946Choice B: Quantity B is greater.
10 62 Choice D: 8 to 9
11 26 Choice B: 115
12 51 121
13 52 Choice C: 75
14 60 Choice C: 4,000
15 18 Choice C: The sales tax was less than $0.45.
16 19 Choice B: Tw o
17 69 Choice D: 40
18 63 9
19 41 Choice B: 190
20 60 Choice B: Decaffeinated, instant, drip-brewed, percolated
21 70 Choice A: 2.5 and 3.0
22 35 Choice D: 224
23 17 Choice A: DBC is an equilateral triangle.
AND
Choice D: The length of BC is 10.
24 51 18
25 34 Choice C: 35.0
349
Evaluating Your Performance
This ebook was issued to HUAN-LIN CHANG, order #14803770223. Unlawful distribution of this ebook is prohibited.
Customer name HUAN-LIN CHANG , Order Id: 14803770223
Answer Key
Section 6. Quantitative Reasoning
Question
Number P+ Correct Answer
166Choice A: Quantity A is greater.
273Choice D: The relationship cannot be determined from the information
given.
352Choice A: Quantity A is greater.
446Choice C: The two quantities are equal.
548Choice B: Quantity B is greater.
633Choice C: The two quantities are equal.
740Choice D: The relationship cannot be determined from the information
given.
826Choice B: Quantity B is greater.
931Choice D: The relationship cannot be determined from the information
given.
10 67 Choice A: $40.00
11 10
12 49 Choice B: 3:30
13 27 Choice E: The fifth-lowest salary in the second quartile
14 38 13
15 40 Choice E: mis an even integer.
16 21
AND
Choice B:
17 79 Choice B: 47
18 44 Choice A: 130
19 41 Choice A: 50
20 30 Choice C: 38%
21 11 Choice E: 58
22 58
23 24
24 15
25 30 Choice E: 304
350
GRE Practice Test 1
153
190
(or any equivalent fraction)
This ebook was issued to HUAN-LIN CHANG, order #14803770223. Unlawful distribution of this ebook is prohibited.
Customer name HUAN-LIN CHANG , Order Id: 14803770223
Score Conversion Table
Raw Score Verbal Reasoning Scaled Score Quantitative Reasoning Scaled Score
50 170 170
49 170 170
48 170 170
47 170 170
46 169 169
45 168 168
44 167 167
43 166 165
42 165 164
41 164 163
40 163 162
39 162 161
38 162 160
37 161 159
36 160 159
35 159 158
34 159 157
33 158 156
32 157 155
31 156 155
30 156 154
29 155 153
28 154 153
27 154 152
26 153 151
25 152 150
24 152 150
23 151 149
22 150 148
21 149 148
20 149 147
19 148 146
18 147 145
17 146 144
16 145 143
351
Evaluating Your Performance
This ebook was issued to HUAN-LIN CHANG, order #14803770223. Unlawful distribution of this ebook is prohibited.
Customer name HUAN-LIN CHANG , Order Id: 14803770223
Score Conversion Table
Raw Score Verbal Reasoning Scaled Score Quantitative Reasoning Scaled Score
15 144 143
14 143 142
13 142 141
12 141 140
11 140 138
10 139 137
9 137 136
8 136 134
7 134 133
6 132 131
5 130 130
4 130 130
3 130 130
2 130 130
1 130 130
0 130 130
352
GRE Practice Test 1
This ebook was issued to HUAN-LIN CHANG, order #14803770223. Unlawful distribution of this ebook is prohibited.
Customer name HUAN-LIN CHANG , Order Id: 14803770223
Analytical Writing Sample Responses
and Reader Commentaries
SECTION 1
Analytical Writing
ANALYZE AN ISSUE
The best way to teach is to praise positive actions and ignore negative ones.
Write a response in which you discuss the extent to which you agree or disagree with the
recommendation and explain your reasoning for the position you take. In developing and
supporting your position, describe specific circumstances in which adopting the
recommendation would or would not be advantageous and explain how these examples
shape your position.
Score 6 Response*
The recommendation presents a view that I would agree is successful most of the
time, but one that I cannot fully support due to the “all or nothing” impression it gives.
Certainly as an educator I agree fully that the best way to elicit positive response
from students is to make use of students’ positive energy and then encourage actions
that you would like to see repeated. It is human nature that we all want to be accepted
and achieve on some level, and when people in authority provide feedback that we
have done something well, the drive to repeat the action that was praised is bound to
be particularly strong.
This blanket statement would obviously pay dividends in situations in which a
teacher desires to have students repeat particular behaviors. For example, if an
educator is attempting to teach students proper classroom etiquette, it would be
appropriate to openly praise a student who raises his or her hand when wishing to
speak or address the class. In such cases, the teacher may also help shape positive
behaviors by ignoring a student who is trying to interject without approval from the
teacher. In fact, the decision to ignore students who are exhibiting inappropriate
behaviors of this type could work very well in this situation, as the stakes are not very
high and the intended outcome can likely be achieved by such a method. However, it
is important to note here that this tactic would only be effective in such a “low-stakes”
situation, as when a student speaks without raising her hand first. As we will discuss
below, ignoring a student who hits another student, or engages in more serious
misbehaviors, would not be effective or prudent.
To expand on this point, it is important for teachers to be careful when working with
353
Analytical Writing Sample Responses
* NOTE: All responses are reproduced exactly as written, including errors, misspellings, etc., if
any.
This ebook was issued to HUAN-LIN CHANG, order #14803770223. Unlawful distribution of this ebook is prohibited.
Customer name HUAN-LIN CHANG , Order Id: 14803770223
the second half of this statement, only ignoring negative actions that are not serious.
Take for instance a student who is misbehaving just by chatting with a fellow class -
mate. This student might not be presenting much of a problem and may be simply
seeking attention. Ignoring the student might, in fact, be the best solution. Now
assume the negative action is the improper administering of chemicals in a science
experiment or the bullying of a fellow student. To ignore these negative actions would
be absurd and negligent. Now you are allowing a problem to persist, one that could
potentially lead to much bigger and more dangerous issues. In a more serious
situation, addressing the negative actions quickly and properly could stop the problem
it in its tracks. It is for reasons like this that I do not advocate the idea that a teacher can
be successful by simply ignoring negative actions.
I do, however, greatly support the idea that the central focus of teaching should be
to build on and encourage positive actions. However, the author’s all-encompasing
statement leaves too many negative possibilities for the classroom. Perhaps a better
way to phrase this statement would be to say, “The best way to teach is to praise
positive actions and ignore negative ones that are not debilitating to class efficiency
or the safety of any individual”.
Thus, in the original statement, there are indeed some good intentions, and there
could be a lot of merit in adopting its basic principles. Data proves that positive support
can substantially increase motivation and desire in students and contribute to positive
achievements. In fact, most studies of teaching efficacy indicate that praising positive
actions and ignoring negative ones can create a more stable and efficient classroom.
It needs to be stressed, however, that this tool is only effective at certain levels of
misbehavior. As mentioned above, when the behavior is precipitated by feelings of
revenge, power or total self-worthlessness, this methodology will likely not work. It is
likely to be very successful, however, when the drive behind the misbehavior is simple
attention seeking. In many of these instances, if the teacher demonstrates clearly that
inappropriate behavior does not result in the gaining of attention, students are more
likely to seek attention by behaving properly. Should the student choose this path, then
the ignoring has worked and when the positive behavior is exhibited, then the teacher
can utilize the first part of the theory and support or praise this behavior. Now it is
much more likely to be repeated. If the student does not choose this path and instead
elects to raise the actions to a higher level that presents a more serious issue, then
ignorance alone cannot work and other methods must be employed.
In conclusion, one can appreciate the credo expressed in this instance, but surely
we all can see the potential error of following it through to the extreme.
Reader Commentary
This response receives a 6 for its well-articulated, insightful analysis of the issue.
Rather than simply rejecting or accepting the prompt, the writer argues that the rec-
ommendation made by the prompt can often be true but is too “all or nothing” to be
endorsed without qualification. The writer turns this idea into an insightful position by
providing examples and evidence to fully and persuasively support its nuanced argu-
ment. The response offers nicely detailed situations that provide compelling support
for a claim that the recommendation can, in fact, work. At the same time, it also high-
lights the recommendation’s limits using additional specific, detailed examples. Partic-
ularly persuasive is the fourth paragraph, in which the writer compares the impact of
ignoring minor behavioral problems like talking in class to the potential costs of ignor-
ing more serious issues like bullying. Thus, the writer recognizes that the prompt’s
claim, as well as his/her own, is inevitably dependent on the specific context for its suc-
354
GRE Practice Test 1
This ebook was issued to HUAN-LIN CHANG, order #14803770223. Unlawful distribution of this ebook is prohibited.
Customer name HUAN-LIN CHANG , Order Id: 14803770223
cess or failure. Throughout the response, the writer demonstrates the ability to convey
ideas fluently and precisely, using effective vocabulary and sentence variety. This sen-
tence demonstrates the level of language facility seen throughout the response: “It is
human nature that we all want to be accepted and achieve on some level, and when
people in authority provide feedback that we have done something well, the drive to
repeat the action that was praised is bound to be particularly strong.”
Score 5 Response
I partially agree with the statement “The best way to teach is to praise positive actions
and ignore negative ones”. Children should be rewarded when they perform well;
however, they should not be ignored for performing sub-optimally. For purposes of
this essay, the term “actions” is defined as behaviors within the classroom.
Utilizing positive reinforcements, such as tangible rewards, can be a good method
to teach children. If the teacher praises children for actions that are desirable, then the
children are more likely to repeat those actions. For example, a student who completes
an assignment on time and does a good job is likely to want to do a good job on the
next assignment if he gets positive feedback. Likewise, the children who are not
currently engaging in the desirable actions may be more inclined to do so in order to
recieve the positive reinforcement.
Conversely, children should not be ignored for negative actions. If a child is not
exhibiting appropriate behavior in the classroom, then it is the teacher’s responsibility
to encourage the child to perform optimally. Ignoring something doesn’t make it go
away, actions and consequences do. A student who is being disruptive in class will
continue to be disruptive unless the teacher does something about it. However, the
teacher’s actions need be appropriate.
Before the teacher attempts to modify a child’s behavior, the teacher needs to try
and identify the reason behind the behavior. For instance, children who leave their
seat often, stare in to space, or call out of turn may be initially viewed as having poor
behavior. However, the teacher may suspect that the child has an attentional problem,
and request that the child be tested. If the child does have an attentional problem, then
the teacher can work with a related service, such as occupational therapy, to alter the
classroom environment in order to cater to the needs of the child. For instance, the
teacher could remove some of the stimulating bulliten board displays to make the
room more calming to the child. If the child becomes more attentive in class then the
teacher was able to assist the child without scorning them or ignoring them. The
teacher met the needs of the child and created an enviornment to enable the child to
optimally perform in the educational setting.
On the other hand, if the child is tested, and does not have any areas of concern
that may be impacting the educational performance in the classroom, then the
negative behavior may strictly be due to defiance. In such a case, the teacher still
should not ignore the child, because the negative actions may hinder the learning
opportunity for the remaining children in the class. As a result, a child who is being
disruptive to the learning process of the class should be set apart from the class so
that they do not receive the positive reinforcement of peer attention.
The teacher should not ignore the student who is misbehaving, but that does not
mean that the teacher just needs to punish. It is better to address the child privately
and make sure the child is aware of the negative actions. Once the child is aware, then
the teacher should once again try to determine the reason why the child is behaving in
355
Analytical Writing Sample Responses
This ebook was issued to HUAN-LIN CHANG, order #14803770223. Unlawful distribution of this ebook is prohibited.
Customer name HUAN-LIN CHANG , Order Id: 14803770223
a negative manner. Perhaps the child’s parents are in the middle of a divorce and the
child is outwardly expressing his frustration in the classroom. Or the academic content
of the class may not be challenging enough for the child and so he is misbehaving out
of boredom. Whatever the reason behind the behavior, the key factor is that the teacher
works with the child to try and identify it. Simply punnishing or ignoring the child
would not solve the problem, whereas working to create a plan for success in the
classroom would. Likewise, rather than punnishing and defeating the child, the teacher
is working with and empowering the child; a much more positive outcome to the
situation.
Reader Commentary
This strong response presents a thoughtful and well-developed analysis of the issue. In
this case the writer argues that teachers need to modify their approach based on con-
text and observation, meaning that a blanket approach cannot be successful. The writer
supports this position with relevant reasons and examples that present logically sound
support. Note that the task instructions ask writers to discuss circumstances in which
adopting the recommendation might or might not prove advantageous, and this
response does that quite clearly. In the second paragraph, the writer gives an example
of a student who completes an assignment on time and receives positive feedback,
showing how the recommendation could prove advantageous. Other examples show
circumstances in which adopting the recommendation would not be a good idea, and
these various points are brought together to support the writer’s position that teachers
have to look at the context of the situation and cannot rely on simply ignoring negative
actions. This response also demonstrates facility with language, using appropriate
vocabulary and sentence variety. Sentences like this one demonstrate the writer’s com-
mand of the conventions of standard written English: “If the child does have an atten-
tional problem, then the teacher can work with a related service, such as occupational
therapy, to alter the classroom environment in order to cater to the needs of the child.”
There are some minor errors, but overall the response demonstrates strong control of
language. Although the response is clearly stronger than a 4, which would simply pre -
sent a clear position on the issue according to the task instructions, it does not reach
the level of a 6 because it does not develop its points in a way that creates a cogent and
insightful position. It does, however, present a generally thoughtful and well-developed
analysis of the issue, leading to a score of 5.
Score 4 Response
I absolutely agree with the first section of the statement above, but find fault with the
latter half.
There is no doubt that praising positive actions is an excellent way to teach, and this
method is most clearly exemplified when dealing with much younger children. When
a young child is learning basic social behavior, it is imperative that he is encouraged
to repeat positive actions. For example, when a child voluntarily shares his toys with
another, if a teacher rewards that behavior, the child will understand that this is a good
practice, and likely share again in the future.
In contrast, if a child displays negative behavior by stealing a toy away from his
playmate, it would be very dangerous for the teacher to ignore this action, for then the
child may never recognize that this is unacceptable. In this instance, the child has not
356
GRE Practice Test 1
This ebook was issued to HUAN-LIN CHANG, order #14803770223. Unlawful distribution of this ebook is prohibited.
Customer name HUAN-LIN CHANG , Order Id: 14803770223
learned from the situation at all. So what should a teacher do when faced with such
a situation? Punishment is not necessarily the optimal choice, either. Rather than
scolding a child for mistreating his playmates and sending him off to a corner, a
teacher would be wise to demonstrate the positive alternative: to share his toys
instead. In this case, rather than ignoring or punishing negative actions, the teacher
could seize the opportunity to reinforce positive behavior, and further extend the child’s
learning experience.
In summary, positive reinforcement is certainly an excellent method for teaching
new methods or behaviors, and encouraging a student to learn more. However to
ignore, rather than recognize and correct negative actions, would be a disservice to
the student, for he would not know what conclusion to draw from his action.
Reader Commentary
This adequate response follows the task directions and presents a clear position on the
issue, supporting its main points with examples that are relevant, if only adequately
developed. For instance, the discussion in the second paragraph of a teacher who rein-
forces the positive behavior of sharing a toy is certainly relevant and on-task (i.e., it
describes a situation in which adopting the recommendation would be advantageous).
However, the development of this idea does not lead to generally thoughtful or insight-
ful analysis. Instead, it is simply presented as an example. In addition to its adequate
development, this response also demonstrates sufficient control of the conventions of
standard written English, and its main points are made with reasonable clarity. Some
of the sentences demonstrate the syntactical variety normally seen in responses that
receive higher scores (e.g., “Rather than scolding a child for mistreating his playmates
and sending him off to a corner, a teacher would be wise to demonstrate the positive
alternative: to share his toys instead”). However, the overall use of language in this
response is merely adequate.
Score 3 Response
Praising postive actions and ignoring negative ones may be a good way to teach but
not the best way. Ignoring negative actions could negate all the postive praises given
to an individual, having negative actions go unchecked will lead to habits formed that
would overwhelm any positive actions that are complementary to an individuals
learning process.
For instance, in a classroom full of eight-year old kids; if during a lesson they are
making alot of noise, having this ignored would tell the kids that it is okay to be
disruptive in class. The individuals in that class would develop the habit of being
distruptive hence hindering their learning process. However if the eight-year old kids
were immediately told to stop the distruption then it will never become a habit.
Every action needs to have a related consequence follow in a learning environment.
In the early years of education, the way they are taught becomes a lifelong habit which
is hard to change in later years. If negative actions are not assigned a related
consequences then teaching becomes ineffective because the students negative
actions soon diminish the ability to do well in school. The way postive actions are dealt
with should also be done with negative actions rather than being ignored which in turn
enhance the learning environment.
357
Analytical Writing Sample Responses
This ebook was issued to HUAN-LIN CHANG, order #14803770223. Unlawful distribution of this ebook is prohibited.
Customer name HUAN-LIN CHANG , Order Id: 14803770223
Reader Commentary
Although this response has minor errors in its use of language, it receives a 3 primarily
for insufficient overall clarity and for the limited development of its claims. The writer
does make an attempt to follow the specific task instructions, and the response has a
clear position on the issue, arguing that it is not acceptable practice to ignore negative
behaviors. However, the development provided in support of that position is limited.
The example of “eight-year old kids” making noise during class can be seen as a situa-
tion in which following the recommendation is not advantageous. Instead of develop-
ing that point in a logically persuasive way, however, the writer proceeds to make an
unsupported assertion about the consequences of following the recommendation (“The
individuals in that class would develop the habit of being distruptive hence hindering
their learning process”). Another issue that keeps this response at the 3 level is a lack of
clarity, particularly in the final paragraph. The final sentence demonstrates this prob-
lem with clarity: “The way postive actions are dealt with should also be done with neg-
ative actions rather than being ignored which in turn enhance the learning
en vironment.” Problems with the structure of this sentence make it difficult to deter-
mine the writer’s intended meaning.
Score 2 Response
I don’t agree with this afirmation, because I think is very important to praise positive
actions but also is important to sign the negative ones, in some situations acording to
the students level, grade, etc., could be better to put more emphasis in the positive
things and if not ignore all the negative ones, do not give so much importance to them,
this is particulary important in the lowest levels of education.
But in another situations you must sign the negative things, trying to avoid that the
students can repeat them in the future, because I think you can also learn from the
negative situations.
For this reason I believe that is important to praise positive actions but is also
important no to ignore the negative ones, because in a given situation the student can
have troubles recongnising what is right and what is wrong. And finally as a conclusion
I think that the best way to teach is combination of praise positive things but also to
sign the negative ones.
Reader Commentary
This response clearly fits several characteristics of a 2, as defined by the scoring guide.
It is seriously limited in its development, organization, and focus. The response repeats
itself rather than developing any of its statements, pointing to an inability to organize
a response capable of supporting any specific claims with relevant reasons or exam-
ples. Additionally, serious language control problems frequently interfere with mean-
ing. Thus, even though the writer does seem to be making an attempt to respond to the
specific task instructions, the response merits a score of 2.
358
GRE Practice Test 1
This ebook was issued to HUAN-LIN CHANG, order #14803770223. Unlawful distribution of this ebook is prohibited.
Customer name HUAN-LIN CHANG , Order Id: 14803770223
359
Analytical Writing Sample Responses
Score 1 Response
Write a response in which you disuss the extent to which you agree or disagree
with the recommendation and explain your reasoning for the position you take. In
developing and supporting position, describe specific circumstances in which adopting
the recommendation would or would not be advantageous and explain how these
examples shape your position.
Author says that The best way to teach is to praise positive actions and ignore
negative ones. I agree to this recommendation. Explaining, I strongly believe that the
best way to teach is not to praise positive action and ignore negative ones but is
makeing everyone to be a good ones. Specific crimstances lead me which adopting
the recommendation as the following:
First, we will lost the good children who have negative maner if we ignore them.
Children are future, not all. Praise in negative should not be, teaching to children to
best way. I strongly believe adopting this recommeindation would be not advantages.
second, negative ones in today may be a great people in the future. Not only ones
behave do worse they are teenage. Teenage in today is not easy for all! Negative ones
can not better, if only prainse positive actions, ignore negativeone. Negative ones may
not positive be having, but if we praise them only, they not think they should be
positive person later.
conclusion, specific circumstances are which adopting the recommendation would
not be advantage, I am not agree to the the recommendation. Ignore negative manor
when they will not be positive behavrio in futre. But they can, if do not ignore them. we
should not ignor negative person but should make them think that they can be a good
man future like positive person.
Reader Commentary
This response has severe and pervasive problems in language and sentence structure
that, as stated in the scoring guide, consistently interfere with meaning and result in
incoherence. The response begins by repeating the prompt, but then the severe prob-
lems with language control and organization undermine any evidence of the ability to
understand the prompt or to present and develop a clear position. For example, it is
not clear what the writer means by the claim that the best way to teach is “makeing
everyone to be a good ones.” Severe problems with language control in that sentence
and throughout the response prevent it from developing a coherent position on the
issue or responding to the specific task instructions. Although the writer makes an
attempt at organization, with points marked as first, second, and conclusion, the
response actually exhibits little or no evidence of the ability to develop any potential
understanding of the prompt into a logical position on the issue.
This ebook was issued to HUAN-LIN CHANG, order #14803770223. Unlawful distribution of this ebook is prohibited.
Customer name HUAN-LIN CHANG , Order Id: 14803770223
SECTION 2
Analytical Writing
ANALYZE AN ARGUMENT
The following appeared in an article written by Dr. Karp, an anthropologist.
"Twenty years ago, Dr. Field, a noted anthropologist, visited the island of Tertia and
concluded from his observations that children in Tertia were reared by an entire village
rather than by their own biological parents. However, my recent interviews with children
living in the group of islands that includes Tertia show that these children spend much more
time talking about their biological parents than about other adults in the village. This
research of mine proves that Dr. Field's conclusion about Tertian village culture is invalid
and thus that the observation-centered approach to studying cultures is invalid as well. The
interview-centered method that my team of graduate students is currently using in Tertia will
establish a much more accurate understanding of child-rearing traditions there and in other
island cultures."
Write a response in which you discuss what specific evidence is needed to evaluate the
argument and explain how the evidence would weaken or strengthen the argument.
Score 6 Response
It might seem logical, at first glance, to agree with the argument in Dr. Karp’s article
that children in Tertia actually are raised by their biological parents (and perhaps even,
by implication, that an observation-centered approach to anthropological study is
not as valid as an interview-centered one). However, in order to fully evaluate this
argument, we need to have a significant amount of additional evidence. The argument
could end up being much weaker than it seems, or it might actually be quite valid. In
order to make that determination, we need to know more then analyze what we learn.
The first piece of evidence that we would need in order to evaluate Dr. Karp’s claims
is information about whether or not Tertia and the surrounding island group have
changed significantly in the past 20 years. Dr. Field conducted his observational study
20 years ago, and it is possible that Tertia has changed significantly since then. For
example, if we had evidence that in teh intervening years Westerners had settled on
the island and they introduced a more typical Western-style family structure, it would
certainly weaken Dr. Karp’s argument. In that case, the original study could have been
accurate, and Dr. Karp’s study could be correct, as well, though his conclusion that Dr.
Field’s method is ineffective would be seriously weakened.
Another piece of evidence that might help us evaluate this claim involves the exact
locations where Dr. Karp’s interviews took place. According to this article, Dr. Karp and
his graduate students conducted interviews of “children living in the group of islands
that includes Tertia.” If we were to learn that they never interviewed a single Tertian
360
GRE Practice Test 1
This ebook was issued to HUAN-LIN CHANG, order #14803770223. Unlawful distribution of this ebook is prohibited.
Customer name HUAN-LIN CHANG , Order Id: 14803770223
child, it would significantly weaken the conclusion. It could turn out to be the case, for
example, that children on Tertia are raised communally, whereas children on other
islands nearby are raised by their biological parents.
In order to fully evaluate this article, we would also need to learn more about the
interview questions that Dr. Karp’s team used. What exactly did they ask? We don’t
know, nor do we know what the children’s responses actually were. What did they say
about their biological parents? The mere fact that they speak more frequently about
their biological parents than they do about other adults does not meant hat they are
raised by their biological parents. It would significantly undermine Dr. Karp’s argument
if it turned out that the children said things like how much they missed their parents or
how their parents had left them in a communal environment. Without knowing WHAT
the children said, it is hard to accept Dr. Karp’s conclusion.
It is slightly more difficult to discuss teh evidence we might need in order to
evaluate the more interesting claims in Dr. Karp’s article, namely his extension of the
results of his study to a conclusion that interview-centered methods are inherently
more valid than observational-centered approaches. In order to fully evaluate this
claim, in fact, we would need to look at many more examples of interview-based and
observation-based anthropological studies and we would also need to look into
different study designs. Perhaps Dr. Field did not conduct an effective observational
study, but other observational approaches could be effective. In order to make such
grandiose claims, Dr. Karp really needs a lot of additional evidence (ideally a meta -
analysis of hundreds of anthropological studies).
Clearly, then, we need to have additional evidence in order to get a more complete
understanding of the strengths and weaknesses of Dr. Karp’s article. We need to know
about Tertia and the surrounding islands, whether or not they have changed over the
past 20 years. We also need to know about study design (Dr. Karp’s and Dr. Field’s).
And we really need a lot more information if we want to extend the results of a study
about one island culture to all anthropological fieldwork.
Reader Commentary
This outstanding response clearly addresses the specific task directions and presents a
cogent, insightful analysis by specifically detailing the impact that different pieces of
evidence would have on the argument. The introductory paragraph sets up the organi-
zation of the response, and each body paragraph provides the sort of compelling devel-
opment typical in responses that receive a score of 6. For example, after the writer
discusses possible evidence that Tertian child-rearing practices have changed over the
past 20 years, he or she clearly explains the impact information about those changes
might have on the argument, saying, “In that case, the original study could have been
accurate, and Dr. Karp’s study could be correct, as well, though his conclusion that Dr.
Field’s method is ineffective would be seriously weakened.” Not only is this argument
compelling, but it also demonstrates sophisticated syntax and facility with language.
There is more insightful development in the fifth paragraph, in which the writer exam-
ines Dr. Karp’s claims about interview-based studies. Although there are a few typos
and minor errors here, nothing in the response distracts from the overall fluency of the
writing. Sentences like this one demonstrate the fluent and precise diction and varied
syntax that are evident throughout the response: “It could turn out to be the case, for
example, that children on Tertia are raised communally, whereas children on other
islands nearby are raised by their biological parents.” Because of its compelling and
insightful development and fluent and precise language, this response fits all of the bul-
let points for a 6.
361
Analytical Writing Sample Responses
This ebook was issued to HUAN-LIN CHANG, order #14803770223. Unlawful distribution of this ebook is prohibited.
Customer name HUAN-LIN CHANG , Order Id: 14803770223
Score 5 Response
There seems to be an abundance of evidence that, if we were to examine it closely,
might make us reconsider Dr. Karp’s argument here. If we look first at the evidence
that might weaken this argument, we can see a lot of the problems with Dr. Karp’s
article. It would certainly weaken the argument if we were to discover that Dr. Karp
and his students did not actually conduct any of their interviews on the island of Tertia
itself. Looking closely at the article, we see that Dr. Karp claims the interviews were
conducted with children from the island group that includes Tertia. There is no
evidence that they interviewed Tertian children. It would definitely weaken the
argument if we were to learn that they interviewed children only on islands close
to Tertia. Those islands may or may not have similar child-rearing traditions, and
geographic proximity does not guarantee societal similarity.
Another piece of evidence that would weaken the argument could come from
transcripts of the interviews themselves. Dr. Karp’s article makes the claim that the
children “spend much more time talking about their biological parents than about
other adults,” but he gives no indication of what exactly they say about their biological
parents. After all, the children may be talking about how they never see their parents.
One more important piece of evidence that might undermine the argument Dr. Karp
is making in this article. He admits that twenty years have passed since Dr. Field’s study
was conducted, but he does not provide evidence that proves child-rearing techniques
have not changed significantly in that time. Any number of factors could have led to a
significant shift in how children are raised. Influences from other cultures, significant
catastrophic events, or a change in government structures could have led to a change
in family dynamics. Any evidence of such changes would clearly undermine Dr. Karp’s
argument.
If we went looking for evidence that could strengthen the argument, we might also
find something interesting. Obviously, some of the evidence above might strengthen
the argument if they were NOT as discussed above (e.g., if there were evidence that the
Tertian islands have NOT changed since Dr. Field’s study or if there were transcripts
that showed the children spoke about how much they loved living with their biological
parents). However, if we discovered that there are numerous interview-based studies
that confirm Dr. Karp’s findings, it would go a long way toward bolstering his claim
about Tertian child-rearing AND his claim about interview-centered studies being more
effective. Another piece of evidence that would strengthen Dr. Karp’s argument is
undermining Dr. Field’s original study. Maybe Dr. Field was sloppy, for example.
Dr. Karp’s article, then, ends up looking like something of an empty shell. Depending
on the evidence we find to fill it out, we may discover that it is quite convincing, or we
could determine that he is full of hot air.
Reader Commentary
This strong response presents a generally thoughtful and well-developed analysis of
the argument, and it follows the specific task directions quite clearly. This writer
approaches the task by first discussing the evidence that might weaken Dr. Karp’s argu-
ment and then, in somewhat less depth, considering the evidence that could strengthen
it. In both cases the writer analyzes the ways in which the evidence would bear on the
argument. For example, the writer notes, “Influences from other cultures, significant
catastrophic events, or a change in government structures could have led to a change
in family dynamics. Any evidence of such changes would clearly undermine Dr. Karp’s
argument.” Although the development presented here is strong, the response does not
362
GRE Practice Test 1
This ebook was issued to HUAN-LIN CHANG, order #14803770223. Unlawful distribution of this ebook is prohibited.
Customer name HUAN-LIN CHANG , Order Id: 14803770223
present the compelling development required for a 6. For instance, in the first para-
graph there is some repetition, and in the third paragraph the reader must fill in the
implications of potential “changes” in Tertia, which are not fully fleshed out. How
could a catastrophic event or a change in governmental structure have led to changes
in child-rearing traditions? The development, then, is strong but not outstanding.
Also, the response demonstrates some facility with language, though it does not convey
meaning skillfully enough to merit a score of 6. In general, the response demonstrates
strong writing skills, in spite of some minor errors like the sentence fragment that
begins paragraph three. Sentences like this one demonstrate the quality of the writing
seen throughout the response: “Those islands may or may not have similar child-
rearing traditions, and geographic proximity does not guarantee societal similarity.” In
terms of writing skill and analysis, then, this response earns a score of 5.
Score 4 Response
Dr. Karp’s arguments that his research proves that obervation-centered research is
invalid and that his interview-centered method “will establish a much more accurate
understanding of child-rearing traditions there and in other island cultures” need more
support. While the findings from Dr. Karp’s interviews do challenge Dr. Field’s results,
one then cannot make the assumption that Dr. Field’s research is invalid. This essay
will attempt to explain three ways in which Dr. Karp can strengthen his argument.
First, Dr. Karp should provide more information about the content of the interviews.
Misinterpretation from observation can be as likely as misinterpretation in interivews. It
is possible that while children may spend more time talking about their own biological
parents, other people from the village are still assisting in most of the rearing of the
child. Perhaps asking the children how much time they spend with their parents, who
disciplines them, and other specific questions about rearing would provide a more
complete answer about who exactly is raising the children.
Second, Dr. Karp could provide some information about societal changes in the
past twenty years. If there have been significant changes on the island of Tertia, it is
possible that both anthropologists are correct. Twenty years ago, the entire village
raised children, and now, biological parents raise their own children. Recents events
could explain the change - such as introduction of Western mass media or changes in
government (monarchy to democracy). Perhaps even interviewing adults to get a
better understanding on child rearing. Not to mention, interpretting information from
children and using that information to generalize about an entire island is not the most
effective means.
Thirdly, Dr. Karp needs more proof that the observation-centered approach to
studying cultures in invalid. A potential mistake in one article can hardly invalidate an
entire method of research. Other anthropologists who employ the interview-centered
method need to dispute the work of anthropoligsts who use the observation-centered
approach. Only when a significant amount of research can be disproved can an entire
method of research be invalidated.
To conclude, Dr. Karp needs to do more research and provide more evidence
before his large claims can be fully supported. In fact, it will take more than Dr. Karp
alone to prove observation-centered method of research is invalid and further, that the
interview-centered method is better. In terms of his own research, Dr. Karp needs to
conduct more interviews on the Tertia islands and scientifically prove Dr. Field’s
research wrong.
363
Analytical Writing Sample Responses
This ebook was issued to HUAN-LIN CHANG, order #14803770223. Unlawful distribution of this ebook is prohibited.
Customer name HUAN-LIN CHANG , Order Id: 14803770223
364
GRE Practice Test 1
Reader Commentary
This adequate response manages to identify some important features of the argument,
presenting a competent examination and generally following the task directions. The
response does not merit a score of 5 or 6, however, because it does not present com-
pelling or insightful development. The response identifies basic points about the
content of the interviews, possible changes in Tertia, and observation-centered studies,
but these points are developed only adequately. Development in paragraph four
(“Thirdly . . . ”) is generic and thin, and the final paragraph just recapitulates the asser-
tions made earlier. The response does follow the specific task instructions, but it does
not develop its discussion of specific evidence fully. For example, there is a claim that
“specific questions about rearing would provide a more complete answer about who
exactly is raising the children,” but the response does not explain what sorts of ques-
tions would give which answers or how those answers would strengthen or weaken the
argument. Also, language control in this response is merely adequate, not strong. There
are some typos and other errors (e.g., a sentence fragment in paragraph 3: “Perhaps
even interviewing adults to get a better understanding on child rearing”), but the
response generally demonstrates control of the conventions of standard written Eng-
lish, and main points are made with reasonable clarity. Because of its adequate control
of language and competent analysis, this response earns a score of 4.
Score 3 Response
It will be very inappropriate to condemn Dr. Field’s observations and findings. A critical
look and analyses of the argument shows that details of Dr. Field’s work was not given
out. In fact, it is sad on the side of the writer to think that Dr.Fields work is invalid.
First, the fact that the children of Tertia spend much time talking about their
biological parents than about other adults in the village can be interpreted in a different
way. The writer did not give any clue on what exactly the children were saying about
their biological parents. It could be that they were talking about their parents
irresponsibility of rearing them by themselves than leaving them in the hands of
the whole community to bring them up. In fact, the argument could have been
strengthened if the writer gave what exactly the children were talking about.
On the other hand, the writer failed on his or her part as a researcher to look at
the time frame from the time Dr. Field did his analyses to the the time writer also
conducted His or Her research. This would have given him the insight as what new
developments has taken place within the twenty years gap that Dr. Field did His
analyses. The writer’s argument would have given a lot of meaning if the writer had
research into the cultural developments that has taken place since the time Dr. fields
last visited and didcompleted His work at Tertia.
Also, as a reader, the tone this writing is not very convincing. It almost seems like
Dr. Karp is making Dr. Fields look bad, instead of supporting his own research with
information. He really only says one sentence about his own research, the rest of it is
about how Fields work is not as good and saying things about Fields work. He needs to
have more details about his own work to really sell the reader on it. He needs to write
more about what the interview-centered method is, since he does not even say what it
is. This will be more convincing if it is less of an attack on Dr. Field and more about the
researches.
On the whole the writer’s work is incomplete and His or Her criticisms are
This ebook was issued to HUAN-LIN CHANG, order #14803770223. Unlawful distribution of this ebook is prohibited.
Customer name HUAN-LIN CHANG , Order Id: 14803770223
unfounded. The writer needs to change the qualitative way of His or Her research into a
more quatitative approach. If done in this way the impact of His or Her findings will be
very strong and convincing.
Reader Commentary
Although this response analyzes some important features of the argument, it is limited
in development and often lacks acceptable clarity in expressing its ideas. In particular,
this response contains occasional major errors and frequent minor errors that can
interfere with meaning. Misused words, subject/verb agreement problems, and other
lapses occur throughout the response. In addition to the problems with language con-
trol, the response demonstrates limited relevant development. It is true that the
response makes an attempt to follow the specific task instructions, identifying the fact
that the argument might be strengthened by evidence that the children were talking in
a positive manner about their parents. However, the response does not explain exactly
how this evidence would strengthen the argument. Similarly, there is discussion of the
elapsed time between the two studies, but the response does not clarify how informa-
tion about the “cultural developments” over the past 20 years would strengthen the
argument Dr. Karp is making. Additionally, some of the points the response is making
are not actually relevant to an analysis of the logic of the argument. The discussion of
Dr. Karp’s tone in the fourth paragraph, for example, is a rhetorical critique, not
a logical one. There is an attempt to talk about evidence (“He needs to have more
details . . . ”), but the focus in this paragraph is on “selling” the reader, not creating a per-
suasive argument. Because of its limited development and language control, this
response earns a score of 3.
Score 2 Response
The argument is on the article written by Dr. Karp , an anthropologist and his study
and the new plan to study the same in the tertia region.Dr.Karp has written an article
on Children in Tertia and about the culture.
The arguement is that they have not mention the type of intreview and the type of
questions of the interviwes.They haven’t indicate the education level in the children
and the background of the children. What are all the things the team is going to
observe and study on the child rearing tradition is not clearly mention.
The team is going to study and correlate the tradition with the other island culture
but there is a possibility of different environment of other island or differnt biological
parents. The resource availability on one island is different than the other is also a
possibility . In that situation it is not possible to correlate the culture between to
iceland.
There is a possibility , Dr. Field’s interview time , lacking of infrastructure in the
tartia. There was no developement of schools and other refreshment activity or the
parents may not spent enough time with the children due to various reasons and that
effect to the children , so they might have spend more time talking about their
biological parent.
To support the argument more information about the nature, cultural background
and also the type of infrastructure presence in the area is require, the kind of study
carring out in the study area is require. Which would help to give more support the
argument.
365
Analytical Writing Sample Responses
This ebook was issued to HUAN-LIN CHANG, order #14803770223. Unlawful distribution of this ebook is prohibited.
Customer name HUAN-LIN CHANG , Order Id: 14803770223
Reader Commentary
This response demonstrates serious weaknesses in analytical writing. There seem to be
some attempts at logical analysis, though none that specifically and clearly examine
the evidence that might weaken or strengthen the argument. Additionally, there is little
or no relevant or reasonable support for the writer’s points. In large part, the lack of
logical development seems to be due to the serious and frequent problems with lan-
guage control seen throughout the response. There are basic errors in just about every
sentence of the response, and these errors frequently interfere with meaning. This sen-
tence exemplifies the problems seen throughout the response: “There was no develope-
ment of schools and other refreshment activity or the parents may not spent enough
time with the children due to various reasons and that effect to the children , so they
might have spend more time talking about their biological parent.” The writer is
attempting to discuss some points that are relevant to an analysis of this argument, but
meaning is obscured by all of the errors present. However, some meaning can be dis-
cerned, and these errors are not severe enough to drop the score to a 1.
Score 1 Response
Twenty years ago Dr field an anthropologist found result after reserch that in small
village of tertia children reared by entire village but according to dr karp he talked most
of the children that they talk about there boilogical parents. so it conclude that the
reserch of dr field is unvalid now and what type of methods dr field used may be not
cover all aspects of there culture and also other cultures of other islands. reared the
children by entire village is not logical but in some cultures there are some surprizing
customs . so may be dr field did not anlysed the culture of that island on various
parameters , which we are using now a days.intrveiw with children and observing their
behaviour is important because some time the person talk one thing and behave in
different way look like either he not telling correct or he is showing his altitude in
misguiding way. i think the behaviour of the children shows proper report of reserch
and you can observe their altitude to the other adult peoples of the village and to their
own biological parents.The expert reserch scholer can easily feel their emotions and
behavour during some time stay with their culture. dr field maybe more research time,
maybe, for longer.
Reader Commentary
This fundamentally deficient response mainly consists of a summary of the prompt,
and although there is some evidence of understanding, the response provides little evi-
dence of the ability to develop and organize an analysis of the argument. Also, severe
problems in language persistently interfere with meaning. In fact, the material that
does not come directly from the prompt is more or less incomprehensible.
366
GRE Practice Test 1
This ebook was issued to HUAN-LIN CHANG, order #14803770223. Unlawful distribution of this ebook is prohibited.
Customer name HUAN-LIN CHANG , Order Id: 14803770223
367
Answers and Explanations
Answers and Explanations
SECTION 3
Verbal Reasoning
25 Questions with Explanations
For each of Questions 1 to 4, select one answer choice unless otherwise instructed.
Questions 1 to 3 are based on the following reading passage.
Whether the languages of the ancient American peoples were used for expressing
abstract universal concepts can be clearly answered in the case of Nahuatl. Nahuatl,
like Greek and German, is a language that allows the formation of extensive com-
pounds. By the combination of radicals or semantic elements, single compound words
can express complex conceptual relations, often of an abstract universal character.
The tlamatinime (those who know) were able to use this rich stock of abstract terms
to express the nuances of their thought. They also availed themselves of other forms of
expression with metaphorical meaning, some probably original, some derived from
Toltec coinages. Of these forms, the most characteristic in Nahuatl is the juxtaposition
of two words that, because they are synonyms, associated terms, or even contraries,
complement each other to evoke one single idea. Used metaphorically, the juxtaposed
terms connote specific or essential traits of the being they refer to, introducing a mode
of poetry as an almost habitual form of expression.
Description
This passage claims that Nahuatl was used to express abstract universal concepts, by
combining semantic elements, and goes on to explain that the tlamatinime used these
terms to express subtle distinctions.
For the following question, consider each of the choices separately and select all that
apply.
1. Which of the following can be inferred from the passage regarding present-day
research relating to Nahuatl?
A
Some record or evidence of the thought of the tlamatinime is available.
B
For at least some Nahuatl expressions, researchers are able to trace their
derivation from another ancient American language.
C
Researchers believe that in Nahuatl, abstract universal concepts are
always expressed metaphorically.
Explanation
Choices A and B are correct.
Choice A is correct: the tlamatinime are mentioned in the first sentence of the sec-
ond paragraph, where it says they were able to use Nahuatl’s stock of abstract terms “to
express the nuances of their thought.” This suggests that there is some evidence of
what those thoughts were, and therefore Choice A can be inferred.
line
5
10
This ebook was issued to HUAN-LIN CHANG, order #14803770223. Unlawful distribution of this ebook is prohibited.
Customer name HUAN-LIN CHANG , Order Id: 14803770223
Choice B is correct: according to the next sentence, Nahuatl speakers used “forms
of expression with metaphorical meaning,” some of which were probably “original”
and others “derived from Toltec coinages.” That researchers know certain Nahuatl
expressions are derived from Toltec suggests that they are able to trace the derivation
of some Nahuatl expressions from another language besides Nahuatl, and therefore
Choice B may be inferred.
Choice C is incorrect: the passage says that in Nahuatl there are single compound
words that can express conceptual relations of an “abstract universal character” and
mentions “other forms of expression with metaphorical meaning,” but it does not indi-
cate whether metaphorical words or phrases are the only way that abstract universal
concepts are expressed in Nahuatl, or whether researchers believe this about Nahuatl.
Therefore Choice C cannot be inferred.
2. Select the sentence in the passage in which the author introduces a specific
Nahuatl mode of expression that is not identified as being shared with certain
European languages.
Explanation
The passage introduces two specific Nahuatl modes of expression. One is the formation
of single compound words that are capable of expressing complex conceptual relations
(first paragraph); the other is the juxtaposition of two related words to evoke a single
idea (second paragraph). In the formation of compounds Nahuatl is described as being
“like Greek and German,” but the second mode is not identified as being shared with
other languages. Therefore the sixth sentence (“Of these forms . . . one single idea”) is
the best choice.
3. In the context in which it appears, “coinages” (line 9) most nearly means
A
adaptations
B
creations
C
idiosyncrasies
D
pronunciations
E
currencies
Explanation
“Coinage” has two senses that are represented among the answer choices: in one sense
it denotes coins and currency, while in the other it denotes things especially words
that are invented. The fifth sentence draws a contrast between linguistic expressions
original to Nahuatl and those derived from Toltec. In this context of original versus
derived language, “coinages” means “inventions,” not “currencies.” Of the answer
choices given, “creations” is the nearest equivalent of “coinages” in the sense of “inven-
tions,” and therefore Choice B is the best answer.
368
GRE Practice Test 1
This ebook was issued to HUAN-LIN CHANG, order #14803770223. Unlawful distribution of this ebook is prohibited.
Customer name HUAN-LIN CHANG , Order Id: 14803770223
Question 4 is based on the following reading passage.
At a certain period in Earth’s history, its atmosphere contained almost no oxygen,
although plants were producing vast quantities of oxygen. As a way of reconciling these
two facts, scientists have hypothesized that nearly all of the oxygen being produced was
taken up by iron on Earth’s surface. Clearly, however, this explanation is inadequate.
New studies show that the amount of iron on Earth’s surface was not sufficient
to absorb anywhere near as much oxygen as was being produced. Therefore,
something in addition to the iron on Earth’s surface must have absorbed much of the
oxygen produced by plant life.
4. In the argument given, the two portions in boldface play which of the following
roles?
A
The first is a claim made by the argument in support of a certain position;
the second is that position.
B
The first is a judgment made by the argument about a certain explanation;
the second is that explanation.
C
The first expresses the argument’s dismissal of an objection to the position
it seeks to establish; the second is that position.
D
The first sums up the argument’s position with regard to a certain
hypothesis; the second provides grounds for that position.
E
The first is a concession by the argument that its initial formulation of the
position it seeks to establish requires modification; the second presents
that position in a modified form.
Explanation
The passage presents an argument and the question asks you to identify the role the
portions highlighted in boldface play in that argument. The first step in responding is
to read through the passage quickly to get an understanding of what is being said. Then
it is possible to go back and assess how the parts of the passage fit together into an
argument.
In this passage the first sentence presents two pieces of information that seem to be
in conflict the atmosphere contained almost no oxygen even though plants were pro-
ducing so much of it. The second sentence presents a hypothetical explanation that has
been proposed for reconciling the discrepancy that oxygen was absorbed by iron.
The next sentence calls this hypothetical explanation inadequate and the following sen-
tence gives a reason for that judgment that there was insufficient iron for the pro-
posed explanation to work. Finally, the last sentence draws the conclusion that there
must have been something in addition to iron to absorb the oxygen.
Since the highlighted portions in the passage represent the main content of the
third and fourth sentences, the task in this question is to find the answer choice whose
two parts fit those sentences’ roles. It can be seen that answer Choice D fits the require-
ment: the third sentence does sum up the argument’s position about a hypothesis, and
the fourth sentence gives grounds for the third. Therefore Choice D is the correct
answer.
369
Answers and Explanations
This ebook was issued to HUAN-LIN CHANG, order #14803770223. Unlawful distribution of this ebook is prohibited.
Customer name HUAN-LIN CHANG , Order Id: 14803770223
370
GRE Practice Test 1
For Questions 5 to 8, select one entry for each blank from the corresponding column
of choices. Fill all blanks in the way that best completes the text.
5. In her later years, Bertha Pappenheim was an apostle of noble but already
(i)__________ notions, always respected for her integrity, her energy, and her
resolve but increasingly out of step and ultimately (ii)__________ even her own
organization.
Blank (i) Blank (ii)
A
anachronistic
D
emulated by
B
accepted
E
appreciated by
C
exotic
F
alienated from
Explanation
The sentence is clearly conveying a contrast since “but” is used twice to indicate some-
thing positive and something negative about Pappenheim. The clue to the negative
aspect is in the later part of the sentence, where “out of step” leads both to “anachro-
nistic” as the answer for the first blank and “alienated from” as the answer for the
second.
Thus, the correct answer is anachronistic (Choice A) and alienated from (Choice F).
6. The reception given to Kimura’s radical theory of molecular evolution shows that
when __________ fights orthodoxy to a draw, then novelty has seized a good
chunk of space from convention.
A
imitation
B
reaction
C
dogmatism
D
invention
E
caution
Explanation
The sentence sets up two parallel, contrasting concepts. The word in the blank must
contrast with “orthodoxy,” and since “convention” in the second contrasting pair is syn-
onymous with “orthodoxy,” the correct answer should be roughly synonymous with
“novelty.” The word “invention” is the best choice.
Thus, the correct answer is invention (Choice D).
7. The (i)__________ of Vladimir Nabokov as one of North America’s literary giants
has thrown the spotlight on his peripheral activities and has thus served to
(ii)__________ his efforts as an amateur entomologist.
Blank (i) Blank (ii)
A
stigmatization
D
foreground
B
lionization
E
transcend
C
marginalization
F
obscure
This ebook was issued to HUAN-LIN CHANG, order #14803770223. Unlawful distribution of this ebook is prohibited.
Customer name HUAN-LIN CHANG , Order Id: 14803770223
371
Answers and Explanations
Explanation
It is possible to analyze this sentence by starting with either blank. Broadly, it states
that something that has happened to Nabokov has called attention to some of his
peripheral activities. It would hardly make sense for what had happened to be either
stigmatization or marginalization, since both of those activities represent a turning
away from him, not a calling attention to him. So the correct answer for the first blank
is “lionization,” since to lionize means to treat as important. Then, since we are told
that Nabokov is a literary giant, entomology must be one of his peripheral activities, so
the correct answer for the second blank must be “foreground,” which also means “call
attention to.” Spotlighting something would not result in transcending it or obscuring
it, so neither of the other choices is correct.
Thus, the correct answer is lionization (Choice B) and foreground (Choice D).
8. Mathematicians have a distinctive sense of beauty: they strive to present their
ideas and results in a clear and compelling fashion, dictated by __________ as
well as by logic.
A
caprice
B
aesthetics
C
obligation
D
methodologies
E
intellect
Explanation
The opening statement attributes a “sense of beauty” to mathematicians, and the
remainder of the sentence after the colon spells out that observation. Filling in the
blank will supply some aspect that balances “logic” and reinforces the view that math-
ematicians have a sense of beauty; “aesthetics” is the best choice.
Thus, the correct answer is aesthetics (Choice B).
For each of Questions 9 to 13, select one answer choice unless otherwise instructed.
Questions 9 to 12 are based on the following reading passage.
Animal signals, such as the complex songs of birds, tend to be costly. A bird, by singing,
may forfeit time that could otherwise be spent on other important behaviors such as
foraging or resting. Singing may also advertise an individual’s location to rivals or pred-
ators and impair the ability to detect their approach. Although these types of cost may
be important, discussions of the cost of singing have generally focused on energy costs.
Overall the evidence is equivocal: for instance, while Eberhardt found increases in
energy consumption during singing for Carolina wrens, Chappell found no effect of
crowing on energy consumption in roosters.
To obtain empirical data regarding the energy costs of singing, Thomas examined
the relationship between song rate and overnight changes in body mass of male
nightingales. Birds store energy as subcutaneous fat deposits or “body reserves”;
changes in these reserves can be reliably estimated by measuring changes in body
mass. If singing has important energy costs, nightingales should lose more body mass
line
5
10
This ebook was issued to HUAN-LIN CHANG, order #14803770223. Unlawful distribution of this ebook is prohibited.
Customer name HUAN-LIN CHANG , Order Id: 14803770223
372
GRE Practice Test 1
on nights when their song rate is high. Thomas found that nightingales reached a sig-
nificantly higher body mass at dusk and lost more mass overnight on nights when their
song rate was high.
These results suggest that there may be several costs of singing at night associated
with body reserves. The increased metabolic cost of possessing higher body mass con-
tributes to the increased overnight mass loss. The strategic regulation of evening body
reserves is also likely to incur additional costs, as nightingales must spend more time
foraging in order to build up larger body reserves. The metabolic cost of singing itself
may also contribute to increased loss of reserves. This metabolic cost may arise from
the muscular and neural activity involved in singing or from behaviors associated with
singing. For example, birds may expend more of their reserves on thermoregulation if
they spend the night exposed to the wind on a song post than if they are in a sheltered
roost site. Thomas’s data therefore show that whether or not singing per se has an
important metabolic cost, metabolic costs associated with singing can have an impor-
tant measurable effect on a bird’s daily energy budget, at least in birds with high song
rates such as nightingales.
Description
The passage mentions various ways in which singing is costly to a bird, but soon
focuses on the main topic: the energy costs of singing. The second paragraph then dis-
cusses a particular experiment designed to assess the energy costs of singing for
nightingales, and the third paragraph identifies a range of different associated costs.
9. The primary purpose of the passage is to
A
compare the different types of cost involved for certain birds in singing
B
question a hypothesis regarding the energy costs of singing for certain
birds
C
present evidence suggesting that singing has an important energy cost for
certain birds
D
discuss the benefits provided to an organism by a behavior that is costly in
energy
E
describe an experiment that supports an alternative model of how
birdsong functions
Explanation
As mentioned above, the first paragraph mentions various costs associated with bird-
song, but from that point onward, the focus of the passage is on evidence concerning
the energy costs of singing, for nightingales in particular. Thus, the correct answer is
Choice C. Although the passage mentions other costs, it does not compare them with
one another, so Choice A is incorrect. Because the passage does not question any
hypotheses, discuss benefits, or advance an alternative model of birdsong, Choices B,
D, and E are incorrect.
line
15
20
25
This ebook was issued to HUAN-LIN CHANG, order #14803770223. Unlawful distribution of this ebook is prohibited.
Customer name HUAN-LIN CHANG , Order Id: 14803770223
373
Answers and Explanations
For the following question, consider each of the choices separately and select all that
apply.
10. The passage implies that during the day before a night on which a male
nightingale’s song rate is high, that nightingale probably does which of the
following?
A
Expends less of its reserves on thermoregulation than on other days
B
Stores more energy as body reserves than on other days
C
Hides to avoid predators
Explanation
Choice B is correct.
Choice A is incorrect: the only reference to thermoregulation comes in line 24 and
discusses nighttime activity, not daytime activity.
Choice B is correct: the second paragraph explains that birds store energy as fat
deposits that can be estimated by measuring body mass, and that body mass at dusk
was significantly higher in nightingales on nights when their song rate was higher.
Choice C is incorrect: while the passage does say that singing exposes birds to pred-
ators (line 3), it says nothing to suggest that they make special efforts to hide before
singing, and in fact it says that nightingales spend extra time foraging (line 20).
11. Select the sentence in the first or second paragraph that presents empirical
results in support of a hypothesis about the energy costs of singing.
Explanation
Only two sentences in the relevant portion of the passage contain information that
might be considered to be empirical results. The last sentence of the first paragraph
contains information about increases in energy consumption but only the last sentence
of the second paragraph provides results in support of the only hypothesis in the pas-
sage, that nightingales should lose more body mass on nights when their song rate is
high. Thus, sentence 9 (“Thomas found . . . high”) is the correct choice.
For the following question, consider each of the choices separately and select all that
apply.
12. It can be inferred from the passage that compared with other costs of singing,
which of the following is true of the energy costs of singing?
A
They are the single greatest cost to an individual bird.
B
They have generally received more attention from scientists.
C
They vary less from one bird species to another.
Explanation
Choice B is correct.
Choice A is incorrect: you might infer that energy costs of singing are significant
but no information is given to suggest that they are greater than other costs.
Choice B is correct: lines 4–5 say that discussions of the costs of singing have gen-
erally focused on energy costs.
This ebook was issued to HUAN-LIN CHANG, order #14803770223. Unlawful distribution of this ebook is prohibited.
Customer name HUAN-LIN CHANG , Order Id: 14803770223
Choice C is incorrect: the only mention of differences across species occurs in the
discussion of the findings of Eberhardt and Chappell. These findings relate to energy
costs alone and, if anything, suggest that energy costs vary considerably.
Question 13 is based on the following reading passage.
In the past ten years, there have been several improvements in mountain-climbing
equipment. These improvements have made the sport both safer and more enjoyable
for experienced climbers. Despite these improvements, however, the rate of mountain-
climbing injuries has doubled in the past ten years.
13. Which of the following, if true, best reconciles the apparent discrepancy
presented in the passage?
A
Many climbers, lulled into a false sense of security, use the new equipment
to attempt climbing feats of which they are not capable.
B
Some mountain-climbing injuries are caused by unforeseeable weather
conditions.
C
Mountain climbing, although a dangerous sport, does not normally result
in injury to the experienced climber.
D
In the past ten years there have been improvements in mountain-climbing
techniques as well as in mountain-climbing equipment.
E
Although the rate of mountain-climbing injuries has increased, the rate of
mountain-climbing deaths has not changed.
Explanation
In this question you are asked to identify the fact that would best reconcile the appar-
ent discrepancy that the passage presents. The discrepancy is that despite improve-
ments in mountain climbing equipment that have made climbing safer, the incidence
of mountain-climbing injuries has greatly increased. Choice A explains how this could
have happened the improvements in equipment have led climbers to attempt feats
that are beyond their level of skill. Therefore, Choice A is the correct answer.
None of the other choices provides information that resolves the discrepancy. Nei-
ther Choice B nor Choice C relates to conditions that have changed over the relevant
ten-year period. Choices D and E do relate to the relevant period. But if, as Choice D
says, techniques as well as equipment have improved, that fact by itself only makes the
increase in injuries more puzzling. Choice E provides more data about the conse-
quences of climbing accidents, but doesn’t suggest any explanation for the increase in
injuries.
374
GRE Practice Test 1
This ebook was issued to HUAN-LIN CHANG, order #14803770223. Unlawful distribution of this ebook is prohibited.
Customer name HUAN-LIN CHANG , Order Id: 14803770223
375
Answers and Explanations
For Questions 14 to 17, select one entry for each blank from the corresponding
column of choices. Fill all blanks in the way that best completes the text.
14. Unenlightened authoritarian managers rarely recognize a crucial reason for the
low levels of serious conflict among members of democratically run work groups:
a modicum of tolerance for dissent often prevents __________.
A
demur
B
schism
C
cooperation
D
compliance
E
shortsightedness
Explanation
The blank must be filled with a word that describes a problem that a work group can
suffer, a problem that can be a cause of (or associated with) serious conflict. Of the
answer choices, only “schism” fits this description.
Thus, the correct answer is schism (Choice B).
15. The novelist devotes so much time to avid descriptions of his characters’ clothes
that the reader soon feels that such __________ concerns, although worthy of
attention, have superseded any more directly literary aims.
A
didactic
B
syntactical
C
irrelevant
D
sartorial
E
frivolous
Explanation
The “concerns” described by the adjective that fills the blank relate to clothing, so “sar-
torial” fits the blank. Although these concerns could also be described as “irrelevant” or
“frivolous,” neither of these choices is correct because the sentence identifies the con-
cerns as “worthy of attention.”
Thus, the correct answer is sartorial (Choice D).
16. Belanger dances with an (i)__________ that draws one’s attention as if by seeking
to (ii)__________ it; through finesse and understatement, he manages to seem at
once intensely present and curiously detached.
Blank (i) Blank (ii)
A
undemonstrative panache
D
focus
B
unrestrained enthusiasm
E
overwhelm
C
unattractive gawkiness
F
deflect
This ebook was issued to HUAN-LIN CHANG, order #14803770223. Unlawful distribution of this ebook is prohibited.
Customer name HUAN-LIN CHANG , Order Id: 14803770223
376
GRE Practice Test 1
Explanation
The point of the sentence is to emphasize contradictory aspects of Belanger’s dancing:
we are told, for example, that he seems “at once intensely present and curiously
detached.” Looking at the second blank with this point in mind, we can see that the
sentence is saying that Belanger draws attention in some way that would not normally
be a means of doing so. The only choice that fits, therefore, is “deflect”; focusing or
overwhelming attention would certainly be expected to draw it. And since employing
“unrestrained enthusiasm” or “unattractive gawkiness” would not be ways of deflecting
attention, the correct choice for the first blank is “undemonstrative panache,” another
paradoxical term, since “panache” means “dash or flamboyance in style.”
Thus, the correct answer is undemonstrative panache (Choice A) and deflect
(Choice F).
17. The most striking thing about the politician is how often his politics have been
(i)__________ rather than ideological, as he adapts his political positions at any
particular moment to the political realities that constrain him. He does not,
however, piously (ii)__________ political principles only to betray them in
practice. Rather, he attempts in subtle ways to balance his political self-interest
with a (iii) __________, viewing himself as an instrument of some unchanging
higher purpose.
Blank (i) Blank (ii) Blank (iii)
A
quixotic
D
brandish
G
profound
cynicism
B
self-righteous
E
flout
H
deeply felt moral
code
C
strategic
F
follow
I
thoroughgoing
pragmatism
Explanation
Since the politician is portrayed as adapting political positions to political realities,
blank (i) should be filled with “strategic,” which is also the only choice that provides
the required contrast with “ideological.” The second blank, brandishing political prin-
ciples is what a politician might do piously, while flouting is not pious and following
principles does not make sense when combined with “betray[ing] them in practice.”
The third blank requires something that would have to be balanced against “political
self-interest” and that would be embraced in service of an “unchanging higher pur-
pose,” making “deeply felt moral code” the only viable choice.
Thus, the correct answer is strategic (Choice C), brandish (Choice D), and deeply
felt moral code (Choice H).
For each of Questions 18 to 20, select one answer choice unless otherwise instructed.
Questions 18 to 20 are based on the following reading passage.
The condition of scholarship devoted to the history of women in photography is con-
founding. Recent years have witnessed the posthumous inflation of the role of the hob-
byist Alice Austen into that of a pioneering documentarian while dozens of notable
This ebook was issued to HUAN-LIN CHANG, order #14803770223. Unlawful distribution of this ebook is prohibited.
Customer name HUAN-LIN CHANG , Order Id: 14803770223
377
Answers and Explanations
senior figures Marion Palfi, whose photographs of civil-rights activities in the South
served as early evidence of the need for protective legislation, to name one received
scant attention from scholars. And, while Naomi Rosenblum’s synoptic History of
Women Photographers covers the subject through 1920 in a generally useful fashion,
once she reaches the 1920s, when the venues, forms, applications, and movements of
the medium expanded exponentially, she resorts to an increasingly terse listing of un -
familiar names, with approaches and careers summarized in a sentence or two.
Description
The passage expresses dismay at the current state of scholarship concerning the history
of women in photography: some figures receive disproportionate attention, and past
1920 Rosenblum’s book is too sketchy to be useful.
18. The author of the passage cites Rosenblum’s book most likely in order to
A
suggest that the works documented most thoroughly by historians of
women in photography often do not warrant that attention
B
offer an explanation for the observation that not all aspects of the history
of women in photography have received the same level of attention
C
provide an example of a way in which scholarship on the history of
women in photography has been unsatisfactory
D
suggest that employing a strictly chronological approach when studying
the history of women in photography may be unproductive
E
provide support for the notion that certain personalities in women’s
photography have attained undue prominence
Explanation
As mentioned above, the topic of the passage is the unsatisfactory condition of schol-
arship devoted to the history of women in photography. Since Rosenblum’s book is
clearly presented as an example of this unsatisfactory scholarship, Choice C is the cor-
rect answer. Choice D may seem appealing, because a strictly chronological approach
might be inadequate to represent the explosive growth of the field in the 1920s. How-
ever, the sentence does not develop this idea, and this is not the reason for mentioning
Rosenblum.
For the following question, consider each of the choices separately and select all that
apply.
19. Which of the following statements about Marion Palfi is supported by the
passage?
A
Marion Palfi’s photographs would have received greater recognition from
historians had her work been done in an era when most aspects of
photography were static rather than in a state of transition.
B
Alice Austen has achieved greater notoriety than has Marion Palfi
primarily because the subjects that Austen photographed were more
familiar to her contemporaries.
C
In addition to providing a record of certain historical events, Marion
Palfi’s photographs played a role in subsequent events.
line
5
10
This ebook was issued to HUAN-LIN CHANG, order #14803770223. Unlawful distribution of this ebook is prohibited.
Customer name HUAN-LIN CHANG , Order Id: 14803770223
Explanation
Choice C is correct.
Choice A is incorrect: the passage does not state whether the period in which Palfi
was working was an era when photography was static or in transition.
Choice B is incorrect: the passage does not state the nature of the subjects Austen
photographed, nor compare their relative familiarity to those photographed by Palfi.
Choice C is correct: Palfi’s photographs played a role in subsequent events because
they served as early evidence of the need for protective legislation.
20. In the context in which it appears, “inflation” (line 2) most nearly means
A
exaggeration
B
acquisition
C
evaluation
D
distortion
E
attenuation
Explanation
The term “hobbyist” suggests Austen’s relative lack of seriousness as a photographer
when compared with “senior figures,” yet her role has been elevated to that of a “pio-
neering documentarian” at the expense of these other figures. Choice D may be appeal-
ing in that this elevation could be considered a form of distortion, but Choice A is more
specific as well as more in line with the dictionary definition of “inflated” as “expanded
to an abnormal or unjustifiable volume or level.” Thus, Choice A, “exaggeration,” is the
correct answer.
For Questions 21 to 25, select the two answer choices that, when used to complete
the sentence, fit the meaning of the sentence as a whole and produce completed
sentences that are alike in meaning.
21. The plan, which the engineers said would save the aquifer by reducing pumping
to __________ levels, has passed a governmental environmental review but faces
opposition from outdoor and environmental groups.
A
innocuous
B
feasible
C
practicable
D
minimal
E
remedial
F
benign
Explanation
If the engineers think that the reduced levels will save the aquifer, they may describe
the reduced levels as innocuous, minimal, remedial, or benign. Of these words, only
“innocuous” and “benign” produce sentences with the same meaning. The two words
“feasible” and “practicable” are similar in meaning, but do not fit the context well,
because they imply that the current levels are not feasible or practicable, conflicting
with the implication that the current levels, though perhaps undesirable, are neverthe-
less entirely feasible.
Thus, the correct answer is innocuous (Choice A) and benign (Choice F).
378
GRE Practice Test 1
This ebook was issued to HUAN-LIN CHANG, order #14803770223. Unlawful distribution of this ebook is prohibited.
Customer name HUAN-LIN CHANG , Order Id: 14803770223
22. Though feminist in its implications, Yvonne Rainer’s 1974 film __________ the
filmmaker’s active involvement in feminist politics.
A
antedated
B
cloaked
C
portrayed
D
preceded
E
renewed
F
represented
Explanation
The words that fill the blank must fit with the idea that Rainer’s film has some feminist
implications, but that those are limited compared with her other activities. From the
six words offered as answer choices, the pair “antedated” and “preceded” and the pair
“portrayed” and “represented” each produce sentences that are similar in meaning.
However, only “antedated” and “preceded” make sense in the context of the sentence:
Rainer’s 1974 film exhibits feminist themes in a limited way because it came before she
became active in feminist politics.
Thus, the correct answer is antedated (Choice A) and preceded (Choice D).
23. Congress is having great difficulty developing a consensus on energy policy,
primarily because the policy objectives of various members of Congress rest on
such __________ assumptions.
A
commonplace
B
disparate
C
divergent
D
fundamental
E
trite
F
trivial
Explanation
The words that fill the blank must help explain the difficulty of developing a consensus.
A lack of agreement on the assumptions that underlie Congress members’ policy objec-
tives would contribute to such a difficulty. Accordingly, “disparate” and “divergent” are
the best choices because they both indicate disagreement among the members.
Although the words “trite” and “trivial” are similar in meaning, triteness and triviality
do not help to explain the difficulty in developing a consensus.
Thus, the correct answer is disparate (Choice B) and divergent (Choice C).
24. During the opera’s most famous aria, the tempo chosen by the orchestra’s
conductor seemed __________, without necessary relation to what had gone
before.
A
arbitrary
B
capricious
C
cautious
D
compelling
E
exacting
F
meticulous
379
Answers and Explanations
This ebook was issued to HUAN-LIN CHANG, order #14803770223. Unlawful distribution of this ebook is prohibited.
Customer name HUAN-LIN CHANG , Order Id: 14803770223
Explanation
Any of the offered words could possibly describe a conductor’s choice of tempo. How-
ever, the phrase “without necessary relation to what had gone before” is presented as
an elaboration on the word in the blank. Among the answer choices, only “arbitrary”
and “capricious” could be elaborated that way; none of the other choices would be
explained by the final phrase.
Thus, the correct answer is arbitrary (Choice A) and capricious (Choice B).
25. Because they had expected the spacecraft Voyager 2 to be able to gather data only
about the planets Jupiter and Saturn, scientists were __________ the wealth of
information it sent back from Neptune twelve years after leaving Earth.
A
anxious for
B
confident in
C
thrilled about
D
keen on
E
elated by
F
eager for
Explanation
In the sentence, the words “expected” and “only” imply that the data received from the
spacecraft exceeded scientists’ expectations. Therefore, the words that fill the blank
should describe a reaction to results that are better than hoped for, and the choices
“thrilled about” and “elated by” both express such a reaction. The scientists may well
also have been eager for, or keen on, the information, but their eagerness is not well
explained by the unexpectedness of the information.
Thus, the correct answer is thrilled about (Choice C) and elated by (Choice E).
380
GRE Practice Test 1
This ebook was issued to HUAN-LIN CHANG, order #14803770223. Unlawful distribution of this ebook is prohibited.
Customer name HUAN-LIN CHANG , Order Id: 14803770223
381
Answers and Explanations
SECTION 4
Verbal Reasoning
25 Questions with Explanations
For Questions 1 to 4, select the two answer choices that, when used to complete
the sentence, fit the meaning of the sentence as a whole and produce completed
sentences that are alike in meaning.
1. Only by ignoring decades of mismanagement and inefficiency could investors
conclude that a fresh infusion of cash would provide anything other than a
__________ solution to the company’s financial woes.
A
complete
B
fleeting
C
momentary
D
premature
E
trivial
F
total
Explanation
The key phrases that indicate how the blank for this question should be completed are
“Only by ignoring decades of mismanagement and inefficiency” and “provide anything
other than.” Taken together, these phrases indicate that the sentence will not envision a
very beneficial or successful resolution of the “financial woes.” Among the answer
choices, “complete” and “total” are quite close in meaning and would clearly create two
sentences very similar in meaning. But those two sentences would be internally con-
tradictory, suggesting that doing something unwise would completely solve a problem.
“Fleeting” and “momentary” suggest that the event referred to (“a fresh infusion of
cash”) might have some beneficial effect, but that it would ultimately not resolve the
problem.
Thus, the correct answer is fleeting (Choice B) and momentary (Choice C).
2. Some scientists argue that carbon compounds play such a central role in life on
Earth because of the possibility of __________ resulting from the carbon atom’s
ability to form an unending series of different molecules.
A
diversity
B
deviation
C
variety
D
reproduction
E
stability
F
invigoration
Explanation
The key phrase that indicates how the blank for this question should be completed is
“the ability to form an unending series of different molecules.” Among the answer
choices, “diversity” and “variety” clearly fit logically with “unending” and “different”
and create two very similar sentences. No other pair of choices here would produce
two sentences as similar in meaning as those created by placing “diversity” and “variety”
in the blank. Thus, the correct answer is diversity (Choice A) and variety (Choice C).
This ebook was issued to HUAN-LIN CHANG, order #14803770223. Unlawful distribution of this ebook is prohibited.
Customer name HUAN-LIN CHANG , Order Id: 14803770223
3. Given the flood of information presented by the mass media, the only way for
someone to keep abreast of the news is to rely on __________ accounts.
A
synoptic
B
abridged
C
sensational
D
copious
E
lurid
F
understated
Explanation
The key phrase that indicates how the blank for this question should be completed is
“the only way for someone to keep abreast of the news.” Among the answer choices,
“synoptic” and “abridged,” while not synonymous in the strict sense, both fit the logic
of this description, “synoptic” because of its emphasis on breadth and generality as
opposed to detail, and “abridged” because of its obvious focus on brevity. “Sensational”
and “lurid” would create two similar sentences but do not fit the logic for completing
the blank, since we would not be relying on sensational or lurid accounts in order to
keep abreast of the news.
Thus, the correct answer is synoptic (Choice A) and abridged (Choice B).
4. Always circumspect, she was reluctant to make judgments, but once arriving at a
conclusion, she was __________ in its defense.
A
deferential
B
intransigent
C
lax
C
negligent
E
obsequious
F
resolute
Explanation
The key phrases that indicate how the blank for this question should be completed are:
“circumspect,” “reluctant,” and “but once.” Taken together they point to completing the
blank with something that is opposite in some way to the two cited adjectives. Among
the answer choices, “intransigent” and “resolute,” although not strictly synonymous,
both fit the logic of the description given here for completing the blank and create sen-
tences that are similar in meaning. “Lax” and “negligent” are clearly similar in meaning
and would create sentences similar in meaning, but they continue the sentiment voiced
in the initial clause rather than contrasting with it. “Deferential” and “obsequious” are
also similar in meaning, but their emphasis on “politeness,” while not strictly synony-
mous with reluctance and circumspection, like “lax” and “negligent” fail to pick up on
the expected contrast.
Thus, the correct answer is intransigent (Choice B) and resolute (Choice F).
382
GRE Practice Test 1
This ebook was issued to HUAN-LIN CHANG, order #14803770223. Unlawful distribution of this ebook is prohibited.
Customer name HUAN-LIN CHANG , Order Id: 14803770223
383
Answers and Explanations
For each of Questions 5 to 9, select one answer choice unless otherwise instructed.
Questions 5 and 6 are based on the following reading passage.
When marine organisms called phytoplankton photosynthesize, they absorb carbon
dioxide dissolved in seawater, potentially causing a reduction in the concentration of
atmospheric carbon dioxide, a gas that contributes to global warming. However, phy-
toplankton flourish only in surface waters where iron levels are sufficiently high. Mar-
tin therefore hypothesized that adding iron to iron-poor regions of the ocean could
help alleviate global warming. While experiments subsequently confirmed that such a
procedure increases phytoplankton growth, field tests have shown that such growth
does not significantly lower atmospheric carbon dioxide. When phytoplankton utilize
carbon dioxide for photosynthesis, the carbon becomes a building block for organic
matter, but the carbon leaks back into the atmosphere when predators consume the
phytoplankton and respire carbon dioxide.
Description
The paragraph presents a hypotheses about reducing global warming by adding iron
to iron-poor areas of the ocean and explains why adding the iron does not have the
hoped-for benefit.
For the following question, consider each of the choices separately and select all that
apply.
5. It can be inferred from the passage that Martin’s hypothesis includes which of the
following elements?
A
A correct understanding of how phytoplankton photosynthesis utilizes
carbon dioxide
B
A correct prediction about how the addition of iron to iron-poor waters
would affect phytoplankton growth
C
An incorrect prediction about how phytoplankton growth would affect the
concentration of atmospheric carbon dioxide
Explanation
All three choices are correct. Martin’s hypothesis was that adding iron to iron-poor
regions of the ocean could help alleviate global warming.
Choice A is correct: the passage presents Martin as using the standard under-
standing of how phytoplankton photosynthesize as a basis for the hypothesis.
Choice B is correct: the passage states that experiments confirmed that adding
iron to iron-poor regions increases phytoplankton growth in those regions. Therefore,
Martin’s prediction about this was correct.
Choice C is correct: it can be inferred that in Martin’s hypothesis the means by
which adding iron in certain regions could alleviate global warming is that phyto-
plankton increase in those regions and absorb atmospheric carbon dioxide. The pas-
sage states that predators who consume phytoplankton respire carbon dioxide, so that
the carbon dioxide absorbed by phytoplankton reenters the atmosphere. Therefore,
Martin’s prediction about this was incorrect.
line
5
10
This ebook was issued to HUAN-LIN CHANG, order #14803770223. Unlawful distribution of this ebook is prohibited.
Customer name HUAN-LIN CHANG , Order Id: 14803770223
6. It can be inferred that the author of the passage mentions predators (line 10)
primarily in order to
A
help explain why Martin’s hypothesis is incorrect
B
identify one effect of adding iron to iron-poor waters
C
indicate how some carbon dioxide is converted to solid organic matter
D
help account for differences in the density of phytoplankton between
different regions of the ocean
E
point out a factor that was not anticipated by the scientists who
conducted the field tests mentioned in the passage
Explanation
Lines 7–11 of the paragraph present the evidence against Martin’s hypothesis. Lines
7–8 present field test results showing that Martin’s hypothesis is incorrect, and the last
sentence explains these results: the reason the increased phytoplankton resulting from
the addition of iron do not reduce atmospheric carbon dioxide is that while the phyto-
plankton absorb carbon dioxide, the gas reenters the atmosphere when it is respired by
phytoplankton predators. Therefore Choice A is correct: predators are mentioned to
explain why Martin’s hypothesis is incorrect. Choice B is not correct because while
predators’ consumption of phytoplankton and respiration of carbon dioxide might be
considered one indirect consequence of adding iron to iron-poor waters, identifying a
consequence is not the primary function of the mention of predators. Choice C is incor-
rect because the reference to predators is used to explain how carbon dioxide reap-
pears as a gas, and Choice D is incorrect because no connection is suggested between
predators and the distribution of phytoplankton. Choice E is not correct because it is
Martin who did not anticipate this factor, rather than the scientists who conducted the
field tests.
Question 7 is based on the following reading passage.
Sparva, unlike Treland’s other provinces, requires automobile insurers to pay for any
medical treatment sought by someone who has been involved in an accident; in the
other provinces, insurers pay for nonemergency treatment only if they preapprove the
treatment. Clearly, Sparva’s less restrictive policy must be the explanation for the fact
that altogether insurers there pay for far more treatments after accidents than insurers
in other provinces, even though Sparva does not have the largest population.
Description
The passage tells us that in Sparva automobile insurers pay for far more medical treat-
ments after accidents than they do in Treland’s other provinces. The passage concludes
that the explanation is to be found in the difference in legal requirements for insurers
in Sparva as compared to other provinces.
7. Which of the following, if true, most strengthens the argument?
A
Car insurance costs more in Sparva than in any other province.
B
The cost of medical care in Sparva is higher than the national average.
C
Different insurance companies have different standards for determining
what constitutes emergency treatment.
D
Fewer insurance companies operate in Sparva than in any other province.
E
There are fewer traffic accidents annually in Sparva than in any of the
provinces of comparable or greater population.
384
GRE Practice Test 1
This ebook was issued to HUAN-LIN CHANG, order #14803770223. Unlawful distribution of this ebook is prohibited.
Customer name HUAN-LIN CHANG , Order Id: 14803770223
385
Answers and Explanations
Explanation
The question asks you to identify among the answer choices a fact that would support
the passage’s argument. The explanation offered in the passage can be supported by rul-
ing out other explanations that might, given the information presented in the passage,
appear likely. One obvious explanation for there being more medical treatments in
Sparva is that there are more accidents there. Choice E rules out that explanation. So
Choice E strengthens the argument in the passage and is the correct answer. Choices A
and D each present consequences that are likely results of insurers in Sparva having to
pay for more medical treatments. But neither bears on the cause of insurers having to
pay for more treatments. Choice B does not strengthen the argument and may weaken
it. A higher cost of medical care provides additional motivation for people to seek insur-
ance payments to cover whatever post-accident care they receive. So Choice B might
weaken the argument by providing an alternative explanation for insurers paying for
more medical treatments in Sparva. According to the passage, whether treatment is
emergency treatment is, in other provinces, an important criterion in determining insur-
ers’ responsibility. But since this criterion does not apply in Sparva, Choice C is not
directly relevant to the point that the passage is trying to establish.
Questions 8 and 9 are based on the following reading passage.
Elements of the Philosophy of Newton, published by Voltaire in 1738, was an early
attempt to popularize the scientific ideas of Isaac Newton. In the book’s frontispiece,
Voltaire is seen writing at his desk, and over him a shaft of light from heaven, the light
of truth, passes through Newton to Voltaire’s collaborator Madame du Châtelet; she
reflects that light onto the inspired Voltaire. Voltaire’s book commanded a wide audi-
ence, according to Feingold, because “he was neither a mathematician nor a physicist,
but a literary giant aloof from the academic disputes over Newtonian ideas.” In other
words, Voltaire’s amateurism in science “was a source of his contemporary appeal,
demonstrating for the first time the accessibility of Newton’s ideas to nonspecialists.”
Description
The paragraph describes Voltaire’s book and gives some reasons for its success.
For the following question, consider each of the choices separately and select all that
apply.
8. Which of the following statements about Voltaire’s Elements of the Philosophy of
Newton can be inferred from the passage?
A
Voltaire’s literary stature helped secure a large audience for this attempt to
popularize Newton’s ideas.
B
Voltaire’s status as a nonscientist was an advantage in this effort to bring
Newtonian science to the attention of the general public.
C
The frontispiece of the book implies that Voltaire’s understanding of
Newton’s ideas was not achieved without assistance.
Explanation
All three choices are correct.
Choice A is correct: the paragraph states that one of the reasons Voltaire’s book
commanded a wide audience is that he was “a literary giant.”
This ebook was issued to HUAN-LIN CHANG, order #14803770223. Unlawful distribution of this ebook is prohibited.
Customer name HUAN-LIN CHANG , Order Id: 14803770223
Choice B is correct: the paragraph states that Voltaire’s amateurism in science
demonstrated that nonspecialists could also understand Newton’s ideas.
Choice C is correct: the paragraph refers to Voltaire’s collaborator, Madame du
Châtelet. In the image described, she serves as the intermediary between Newton and
Voltaire, conveying Newton’s ideas to Voltaire.
9. Select the sentence that describes an allegory for Voltaire’s acquisition of
knowledge concerning Newton’s ideas.
Explanation
In the image described in the second sentence, Voltaire acquires his knowledge of New-
ton through Madame du Châtelet, who conveys Newton’s ideas his “light of truth”
to Voltaire. The only other sentence that contains figurative language is the next sen-
tence, in which Voltaire is described as “a literary giant aloof . . . from disputes.” How-
ever, this image refers not to Voltaire’s acquisition of knowledge of Newtonian ideas,
but rather to his attitude regarding Newtonian ideas. Therefore sentence 2 (“In the
book’s . . . Voltaire”) is the correct choice.
For Questions 10 to 13, select one entry for each blank from the corresponding
column of choices. Fill all blanks in the way that best completes the text.
10. Ironically, the writer so wary of (i)__________ was (ii)__________ with ink and
paper, his novel running to 2,500 shagreen-bound folio pages a fortune in
stationery at the time.
Blank (i) Blank (ii)
A
probity
D
acquisitive
B
extravagance
E
illiberal
C
disapprobation
F
profligate
Explanation
The last part of the sentence provides most of the context needed to fill in the two
blanks. The novel was extremely long and required vast amounts of paper. Among the
choices for the second blank, only “profligate” matches this lack of restraint. The word
“Ironically” indicates that what the writer was “wary of” was something similar to
profligacy; of the choices for the first blank, “extravagance” is the closest.
Thus, the correct answer is extravagance (Choice B) and profligate (Choice F).
386
GRE Practice Test 1
This ebook was issued to HUAN-LIN CHANG, order #14803770223. Unlawful distribution of this ebook is prohibited.
Customer name HUAN-LIN CHANG , Order Id: 14803770223
387
Answers and Explanations
11. What readers most commonly remember about John Stuart Mill’s classic
exploration of the liberty of thought and discussion concerns the danger of
(i)__________: in the absence of challenge, one’s opinions, even when they are
correct, grow weak and flabby. Yet Mill had another reason for encouraging the
liberty of thought and discussion: the danger of partiality and incompleteness.
Since one’s opinions, even under the best circumstances, tend to (ii)__________,
and because opinions opposed to one’s own rarely turn out to be completely
(iii)__________, it is crucial to supplement one’s opinions with alternative points
of view.
Blank (i) Blank (ii) Blank (iii)
A
tendentiousness
D
embrace only a
G
erroneous
portion of the truth
B
complacency
E
change over
H
antithetical
time
C
fractiousness
F
focus on matters
I
immutable
close at hand
Explanation
An overview of the passage suggests that the first sentence is relatively self-contained
and that the blank is answerable without the succeeding sentences, where the topic
shifts slightly. The colon after the first blank signals that what follows will define the
word in the blank and will explain what danger Mill was concerned about. It says that
without challenge, one’s opinions grow “weak and flabby” and therefore one becomes
complacent, not tendentious or fractious. A quick reading of the next two sentences
suggests that the topic will be another danger that Mill described, “the danger of par-
tiality and incompleteness.” Free and open discussion needs to take place because each
person’s opinion tends to “embrace only a portion of the truth” and others’ views are
partially right, or never completely “erroneous.” The other choices for the second and
third blanks deal with change, immediacy, or antithesis, none of which relate to the
second danger of “partiality” or “incompleteness.”
Thus, the correct answer is complacency (Choice B), embrace only a portion of
the truth (Choice D), and erroneous (Choice G).
12. Just as the authors’ book on eels is often a key text for courses in marine
vertebrate zoology, their ideas on animal development and phylogeny __________
teaching in this area.
A
prevent
B
defy
C
replicate
D
inform
E
use
Explanation
The “just as” structure indicates that the second half of the sentence should somehow
parallel the idea presented in the first half (i.e., the idea that the authors’ book on eels
This ebook was issued to HUAN-LIN CHANG, order #14803770223. Unlawful distribution of this ebook is prohibited.
Customer name HUAN-LIN CHANG , Order Id: 14803770223
388
GRE Practice Test 1
is a “key text” in marine vertebrate zoology). Among the choices given, “inform” is
clearly the best choice. “Prevent” and “defy” work in the opposite direction, while “use”
and “replicate” would suggest that the authors’ ideas are drawing upon the teaching in
this area rather than the other way around. “Inform” leads to a meaning that nicely
matches the first half of the sentence.
Thus, the correct answer is inform (Choice D).
13. Mechanisms develop whereby every successful species can __________ its innate
capacity for population growth with the constraints that arise through its
interactions with the natural environment.
A
enhance
B
replace
C
produce
D
surpass
E
reconcile
Explanation
A quick overview of the sentence indicates that the blank should be filled with a verb
that indicates what a successful species does with its “innate capacity for population
growth” in the face of certain constraints on that growth. This analysis suggests that
the correct answer will have something to do with adjusting that capacity in the face of
these constraints. Of the choices given, “reconcile” is closest to that meaning. None of
the other options make for a meaningful, coherent sentence. “Enhance,” for example,
may fit nicely with “its innate capacity,” but it does not make sense with “constraints.”
Thus, the correct answer is reconcile (Choice E).
For each of Questions 14 to 16, select one answer choice unless otherwise instructed.
Questions 14 to 16 are based on the following reading passage.
It would be expected that a novel by a figure as prominent as W. E. B. DuBois would
attract the attention of literary critics. Additionally, when the novel subtly engages the
issue of race, as DuBois’ The Quest of the Silver Fleece (1911) does, it would be a sur-
prise not to encounter an abundance of scholarly work about that text. But though
valuable scholarship has examined DuBois’ political and historical thought, his novels
have received scant attention. Perhaps DuBois the novelist must wait his turn behind
DuBois the philosopher, historian, and editor. But what if the truth lies elsewhere: what
if his novels do not speak to current concerns?
Description
The paragraph first presents reasons for critical interest in DuBois’ novels, but then
goes on to explain that there has in fact been very little such interest and speculates as
to why that might be.
This ebook was issued to HUAN-LIN CHANG, order #14803770223. Unlawful distribution of this ebook is prohibited.
Customer name HUAN-LIN CHANG , Order Id: 14803770223
389
Answers and Explanations
14. Which of the following can be inferred from the passage regarding DuBois’ The
Quest of the Silver Fleece?
A
The lack of attention devoted to The Quest of the Silver Fleece can be
attributed to the fact that it was DuBois’ first novel.
B
Among DuBois’ novels, The Quest of the Silver Fleece is unusual in that it
has received scant attention from scholars.
C
The Quest of the Silver Fleece has at least one feature that typically would
attract the attention of literary scholars.
D
The Quest of the Silver Fleece, given its subtle exploration of race, is
probably the best novel written by DuBois.
E
Much of the scholarly work that has focused on The Quest of the Silver
Fleece has been surprisingly critical of it.
Explanation
Choice C is correct. The second sentence states that The Quest of the Silver Fleece sub-
tly engages the issue of race and implies that such an issue would attract the attention
of literary scholars. The passage provides no information about whether The Quest of
the Silver Fleece is DuBois’ first novel (Choice A), whether it received more or less schol-
arly attention than his other novels (Choice B), whether it is better than any of his
other novels (Choice D), nor about what scholars have said about it (Choice E).
15. In the fourth sentence (“Perhaps DuBois . . . editor.”), the author of the passage is
most likely suggesting that
A
scholars will find that DuBois’ novels are more relevant to current
concerns than is his work as philosopher, historian, and editor
B
more scholarly attention will be paid to The Quest of the Silver Fleece than
to DuBois’ other novels
C
DuBois’ novels will come to overshadow his work as philosopher,
historian, and editor
D
DuBois’ novels may eventually attract greater scholarly interest than they
have to date
E
it will be shown that DuBois’ work as philosopher, historian, and editor
had an important influence on his work as novelist
Explanation
The fourth sentence speculates that once DuBois scholars have exhausted potential
avenues of research in the fields of philosophy, history, and editing, they will turn to his
novels, so Choice D is the correct answer. None of the other choices fits the metaphor
in “Perhaps DuBois the novelist must wait his turn.”
This ebook was issued to HUAN-LIN CHANG, order #14803770223. Unlawful distribution of this ebook is prohibited.
Customer name HUAN-LIN CHANG , Order Id: 14803770223
390
GRE Practice Test 1
16. Which of the following best describes the central issue with which the passage is
concerned?
A
The perfunctoriness of much of the critical work devoted to DuBois’
novels
B
The nature of DuBois’ engagement with the issue of race in The Quest of
the Silver Fleece
C
Whether DuBois’ novels are of high quality and relevant to current
concerns
D
The relationship between DuBois the novelist and DuBois the philosopher,
historian, and editor
E
The degree of consideration that has been given to DuBois’ novels,
including The Quest of the Silver Fleece
Explanation
The passage focuses on the scant attention given to DuBois’ novels, The Quest of the Sil-
ver Fleece in particular. The first two sentences give reasons to expect greater attention,
while the last two offer speculations about the explanation for the scant attention.
Thus, Choice E is correct. The issues described in the other answer choices are all mar-
ginal to the passage, if they are mentioned at all.
For Questions 17 to 20, select one entry for each blank from the corresponding
column of choices. Fill all blanks in the way that best completes the text.
17. In the midst of so many evasive comments, this forthright statement, whatever its
intrinsic merit, plainly stands out as __________.
A
a paradigm
B
a misnomer
C
a profundity
D
an inaccuracy
E
an anomaly
Explanation
The sentence offers a contrast between “many evasive statements” and a single “forth-
right statement.” On that basis alone, one might expect an answer such as “an anom-
aly.” Do any of the other options make for a meaningful, coherent sentence? “A
paradigm” is appealing, as is “a profundity,” since the forthright statement is clearly
presented as something positive. However, we are not in a position to call it paradig-
matic or profound, since the sentence withholds judgment on “its intrinsic merit.” The
same reasoning allows us to eliminate “inaccuracy” and “misnomer.” The straightfor-
wardly descriptive “anomaly” is clearly the best choice.
Thus, the correct answer is an anomaly (Choice E).
This ebook was issued to HUAN-LIN CHANG, order #14803770223. Unlawful distribution of this ebook is prohibited.
Customer name HUAN-LIN CHANG , Order Id: 14803770223
391
Answers and Explanations
18. The activists’ energetic work in the service of both woman suffrage and the
temperance movement in the late nineteenth century (i)__________ the assertion
that the two movements were (ii)__________.
Blank (i) Blank (ii)
A
undermines
D
diffuse
B
supports
E
inimical
C
underscores
F
predominant
Explanation
The sentence is about the implications of the activists’ energetic work for some asser-
tion about the woman suffrage and temperance movements. The second blank, how-
ever, obscures the nature of that assertion. But it is clear that the “energetic work”
could either support an assertion that the two movements were similar, or undermine
an assertion that the two movements were opposed. “Supports” is offered as a choice
for the first blank (as is the somewhat similar “underscores”), but there is no corre-
sponding term in the second blank, nothing along the lines of “similar” or “compati-
ble.” “Undermines” and “inimical” make for the only meaningful statement.
Thus, the correct answer is undermines (Choice A) and inimical (Choice E).
19. There is nothing quite like this movie, and indeed I am not altogether sure there
is much more to it than its lovely (i) __________. At a moment when so many
films strive to be as (ii)__________ as possible, it is gratifying to find one that is so
subtle and puzzling.
Blank (i) Blank (ii)
A
peculiarity
D
indirect
B
pellucidity
E
assertive
C
conventionality
F
enigmatic
Explanation
The two sentences provide the reader with quite a bit of information about the movie.
There is “nothing quite like it” and it is “subtle and puzzling.” “Peculiarity” is clearly a
solid fit for the first blank, while “conventionality” clearly does not work, given the fact
that there is “nothing quite like it.” That leaves “pellucidity,” which, while it could fit
logically in the first sentence in isolation, does not fit the later claim that the movie is
“subtle and puzzling.” The second blank needs simply to provide a contrast with “sub-
tle and puzzling.” Of the choices offered, only “assertive” clearly does that.
Thus, the correct answer is peculiarity (Choice A) and assertive (Choice E).
This ebook was issued to HUAN-LIN CHANG, order #14803770223. Unlawful distribution of this ebook is prohibited.
Customer name HUAN-LIN CHANG , Order Id: 14803770223
392
GRE Practice Test 1
20. Wills argues that certain malarial parasites are especially (i)__________ because
they have more recently entered humans than other species and therefore have
had (ii)__________ time to evolve toward (iii)__________. Yet there is no reliable
evidence that the most harmful Plasmodium species has been in humans for a
shorter time than less harmful species.
Blank (i) Blank (ii) Blank (iii)
A
populous
D
ample
G
virulence
B
malignant
E
insufficient
H
benignity
C
threatened
F
adequate
I
variability
Explanation
The “Yet” that begins the second sentence indicates that Wills’ position would be sup-
ported by evidence that the newer parasites are in humans, the more harmful they are.
So Wills’ position must be that more recent parasites are especially harmful, implying
that “malignant” is the correct choice for the first blank. What follows “therefore” is a
potential explanation for the trend that Wills expects, namely an evolution toward
harmlessness, implying “benignity” for the third blank, with newer species having had
“insufficient” time (second blank) to evolve toward harmlessness.
Thus, the correct answer is malignant (Choice B), insufficient (Choice E), and
benignity (Choice H).
For each of questions 21 to 25, select one answer choice unless otherwise instructed.
Question 21 is based on the following reading passage.
Saturn’s giant moon Titan is the only planetary satellite with a significant atmosphere
and the only body in the solar system other than Earth that has a thick atmosphere
dominated by molecular nitrogen. For a long time, the big question about Titan’s
atmosphere was how it could be so thick, given that Jupiter’s moons Ganymede and
Callisto, which are the same size as Titan, have none. The conditions for acquiring and
retaining a thick nitrogen atmosphere are now readily understood. The low tempera-
ture of the protosaturnian nebula enabled Titan to acquire the moderately volatile com-
pounds methane and ammonia (later converted to nitrogen) in addition to water. The
higher temperatures of Jupiter’s moons, which were closer to the Sun, prevented them
from acquiring such an atmosphere.
Description
The paragraph discusses Titan’s thick atmosphere and explains the conditions under
which a body can have a thick atmosphere.
21. According to the passage, Titan differs atmospherically from Ganymede and
Callisto because of a difference in
A
rate of heat loss
B
proximity to the Sun
C
availability of methane and ammonia
D
distance from its planet
E
size
This ebook was issued to HUAN-LIN CHANG, order #14803770223. Unlawful distribution of this ebook is prohibited.
Customer name HUAN-LIN CHANG , Order Id: 14803770223
393
Answers and Explanations
Explanation
According to the last two sentences of the paragraph, Titan was able to acquire an
atmosphere because of a prevailing low temperature, but Ganymede and Callisto could
not because they were at a higher temperature. Because the reason for this difference
in temperature was their respective distances from the Sun, Choice B is correct. The
passage says nothing about differences in rate of heat loss, availability of methane and
ammonia, or distance from their planets, and it explicitly states that the three moons
are the same size.
Question 22 is based on the following reading passage.
Observations of the Arctic reveal that the Arctic Ocean is covered by less ice each sum-
mer than the previous summer. If this warming trend continues, within 50 years the
Arctic Ocean will be ice free during the summer months. This occurrence would in
itself have little or no effect on global sea levels, since the melting of ice floating in
water does not affect the water level. However, serious consequences to sea levels
would eventually result, because __________.
22. Which of the following most logically completes the passage?
A
large masses of floating sea ice would continue to form in the wintertime
B
significant changes in Arctic sea temperatures would be accompanied by
changes in sea temperatures in more temperate parts of the world
C
such a warm Arctic Ocean would trigger the melting of massive land-
based glaciers in the Arctic
D
an ice-free Arctic Ocean would support a very different ecosystem than it
does presently
E
in the spring, melting sea ice would cause more icebergs to be created and
to drift south into shipping routes
Explanation
To logically complete the passage’s open-ended “because,” something is needed that
will explain why the continuation of the warming trend would have serious conse-
quences for sea levels. The passage explains that the melting of the Arctic Ocean ice
will not affect sea levels because the contribution that the water contained in that ice
makes to sea levels is the same whether the water is frozen or liquid. But Choice C
points to a way in which increasing temperatures in the Arctic could add water to the
ocean, namely by melting ice on the land. So Choice C logically completes the passage
and is the correct answer.
Given that the passage has already explained that melting sea ice does not affect
sea levels, the formation of sea ice described in Choice A does not explain why there
would be consequences for sea levels.
Choices B, D, and E all describe possible consequences of increased temperatures
in the Arctic, but none of these consequences suggests a mechanism by which sea
levels would change. So none of these options provides a logical completion for the
passage.
This ebook was issued to HUAN-LIN CHANG, order #14803770223. Unlawful distribution of this ebook is prohibited.
Customer name HUAN-LIN CHANG , Order Id: 14803770223
Questions 23 to 25 are based on the following reading passage.
In a recent study, David Cressy examines two central questions concerning English
immigration to New England in the 1630s: what kinds of people immigrated and why?
Using contemporary literary evidence, shipping lists, and customs records, Cressy finds
that most adult immigrants were skilled in farming or crafts, were literate, and were
organized in families. Each of these characteristics sharply distinguishes the 21,000
people who left for New England in the 1630s from most of the approximately 377,000
English people who had immigrated to America by 1700.
With respect to their reasons for immigrating, Cressy does not deny the frequently
noted fact that some of the immigrants of the 1630s, most notably the organizers and
clergy, advanced religious explanations for departure, but he finds that such explana-
tions usually assumed primacy only in retrospect. When he moves beyond the princi-
pal actors, he finds that religious explanations were less frequently offered, and he
concludes that most people immigrated because they were recruited by promises of
material improvement.
Description
The passage discusses Cressy’s answers to the questions posed in the first sentence. The
immigrants were skilled, literate, and in families, and they apparently immigrated to
have a better life materially, rather than religiously.
For the following question, consider each of the choices separately and select all that
apply.
23. The passage indicates that Cressy would agree with which of the following
statements about the organizers among the English immigrants to New England
in the 1630s?
A
Some of them offered a religious explanation for their immigration.
B
They did not offer any reasons for their immigration until some time after
they had immigrated.
C
They were more likely than the average immigrant to be motivated by
material considerations.
Explanation
Choice A is correct.
Choice A is correct: the organizers are mentioned in the second paragraph, where
the passage says that Cressy “does not deny” that organizers “advanced religious expla-
nations” for leaving England and immigrating to New England in the 1630s. This
suggests that Cressy would agree with the statement in choice A about the organizers.
Choice B is incorrect: in lines 10–11, the passage says that Cressy finds that reli-
gious reasons for immigration “assumed primacy” only in retrospect, but this is not the
same as Cressy’s concluding that no reasons were given at the time of immigration.
Therefore it cannot be inferred that Cressy would agree with the statement in Choice B.
Choice C is incorrect: the passage refers in lines 13–14 to “promises of material
improvement” as a factor that in Cressy’s view motivated most immigrants other than
“the principal actors.” This suggests that Cressy regards the principal actors, such as
organizers, as having been less, not more, motivated by material considerations than
average immigrants were. Therefore it cannot be inferred that Cressy would agree with
the statement in Choice C.
394
GRE Practice Test 1
line
5
10
This ebook was issued to HUAN-LIN CHANG, order #14803770223. Unlawful distribution of this ebook is prohibited.
Customer name HUAN-LIN CHANG , Order Id: 14803770223
24. Select the sentence that provides Cressy’s opinion about what motivated English
immigrants to go to New England in the 1630s.
Explanation
The last sentence says that Cressy “concludes that most people immigrated because
they were recruited by promises of material improvement.” Because this suggests that
Cressy believes immigrants were motivated by these promises to go to New England,
sentence 5 (“When he . . . improvement) is the correct choice. The preceding sentence
suggests that Cressy does not believe religion was a primary motive influencing immi-
grants’ decision to immigrate in the 1630s. Thus, although this sentence provides an
opinion of Cressy’s concerning some immigrants’ stated reasons for immigrating, it
does not say what motive he believes was actually behind the immigration, and there-
fore does not answer the question.
25. In the passage, the author is primarily concerned with
A
summarizing the findings of an investigation
B
analyzing a method of argument
C
evaluating a point of view
D
hypothesizing about a set of circumstances
E
establishing categories
Explanation
The passage is about Cressy’s investigation of English immigration to New England in
the 1630s, and it summarizes his findings concerning who immigrated and why.
Choice A, “summarizing the findings of an investigation,” is therefore the best descrip-
tion of the author’s primary concern in the passage. The passage does not analyze a
method of argument, so Choice B is incorrect. Choice C is incorrect because the pas-
sage is not primarily concerned with evaluating a point of view: it does not assess the
merits or demerits of Cressy’s viewpoint. The passage is concerned with reporting
Cressy’s findings, not with hypothesizing or with establishing categories, so Choices D
and E are incorrect.
395
Answers and Explanations
This ebook was issued to HUAN-LIN CHANG, order #14803770223. Unlawful distribution of this ebook is prohibited.
Customer name HUAN-LIN CHANG , Order Id: 14803770223
GRE Practice Test 1
396
SECTION 5
Quantitative Reasoning
25 Questions with Explanations
AQuantity A is greater.
BQuantity B is greater.
CThe two quantities are equal.
DThe relationship cannot be determined from the information given.
O
3
3
4
x
Ois the center of the circle above.
Quantity A Quantity B
1. x5ABCD
Explanation
In this question you are asked to compare xwith 5, where xis the length of a
line segment from the center of the circle to a point inside the circle. In a circle
the easiest line segments to deal with are the radius and the diameter. Looking at
the figure in the question, you can see that you can draw two radii, each of
which “completes” a right triangle, as shown in the figure below.
O
3
3
4
x
r
r
Since in one of the triangles the lengths of both legs are known, you can use
that triangle to determine the length of the radius of the circle. The triangle has
legs of length 3and 4. If the length of the radius is r, then, using the
Pythagorean theorem, you can see that
222
r=3+4or
2
r=9+16 or
2
r=25, and thus, r=5
Since the length of the radius of the circle is 5 and the line segment of length x
is clearly shorter than the radius, you know that and the correct answer isx<5,
Choice B.
You could also see that the two triangles are congruent, and so againx=4,
yielding Choice B.
This ebook was issued to HUAN-LIN CHANG, order #14803770223. Unlawful distribution of this ebook is prohibited.
Customer name HUAN-LIN CHANG , Order Id: 14803770223
Answers and Explanations
397
Runner Aran kilometer and Runner Bran 800 meters.
4
5
Quantity A Quantity B
2. The distance that Aran The distance that Bran ABCD
Explanation
In this question you are asked to compare two measurements, one given in
kilometers and the other in meters. It would be easier to compare these
measurements if they were both given in meters or both given in kilometers.
If you choose to convert the distance that Runner Bran from meters to
kilometers, you need to use the conversion 1 meter is equal to kilometer.
1
1,000
Since Bran 800 meters, it follows that Bran or kilometer, which
14
(800) ,
冢冣
1,000 5
is the same distance that Aran.
If you choose to convert the distance that Runner Aran from kilometers to
meters, you need to use the conversion 1 kilometer is equal to 1,000 meters.
Since Aran kilometer, it follows that Aran or 800 meters, which is
44
(1,000),
冢冣
55
the same distance that Bran. Either way, Aand Bran the same distance, and
the correct answer is Choice C.
x<y<z
Quantity A Quantity B
3.x+y+z
3
yABCD
Explanation
In this question you are given that and you are asked to comparex<y<z,
with y.
x+y+z
3
Two approaches that you could use to solve this problem are:
1: Search for a mathematical relationship between the two quantities.
2: Plug in numbers for the variables.
Approach 1: Note that is the average of the three numbers x,y, and
x+y+z
3
zand that yis the median. Is the average of 3numbers always equal to the
median? The average could equal the median, but in general they do not have to
be equal. Therefore, the correct answer is Choice D.
Approach 2: When you plug in numbers for the variables, it is a good idea to
consider what kind of numbers are appropriate to plug in and to choose
numbers that are easy to work with, if possible.
Since is the average of the three numbers x,y, and zand you are
x+y+z
3
comparing it to the median, you may want to try plugging in numbers that are
evenly spaced and plugging in numbers that are not evenly spaced.
This ebook was issued to HUAN-LIN CHANG, order #14803770223. Unlawful distribution of this ebook is prohibited.
Customer name HUAN-LIN CHANG , Order Id: 14803770223
GRE Practice Test 1
398
You can plug in numbers that are both evenly spaced and easy to work with.
For example, you can plug in and In this case,x=1, y=2, z=3.
, and so .
x+y+z1+2+36x+y+z
===2=y
333 3
You can also plug in numbers that not are not evenly spaced and are easy to
work with. For example, you can plug in and In this case,x=3,y=6, z=12.
and Since in the first case,
x+y+z3+6+12 21 x+y+zx+y+z
===7, >y.
333 3 3
is equal to yand in the second case, it is greater than y, the relationship between
the two quantities and ycannot be determined from the information
x+y+z
3
given. The correct answer is Choice D.
x
y
50°
Quantity A Quantity B
4.x
y
1ABCD
Explanation
One way you can solve this problem is by using the following fact:
If ABC is a triangle and the measure of angle Ais greater than the
measure of angle B, then the side opposite angle Ais longer than the side
opposite angle B.
Since the third angle of the triangle measures you can use the fact above40,
to conclude that the side opposite the angle is longer than the side opposite50
the angle. So , and which yields Choice A.
x
40x>y>1,
y
You can also solve this problem without using the fact above. Instead, you
can use the strategy of adapting solutions to related problems to determine the
relationship between xand y.
Note that the angles in the triangle differ only a little from the angles in a
triangle. How do the lengths of the legs of a tri-454590454590
angle compare to the lengths of the legs of the triangle in the question? To make
the comparison, add a line segment to the triangle so that the line segment cuts
the angle in two parts, making a angle with the horizontal base, as5045
shown in the following figure:
This ebook was issued to HUAN-LIN CHANG, order #14803770223. Unlawful distribution of this ebook is prohibited.
Customer name HUAN-LIN CHANG , Order Id: 14803770223
Answers and Explanations
399
z
x
y
45°
45°
The triangle has two angles, so , and Since
z
45459045z=y=1.
y
and it follows that The correct answer is Choice A.
zx
=1x>z,>1.
yy
0<x<y<1
Quantity A Quantity B
5. 1 yyxABCD
Explanation
Two approaches that you could use to solve this problem are:
1: Translate from algebra to a number line.
2: Plug in values for the variables.
Approach 1: The following figure represents the information given in the
problem on a number line.
1
0xy
On the number line, is the distance between 1 and y, and is the1 yyx
distance between yand x.Ifyis exactly halfway between xand 1, then is1 y
equal to and if yis not halfway between xand 1, then is not equal toyx;1y
But ycan be any number between xand 1, so the correct answer isyx.
Choice D.
Approach 2: Since this problem involves subtraction, it is a good idea to
choose values for xand ythat are close to each other as well as values that are
far apart. For example, if and then andx=0.4y=0.5, 1 y=0.5 yx=0.1 ;
and if and then and This shows that thex=0.1 y=0.9, 1 y=0.1 yx=0.8.
relationship cannot be determined, and the correct answer is Choice D.
This ebook was issued to HUAN-LIN CHANG, order #14803770223. Unlawful distribution of this ebook is prohibited.
Customer name HUAN-LIN CHANG , Order Id: 14803770223
GRE Practice Test 1
400
pis the probability that event Ewill occur, and sis the
probability that event Ewill not occur.
Quantity A Quantity B
6. p+sps
ABCD
Explanation
Since event Ewill either occur or not occur, it follows that and thep+s=1,
value of Quantity A is always 1. Since Quantity B is the product of the two
probabilities pand s, you need to look at its value for the cases andp=1, p=0,
0<p<1.
If then similarly, if then In both cases, is equalp=1, s=0; p=0, s=1. ps
to 0.
If both pand sare positive and less than 1, so is positive and0 <p<1, ps
less than 1. Since Quantity A is equal to 1 and Quantity B is less than 1, the
correct answer is Choice A.
Xis the set of all integers nthat satisfy the inequality
2≤⎪n⎪≤5.
Quantity A Quantity B
7. The absolute value of the
greatest integer in X
The absolute value of the
least integer in X
ABCD
Explanation
When comparing these quantities, it is important to remember that a nonzero
number and its negative have the same absolute value. For example,
Keeping this in mind, you can see that the positive integers 2, 3,2=2=2.
4, and 5 and the negative integers and all satisfy the inequalities2, 3,4,5
and that these are the only such integers. Thus, the set Xconsists of2 ≤⎪n⎪≤5,
the integers 2, 3,4, and 5. The greatest of these integers is 5,5, 4,3,2,
and its absolute value is 5. The least of these integers is and its absolute5,
value is also 5. Therefore, Quantity A is equal to Quantity B. The correct answer
is Choice C.
xand mare positive numbers, and mis a multiple of 3.
Quantity A Quantity B
8.
m
x
3
x
m/3
xABCD
Explanation
Since you need to compare with Since the base in both
m
x
m3m3m/3
=x,xx.
3
x
expressions is the same, a good strategy to use to solve this problem is to plug in
numbers for min both expressions and compare them.
You know that mis a multiple of 3, so the least positive integer you can plug
in for mis 3.
This ebook was issued to HUAN-LIN CHANG, order #14803770223. Unlawful distribution of this ebook is prohibited.
Customer name HUAN-LIN CHANG , Order Id: 14803770223
Answers and Explanations
401
If then and Since xcan be any real number, its
m3m/3
m=3,x=1x=x.
relationship to 1 cannot be determined from the information given. This example
is sufficient to show that the relationship between and cannot be
m
x
m/3
x
3
x
determined from the information given. The correct answer is Choice D.
A random variable Yis normally distributed with a
mean of 200 and a standard deviation of 10.
Quantity A Quantity B
9. The probability of the event
that the value of Yis
greater than 220
1
6
ABCD
Explanation
This problem involves a normal distribution with mean 200 and standard
deviation 10. Thus, the value of 210 is 1 standard deviation above the mean, and
the value of 220 is 2 standard deviations above the mean. To compare Quantity A
with Quantity B, it is not necessary to exactly determine the probability of the
event that the value of Yis greater than 220. Remember that in any normal
distribution, almost all of the data values, or about 95% of them, fall within 2
standard deviations on either side of the mean. This means that less than 5% of
the values in this distribution will be greater than 220. Thus, the probability of
the event that the value of Yis greater than 220 must be less than 5%, or and
1,
20
this is certainly less than The correct answer is Choice B.
1.
6
Another approach to this problem is to draw a normal curve, or “bell-shaped
curve,” that represents the probability distribution of the random variable Y,as
shown in the figure below.
200170 230220210190180
The curve is symmetric about the mean 200. The values of 210, 220, and 230
are equally spaced to the right of 200 and represent 1, 2, and 3standard
deviations, respectively, above the mean. Similarly, the values of 190, 180, and
170 are 1, 2, and 3standard deviations, respectively, below the mean. Quantity A,
the probability of the event that the value of Yis greater than 220, is equal to the
area of the shaded region as a fraction of the total area under the curve.
From the figure, you can see that the area under the normal curve has been
divided into 6 regions and that these regions are not equal in area. The
This ebook was issued to HUAN-LIN CHANG, order #14803770223. Unlawful distribution of this ebook is prohibited.
Customer name HUAN-LIN CHANG , Order Id: 14803770223
GRE Practice Test 1
402
shaded region is one of the two smallest of the 6 regions, so its area must be less
than of the total area under the curve. The correct answer is Choice B.
1
6
10. The ratio of is equal to the ratio of
13
to
38
A1to8
B8to1
C8to3
D8to9
E9to8
Explanation
Multiplying both parts of a ratio by the same number produces an equivalent
ratio. While you could multiply both fractions in the ratio by any number, 24is a
good number to choose because it is the least common multiple of 3and 8.
Thus, multiplying both and by 24, you get that the ratio of to is equal to
1313
3838
the ratio of 8 to 9. The correct answer is Choice D.
An alternate approach to this problem is to express the ratio of to as the
13
38
fraction This fraction is equivalent to or The correct answer is
1
318 8
.,.
冢冣冢冣
3339
8
Choice D.
11. A reading list for a humanities course consists of 10 books, of which 4are
biographies and the rest are novels. Each student is required to read a
selection of 4books from the list, including 2 or more biographies. How
many selections of 4books satisfy the requirements?
A90
B115
C130
D144
E195
Explanation
The requirement to select 4books, including 2 or more biographies, means that
you have to consider three cases. A student can choose 4biographies and no
novels, or 3biographies and 1 novel, or 2 biographies and 2 novels.
Case 1: Choose 4biographies. This case is easy, as there is only 1 way to
choose all four biographies and no novels.
In the other two cases, you have to find the number of ways of choosing the
biographies and the number of ways of choosing the novels and then multiply
these two numbers.
This ebook was issued to HUAN-LIN CHANG, order #14803770223. Unlawful distribution of this ebook is prohibited.
Customer name HUAN-LIN CHANG , Order Id: 14803770223
Answers and Explanations
403
Case 2: Choose 3biographies and 1 novel. First, you need to find the number
of ways of choosing 3biographies out of 4. If you think of this as not choosing 1
out of the 4, you see that there are 4choices. The number of ways of choosing 1
novel out of the 6 novels is 6. Therefore, the total number of choices is
(4)(6) =24.
Case 3: Choose 2 biographies and 2 novels. First, you need to find the
number of ways of choosing 2 biographies out of 4. This number is sometimes
called “4choose 2” or the number of combinations of 4objects taken 2 at a
time. If you remember the combinations formula, you know that the number of
combinations is (which is denoted symbolically as or ). The
4
4!C
42
冢冣
2
2!(42)!
value of is Thus, there are 6 ways to choose 2
4!(4)(3)(2!) (4)(3)
==6.
2!(42)! (2)(2!) 2
biographies out of 4. Similarly, the number of ways to choose 2 novels out of 6 is
Thus, the total number of ways to choose 2 biographies and 2
6! (6)(5)
==15.
2!4!2
novels is (6)(15) =90.
Adding the number of ways to choose the books for each of the three cases,
you get a total of . The correct answer is Choice B.1 +24+90 =115
For the following question, enter your answer in the box.
12. In a graduating class of 236 students, 142 took algebra and 121 took chemistry.
What is the greatest possible number of students that could have taken both
algebra and chemistry?
students
Explanation
This is the type of problem for which drawing a Venn diagram is usually helpful.
The Venn diagram below is one you could draw to represent the information
given in the question.
Total Students: 236
Chemistry
121
Algebra
142
Note that the algebra and chemistry numbers given do not separate out the
number of students who took both algebra and chemistry, and that this question
asks for the greatest possible number of such students. It is a good idea,
therefore, to redraw the Venn diagram with the number of students who took
This ebook was issued to HUAN-LIN CHANG, order #14803770223. Unlawful distribution of this ebook is prohibited.
Customer name HUAN-LIN CHANG , Order Id: 14803770223
GRE Practice Test 1
404
both algebra and chemistry separated out. The revised Venn diagram looks like
the one below.
Total Students: 236
Chemistry
only
121x
x
Algebra
only
142x
Algebra and
Chemistry
To solve this problem you want the greatest possible value of x. It is clear
from the diagram that xcannot be greater than 142 nor greater than 121,
otherwise or would be negative. Hence, xmust be less than or142x121 x
equal to 121. Since there is no information to exclude the correctx=121,
answer is 121.
m
k
s°
t°
13. In the figure above, if mkand s=t+30, then t=
A30
B60
C75
D80
E105
Explanation
When trying to solve a geometric problem, it is often helpful to add any known
information to the figure. Since corresponding angles have equal measures, you
could place two more angle measures on the figure, as shown below.
m
k
s°
s°t°
t°
This ebook was issued to HUAN-LIN CHANG, order #14803770223. Unlawful distribution of this ebook is prohibited.
Customer name HUAN-LIN CHANG , Order Id: 14803770223
Answers and Explanations
405
Now, from the figure, you can see that Therefore, since it is givens+t=180.
that you can substitute for sinto the equation ands=t+30, t+30s+t=180
get that which can be simplified as follows.(t+30) +t=180,
(t+30) +t=180
2t=150
t=75
The correct answer is Choice C.
14. If then2x=3y=4z=20, 12xyz =
A16,000
B8,000
C4,000
D800
E10
Explanation
One approach you can use to solve this problem is to find the value of all three
variables.
2x=20, or x=10
20
3y=20, or y=3
4z=20, or z=5
So and the correct answer is Choice C.
20
12xyz =12(10) (5) =4,000,
冢冣
3
Another approach you can use to solve this problem is to notice that
Therefore, the correct answer is
(2x)(3y)(4z) (20)(20)(20)
12xyz == =4,000.
22
Choice C.
For the following question, select all the answer choices that apply.
15. The total amount that Mary paid for a book was equal to the price of the book
plus a sales tax that was 4percent of the price of the book. Mary paid for the
book with a $10 bill and received the correct change, which was less than $3.00.
Which of the following statements must be true?
Indicate all such statements.
AThe price of the book was less than $9.50.
BThe price of the book was greater than $6.90.
CThe sales tax was less than $0.45.
Explanation
For this problem you may find it helpful to translate the given information into
an algebraic expression. Since the price of the book is unknown, you can call it x
dollars, and then the total amount that Mary paid is xdollars plus 4%ofx
dollars, or 1.04xdollars. The problem states that Mary received some change
from a $10 bill, so 1.04xdollars must be less than $10. Since the change was less
This ebook was issued to HUAN-LIN CHANG, order #14803770223. Unlawful distribution of this ebook is prohibited.
Customer name HUAN-LIN CHANG , Order Id: 14803770223
GRE Practice Test 1
406
than $3.00, the total amount Mary paid for the book must have been greater
than $7.00. You can express this information algebraically by the inequality
7.00 <1.04x<10.00
Solving the inequality for xby dividing by 1.04, and rounding, you get
6.73<x<9.62
So you see that x, the price of the book, must be between $6.73and $9.62. With
this information, you can quickly examine the first two statements. Choice A is
not necessarily true because the price could be as high as $9.61, and Choice B is
not necessarily true because the price could be as low as $6.74.
To examine Choice C, you could compute the tax for the greatest possible
price, which would be 4% of 9.61, or Since this greatest(0.04)(9.61) =0.38.
possible tax is less than $0.45, Choice Cmust be true.
You can also quickly see that Choice Cmust be true if you note that 4%of
$10.00 would only be $0.40, and since the price must be less than $10.00, the tax
must be less than $0.40. The correct answer consists of Choice C.
16. If is expressed as a terminating decimal, how many nonzero digits
1
11 17
(2 )(5 )
will the decimal have?
AOne
BTw o
CFour
DSix
EEleven
Explanation
To convert the fraction to a decimal, it is helpful to first write the fraction in
powers of 10. Using the rules of exponents, you can write the following.
11
=
11 17 11 11+pl6
(2 )(5 ) (2 )(5 )
1
=
11 11 6
(2 )(5 )(5 )
1
=
11 6
(10 )(5 )
6
1
11
=(10 )
冢冣
5
611
=(0.2) (10 )
16 11
=((2)(10) ) (10 )
6611
=(2 )(10 )(10 )
617
=(2 )(10 )
17
=(64)(10 )
So the decimal has two nonzero digits, 6 and 4. The correct answer is Choice B.
This ebook was issued to HUAN-LIN CHANG, order #14803770223. Unlawful distribution of this ebook is prohibited.
Customer name HUAN-LIN CHANG , Order Id: 14803770223
Answers and Explanations
407
Questions 17 to 20 are based on the following data.
VARIATION IN THE AMOUNT OF CAFFEINE IN COMMON BEVERAGES AND DRUGS*
Amount of Caffeine (milligrams)
Decaffeinated coffee
Source: Food and Drug Administration
Coffee
Drugs
Other
beverages
Percolated coffee
Drip-brewed coffee
Instant coffee
Brewed tea
Instant tea
Cocoa
Caffeinated soft drinks
Weight-loss drugs,
diuretics, and stimulants
Pain relievers
Cold/allergy remedies
025 50 75 100 125 150 175 200
*Based on 5-ounce cups of coffee, tea, and cocoa; 12-ounce cups of soft drinks;
and sin
g
le doses of dru
g
s.
17. The least amount of caffeine in a 5-ounce cup of drip-brewed coffee exceeds
the greatest amount of caffeine in a 5-ounce cup of cocoa by approximately
how many milligrams?
A160
B80
C60
D40
E20
Explanation
Each horizontal bar in the bar graph shows the possible number of milligrams
of caffeine in each of the common beverages and drugs. The least possible
amount of caffeine in a 5-ounce cup of drip-brewed coffee is about 60
milligrams, and the greatest possible amount of caffeine in a 5-ounce cup of
cocoa is about 20 milligrams. So, the difference is approximately or 4060 20,
milligrams. The correct answer is Choice D.
To check your answer, it is useful to try to solve the problem using another
method as well to see if you get the same answer. To solve this problem in
another way, note that the distance between each pair of adjacent vertical grid
lines represents 25 milligrams of caffeine, and the distance between the high end
of the cocoa bar and the low end of the drip-brewed coffee bar is a little more
than the distance between a pair of adjacent grid lines. Therefore, the answer is
between 25 and 50. Among the choices, only Choice Dis between 25 and 50, so
the correct answer is Choice D.
This ebook was issued to HUAN-LIN CHANG, order #14803770223. Unlawful distribution of this ebook is prohibited.
Customer name HUAN-LIN CHANG , Order Id: 14803770223
GRE Practice Test 1
408
For the following question, enter your answer in the box.
18. For how many of the 11 categories of beverages and drugs listed in the graph
can the amount of caffeine in the given serving size be less than 50 milligrams?
categories
Explanation
In the graph, the left edge of each bar tells you what is the least possible amount
of caffeine in the corresponding beverage or drug. A beverage or drug can have
less than 50 milligrams of caffeine if the left edge of its bar lies to the left of the
vertical line corresponding to 50 milligrams of caffeine. From the graph, you see
that there are 9 bars for which this is true. There are only 2 bars that lie entirely
to the right of the 50-milligram linethe bar for drip-brewed coffee and the bar
for weight-loss drugs, diuretics, and stimulants. So there are 9 categories of
beverages and drugs that can have less than 50 milligrams of caffeine in the
given serving size. The correct answer is 9.
19. Approximately what is the minimum amount of caffeine, in milligrams,
consumed per day by a person who daily drinks two 10-ounce mugs of
percolated coffee and one 12-ounce cup of a caffeinated soft drink?
A230
B190
C140
D110
E70
Explanation
According to the bar graph, the minimum amount of caffeine in a 5-ounce cup
of percolated coffee is approximately 40 milligrams. Therefore, the minimum
amount of caffeine in two 10-ounce cups of percolated coffee, which is the same
as the minimum amount of caffeine in four 5-ounce cups, is approximately
or 160 milligrams. The minimum amount of caffeine in a 12-ounce(40)(4),
caffeinated soft drink is approximately 30 milligrams. So, the minimum amount
of caffeine in two 10-ounce mugs of percolated coffee and one 12-ounce
caffeinated soft drink is approximately or 190 milligrams. The correct160 +30,
answer is Choice B.
20. Which of the following shows the four types of coffee listed in order
according to the range of the amounts of caffeine in a 5-ounce cup, from
the least range to the greatest range?
ADecaffeinated, instant, percolated, drip-brewed
BDecaffeinated, instant, drip-brewed, percolated
CInstant, decaffeinated, drip-brewed, percolated
DInstant, drip-brewed, decaffeinated, percolated
EInstant, percolated, drip-brewed, decaffeinated
This ebook was issued to HUAN-LIN CHANG, order #14803770223. Unlawful distribution of this ebook is prohibited.
Customer name HUAN-LIN CHANG , Order Id: 14803770223
Answers and Explanations
409
Explanation
For each of the four types of coffee, the range of the amounts of caffeine is the
greatest possible amount minus the least possible amount. In the graph, this
difference is represented by the length of the corresponding bar, so you can
order the four types of coffee according to the lengths of their corresponding
bars, from shortest to longest. From the graph, you can see that the order is
decaffeinated coffee, instant coffee, drip-brewed coffee, percolated coffee. The
correct answer is Choice B.
300
200
100
0102
S
p
eed (miles
p
er hour)
ENERGY USED PER METER VERSUS SPEED
DURING RUNNING AND WALKING
Energy Used per Meter
(joules per meter)
34 56789
Running
Walking
21. If sis a speed, in miles per hour, at which the energy used per meter during
running is twice the energy used per meter during walking, then, according
to the graph above, sis between
A2.5 and 3.0
B3.0 and 3.5
C3.5 and 4.0
D4.0 and 4.5
E4.5 and 5.0
Explanation
This question is about the speed at which the energy used per meter during
running is twice that used per meter during walking. Graphically, this is the
speed for which the running energy is twice as high as the walking energy.
Looking at the graph, you can see that for speeds greater than or equal to 3.0
miles per hour, the running energy is less than twice the walking energy, so the
desired speed must be less than 3.0. In fact, the desired speed is between 2.0 (the
lowest speed on the graph) and 3.0. Looking at the answer choices, you see that
there is only one answer choice that is between 2.0 and 3.0; namely, Choice A,
which says the desired speed is between 2.5 and 3.0. The correct answer is
Choice A.
This ebook was issued to HUAN-LIN CHANG, order #14803770223. Unlawful distribution of this ebook is prohibited.
Customer name HUAN-LIN CHANG , Order Id: 14803770223
GRE Practice Test 1
410
22. If then
3n
n=2, n=
A
6
2
B
11
2
C
18
2
D
24
2
E
27
2
Explanation
When answering a question in which you are asked to calculate the value of an
expression, it is often helpful to look at the answer choices first to see what form
they are in. In this question the answer choices are all in the form 2 raised to a
power, so you should try to achieve that form. It is given that
3
n=2=8.
Therefore, . The correct answer is Choice D.
n3824
n=(2 ) =2
For the following question, select all the answer choices that apply.
A
C
B
D
The length of AB is 10 3.
23. Which of the following statements individually provide(s) sufficient
additional information to determine the area of triangle ABC above?
Indicate all such statements.
ADBC is an equilateral triangle.
BABDis an isosceles triangle.
CThe length of BC is equal to the length of AD.
DThe length of BC is 10.
EThe length of ADis 10.
Explanation
From the figure you know that ABC is a right triangle with its right angle at
vertex B. You also know that point Dis on the hypotenuse AC. You are given that
the length of AB is However, because the figure is not necessarily drawn to
10 3.
scale, you don’t know the lengths of AD,DC, and BC. In particular, you don’t
know where Dis on AC.
The area of a triangle is Thus, the area of right triangle ABC
1(base)(height).
2
is equal to of the length of AB times the length of BC. You already know that
1
2
the length of AB is Any additional information that would allow you to
10 3.
calculate the length of BC would be sufficient to find the area of triangle ABC.
You need to consider each of the five statements individually, as follows.
StatementA: DBC is an equilateral triangle. This statement implies that angle
DCB is a angle; and therefore, triangle ABC is a triangle. Thus60306090
This ebook was issued to HUAN-LIN CHANG, order #14803770223. Unlawful distribution of this ebook is prohibited.
Customer name HUAN-LIN CHANG , Order Id: 14803770223
Answers and Explanations
411
the length of BC can be determined, and this statement provides sufficient
additional information to determine the area of triangle ABC.
StatementB: ABDis an isosceles triangle. There is more than one way in
which triangle ABDcan be isosceles. Below are two redrawn figures showing
triangle ABDas isosceles. In the figure on the left, the length of ADis equal to
the length of DB; and in the figure on the right, the length of AB is equal to the
length of AD.
A
B
C
D
A
B
C
D
Either of the figures could have been drawn with the length of BC even
longer. So, statement B does not provide sufficient additional information to
determine the area of triangle ABC.
StatementC: The length of BC is equal to the length of AD. You have no way
of finding the length of ADwithout making other assumptions, so statement C
does not provide sufficient additional information to determine the area of
triangle ABC.
StatementD:The length of BC is 10. The length of BC is known, so the area
of triangle ABC can be found. Statement Dprovides sufficient additional
information to determine the area of triangle ABC.
StatementE: The length of ADis 10. The relationship between ADand BC is
not known, so statement E does not provide sufficient additional information to
determine the area of triangle ABC.
StatementsA and Dindividually provide sufficient additional information to
determine the area of triangle ABC. Therefore, the correct answer consists of
Choices A and D.
For the following question, enter your answer in the box.
a,a,a,...,a,...
123n
24. In the sequence above, each term after the first term is equal to the preceding
term plus the constant c. If what is the value ofa+a+a=27, a+a?
13524
a+a=
24
Explanation
Note that answering this question requires information only about the first five
terms of the sequence. So it is a good idea to work with the relationships among
these five terms to see what is happening.
Since you are given that in this sequence each term after is cgreater thana
1
the previous term, you can rewrite the first five terms of the sequence in terms of
and cas follows.a
1
a=a+c
21
a=a+c=a+2c
321
a=a+3c
41
a=a+4c
51
This ebook was issued to HUAN-LIN CHANG, order #14803770223. Unlawful distribution of this ebook is prohibited.
Customer name HUAN-LIN CHANG , Order Id: 14803770223
GRE Practice Test 1
412
From the question, you know that and from the equationsa+a+a=27,
135
above, . So you can concludea+a+a=a+(a+2c)+(a+4c)=3a+6c
1351 1 1 1
that , or3a+6c=27 a+2c=9.
11
To find you can express and in terms of and cand simplifya+a,aa a
24241
as follows.
a+a=(a+c)+(a+3c)
2411
=2a+4c
1
=2(a+2c)
1
But so The correct answer is 18.a+2c=9, a+a=2(9) =18.
124
25. A desert outpost has a water supply that is sufficient to last 21 days for 15
people. At the same average rate of water consumption per person, how
many days would the water supply last for 9 people?
A28.0
B32.5
C35.0
D37.5
E42.0
Explanation
The water supply is enough for 15 people to survive 21 days. Assuming the same
average rate of water consumption per person, 1 person would have enough
water to last for days. Therefore, 9 people would have enough(15)(21) =315
water for of the 315 days, or 35 days. The correct answer is Choice C.
1
9
Another approach to solving this problem is to recognize that the water
supply would last as many days for 9 people as it would for 15 people.
15
9
Therefore, since the water supply would last 21 days for 15 people, it would last
or 35 days for 9 people. The correct answer is Choice C.
15 (21),
冢冣
9
This ebook was issued to HUAN-LIN CHANG, order #14803770223. Unlawful distribution of this ebook is prohibited.
Customer name HUAN-LIN CHANG , Order Id: 14803770223
Answers and Explanations
413
SECTION 6
Quantitative Reasoning
25 Questions with Explanations
AQuantity A is greater.
BQuantity B is greater.
CThe two quantities are equal.
DThe relationship cannot be determined from the information given.
Country Value of 1 United States Dollar
Argentina 0.93 peso
Kenya 32.08 shillings
Quantity A Quantity B
1. The dollar value of 1
Argentine peso according
to the table above
The dollar value of 1
Kenyan shilling according
to the table above
ABCD
Explanation
When you are answering Quantitative Comparison questions, it is a good time-
saving idea to see whether you can determine the relative sizes of the two
quantities being compared without doing any calculations.
In the table accompanying this question, both the value of the Argentine
peso and the value of the Kenyan shilling are compared to the United States
dollar.
Without doing any calculations, you can see from the information given that
1 United States dollar is worth a little less than 1 Argentine peso, so 1 peso is
worth more than 1 United States dollar. On the other hand, 1 United States
dollar is worth 32.08 Kenyan shillings, so 1 Kenyan shilling is worth only a small
fraction of 1 United States dollar. The correct answer is Choice A.
kis a digit in the decimal 1.3k5, and 1.3k5 is less
than 1.33.
Quantity A Quantity B
2. k1ABCD
Explanation
In this question, you are given that kis a digit in the decimal 1.3k5 and that
1.3k5 is less than 1.33. So you can see that Therefore, 1.3k51.30 <1.3k5<1.33.
must equal 1.305 or 1.315 or 1.325, and the digit kmust be 0, 1, or 2. The
correct answer is Choice D.
This ebook was issued to HUAN-LIN CHANG, order #14803770223. Unlawful distribution of this ebook is prohibited.
Customer name HUAN-LIN CHANG , Order Id: 14803770223
GRE Practice Test 1
414
A
CBD
AB is a diameter of the circle above.
Quantity A Quantity B
3. The length of AB The average (arithmetic
mean) of the lengths of AC
and AD
ABCD
Explanation
Recall that in a circle, any diameter is longer than any other chord that is not a
diameter. You are given that AB is a diameter of the circle. It follows that AC and
AD are chords that are not diameters, since there is only one diameter with
endpoint A. Hence, AB is longer than both AC and AD. Note that the average of
two numbers is always less than or equal to the greater of the two numbers.
Therefore, the average of the lengths of AC and AD, which is Quantity B, must be
less than the length of AB, which is Quantity A. The correct answer is Choice A.
st=10
Quantity A Quantity B
4.
2
s10
2
t
ABCD
Explanation
In this question you are asked to compare with Since it is given that
10
2
s.
2
t
it follows that and Dividing both sides of the
2
222
st =10, (st)=
(
10
)
,st =10.
冪冪
equation by you get The correct answer is Choice C.
10
22 2 2
st =10 t,s=.
2
t
You can look at this problem in another way. You can use the fact that
to express Quantity A in terms of t. Since it follows that
st =10 st =10,
冪冪
, and Quantity A is equal to which is the same as Quantity B.
2
10 10
冪冪
10
s=冢冣=,
2
ttt
The correct answer is Choice C.
This ebook was issued to HUAN-LIN CHANG, order #14803770223. Unlawful distribution of this ebook is prohibited.
Customer name HUAN-LIN CHANG , Order Id: 14803770223
Answers and Explanations
415
Three consecutive integers have a sum of 84.
Quantity A Quantity B
5. The least of the three
integers
28 ABCD
Explanation
Two approaches you could use to solve this problem are:
1: Translate from words to algebra.
2: Determine a mathematical relationship between the two quantities.
Approach 1: You can represent the least of the three consecutive integers by
x, and then the three integers would be represented by x, and It isx+1, x+2.
given that the sum of the three integers is so 84.84, x+(x+1) +(x+2) =
You can solve this equation for xas follows.
x+(x+1) +(x+2) =−84
3x+3=−84
3x=−87
x=−29
Since the least of the three integers, is less than the correct answer is29, 28,
Choice B.
Approach 2: You could ask yourself what would happen if the least of the
three consecutive integers was The three consecutive integers would then28.
be and and their sum would be But you were given that28, 27, 26, 81.
the sum of the three consecutive integers is which is less than84, 81.
Therefore, is greater than the least of the three consecutive integers, and the28
correct answer is Choice B.
In the xy-plane, the equation of line kis 3x2y=0.
Quantity A Quantity B
6. The x-intercept of line kThe y-intercept of line kABCD
Explanation
Two approaches you could use to solve this problem are:
1: Reason algebraically.
2: Reason geometrically.
Approach 1: To solve this problem algebraically, note that given the equation
of a line in the xy-plane, the x-intercept of the line is the value of xwhen y
equals 0, and the y-intercept of the line is the value of ywhen xequals 0. The
equation of line kis If then and if then3x2y=0. y=0, x=0; x=0, y=0.
Therefore, both the x-intercept and y-intercept of the line are equal to 0, which
means that the line passes through the origin. The correct answer is Choice C.
Approach 2: To solve this problem geometrically, graph the line with equation
in the xy-plane. Since two points determine a straight line, you can3x2y=0
do this by plotting two points on the line and drawing the line they determine.
The points and lie on the line, and the graph of the line in the xy-(0,0) (2,3)
plane is shown in the following figure.
This ebook was issued to HUAN-LIN CHANG, order #14803770223. Unlawful distribution of this ebook is prohibited.
Customer name HUAN-LIN CHANG , Order Id: 14803770223
GRE Practice Test 1
416
x
(2, 3)
(0, 0)
y
As you can see, the line passes through the origin, and so it crosses both the
x-axis and the y-axis at The correct answer is Choice C.(0,0).
nis a positive integer that is divisible by 6.
Quantity A Quantity B
7. The remainder when nis
divided by 12
The remainder when nis
divided by 18
ABCD
Explanation
One way to compare the two quantities is to plug in a few values of n.Ifyou
plug in you find that both the remainder when nis divided by 12 and then=36,
remainder when nis divided by 18 are equal to 0, so Quantity A is equal to
Quantity B. However, if you plug in you find that the remainder when nn =18,
is divided by 12 is 6 and the remainder when nis divided by 18 is 0, so Quantity
B is greater than Quantity A. Therefore, the correct answer is Choice D.
Another way to compare the two quantities is to find all of the possible
values of Quantity A and Quantity B. The positive integers that are divisible by 6
are 6, 12, 18, 24, 30, 36, etc. When dividing each of these integers by 12, you get
a remainder of either 0 or 6, so Quantity A is either 0 or 6. When dividing each
of these integers by 18, you get a remainder of either 0 or 6 or 12, so Quantity B
is either 0 or 6 or 12. Note that when the value of Quantity B is 12, the value of
Quantity A, 0 or 6, is less than the value of Quantity B; but when the value of
Quantity B is 0, the value of Quantity A is greater than or equal to the value of
Quantity B. Thus, the correct answer is Choice D.
1x1
=
x1x
Quantity A Quantity B
8. x1
2
ABCD
Explanation
One approach you could use to solve this problem is to solve the equation
for x. Since fractions are defined only when the denominator is not
1x1
=
x1x
This ebook was issued to HUAN-LIN CHANG, order #14803770223. Unlawful distribution of this ebook is prohibited.
Customer name HUAN-LIN CHANG , Order Id: 14803770223
Answers and Explanations
417
equal to 0, the denominators of both of the fractions in the equation are
nonzero. Therefore, andx0x1.
To solve the equation for x, begin by multiplying both sides of the equation
by the common denominator to get Then proceedx(x+1) x(1 x)=(x1)(1).
as follows.
x(1 x)=(x1)(1)
2
xx=x1
2
x=1
Since and it follows that
2
x=1x1, x=−1.
Quantity A is equal to and Quantity B is equal to Therefore, Quantity
1
1.
2
B is greater, and the correct answer is Choice B.
Another approach is to notice that for all values of the value of is
1x
x1, x1
equal to You can try plugging in a few numbers for xto see that this is true.1.
For example, if you plug in you get 716
x=7, ==1.
176
You can also show that for all values of the value of is equal to
1x
x1, x1
algebraically by rewriting as Thus, 1x(x1)
11x(x1). ==1.
x1(x1)
Because the left side of the equation is equal to it follows that
1x1
=−1,
x1x
and so Therefore, Quantity A is equal to which is less than
1
1=,x=−1. 1,
x
Quantity B, and the correct answer is Choice B.
1
,
2
In a set of 24 positive integers, 12 of the integers are
less than 50. The rest are greater than 50.
Quantity A Quantity B
9. The median of the 24
integers
50 ABCD
Explanation
In general, the median of a set of npositive integers, where nis even, is
obtained by ordering the integers from least to greatest and then calculating the
average (arithmetic mean) of the two middle integers. For this set of 24 integers,
you do not know the values of the two middle integers; you know only that half
of the integers are less than 50 and the other half are greater than 50. If the two
middle integers in the list are 49 and 51, the median is 50; and if the two middle
integers are 49 and 53, the median is 51. Thus the relationship cannot be
determined from the information given, and the correct answer is Choice D.
This ebook was issued to HUAN-LIN CHANG, order #14803770223. Unlawful distribution of this ebook is prohibited.
Customer name HUAN-LIN CHANG , Order Id: 14803770223
GRE Practice Test 1
418
10. The fabric needed to make 3 curtains sells for $8.00 per yard and can be
purchased only by the full yard. If the length of fabric required for each
curtain is 1.6 yards and all of the fabric is purchased as a single length,
what is the total cost of the fabric that needs to be purchased for the 3
curtains?
A$40.00
B$38.40
C$24.00
D$16.00
E$12.80
Explanation
Since 1.6 yards of fabric are required for each curtain, it follows that or(3)(1.6),
4.8, yards of fabric are required to make the 3 curtains. The fabric can be
purchased only by the full yard, so 5 yards of fabric would need to be purchased.
Since the fabric sells for $8.00 per yard, the total cost of the fabric is $40.00. The
correct answer is Choice A.
For the following question, select all the answer choices that apply.
11. In the xy-plane, line kis a line that does not pass through the origin.
Which of the following statements individually provide(s) sufficient additional
information to determine whether the slope of line kis negative?
Indicate all such statements.
AThe x-intercept of line kis twice the y-intercept of line k.
BThe product of the x-intercept and the y-intercept of line kis positive.
CLine kpasses through the points and where(a,b)(r,s),
(ar)(bs)<0.
Explanation
For questions involving a coordinate system, it is often helpful to draw a figure
to visualize the problem situation. If you draw some lines with negative slopes in
the xy-plane, such as those in the figure below, you see that for each line that
does not pass through the origin, the x- and y-intercepts are either both positive
or both negative. Conversely, you can see that if the x- and y-intercepts of a line
have the same sign then the slope of the line is negative.
This ebook was issued to HUAN-LIN CHANG, order #14803770223. Unlawful distribution of this ebook is prohibited.
Customer name HUAN-LIN CHANG , Order Id: 14803770223
Answers and Explanations
419
Ox
y
rise
run
(a, b)
(r, s)
You can use this fact to examine the information given in the first two
statements. Remember that you need to evaluate each statement by itself.
Choice A states that the x-intercept is twice the y-intercept, so you can
conclude that both intercepts have the same sign, and thus the slope of line kis
negative. So the information in Choice A is sufficient to determine that the slope
of line kis negative.
Choice B states that the product of the x-intercept and the y-intercept is
positive. You know that the product of two numbers is positive if both factors
have the same sign. So this information is also sufficient to determine that the
slope of line kis negative.
To evaluate Choice C, it is helpful to recall the definition of the slope of a
line passing through two given points. You may remember it as “rise over run.”
If the two points are and then the slope is bs
(a,b)(r,s), .
ar
Choice C states that the product of the quantities and is(ar)(bs)
negative. Note that these are the denominator and the numerator, respectively, of
the slope of line k. So you can conclude that and have
bs,(ar)(bs)
ar
opposite signs and the slope of line kis negative. The information in Choice C is
sufficient to determine that the slope of line kis negative.
So each of the three statements individually provides sufficient information
to determine whether the slope of line kis negative. The correct answer consists
of Choices A, B, and C.
This ebook was issued to HUAN-LIN CHANG, order #14803770223. Unlawful distribution of this ebook is prohibited.
Customer name HUAN-LIN CHANG , Order Id: 14803770223
GRE Practice Test 1
420
Distance from Centerville
(miles)
Freight train 10t+115
Passenger train 20t+150
12. The expressions in the table above give the distance from Centerville to
each of two trains thours after 12:00 noon. At what time after 12:00 noon
will the trains be equidistant from Centerville?
A1:30
B3:30
C5:10
D8:50
E11:30
Explanation
The distance between the freight train and Centerville at thours past noon is
The distance between the passenger train and Centerville at thours10t+115.
past noon is To find out at what time the distances will be the same20t+150.
you need to equate the two expressions and solve for tas follows.
10t+115 =−20t+150
10t+115 =150
10t=35
t=3.5
Therefore, the two trains will be the same distance from Centerville at 3.5 hours
past noon, or at 3:30. The correct answer is Choice B.
13. The company at which Mark is employed has 80 employees, each of whom
has a different salary. Mark’s salary of $43,700 is the second-highest salary
in the first quartile of the 80 salaries. If the company were to hire 8 new
employees at salaries that are less than the lowest of the 80 salaries, what
would Mark’s salary be with respect to the quartiles of the 88 salaries at the
company, assuming no other changes in the salaries?
AThe fourth-highest salary in the first quartile
BThe highest salary in the first quartile
CThe second-lowest salary in the second quartile
DThe third-lowest salary in the second quartile
EThe fifth-lowest salary in the second quartile
Explanation
In this question you are told that Mark’s salary is the second-highest in the first
quartile. From this you can conclude that the word quartile refers to one of the
four groups that are created by listing the data in increasing order and then
dividing the data into four groups of equal size. When the salaries of the 80
employees are listed in order, the 20 lowest salaries (that is, the salaries in the
first quartile) are the first 20 salaries in the list. Since Mark’s salary is the
This ebook was issued to HUAN-LIN CHANG, order #14803770223. Unlawful distribution of this ebook is prohibited.
Customer name HUAN-LIN CHANG , Order Id: 14803770223
Answers and Explanations
421
second-highest in the first quartile, 18 salaries in that quartile are lower than his,
and one salary in that quartile is higher than his. After the salaries of the 8 new
employees are added, there are 26 salaries that are lower than Mark’s. The lowest
22 of those would be in the first quartile of the 88 salaries, and the remaining 4
(salaries 23 to 26) would be in the second quartile, followed by Mark’s salary.
This puts Mark at the fifth-lowest salary in the second quartile. The correct
answer is Choice E.
Another way to approach this problem is to think of all 80 salaries numbered
in order from least to greatest, the lowest salary at the number 1 position and
the greatest salary at the number 80 position. There are 20 positions in each
quartile, and Mark’s salary is at position 19. The diagram below shows the salary
positions and the quartile into which each position falls. Note that position 19,
where Mark’s salary appears, is second-highest in the first quartile.
First quartile Second quartile Third quartile Fourth quartile
1
2
3
18
19
20
. . .
21
22
23
38
39
40
. . .
41
42
43
58
59
60
. . .
61
62
63
78
79
80
. . .
Mark’s
salary
To see what Mark’s position is with respect to the quartiles of the 88 salaries,
you need add the 8 new salaries to the list, renumber the list from 1 to 88, and
put 22 salaries in each quartile. Because the 8 new salaries are less than the
original 80 salaries, they must be listed in positions 1 through 8, and all salaries
in the original list must move up by 8 positions in the renumbered list. In
particular, Mark’s salary moves from position 19 to position 27. The diagram
below shows the renumbered list. Note that Mark’s salary is in position 27, the
fifth position in the second quartile.
First quartile Second quartile Third quartile Fourth quartile
1
2
8
9
20
21
22
. . .. . .
23
24
25
26
27
42
43
44
. . .
45
46
47
48
49
64
65
66
. . .
67
68
69
70
71
86
87
88
. . .
Salary at
position 1 of
original list
Mark’s
salary
New
salaries
Since Mark’s salary is in the fifth position in the second quartile and the
salaries are listed in order from least to greatest, Mark’s salary would be the
fifth-lowest in the second quartile. The correct answer is Choice E.
This ebook was issued to HUAN-LIN CHANG, order #14803770223. Unlawful distribution of this ebook is prohibited.
Customer name HUAN-LIN CHANG , Order Id: 14803770223
GRE Practice Test 1
422
For the following question, enter your answer in the box.
14. In the xy-plane, the point with coordinates is the center of circle C.(6, 7)
The point with coordinates lies inside C, and the point with(6, 5)
coordinates lies outside C.Ifmis the radius of Cand mis an(8, 7)
integer, what is the value of m?
m=
Explanation
A strategy that is often helpful in working with geometry problems is drawing a
figure that represents the given information as accurately as possible.
In this question you are given that the point with coordinates is the(6, 7)
center of circle C, the point with coordinates lies inside circle C, and the(6, 5)
point with coordinates lies outside circle C, so you could draw the(8, 7)
following figure.
x
O
(–6, 5)
(–6, –7) (8, –7)
y
From the figure, you can see that the distance between and(6, 7) (6, 5)
is or 12, and the radius of Cmust be greater than 12. You can also see7 +5,
that the distance between and is or 14, and the radius of(6, 7) (8, 7) 6 +8,
Cmust be less than 14. Therefore, since the radius is an integer greater than 12
and less than 14, it must be 13. The correct answer is 13.
15. If is an even integer, which of the following must be true?
m
19
Amis a negative number.
Bmis a positive number.
Cmis a prime number.
Dmis an odd integer.
Emis an even integer.
This ebook was issued to HUAN-LIN CHANG, order #14803770223. Unlawful distribution of this ebook is prohibited.
Customer name HUAN-LIN CHANG , Order Id: 14803770223
Answers and Explanations
423
Explanation
An even integer is a multiple of 2. If is an even integer, it must equal 2
m
19
times some integer k. This means that or
m
−=2k,m=−19(2k)=2(19k),
19
which is a multiple of 2. Thus mmust be an even integer, and the correct answer
is Choice E. You can see that none of the other choices can be the correct
answer by evaluating them as follows.
Amdoes not have to be a negative number for to be even. For
m
19
example, if then which is an even number.
m
m=38, −=2,
19
Bmdoes not have to be a positive number for to be even. For
m
19
example, if then which is an even number.
m
m=−38, −=2,
19
CThe number used in (A), shows that mdoes not have to be am=38,
prime number. In fact, because mis the product of at least two prime
numbers (2 and 19), mcannot be a prime number.
DSince mmust be an even integer, mcannot be an odd integer.
For the following question, select all the answer choices that apply.
16. The integer vis greater than 1. If vis the square of an integer, which of the
following numbers must also be the square of an integer?
Indicate all such numbers.
A81v
B25v+10 v+1
C
2
4v+4v+1
Explanation
If vis the square of an integer, then is an integer. You can use this fact,v
together with the fact that the product and the sum of integers are also integers,
to examine the first two choices.
Choice A: The square root of 81vis which is an integer. So 81vis the
9v,
square of an integer.
Choice B: and is an integer. So
2
25v+10 v+1=(5 v+1) 5 v+1
冪冪 冪
is the square of an integer.25v+10 v+1
Choice C: Since there is no obvious way to factor the given expression, you
may suspect that it is not the square of an integer. To show that a given
statement is not true, it is sufficient to find one counterexample. In this case,
you need to find one value of vsuch that vis the square of an integer
but is not the square of an integer. If then
2
4v+4v+1v=4,
which is not the square of an integer. This proves
2
4v+4v+1=64 +8+1=73,
that does not have to be the square of an integer.
2
4v+4v+1
The correct answer consists of Choices A and B.
This ebook was issued to HUAN-LIN CHANG, order #14803770223. Unlawful distribution of this ebook is prohibited.
Customer name HUAN-LIN CHANG , Order Id: 14803770223
GRE Practice Test 1
424
Questions 17 to 20 are based on the following data.
110
100
90
80
70
60
50
40
30
20
10
0
Distance (feet)
Speed (miles per hour)
0 102030405060708090100
Speed (miles per hour)
0 102030405060708090100
Distance Traveled
During Reaction Time*
550
500
450
400
350
300
250
200
150
100
50
0
Distance (feet)
Distance Traveled After Brakes
Have Been Applied
922 49
88
137
198
550
445
352
269
DISTANCE TRAVELED BY A CAR ACCORDING TO THE CAR’S SPEED
WHEN THE DRIVER IS SIGNALED TO STOP
*Reaction time is the time period that begins when the driver is signaled to stop
and ends when the driver applies the brakes.
Note: Total stopping distance is the sum of the distance traveled during reaction time
and the distance traveled after brakes have been applied.
17. The speed, in miles per hour, at which the car travels a distance of 52 feet
during reaction time is closest to which of the following?
A43
B47
C51
D55
E59
Explanation
The data accompanying questions 17 to 20 consists of two graphs. It is a good
idea to look at the graphs before you try to answer the questions, so you can
become familiar with the information contained in the graphs. Then, as you read
each question, you should think about which of the graphs contains the
information you need to solve the problem. It could be that all the information
you need to solve the problem is contained in one of the graphs, or it could be
that you need to get information from both of the graphs.
The graph on the left shows the relationship between the speed of the
automobile and the distance it traveled during the reaction time. Therefore, the
answer to this question is found using this graph by reading the speed, in miles
per hour, corresponding to a distance of 52 feet. A distance of 52 feet is a little
above the distance of 50 feet on the vertical axis of the graph. On the graph, the
This ebook was issued to HUAN-LIN CHANG, order #14803770223. Unlawful distribution of this ebook is prohibited.
Customer name HUAN-LIN CHANG , Order Id: 14803770223
Answers and Explanations
425
speed corresponding to a distance of 52 feet is a little less than 50 miles per
hour. The correct answer is Choice B.
18. Approximately what is the total stopping distance, in feet, if the car is
traveling at a speed of 40 miles per hour when the driver is signaled to
stop?
A130
B110
C90
D70
E40
Explanation
Since the total stopping distance is the sum of the distance traveled during
reaction time and the distance traveled after the brakes have been applied, you
need information from both graphs to answer this question. At a speed of 40
miles per hour, the distance traveled during reaction time is a little less than 45
feet, and the distance traveled after the brakes have been applied is 88 feet.
Since the correct answer is Choice A.45 +88 =133,
19. Of the following, which is the greatest speed, in miles per hour, at which
the car can travel and stop with a total stopping distance of less than 200
feet?
A50
B55
C60
D65
E70
Explanation
Since the total stopping distance is the sum of the distance traveled during
reaction time and the distance traveled after the brakes have been applied, you
need information from both graphs to answer this question. A good strategy for
solving this problem is to calculate the total stopping distance for the speeds
given in the options. For a speed of 50 miles per hour, the distance traveled
during reaction time is about 55 feet, and the distance traveled after the brakes
have been applied is 137 feet; therefore, the total stopping distance is about
or 192 feet. For a speed of 55 miles per hour, the distance traveled55 +137,
during reaction time is about 60 feet, and the distance traveled after the brakes
have been applied is about 170 feet; therefore, the total stopping distance is
about or 230 feet. Since the speeds in the remaining choices are60 +170,
greater than 55 miles per hour and both types of stopping distances increase as
the speed increases, it follows that the total stopping distances for all the
remaining choices are greater than 200 feet. The correct answer is Choice A.
This ebook was issued to HUAN-LIN CHANG, order #14803770223. Unlawful distribution of this ebook is prohibited.
Customer name HUAN-LIN CHANG , Order Id: 14803770223
GRE Practice Test 1
426
20. The total stopping distance for the car traveling at 60 miles per hour is
approximately what percent greater than the total stopping distance for the
car traveling at 50 miles per hour?
A22%
B30%
C38%
D45%
E52%
Explanation
To solve this problem you need to find the total stopping distance at 50 miles per
hour and at 60 miles per hour, find their difference, and then express the
difference as a percent of the shorter total stopping distance. You need to use
both graphs to find the total stopping distances. At 50 miles per hour, the total
stopping distance is approximately feet; and at 60 miles per hour
55 +137 =192
it is approximately feet. The difference of 72 feet as a percent of
66 +198 =264
192 feet is or approximately 38%. The correct answer is Choice C.
72 =0.375,
192
21. What is the least positive integer that is not a factor of and is not a
25!
prime number?
A26
B28
C36
D56
E58
Explanation
Note that is equal to the product of all positive integers from 1 to 25,
25!
inclusive. Thus, every positive integer less than or equal to 25 is a factor of 25!.
Also, any integer greater than 25 that can be expressed as the product of
different positive integers less than 25 is a factor of In view of this, it’s
25!.
reasonable to consider the next few integers greater than 25, including answer
choices A and B.
Choice A, 26, is equal to Both 2 and 13 are factors of so 26 is
(2)(13). 25!,
also a factor of The same is true for 27, or and for Choice B, 28, or
25!. (3)(9),
However, the next integer, 29, is a prime number greater than 25, and as
(4)(7).
such, it has no positive factors (other than 1) that are less than or equal to 25.
Therefore, 29 is the least positive integer that is not a factor of However, the25!.
question asks for an integer that is not a prime number, so 29 is not the answer.
At this point, you could consider 30, 31, 32, etc., but it is quicker to look at
the rest of the choices. Choice C, 36, is equal to Both 4 and 9 are factors(4)(9).
of so 36 is also a factor of Choice D, 56, is equal to Both 4 and25!, 25!. (4)(14).
14 are factors of so 56 is also a factor of Choice E, 58, is equal to25!, 25!.
Although 2 is a factor of the prime number 29, as noted earlier, is(2)(29). 25!,
not a factor of and therefore 58 is not a factor of The correct answer25!, 25!.
must be Choice E.
This ebook was issued to HUAN-LIN CHANG, order #14803770223. Unlawful distribution of this ebook is prohibited.
Customer name HUAN-LIN CHANG , Order Id: 14803770223
Answers and Explanations
427
The explanation above uses a process of elimination to arrive at Choice E,
which is sometimes the most efficient way to find the correct answer. However,
one can also show directly that the correct answer is 58. For if a positive integer
nis not a factor of then one of the following must be true:25!,
(i) nis a prime number greater than 25, like 29 or 31, or a multiple of
such a prime number, like 58 or 62;
(ii) nis so great a multiple of some prime number less than 25, that it
must be greater than 58.
To see that (i) or (ii) is true, recall that every integer greater than 1 has a
unique prime factorization, and consider the prime factorization of The25!.
prime factors of are 2, 3, 5, 7, 11, 13, 17, 19, and 23, some of which occur25!
more than once in the product For example, there are 8 positive multiples of25!.
3 less than 25, namely 3, 6, 9, 12, 15, 18, 21, and 24. The prime number 3 occurs
once in each of these multiples, except for 9 and 18, in which it occurs twice.
Thus, the factor 3 occurs 10 times in the prime factorization of The same25!.
reasoning can be used to find the number of times that each of the prime factors
occur, yielding the prime factorization 25! =
22 10 6 3 2
(2 )(3 )(5 )(7 )(11 )(13)(17)(19)(23).
Any integer whose prime factorization is a combination of one or more of the
factors in the prime factorization of perhaps with lesser exponents, is a25!,
factor of Equivalently, if the positive integer nis not a factor of then,25!. 25!,
restating (i) and (ii) above, the prime factorization of nmust
(i) include a prime number greater than 25; or
(ii) have a greater exponent for one of the prime numbers in the prime
factorization of 25!.
For (ii), the least possibilities are , , , , , , , , and .
23 11 7 4 3 2 2 2 2
2 3 5711131719 23
Clearly, all of these are greater than 58. The least possibility for (i) that is not a
prime number is 58, and the least possibility for (ii) is greater than 58, so 58 is
the correct answer.
22. If which of the following is true about the reciprocals of a0<a<1<b,
and b?
A11
1<<
ab
B11
<1<
ab
C11
<<1
ab
D11
<1<
ba
E11
<<1
ba
Explanation
To answer this question, you must first look at the answer choices. Note that all
of the choices are possible orderings of the quantities and 1 from least to
11
,,
ab
greatest. So to answer the question, you must put the three quantities in order
This ebook was issued to HUAN-LIN CHANG, order #14803770223. Unlawful distribution of this ebook is prohibited.
Customer name HUAN-LIN CHANG , Order Id: 14803770223
GRE Practice Test 1
428
from least to greatest. The inequality tells you that and0 <a<1<b0<a<1
that Since ais a value between 0 and 1, the value of must be greater
1
b>1. a
than 1. Since bis greater than 1, the value of must be less than 1. So you
1
b
know that and that or combined in one expression, and
11 11
>1<1, <1<,
ab ba
the correct answer is Choice D.
OP
23. In the figure above, Oand Pare the centers of the two circles. If each circle
has radius r, what is the area of the shaded region?
A2
2
r
2
B3
2
r
2
C
2
2r
D
2
3r
E
2
23r
Explanation
If a geometric problem contains a figure, it can be helpful to draw additional
lines and add information given in the text of the problem to the figure. For
circles, the helpful additional lines are often radii or diameters. In this case,
drawing radius OP will divide the shaded region into two triangles, as shown in
the figure below.
OP
The two circles have the same radius, r. Therefore, in each of the triangles,
all three sides have length r, and each of the triangles is equilateral. If you
remember from geometry that the height of an equilateral triangle with sides of
length ris you could use that fact in solving the problem. However,
3
r,
2
This ebook was issued to HUAN-LIN CHANG, order #14803770223. Unlawful distribution of this ebook is prohibited.
Customer name HUAN-LIN CHANG , Order Id: 14803770223
Answers and Explanations
429
if you do not remember what the height is, you can use the following figure to
help you find the height.
rr
r
2
h
r
2
Using the Pythagorean theorem, you get
2
r
22
+h=r
冢冣
2
2
r
22
+h=r
4
3
22
h=r
4
3
h=r
2
So the area of the equilateral triangle is .
33
冪冪
11
2
(base)(height) =(r)r=r
2224
冢冣
Since the shaded region consists of 2 equilateral triangles with sides of length r,
the area of the shaded region is and the correct answer is
33
冪冪
22
(2) r=r,
42
冢冣
Choice B.
For the following question, enter your answer in the boxes.
24. Of the 20 lightbulbs in a box, 2 are defective. An inspector will select 2
lightbulbs simultaneously and at random from the box. What is the probability
that neither of the lightbulbs selected will be defective?
Give your answer as a fraction.
Explanation
The desired probability corresponds to the fraction
the number of ways that 2 lightbulbs, both of which are not defective, can be chosen
the number of ways that 2 lightbulbs can be chosen
In order to calculate the desired probability, you need to calculate the values
of the numerator and the denominator of this fraction.
This ebook was issued to HUAN-LIN CHANG, order #14803770223. Unlawful distribution of this ebook is prohibited.
Customer name HUAN-LIN CHANG , Order Id: 14803770223
GRE Practice Test 1
430
In the box there are 20 lightbulbs, 18 of which are not defective. The
numerator of the fraction is the number of ways that 2 lightbulbs can be chosen
from the 18 that are not defective, also known as the number of combinations of
18 objects taken 2 at a time.
If you remember the combinations formula, you know that the number of
combinations is (which is denoted symbolically as or ).
18
18! C
18 2
冢冣
2
2!(182)!
Simplifying, you get
18! (18)(17)(16!) (18)(17)
===153
2!16! (2)(16!) 2
Similarly, the denominator of the fraction is the number of ways
that 2 lightbulbs can be chosen from the 20 in the box, which is
Therefore, the probability that
20 20! (20)(19)(18!) (20)(19)
== = =190.
冢冣
22!18! (2)(18!) 2
neither of the lightbulbs selected will be defective is The correct answer
153 .
190
is (or any equivalent fraction).
153
190
Another approach is to look at the selection of the two lightbulbs separately.
The problem states that lightbulbs are selected simultaneously. However, the
timing of the selection only ensures that the same lightbulb is not chosen twice.
This is equivalent to choosing one lightbulb first and then choosing a second
lightbulb without replacing the first. The probability that the first lightbulb
selected will not be defective is If the first lightbulb selected is not defective,
18 .
20
there will be 19 lightbulbs left to choose from, 17 of which are not defective.
Thus, the probability that the second lightbulb selected will not be defective is
The probability that both lightbulbs selected will not be defective is the
17 .
19
product of these two probabilities. Thus, the desired probability is
The correct answer is (or any equivalent fraction).
18 17 153 153
=.
冢冣冢冣
20 19 190 190
25. What is the perimeter, in meters, of a rectangular playground 24 meters
wide that has the same area as a rectangular playground 64 meters long
and 48 meters wide?
A112
B152
C224
D256
E304
Explanation
The area of the rectangular playground that is 64 meters long and 48 meters
wide is square meters. The second playground, which has the(64)(48) =3,072
same area, is 24 meters wide and meters long. Therefore, the
3,072 =128
24
perimeter of the second playground is meters. The(2)(24) +(2)(128) =304
correct answer is Choice E.
This ebook was issued to HUAN-LIN CHANG, order #14803770223. Unlawful distribution of this ebook is prohibited.
Customer name HUAN-LIN CHANG , Order Id: 14803770223
Taking Practice Test 2
When you have taken Practice Test 1 and used the results to help you in your
preparations, you can use Practice Test 2 to reassess how ready you are to
take the actual GRE revised General Test. Practice Test 2 begins on the
following pages. As with Practice Test 1, the total time that you should allow for this
practice test is 3 hours and 30 minutes. The time allotted for each section appears at
the beginning of that section.
Try to take Practice Test 2 under actual test conditions. Find a quiet place to work, and
set aside enough time to complete the test without being disturbed. Work on only one sec-
tion at a time. Use your watch or a timer to keep track of the time limit for each section.
For the Verbal Reasoning and Quantitative Reasoning portions of this practice test,
mark your answers directly in this book. However, when you take the real GRE revised
General Test on computer, you will click on your answer choices. For the Analytical
Writing sections of this test, how you respond will depend on how you plan to take the
actual test. If you plan to take the actual test on computer, you should type your
responses. If you plan to take the paper-based test, you should handwrite your
responses on paper. You will need a supply of lined paper for this task.
It is important to note that this practice test is structured like the paper-based test. As
a result, the number of questions and time allowed per section are not the same as in the
computer-based test. The results of this practice test will give you an accurate guide to
your readiness to take the test, but for a more realistic experience of the computer-
based test, you should use the practice tests included in the POWERPREP II®software.
Following this practice test you will find an answer key. Check your answers
against the key, then follow the instructions for calculating your Verbal Reasoning and
Quantitative Reasoning scores and evaluating your Analytical Writing performance.
You will also find explanations for each test question. Review the explanations, paying
particular attention to the ones for the questions that you answered incorrectly.
Once you have worked your way through Practice Test 2, you will have a good idea
of how ready you are to take the actual test. You should use the results to check
whether there are any areas in which you still need improvement and whether you
need to improve your test-taking speed and time management skills. For additional
test-preparation materials and suggestions, visit the GRE website at www.ets.org/gre/
prepare.
gTake the second full-length authentic
GRE
®
revised General Test under actual test time limits.
gCheck your answers and read explanations for
every question.
gUse your results to identify your strengths and
weaknesses.
Your goals
for this
chapter
431
9
GRE
®
Practice Test 2
This ebook was issued to HUAN-LIN CHANG, order #14803770223. Unlawful distribution of this ebook is prohibited.
Customer name HUAN-LIN CHANG , Order Id: 14803770223
The Graduate Record
Examinations®
SECTION 1
Analytical Writing
ANALYZE AN ISSUE
30 minutes
You will be given a brief quotation that states or implies an issue of general interest and specific
instructions on how to respond to that issue. You will then have 30 minutes to plan and compose
a response according to the specific instructions. A response to any other issue will receive a
score of zero.
Make sure that you respond according to the specific instructions and support your position on the
issue with reasons and examples drawn from such areas as your reading, experience, observations,
and/or academic studies.
Trained GRE readers will evaluate your response for its overall quality, based on how well you:
bRespond to the specific task instructions
bConsider the complexities of the issue
bOrganize, develop, and express your ideas
bSupport your ideas with relevant reasons and/or examples
bControl the elements of standard written English
Before you begin writing, you may want to think for a few minutes about the issue and the specific
task instructions and then plan your response. Be sure to develop your position fully and organize
it coherently, but leave time to reread what you have written and make any revisions you think are
necessary.
432
GRE Practice Test 2
This ebook was issued to HUAN-LIN CHANG, order #14803770223. Unlawful distribution of this ebook is prohibited.
Customer name HUAN-LIN CHANG , Order Id: 14803770223
433
Section 1. Analytical Writing
Issue Topic
Some people believe that corporations have a responsibility to promote the well-
being of the societies and environments in which they operate. Others believe that
the only responsibility of corporations, provided they operate within the law, is to
make as much money as possible.
Write a response in which you discuss which view more closely aligns with your
own position and explain your reasoning for the position you take. In developing
and supporting your position, you should address both of the views presented.
This ebook was issued to HUAN-LIN CHANG, order #14803770223. Unlawful distribution of this ebook is prohibited.
Customer name HUAN-LIN CHANG , Order Id: 14803770223
The Graduate Record
Examinations®
SECTION 2
Analytical Writing
ANALYZE AN ARGUMENT
30 minutes
You will be given a short passage that presents an argument and specific instructions on how to
respond to that passage. You will have 30 minutes to plan and compose a response in which you
evaluate the passage according to the specific instructions. A response to any other argument will
receive a score of zero.
Note that you are NOT being asked to present your own views on the subject. Make sure that
you respond according to the specific instructions and support your evaluation with relevant
reasons and/or examples.
Trained GRE readers will evaluate your response for its overall quality, based on how well you:
bRespond to the specific task instructions
bIdentify and analyze features of the argument relevant to the assigned task
bOrganize, develop, and express your ideas
bSupport your analysis with relevant reasons and/or examples
bControl the elements of standard written English
Before you begin writing, you may want to think for a few minutes about the argument passage
and the specific task instructions and then plan your response. Be sure to develop your response
fully and organize it coherently, but leave time to reread what you have written and make any
revisions you think are necessary.
434
GRE Practice Test 2
This ebook was issued to HUAN-LIN CHANG, order #14803770223. Unlawful distribution of this ebook is prohibited.
Customer name HUAN-LIN CHANG , Order Id: 14803770223
435
Section 2. Analytical Writing
Argument Topic
The following appeared in a memorandum from the owner of Movies Galore, a chain of
video rental stores.
“In order to reverse the recent decline in our profits, we must reduce operating expenses at
Movies Galore’s ten video rental stores. Since we are famous for our special bargains, raising
our rental prices is not a viable way to improve profits. Last month our store in downtown
Marston significantly decreased its operating expenses by closing at 6:00 P.M. rather than
9:00 P.M. and by reducing its stock by eliminating all movies released more than five years
ago. Therefore, in order to increase profits without jeopardizing our reputation for offering
great movies at low prices, we recommend implementing similar changes in our other nine
Movies Galore stores.”
Write a response in which you discuss what questions would need to be answered in order to
decide whether the recommendation is likely to have the predicted result. Be sure to explain
how the answers to these questions would help to evaluate the recommendation.
This ebook was issued to HUAN-LIN CHANG, order #14803770223. Unlawful distribution of this ebook is prohibited.
Customer name HUAN-LIN CHANG , Order Id: 14803770223
436
GRE Practice Test 2
NO TEST MATERIAL ON THIS PAGE
This ebook was issued to HUAN-LIN CHANG, order #14803770223. Unlawful distribution of this ebook is prohibited.
Customer name HUAN-LIN CHANG , Order Id: 14803770223
437
Section 3. Verbal Reasoning
SECTION 3
Verbal Reasoning
Time — 35 minutes
25 Questions
For questions 1 to 8, select one entry for each blank from the corresponding column
of choices. Fill all blanks in the way that best completes the text.
1. Although plant and animal species that become established in ecosystems where
they did not originate are sometimes referred to by the alarming term “invasive
species,” many such species are ________ in their new environments.
A
innocuous
B
conspicuous
C
robust
D
menacing
E
distinctive
2. Far from being ________ the corporate world because of cutbacks, serious
researchers are playing a growing role in innovation at many firms.
A
lured to
B
enchanted with
C
banished from
D
protected by
E
immured in
3. The brief survey, published under the title The Work of Nature: How the Diversity
of Life Sustains Us, is surprisingly (i)________. Indeed it makes several longer
treatments of the effects of lost biodiversity seem (ii)________.
Blank (i) Blank (ii)
A
distorted
D
redundant
B
objective
E
pithy
C
comprehensive
F
premature
4. The government has no choice but to (i)________ the incessant demands for land
reform, and yet any governmental action that initiated land reform without
requisite attention to agrarian reform would (ii)________ the overall goal of
economic modernization.
Blank (i) Blank (ii)
A
anticipate
D
delineate
B
heed
E
condone
C
silence
F
compromise
GO ON TO NEXT PAGE g
This ebook was issued to HUAN-LIN CHANG, order #14803770223. Unlawful distribution of this ebook is prohibited.
Customer name HUAN-LIN CHANG , Order Id: 14803770223
5. Certain music lovers yearn for (i)________, but when it is achieved, there is
something missing; perhaps they feel uncomfortable in a world where nothing
discernible is (ii)________.
Blank (i) Blank (ii)
A
novelty
D
wrong
B
beauty
E
visionary
C
flawlessness
F
changed
6. Putting a cash value on the ecological services provided by nature—such as the
water filtration “service” provided by a forested watershed—has, historically,
been a (i)________ process. Early attempts at such valuation resulted in
impressive but (ii)________ figures that were seized on by environmental
advocates and then, when these figures were later (iii)________, they were used by
opponents to tar the whole idea.
Blank (i) Blank (ii) Blank (iii)
A
dispassionate
D
redundant
G
ignored
B
problematic
E
unsound
H
discredited
C
straightforward
F
understated
I
confirmed
7. Only with the discovery of an ozone hole over Antarctica in 1985 did chemical
companies finally relinquish their opposition to a ban on chlorofluorocarbons
(CFCs), which destroy ozone. The discovery suggested that strong political action
to halt production of CFCs might be (i)________, and fortunately, the chemical
industry no longer felt compelled to oppose such action: although companies
had recently (ii)________ their research into CFC substitutes, studies they had
initiated years earlier had produced (iii)________ results.
Blank (i) Blank (ii) Blank (iii)
A
imminent
D
corroborated
G
encouraging
B
imprudent
E
publicized
H
inconclusive
C
premature
F
curtailed
I
unsurprising
8. The incipient (i)________ regarding taxes could affect trade between the two
countries much more than the (ii)________ banana imports, which has been
going on for years. Unfortunately, the trade regulators seem to be ignoring both
disagreements.
Blank (i) Blank (ii)
A
row
D
profitable dealing in
B
accord
E
predicament regarding
C
investigation
F
festering dispute over
438
GRE Practice Test 2
This ebook was issued to HUAN-LIN CHANG, order #14803770223. Unlawful distribution of this ebook is prohibited.
Customer name HUAN-LIN CHANG , Order Id: 14803770223
For each of questions 9 to 14, select one answer choice unless otherwise instructed.
Questions 9 and 10 are based on the following reading passage.
Fossil bones of the huge herbivorous dinosaurs known as sauropods were first discov-
ered and studied between 1840 and 1880, providing evidence for the gargantuan
dimensions of the adults. The shape of sauropod teeth suggested what they ate. But
aside from trackways, or series of fossilized footprints which established that sauro -
pods at least occasionally lived in herds fossils incorporating direct evidence of other
behavior, such as reproductive behavior, have been almost nonexistent. Because no
modern land animals even approach sauropod size, scientists have also lacked a living
analogue to use as a guide to possible sauropod behavior. Until the recent discovery of
fossilized sauropod nesting grounds, scientists were thus uncertain whether sauropods
laid eggs or gave birth to live young.
For the following question, consider each of the choices separately and select all that
apply.
9. Which of the following can be inferred from the passage regarding the evidence
provided by sauropod teeth?
A
The teeth allow inferences to be made about sauropod social behavior.
B
The shape of the teeth indicates that sauropods were herbivorous.
C
The teeth have no resemblance to those of any modern land animal.
For the following question, consider each of the choices separately and select all that
apply.
10. Which of the following can be inferred from the passage regarding the recently
discovered fossilized sauropod nesting grounds?
A
They are among the few fossils incorporating direct evidence of sauropod
behavior.
B
They confirm the evidence provided by trackways about sauropod
behavior.
C
They have forced a reevaluation of theories regarding the nature of
sauropod herd behavior.
439
Section 3. Verbal Reasoning
GO ON TO NEXT PAGE g
line
5
10
This ebook was issued to HUAN-LIN CHANG, order #14803770223. Unlawful distribution of this ebook is prohibited.
Customer name HUAN-LIN CHANG , Order Id: 14803770223
Questions 11 to 14 are based on the following reading passage.
Some researchers contend that sleep plays no role in the consolidation of declarative
memory (i.e., memory involving factual information). These researchers note that peo-
ple with impairments in rapid eye movement (REM) sleep continue to lead normal
lives, and they argue that if sleep were crucial for memory, then these individuals
would have apparent memory deficits. Yet the same researchers acknowledge that the
cognitive capacities of these individuals have never been systematically examined, nor
have they been the subject of studies of tasks on which performance reportedly
depends on sleep. Even if such studies were done, they could only clarify our under-
standing of the role of REM sleep, not sleep in general.
These researchers also claim that improvements of memory overnight can be ex -
plained by the mere passage of time, rather than attributed to sleep. But recent stud-
ies of memory performance after sleep including one demonstrating that sleep
stabilizes declarative memories from future interference caused by mental activity
during wakefulness make this claim unsustainable. Certainly there are memory-
consolidation processes that occur across periods of wakefulness, some of which
neither depend on nor are enhanced by sleep. But when sleep is compared with wake-
fulness, and performance is better after sleep, then some benefit of sleep for memory
must be acknowledged.
11. The primary purpose of the passage is to
A
present the evidence that supports a particular claim regarding REM sleep
and memory
B
describe how various factors contribute to the effect of sleep on memory
C
argue against a particular position regarding sleep’s role in memory
D
summarize the most prevalent theory regarding sleep and memory
E
defend the importance of the consolidation of declarative memory
12. According to the author of the passage, which of the following generalizations
about memory and sleep is true?
A
There are some memory-consolidation processes that have nothing to do
with sleep.
B
Sleep is more important to the consolidation of declarative memory than
to the consolidation of other types of memory.
C
REM sleep is more important to memory consolidation than is non-REM
sleep.
D
There are significant variations in the amount of sleep that people require
for the successful consolidation of memory.
E
It is likely that memory is more thoroughly consolidated during
wakefulness than during sleep.
13. Which of the following best describes the function of the sentence in lines 14–16
(“Certainly . . . sleep”)?
A
It provides the reasoning behind a claim about the role of sleep in
memory consolidation.
B
It explains why a previous claim about sleep and memory is
unsustainable.
440
GRE Practice Test 2
line
5
10
15
This ebook was issued to HUAN-LIN CHANG, order #14803770223. Unlawful distribution of this ebook is prohibited.
Customer name HUAN-LIN CHANG , Order Id: 14803770223
C
It demonstrates why wakefulness is central to the process of declarative
memory consolidation.
D
It emphasizes the limited role sleep plays in the process of declarative
memory consolidation.
E
It concedes that the consolidation of declarative memory does not depend
entirely on one factor.
14. The importance of the study mentioned in lines 12–14 is that it
A
reveals the mechanism by which declarative memory is stabilized during
sleep
B
identifies a specific function that sleep plays in the memory-consolidation
process
C
demonstrates that some kinds of mental activity can interfere with
memory consolidation
D
suggests that sleep and wakefulness are both important to memory
consolidation
E
explains how the passage of time contributes to memory consolidation
For questions 15 to 19, select the two answer choices that, when used to complete
the sentence, fit the meaning of the sentence as a whole and produce completed
sentences that are alike in meaning.
15. In American Indian art, the supposed distinction between modern and traditional
was fabricated by critics, and when artists have control over interpretation of
their own work, the distinction appears, happily, to have been ________.
A
eliminated
B
reinforced
C
put to rest
D
intensified
E
recognized
F
established
16. Notwithstanding their ________ regarding other issues, township residents have
consistently passed the board of education’s annual budget.
A
accord
B
indecision
C
consensus
D
disagreement
E
divergence
F
enthusiasm
441
Section 3. Verbal Reasoning
GO ON TO NEXT PAGE g
This ebook was issued to HUAN-LIN CHANG, order #14803770223. Unlawful distribution of this ebook is prohibited.
Customer name HUAN-LIN CHANG , Order Id: 14803770223
442
GRE Practice Test 2
17. Some of the company’s supporters charged that the negative report had been
motivated by a broader political assault on the company that was designed to
help market rivals who would like to see the company ________.
A
reined in
B
bolstered
C
indemnified
D
propped up
E
manacled
F
lionized
18. Skeptics contend that any scheme for charging visitors to Web sites that rewards
the vendor adequately would require steep prices, ________ the kind of frequent,
casual use of Web sites that surfers now take for granted.
A
bridling
B
exciting
C
forbidding
D
inhibiting
E
provoking
F
reversing
19. It seems obvious that Miles Davis’ ________ the Juilliard School, which resulted
in his decision to drop out, was based on the school’s training of musicians for a
kind of music that he did not want to play.
A
disaffection with
B
dislocation of
C
disentanglement from
D
subversion of
E
displacement of
F
estrangement from
This ebook was issued to HUAN-LIN CHANG, order #14803770223. Unlawful distribution of this ebook is prohibited.
Customer name HUAN-LIN CHANG , Order Id: 14803770223
For each of questions 20 to 25, select one answer choice unless otherwise instructed.
Question 20 is based on the following reading passage.
Astronomers found a large body orbiting close to the star Upsilon Andromedae. The
standard theory of planet formation holds that no planet that large could be formed so
close to a star, leading to the suggestion that the body is a companion star. A subse-
quent discovery puts that suggestion in doubt: two other large bodies were found orbit-
ing close to Upsilon Andromedae, and the standard theory of companion stars allows
for at most one companion star.
20. Which of the following, if true, most helps to resolve the status of the orbiting
body without casting doubt on the two standard theories mentioned?
A
The smaller a planet orbiting a star is, and the farther away it is from the
star, the less likely it is to be discovered.
B
If a planet’s orbit is disturbed, the planet can be drawn by gravity toward
the star it is orbiting.
C
The largest of the bodies orbiting Upsilon Andromedae is the farthest
away from the star, and the smallest is the nearest.
D
It is likely that there are many stars, in addition to Upsilon Andromedae
and the Sun, that are orbited by more than one smaller body.
E
In most cases of companion stars, the smaller companion is much fainter
than the larger star.
Question 21 is based on the following reading passage.
In Gilavia, the number of reported workplace injuries has declined 16 percent in the
last five years. However, perhaps part of the decline results from injuries going unre-
ported: many employers have introduced safety-incentive programs, such as prize
drawings for which only employees who have a perfect work-safety record are eligible.
Since a workplace injury would disqualify an employee from such programs, some
employees might be concealing injury, when it is feasible to do so.
21. Which of the following, if true in Gilavia, most strongly supports the proposed
explanation?
A
In the last five years, there has been no decline in the number of workplace
injuries leading to immediate admission to a hospital emergency room.
B
Employers generally have to pay financial compensation to employees who
suffer work-related injuries.
C
Many injuries that happen on the job are injuries that would be impossible
to conceal and yet would not be severe enough to require any change to
either the employee’s work schedule or the employee’s job responsibilities.
D
A continuing shift in employment patterns has led to a decline in the
percentage of the workforce that is employed in the dangerous occupations
in which workplace injuries are likely.
E
Employers who have instituted safety-incentive programs do not in general
have a lower proportion of reported workplace injuries among their
employees than do employers without such programs.
443
Section 3. Verbal Reasoning
GO ON TO NEXT PAGE g
This ebook was issued to HUAN-LIN CHANG, order #14803770223. Unlawful distribution of this ebook is prohibited.
Customer name HUAN-LIN CHANG , Order Id: 14803770223
Questions 22 and 23 are based on the following reading passage.
The attribution of early-nineteenth-century English fiction is notoriously problematic.
Fewer than half of new novels published in Britain between 1800 and 1829 had the
author’s true name printed on the title page. Most of these titles have subsequently
been attributed, either through the author’s own acknowledgment of a previously
anonymous or pseudonymous work, or through bibliographical research. One impor-
tant tool available to researchers is the list of earlier works “by the author” often found
on title pages. But such lists are as likely to create new confusion as they are to solve
old problems. Title pages were generally prepared last in the publication process, often
without full authorial assent, and in the last-minute rush to press, mistakes were fre-
quently made.
For the following question, consider each of the choices separately and select all that
apply.
22. The passage suggests that which of the following factors contributes to the
“notoriously problematic” (line 1) nature of authorial attribution in early-
nineteenth-century English fiction?
A
The unwillingness of any writers to acknowledge their authorship of
works that were originally published anonymously or pseudonymously
B
The possibility that the title page of a work may attribute works written by
other authors to the author of that work
C
The possibility that the author’s name printed on a title page is fictitious
For the following question, consider each of the choices separately and select all that
apply.
23. The passage suggests that which of the following is frequently true of the title
pages of early-nineteenth-century English novels?
A
The title page was prepared for printing in a hurried manner.
B
Material on the title page was included without the author’s knowledge or
approval.
C
Information on the title page was deliberately falsified to make the novel
more marketable.
444
GRE Practice Test 2
line
5
10
This ebook was issued to HUAN-LIN CHANG, order #14803770223. Unlawful distribution of this ebook is prohibited.
Customer name HUAN-LIN CHANG , Order Id: 14803770223
Questions 24 and 25 are based on the following reading passage.
The more definitions a given noun has, the more valuable is each one. Multiple defini-
tions, each subtly different from all the others, convey multiple shades of meaning.
They expand the uses of the word; language is enriched, thought is widened, and inter-
pretations increase or dilate to fill the potentialities of association. The very impossi-
bility of absoluteness in the definition of certain nouns adds to the levels of
connotation they may reach. The inner life of a writer often says more than most read-
ers can know; the mind of a reader can discover truths that go beyond the intent or
perhaps even the comprehension of the writer. And all of it finds expression because a
word can mean many things.
24. In the context in which it appears, “shades” (line 2) most nearly means
A
reminders
B
nuances
C
obscurities
D
coverings
E
degrees
25. The passage suggests that a writer’s use of nouns that have multiple definitions
can have which of the following effects on the relationship between writer and
reader?
A
It can encourage the reader to consider how the writer’s life might have
influenced the work.
B
It can cause the reader to become frustrated with the writer’s failure to
distinguish between subtle shades of meaning.
C
It can allow the reader to discern in a work certain meanings that the
writer did not foresee.
D
It allows the writer to provide the reader with clues beyond the word itself
in order to avoid ambiguity.
E
It allows the writer to present unfamiliar ideas to the reader more
efficiently.
STOP. This is the end of Section 3.
445
Section 3. Verbal Reasoning
line
5
This ebook was issued to HUAN-LIN CHANG, order #14803770223. Unlawful distribution of this ebook is prohibited.
Customer name HUAN-LIN CHANG , Order Id: 14803770223
SECTION 4
Verbal Reasoning
Time — 35 minutes
25 Questions
For questions 1 to 8, select one entry for each blank from the corresponding column
of choices. Fill all blanks in the way that best completes the text.
1. The unexplained digressions into the finer points of quantum electrodynamics
are so ________ that even readers with a physics degree would be wise to keep a
textbook handy to make sense of them.
A
uninteresting
B
controversial
C
unsophisticated
D
frustrating
E
humorless
2. The belief that politicians might become ________ after their election to office led
to the appointment of ethics officers at various levels of government.
A
scrupulous
B
entrenched
C
venal
D
puzzled
E
artificial
3. Even the charisma and technical prowess of two fine actors are not ________ the
task of fully invigorating a gray domestic drama with a tired tale to tell.
A
required for
B
interested in
C
preferred for
D
adequate to
E
inferior to
4. There may be a threshold below which blood pressure reductions become
________ given that a long-running study showed no decreased heart risk for
drops in blood pressure below a certain point.
A
worthwhile
B
indiscernible
C
arduous
D
significant
E
superfluous
446
GRE Practice Test 2
This ebook was issued to HUAN-LIN CHANG, order #14803770223. Unlawful distribution of this ebook is prohibited.
Customer name HUAN-LIN CHANG , Order Id: 14803770223
5. Unlike the problems in recent financial scandals, issues raised by the regulators
in this case appear largely to pertain to unwieldy accounting rules that are open
to widely divergent interpretations not to (i)________ transactions designed to
(ii)________ corporate malfeasance.
Blank (i) Blank (ii)
A
sham
D
cloak
B
unpremeditated
E
ameliorate
C
justifiable
F
illuminate
6. Everyone has routines that govern their work. The myth is that artists are
somehow different, that they reject (i)________, but of course that’s not true: most
artists work as the rest of us do, (ii) ________, day by day, according to their own
customs.
Blank (i) Blank (ii)
A
latitude
D
impetuously
B
habit
E
ploddingly
C
materialism
F
sporadically
7. Repression of painful memories is sometimes called “willed forgetting.” Yet true
forgetting is (i)________ than the phenomenon of repressed memory. In spite of
the effort that it (ii)________, repressing unwanted memories is less (iii) ________
than truly forgetting them, for repressed memories are prone to come back.
Blank (i) Blank (ii) Blank (iii)
A
less controlled
D
eases
G
permanent
B
different in its effect
E
conveys
H
arduous
C
far more common
F
entails
I
immediate
8. Rather than viewing the Massachusetts Bay Colony’s antinomian controversy as
the inevitable (i)________ of the intransigent opposing forces of radical and
(ii)________ beliefs, male and female piety, (iii)________ and secular power, and
the like, as other critics have, Winship argues that the crisis was not “fixed and
structural.”
Blank (i) Blank (ii) Blank (iii)
A
dissolution
D
revolutionary
G
clerical
B
melding
E
orthodox
H
civil
C
collision
F
questionable
I
cerebral
447
Section 4. Verbal Reasoning
GO ON TO NEXT PAGE g
This ebook was issued to HUAN-LIN CHANG, order #14803770223. Unlawful distribution of this ebook is prohibited.
Customer name HUAN-LIN CHANG , Order Id: 14803770223
line
5
10
15
20
25
30
35
40
448
GRE Practice Test 2
For each of questions 9 to 14, select one answer choice unless otherwise instructed.
Questions 9 to 12 are based on the following reading passage.
Until recently, many anthropologists assumed that the environment of what is now the
southwestern United States shaped the social history and culture of the region’s indige-
nous peoples. Building on this assumption, archaeologists asserted that adverse envi-
ronmental conditions and droughts were responsible for the disappearances and
migrations of southwestern populations from many sites they once inhabited.
However, such deterministic arguments fail to acknowledge that local environmen-
tal variability in the Southwest makes generalizing about that environment difficult. To
examine the relationship between environmental variation and sociocultural change in
the Western Pueblo region of central Arizona, which indigenous tribes have occupied
continuously for at least 800 years, a research team recently reconstructed the climatic,
vegetational, and erosional cycles of past centuries. The researchers found it impossi-
ble to provide a single, generally applicable characterization of environmental condi-
tions for the region. Rather, they found that local areas experienced different patterns
of rainfall, wind, and erosion, and that such conditions had prevailed in the Southwest
for the last 1,400 years. Rainfall, for example, varied within and between local valley
systems, so that even adjacent agricultural fields can produce significantly different
yields.
The researchers characterized episodes of variation in southwestern environments
by frequency: low-frequency environmental processes occur in cycles longer than one
human generation, which generally is considered to last about 25 years, and high-
frequency processes have shorter cycles. The researchers pointed out that low-frequency
processes, such as fluctuations in stream flow and groundwater levels, would not usu-
ally be apparent to human populations. In contrast, high-frequency fluctuations such
as seasonal temperature variations are observable and somewhat predictable, so that
groups could have adapted their behaviors accordingly. When the researchers com-
pared sequences of sociocultural change in the Western Pueblo region with episodes of
low- and high-frequency environmental variation, however, they found no simple cor-
relation between environmental process and sociocultural change or persistence.
Although early Pueblo peoples did protect themselves against environmental risk
and uncertainty, they responded variously on different occasions to similar patterns of
high-frequency climatic and environmental change. The researchers identified seven
major adaptive responses, including increased mobility, relocation of permanent
settlements, changes in subsistence foods, and reliance on trade with other groups.
These findings suggest that groups’ adaptive choices depended on cultural and social
as well as environmental factors and were flexible strategies rather than uncomplicated
reactions to environmental change. Environmental conditions mattered, but they were
rarely, if ever, sufficient to account for sociocultural persistence and change. Group size
and composition, culture, contact with other groups, and individual choices and
actions were barring catastrophes such as floods or earthquakes more significant
for a population’s survival than were climate and environment.
This ebook was issued to HUAN-LIN CHANG, order #14803770223. Unlawful distribution of this ebook is prohibited.
Customer name HUAN-LIN CHANG , Order Id: 14803770223
449
Section 4. Verbal Reasoning
GO ON TO NEXT PAGE g
9. The passage is primarily concerned with
A
explaining why certain research findings have created controversy
B
pointing out the flaws in a research methodology and suggesting a
different approach
C
presenting evidence to challenge an explanation and offering an
alternative explanation
D
elucidating the means by which certain groups have adapted to their
environment
E
defending a long-held interpretation by presenting new research findings
10. Which of the following findings would most strongly support the assertion made
by the archaeologists mentioned in line 3?
A
A population remained in a certain region at least a century after erosion
wore away much of the topsoil that sustained grass for their grazing
animals.
B
The range of a certain group’s agricultural activity increased over a
century of gradual decrease in annual rainfall.
C
As winters grew increasingly mild in a certain region, the nomadic
residents of the region continued to move between their summer and
winter encampments.
D
An agricultural population began to trade for supplies of a grain instead of
producing the grain in its own fields as it had in the past.
E
A half century of drought and falling groundwater levels caused a certain
population to abandon their settlements along a riverbank.
11. The fact that “adjacent agricultural fields can produce significantly different
yields” (lines 16–17) is offered as evidence of the
A
unpredictability of the climate and environment of the southwestern
United States
B
difficulty of producing a consistent food supply for a large population in
the Western Pueblo region
C
lack of water and land suitable for cultivation in central Arizona
D
local climatic variation in the environment of the southwestern United
States
E
high-frequency environmental processes at work in the southwestern
United States
This ebook was issued to HUAN-LIN CHANG, order #14803770223. Unlawful distribution of this ebook is prohibited.
Customer name HUAN-LIN CHANG , Order Id: 14803770223
450
GRE Practice Test 2
line
5
12. It can be inferred from the passage that which of the following activities is NOT
an example of a population responding to high-frequency environmental
processes?
A
Developing watertight jars in which to collect and store water during the
rainy season
B
Building multistory dwellings in low-lying areas to avoid the flash
flooding that occurs each summer
C
Moving a village because groundwater levels have changed over the last
generation
D
Trading with other groups for furs from which to make winter clothes
E
Moving one’s herds of grazing animals each year between summer and
winter pastures
Questions 13 and 14 are based on the following reading passage.
Arctic sea ice comes in two varieties. Seasonal ice forms in winter and then melts in
summer, while perennial ice persists year-round. To the untrained eye, all sea ice looks
similar, but by licking it, one can estimate how long a particular piece has been float-
ing around. When ice begins to form in seawater, it forces out salt, which has no place
in the crystal structure. As the ice gets thicker, the rejected salt collects in tiny pockets
of brine too highly concentrated to freeze. A piece of first-year ice will taste salty. Even-
tually, if the ice survives, these pockets of brine drain out through fine, veinlike chan-
nels, and the ice becomes fresher; multiyear ice can even be melted and drunk.
For the following question, consider each of the choices separately and select all that
apply.
13. The passage mentions which of the following as being a characteristic of seasonal
ice?
A
It is similar in appearance to perennial ice.
B
It is typically filled with fine, veinlike channels.
C
It tastes saltier than perennial ice.
14. In the context in which it appears, “fine” (line 7) most nearly means
A
acceptable
B
elegant
C
precise
D
pure
E
small
This ebook was issued to HUAN-LIN CHANG, order #14803770223. Unlawful distribution of this ebook is prohibited.
Customer name HUAN-LIN CHANG , Order Id: 14803770223
451
Section 4. Verbal Reasoning
GO ON TO NEXT PAGE g
For questions 15 to 18, select the two answer choices that, when used to complete
the sentence, fit the meaning of the sentence as a whole and produce completed
sentences that are alike in meaning.
15. It would have been disingenuous of the candidate to appear ________ when her
opponent won the election, but she congratulated the victor nonetheless.
A
gracious
B
ecstatic
C
crestfallen
D
indifferent
E
euphoric
F
disgruntled
16. As market forces penetrate firms and bid up the value of attributes of labor that
are more measurable than is the knowledge born of experience, it can be
expected that trends in wages will not ________ those whose main value lies in
such experiential knowledge.
A
favor
B
aid
C
affect
D
forsake
E
betray
F
differentiate
17. The point we might still take from the First World War is the old one that wars
are always, as one historian aptly put it, ________: they produce unforeseeable
results.
A
unsurprising
B
astounding
C
conventional
D
ruinous
E
stunning
F
devastating
18. This is the kind of movie stuffed with intimations of faraway strife and
people in suits talking frantically on cell phones and walkie-talkies that is
conven tionally described as a political thriller, but the film is as apolitical as it
is ________.
A
intense
B
unprecedented
C
subtle
D
humdrum
E
refined
F
dull
This ebook was issued to HUAN-LIN CHANG, order #14803770223. Unlawful distribution of this ebook is prohibited.
Customer name HUAN-LIN CHANG , Order Id: 14803770223
452
GRE Practice Test 2
For each of questions 19 to 25, select one answer choice unless otherwise instructed.
Questions 19 and 20 are based on the following reading passage.
Historians credit repeated locust invasions in the nineteenth century with reshaping
United States agriculture west of the Mississippi River. Admonished by government
entomologists, farmers began to diversify. Wheat had come to nearly monopolize the
region, but it was particularly vulnerable to the locusts. In 1873, just before the locusts’
most withering offensive, nearly two-thirds of Minnesota farmland was producing
wheat; by the invasions’ last year, that fraction had dropped to less than one-sixth.
Farmers learned that peas and beans were far less vulnerable to the insects, and corn
was a more robust grain than wheat. In addition to planting alternative crops, many
farmers turned to dairy and beef production. Although pastures were often damaged
by the locusts, these lands were almost always left in better shape than the crops were.
For the following question, consider each of the choices separately and select all that
apply.
19. According to the passage, before the recommendations by the government
entomologists, which of the following was true about farming west of the
Mississippi River?
A
Farmers focused primarily on growing wheat.
B
Peas and beans had not yet been planted in the region.
C
A relatively small portion of farmland was devoted to crops other than
wheat.
20. In the context in which it appears, “robust” (line 8) most nearly means
A
crude
B
demanding
C
productive
D
vigorous
E
rich
line
5
10
This ebook was issued to HUAN-LIN CHANG, order #14803770223. Unlawful distribution of this ebook is prohibited.
Customer name HUAN-LIN CHANG , Order Id: 14803770223
Question 21 is based on the following reading passage.
In 1998 the United States Department of Transportation received nearly 10,000 con-
sumer complaints about airlines; in 1999 it received over 20,000. Moreover, the num-
ber of complaints per 100,000 passengers also more than doubled. In both years the
vast majority of complaints concerned flight delays, cancellations, mishandled bag-
gage, and customer service. Clearly, therefore, despite the United States airline indus-
try’s serious efforts to improve performance in these areas, passenger dissatisfaction
with airline service increased significantly in 1999.
21. Which of the following, if true, most seriously weakens the argument?
A
Although the percentage of flights that arrived on time dropped slightly
overall, from 77 percent in 1998 to 76 percent in 1999, some United States
airlines’ 1999 on-time rate was actually better than their 1998 on-time
rate.
B
The number of passengers flying on United States airlines was
significantly higher in 1999 than in 1998.
C
Fewer bags per 1,000 passengers flying on United States airlines were lost
or delayed in 1999 than in 1998.
D
The appearance in 1999 of many new Internet sites that relay complaints
directly to the Department of Transportation has made filing a complaint
about airlines much easier for consumers than ever before.
E
Although the number of consumer complaints increased for every major
United States airline in 1999, for some airlines the extent of the increase
was substantial, whereas for others it was extremely small.
453
Section 4. Verbal Reasoning
GO ON TO NEXT PAGE g
This ebook was issued to HUAN-LIN CHANG, order #14803770223. Unlawful distribution of this ebook is prohibited.
Customer name HUAN-LIN CHANG , Order Id: 14803770223
Questions 22 to 24 are based on the following reading passage.
Nineteenth-century architect Eugène-Emmanuel Viollet-le-Duc contended that Paris’s
Notre-Dame cathedral, built primarily in the late twelfth century, was supported from
the very beginning by a system of flying buttresses a series of exterior arches (flyers)
and their supports (buttresses) which permitted the construction of taller vaulted
buildings with slimmer walls and interior supports than had been possible previously.
Other commentators insist, however, that Notre-Dame did not have flying buttresses
until the thirteenth or fourteenth century, when they were added to update the build-
ing aesthetically and correct its structural flaws. Although post-twelfth-century modi-
fications and renovations complicate efforts to resolve this controversy all
pre-fifteenth-century flyers have been replaced, and the buttresses have been rebuilt
and/or resurfaced it is nevertheless possible to tell that both the nave and the choir,
the church’s two major parts, have always had flying buttresses. It is clear, now that
nineteenth-century paint and plaster have been removed, that the nave’s lower but-
tresses date from the twelfth century. Moreover, the choir’s lower flyers have chevron
(zigzag) decoration. Chevron decoration, which was characteristic of the second half of
the twelfth century and was out of favor by the fourteenth century, is entirely absent
from modifications to the building that can be dated with confidence to the thirteenth
century.
22. The passage is primarily concerned with
A
tracing the development of a controversy
B
discussing obstacles to resolving a controversy
C
arguing in support of one side in a controversy
D
analyzing the assumptions underlying the claims made in a controversy
E
explaining why evidence relevant to a controversy has been overlooked
23. The claim of the “other commentators” (line 6) suggests that they believe which
of the following about Notre-Dame?
A
It was the inspiration for many vaulted cathedrals built in the thirteenth
and fourteenth centuries.
B
Its design flaws were not apparent until flying buttresses were added in
the thirteenth or fourteenth century.
C
Its flying buttresses are embellished with decoration characteristic of the
thirteenth and fourteenth centuries.
D
It had been modified in some respects before flying buttresses were added
in the thirteenth or fourteenth century.
E
It was originally constructed in an architectural style that was considered
outmoded by the thirteenth or fourteenth century.
454
GRE Practice Test 2
line
5
10
15
This ebook was issued to HUAN-LIN CHANG, order #14803770223. Unlawful distribution of this ebook is prohibited.
Customer name HUAN-LIN CHANG , Order Id: 14803770223
24. The author’s argument concerning Notre-Dame’s flying buttresses depends on
which of the following assumptions about the choir’s lower flyers?
A
They accurately reproduce the decoration on the choir’s original lower
flyers.
B
They have a type of decoration used exclusively for exterior surfaces.
C
They were the models for the choir’s original upper flyers.
D
They were the models for the nave’s original lower flyers.
E
They were constructed after the nave’s flyers were constructed.
Question 25 is based on the following reading passage.
The average temperature of the lobster-rich waters off the coast of Foerkland has been
increasing for some years. In warmer water, lobsters grow faster. In particular, lobster
larvae take less time to reach the size at which they are no longer vulnerable to preda-
tion by young cod, the chief threat to their survival. Consequently, the survival rate of
lobster larvae must be going up, and the lobster population in Foerkland’s coastal
waters is bound to increase.
25. Which of the following, if true, most seriously weakens the argument?
A
There are indications that in recent years the fishing fleet operating off the
coast of Foerkland has been taking cod at an unsustainably high rate.
B
The increase in water temperatures off Foerkland has not been as
pronounced as the increase in average soil temperatures in Foerkland.
C
Because of their speeded-up growth, lobsters now get large enough to be
legal catch before they reach reproductive maturity.
D
Even though lobsters grow faster in warmer waters, warmer waters have
no effect on the maximum size to which a lobster can eventually grow.
E
Cod are a cold-water species, and the increasing water temperatures have
caused a northward shift in Foerkland’s cod population.
STOP. This is the end of Section 4.
455
Section 4. Verbal Reasoning
This ebook was issued to HUAN-LIN CHANG, order #14803770223. Unlawful distribution of this ebook is prohibited.
Customer name HUAN-LIN CHANG , Order Id: 14803770223
GRE Practice Test 2
456
SECTION 5
Quantitative Reasoning
Time40 minutes
25 Questions
For each question, indicate the best answer, using the directions given.
Notes: All numbers used are real numbers.
All figures are assumed to lie in a plane unless otherwise indicated.
Geometric figures, such as lines, circles, triangles, and quadrilaterals, are not
necessarily drawn to scale. That is, you should not assume that quantities
such as lengths and angle measures are as they appear in a figure. You
should assume, however, that lines shown as straight are actually straight,
points on a line are in the order shown, and more generally, all geometric
objects are in the relative positions shown. For questions with geometric fig-
ures, you should base your answers on geometric reasoning, not on estimat-
ing or comparing quantities by sight or by measurement.
Coordinate systems, such as xy-planes and number lines, are drawn to scale;
therefore, you can read, estimate, or compare quantities in such figures by
sight or by measurement.
Graphical data presentations, such as bar graphs, circle graphs, and line
graphs, are drawn to scale; therefore, you can read, estimate, or compare
data values by sight or by measurement.
For each of Questions 1 to 9, compare Quantity A and Quantity B, using
additional information centered above the two quantities if such information is
given. Select one of the following four answer choices and fill in the
corresponding oval to the right of the question.
AQuantity A is greater.
BQuantity B is greater.
CThe two quantities are equal.
DThe relationship cannot be determined from the information given.
A symbol that appears more than once in a question has the same meaning
throughout the question.
Quantity A Quantity B Correct Answer
Example 1: (2)(6) 2 +6ABCD
––––––––––––––––––––––––––––––––––––
Q
RP S
Quantity A Quantity B Correct Answer
Example 2: PS SR ABCD
(since equal lengths cannot
be assumed, even though
PS and SR appear equal)
This ebook was issued to HUAN-LIN CHANG, order #14803770223. Unlawful distribution of this ebook is prohibited.
Customer name HUAN-LIN CHANG , Order Id: 14803770223
Section 5. Quantitative Reasoning
457
AQuantity A is greater.
BQuantity B is greater.
CThe two quantities are equal.
DThe relationship cannot be determined from the information given.
Quantity A Quantity B
1.
1
3
1
4
4
3
ABCD
andx<1x0
Quantity A Quantity B
2.
2
x+1
3
x+1ABCD
x>0
Quantity A Quantity B
3. 0.5% of x1x
2
ABCD
The median income of a group of College Cgraduates
six months after graduation was $3,000 higher than the
median income of a group of College Dgraduates six
months after graduation.
Quantity A Quantity B
4. The 75th percentile of the
incomes of the group of
College Cgraduates six
months after graduation
The 75th percentile of the
incomes of the group of
College Dgraduates six
months after graduation
ABCD
GO ON TO THE NEXT PAGE b
This ebook was issued to HUAN-LIN CHANG, order #14803770223. Unlawful distribution of this ebook is prohibited.
Customer name HUAN-LIN CHANG , Order Id: 14803770223
GRE Practice Test 2
458
AQuantity A is greater.
BQuantity B is greater.
CThe two quantities are equal.
DThe relationship cannot be determined from the information given.
Frequency
15
10
5
0123456
24710
16
11
The graph above shows the frequency distribution of 50
integer values varying from 1 to 6.
Quantity A Quantity B
5. The average (arithmetic
mean) of the 50 values
The median of the 50
values
ABCD
3
P
R
Q
S
5
52
Quantity A Quantity B
6. The area of triangle PQR The area of triangle PSR ABCD
Quantity A Quantity B
7. The sum of the odd
integers from 1 to 199
The sum of the even
integers from 2 to 198
ABCD
sand tare positive integers, and
st
32 =2.
Quantity A Quantity B
8. s
t
1
5
ABCD
This ebook was issued to HUAN-LIN CHANG, order #14803770223. Unlawful distribution of this ebook is prohibited.
Customer name HUAN-LIN CHANG , Order Id: 14803770223
Section 5. Quantitative Reasoning
459
AQuantity A is greater.
BQuantity B is greater.
CThe two quantities are equal.
DThe relationship cannot be determined from the information given.
In a quality-control test, 50 boxeseach containing 30
machine partswere examined for defective parts. The
number of defective parts was recorded for each box,
and the average (arithmetic mean) of the 50 recorded
numbers of defective parts per box was 1.12. Only one
error was made in recording the 50 numbers: “1”
defective part in a certain box was incorrectly recorded
as “10”.
Quantity A Quantity B
9. The actual average number
of defective parts per box
0.94 ABCD
Questions 10 to 25 have several different formats. Unless otherwise directed,
select a single answer choice. For Numeric Entry questions, follow the
instructions below.
Numeric Entry Questions
Enter your answer in the answer box(es) below the question.
Your answer may be an integer, a decimal, or a fraction, and it may be
negative.
If a question asks for a fraction, there will be two boxesone for the
numerator and one for the denominator.
Equivalent forms of the correct answer, such as 2.5 and 2.50, are all correct.
Fractions do not need to be reduced to lowest terms.
Enter the exact answer unless the question asks you to round your answer.
10. In year Y, the population of Colorado was approximately half that of New
Jersey, and the land area of Colorado was approximately 14 times that of
New Jersey. The population density (number of persons per unit of land
area) of Colorado in year Ywas approximately how many times the
population density of New Jersey?
A1
28
B1
14
C1
7
D1
4
E1
2GO ON TO THE NEXT PAGE b
This ebook was issued to HUAN-LIN CHANG, order #14803770223. Unlawful distribution of this ebook is prohibited.
Customer name HUAN-LIN CHANG , Order Id: 14803770223
GRE Practice Test 2
460
For the following question, enter your answer in the box.
35°
(2yk
m
11. In the figure above, line kis parallel to line m. What is the value of y?
y=
12. The numbers in data set Shave a standard deviation of 5. If a new data set is
formed by adding 3 to each number in S, what is the standard deviation of the
numbers in the new data set?
A2
B3
C5
D8
E15
13. If which of the following could be the value of y?
2y33y
=,
y2
A4
B1
C1
D3
E5
For the following question, select all the answer choices that apply.
14. List Kconsists of the numbers 0, 5, and 10. Which of the following10, 5,
lists of numbers have the same range as the numbers in list K?
Indicate all such lists.
A0, 1, 1515, 1,
B1, 137, 4, 2,
C0, 1, 2, 5, 8, 10
D2, 3, 5, 15, 19, 22
E4, 5, 6, 24
This ebook was issued to HUAN-LIN CHANG, order #14803770223. Unlawful distribution of this ebook is prohibited.
Customer name HUAN-LIN CHANG , Order Id: 14803770223
Section 5. Quantitative Reasoning
461
15. Aisha’s income in 2004 was 20 percent greater than her income in 2003.
What is the ratio of Aisha’s income in 2004 to her income in 2003 ?
A1to5
B5to6
C6to5
D5to1
E20 to 1
16. Jacob’s weekly take-home pay is ndollars. Each week he uses dollars for
4n
5
expenses and saves the rest. At those rates, how many weeks will it take
Jacob to save $500, in terms of n?
A500
n
B2,500
n
Cn
625
Dn
2,500
E625n
Questions 17 to 20 are based on the following data.
LENGTH OF UNEMPLOYMENT FOR WORKERS IN REGION X FOR TWO INDUSTRIES, 2003
Note: The circle graphs show the distributions of workers who were unemployed for at
least 1 week in 2003, by length of unemployment, rounded to the nearest week.
Manufacturing Industry
Total: 10 million
Service Industry
Total: 8 million
1 to 4
weeks
56%
1 to 4
weeks
40%
16%
5 to 10 weeks
20%
18%
10%
12%
14%
9%
5%
11 to 14 weeks
15 to 25 weeks
26 weeks or more
17. In the circle graphs, the degree measure of the central angle of the sector
representing the number of workers unemployed for 11 to 14 weeks is how
much greater in the manufacturing industry graph than in the service
industry graph?
A5
B10
C15
D18
E20
GO ON TO THE NEXT PAGE b
This ebook was issued to HUAN-LIN CHANG, order #14803770223. Unlawful distribution of this ebook is prohibited.
Customer name HUAN-LIN CHANG , Order Id: 14803770223
GRE Practice Test 2
462
18. Which of the following could be the median length of unemployment, in
weeks, for manufacturing industry workers who were unemployed for at
least 1 week?
A4
B8
C12
D16
E20
19. If one of the workers in the manufacturing and service industries who were
unemployed for at least 1 week will be randomly selected, what is the
probability that the person selected will be a service industry worker who
was unemployed for 26 weeks or more?
A0.04
B0.09
C0.21
D0.40
E0.90
20. The ratio of the number of manufacturing industry workers who were
unemployed for 5 to 10 weeks to the number of service industry workers
who were unemployed for 5 to 10 weeks is closest to which of the
following?
A5to4
B6to5
C3to2
D5to2
E7to6
For the following question, select all the answer choices that apply.
21. If which of the following could be the value of t?t+3>5,
Indicate all such values.
A9
B6
C2
D0
E2
F3
This ebook was issued to HUAN-LIN CHANG, order #14803770223. Unlawful distribution of this ebook is prohibited.
Customer name HUAN-LIN CHANG , Order Id: 14803770223
Section 5. Quantitative Reasoning
463
22. The operation is defined for all integers xand yas If xandxy=xy y.
yare positive integers, which of the following CANNOT be zero?
Axy
Byx
C(x1) y
D(x+1) y
Ex(y1)
23. P, Q , and Rare three points in a plane, and Rdoes not lie on line PQ.
Which of the following is true about the set of all points in the plane that
are the same distance from all three points?
AIt contains no points.
BIt contains one point.
CIt contains two points.
DIt is a line.
EIt is a circle.
24. If which of the following inequalities must be true?x<y<0,
Ay+1<x
By1<x
C
2
xy <x
D
2
xy <y
E
2
xy <x
For the following question, enter your answer in the box.
25. What is the length of a diagonal of a rectangle that has width 5 and
perimeter 34 ?
STOP. This is the end of Section 5.
This ebook was issued to HUAN-LIN CHANG, order #14803770223. Unlawful distribution of this ebook is prohibited.
Customer name HUAN-LIN CHANG , Order Id: 14803770223
GRE Practice Test 2
464
SECTION 6
Quantitative Reasoning
Time40 minutes
25 Questions
For each question, indicate the best answer, using the directions given.
Notes: All numbers used are real numbers.
All figures are assumed to lie in a plane unless otherwise indicated.
Geometric figures, such as lines, circles, triangles, and quadrilaterals, are not
necessarily drawn to scale. That is, you should not assume that quantities
such as lengths and angle measures are as they appear in a figure. You
should assume, however, that lines shown as straight are actually straight,
points on a line are in the order shown, and more generally, all geometric
objects are in the relative positions shown. For questions with geometric fig-
ures, you should base your answers on geometric reasoning, not on estimat-
ing or comparing quantities by sight or by measurement.
Coordinate systems, such as xy-planes and number lines, are drawn to scale;
therefore, you can read, estimate, or compare quantities in such figures by
sight or by measurement.
Graphical data presentations, such as bar graphs, circle graphs, and line
graphs, are drawn to scale; therefore, you can read, estimate, or compare
data values by sight or by measurement.
For each of Questions 1 to 9, compare Quantity A and Quantity B, using
additional information centered above the two quantities if such information is
given. Select one of the following four answer choices and fill in the
corresponding oval to the right of the question.
AQuantity A is greater.
BQuantity B is greater.
CThe two quantities are equal.
DThe relationship cannot be determined from the information given.
A symbol that appears more than once in a question has the same meaning
throughout the question.
Quantity A Quantity B Correct Answer
Example 1: (2)(6) 2+6ABCD
––––––––––––––––––––––––––––––––––––
Q
RP S
Quantity A Quantity B Correct Answer
Example 2: PS SR ABCD
(since equal lengths cannot
be assumed, even though
PS and SR appear equal)
This ebook was issued to HUAN-LIN CHANG, order #14803770223. Unlawful distribution of this ebook is prohibited.
Customer name HUAN-LIN CHANG , Order Id: 14803770223
Section 6. Quantitative Reasoning
465
AQuantity A is greater.
BQuantity B is greater.
CThe two quantities are equal.
DThe relationship cannot be determined from the information given.
A circle is inscribed in a square with sides of length 5.
Quantity A Quantity B
1. The circumference of the
circle
15 ABCD
2u+v=14
uv =0
Quantity A Quantity B
2. uv
ABCD
Quantity A Quantity B
3.
2,000
950
6,000
10 ABCD
Set Aconsists of 40 integers, and set Bconsists of 150
integers. The number of integers that are in both set A
and set Bis 20.
Quantity A Quantity B
4. The total number of integers
that are in set A or set B,
or both
170 ABCD
GO ON TO THE NEXT PAGE b
This ebook was issued to HUAN-LIN CHANG, order #14803770223. Unlawful distribution of this ebook is prohibited.
Customer name HUAN-LIN CHANG , Order Id: 14803770223
GRE Practice Test 2
466
AQuantity A is greater.
BQuantity B is greater.
CThe two quantities are equal.
DThe relationship cannot be determined from the information given.
xis a negative integer.
Quantity A Quantity B
5.
x
2
x+1
3ABCD
(x+3)( y4) =0
Quantity A Quantity B
6. xy 12 ABCD
Geoff used $630 to buy a new guitar. This amount was
15 percent of his earnings last summer.
Quantity A Quantity B
7. The amount of Geoff’s
earnings last summer not
used to buy the new guitar
$3,570 ABCD
Set Sconsists of 5 objects.
Quantity A Quantity B
8. The number of subsets
of set Sthat consist of
1 object
The number of subsets
of set Sthat consist of
4 objects
ABCD
This ebook was issued to HUAN-LIN CHANG, order #14803770223. Unlawful distribution of this ebook is prohibited.
Customer name HUAN-LIN CHANG , Order Id: 14803770223
Section 6. Quantitative Reasoning
467
AQuantity A is greater.
BQuantity B is greater.
CThe two quantities are equal.
DThe relationship cannot be determined from the information given.
A
B
C
45° 60°
2
Quantity A Quantity B
9. The length of line segment
AC
3ABCD
Questions 10 to 25 have several different formats. Unless otherwise directed,
select a single answer choice. For Numeric Entry questions, follow the
instructions below.
Numeric Entry Questions
Enter your answer in the answer box(es) below the question.
Your answer may be an integer, a decimal, or a fraction, and it may be
negative.
If a question asks for a fraction, there will be two boxesone for the
numerator and one for the denominator.
Equivalent forms of the correct answer, such as 2.5 and 2.50, are all correct.
Fractions do not need to be reduced to lowest terms.
Enter the exact answer unless the question asks you to round your answer.
5
–5
x
k
y
O
10. What is the slope of line kin the xy-plane above?
A5
B1
C0
D1
E5
GO ON TO THE NEXT PAGE b
This ebook was issued to HUAN-LIN CHANG, order #14803770223. Unlawful distribution of this ebook is prohibited.
Customer name HUAN-LIN CHANG , Order Id: 14803770223
GRE Practice Test 2
468
b3, b1, b+2, b+3, b+4
11. The median of the five terms listed above is 5, where bis a constant. What
is the average (arithmetic mean) of the five terms?
A3
B4
C5
D6
E7
For the following question, enter your answer in the box.
15
9
12 8
12. What is the area of the region shown above?
13. During a one-year study, biologists observed the number of fish in a certain
pond as well as the percent of the fish that were catfish. At the beginning of
the year, there were 300 fish in the pond, of which 15 percent were catfish;
and at the end of the year, there were 400 fish in the pond, of which 10
percent were catfish. From the beginning of the year to the end of the year,
the number of catfish in the pond
Adecreased by more than 5%
Bdecreased by 5%
Cdid not change
Dincreased by 5%
Eincreased by more than 5%
For the following question, enter your answer in the box.
14. On a radio tower, a red light flashes every 6 seconds and a blue light flashes
every 10 seconds. If both lights flash together at a certain time, how many
seconds later will both lights flash together the next time?
seconds
This ebook was issued to HUAN-LIN CHANG, order #14803770223. Unlawful distribution of this ebook is prohibited.
Customer name HUAN-LIN CHANG , Order Id: 14803770223
Section 6. Quantitative Reasoning
469
For the following question, select all the answer choices that apply.
15. If which of the following numbers must be positive?a<b<0,
Indicate all such numbers.
Aab
B
22
ab
Cab
D
2
ab
E
22
ab+ab
x
x
16. A flat rectangular picture, represented by the unshaded region in the figure
above, is mounted in a flat rectangular frame, represented by the shaded
region. The frame is 1 inch wide on all sides. For what value of x, in inches,
is the area of the frame equal to the area of the picture?
A4
B5
C6
D7
E8
GO ON TO THE NEXT PAGE b
This ebook was issued to HUAN-LIN CHANG, order #14803770223. Unlawful distribution of this ebook is prohibited.
Customer name HUAN-LIN CHANG , Order Id: 14803770223
GRE Practice Test 2
470
Questions 17 to 20 are based on the following data.
PERCENT OF THE 300 PEOPLE IN GROUP 1 AND THE 400 PEOPLE
IN GROUP 2 WHO HAVE SELECTED AILMENTS
Respiratory Ailment
Percent of People
in Group 1 Who
Have Ailment
Percent of People
in Group 2 Who
Have Ailment
Allergic sensitivity to endotoxins 14% 21%
Asthma (allergic) 3% 4%
Asthma (nonallergic) 2% 3%
Hay fever 4% 10%
Sneezing and itchy eyes 8% 11%
Wheezing (allergic) 5% 6%
Wheezing (nonallergic) 2% 5%
17. The number of people in group 2 who have hay fever is how much greater
than the number of people in group 1 who have hay fever?
A37
B35
C32
D28
E24
18. For the seven ailments, what is the median of the numbers of people in
group 2 who have the ailments?
A20
B22
C24
D26
E28
19. The number of people in group 1 who have the ailment wheezing (allergic)
is what percent greater than the number of people in group 1 who have the
ailment wheezing (nonallergic)?
A50%
B75%
C150%
D200%
E300%
This ebook was issued to HUAN-LIN CHANG, order #14803770223. Unlawful distribution of this ebook is prohibited.
Customer name HUAN-LIN CHANG , Order Id: 14803770223
Section 6. Quantitative Reasoning
471
For the following question, enter your answer in the boxes.
20. What is the ratio of the number of people in group 2 with the ailment
sneezing and itchy eyes to the total number of people in both groups with
the ailment sneezing and itchy eyes?
Give your answer as a fraction.
21. Of the people in a certain survey, 58 percent were at most 40 years old and
70 percent were at most 60 years old. If 252 of the people in the survey
were more than 40 years old and at most 60 years old, what was the total
number of people in the survey?
A1,900
B2,100
C2,400
D2,700
E3,000
22. If which of the following is equal to 1.25 percent of x?x>0,
Ax
80
Bx
8
Cx
4
D5x
8
E3x
4
23. Alice earns ddollars and has tpercent of what she earns deducted for taxes.
How much of what she earns does Alice have left after taxes?
Adollarsd(1 100t)
Bdollarsd(1 10t)
Cdollarsd(1 t)
Ddollarsd(1 0.1t)
Edollarsd(1 0.01t)
GO ON TO THE NEXT PAGE b
This ebook was issued to HUAN-LIN CHANG, order #14803770223. Unlawful distribution of this ebook is prohibited.
Customer name HUAN-LIN CHANG , Order Id: 14803770223
GRE Practice Test 2
472
For the following question, select all the answer choices that apply.
24. A student made a conjecture that for any integer n, the integer is a prime4n+3
number. Which of the following values of ncould be used to disprove the
student’s conjecture?
Indicate all such values.
A1
B3
C4
D6
E7
25. Eight points are equally spaced on a circle. If 4 of the 8 points are to be
chosen at random, what is the probability that a quadrilateral having the 4
points chosen as vertices will be a square?
A1
70
B1
35
C1
7
D1
4
E1
2
STOP. This is the end of Section 6.
This ebook was issued to HUAN-LIN CHANG, order #14803770223. Unlawful distribution of this ebook is prohibited.
Customer name HUAN-LIN CHANG , Order Id: 14803770223
Evaluating Your Performance
Now that you have completed Practice Test 2, it is time to evaluate your performance.
Analytical Writing Measure
One way to evaluate your performance on the Issue and Argument topics you answered
on this practice test is to compare your essay responses with the scored sample essay
responses for these topics and review the rater commentary. Scored sample essay
responses and rater commentary are presented starting on page 481 for the one Issue
topic and one Argument topic presented in the Analytical Writing sections of Practice
Test 2. The Issue and Argument scoring guides start on page 37.
To better understand the analytical writing abilities characteristic of particular
score levels, you should review the score level descriptions on page 41.
Verbal Reasoning and Quantitative Reasoning Measures
The tables that follow contain information to help you evaluate your performance on
the Verbal Reasoning and Quantitative Reasoning measures of Practice Test 2. An
answer key with the correct answers to the questions in the Verbal Reasoning and
Quantitative Reasoning sections in this practice test begins on page 475. Compare your
answers with the correct answers given in the table, crossing out questions you
answered incorrectly or omitted. Partially correct answers should be treated as incor-
rect. Knowing which questions you answered incorrectly or omitted can help you iden-
tify content areas in which you need more practice or review.
The answer key contains additional information to help you evaluate your per-
formance. With each answer, the key provides a number, the P+. The P+ is the percent
of a group of actual GRE takers who were administered that same question at a previ-
ous test administration and who answered it correctly. P+ is used to gauge the relative
difficulty of a test question. The higher the P+, the easier the test question. You can use
the P+ to compare your performance on each test question to the performance of other
test takers on that same question. For example, if the P+ for a question is 89, that
means that 89 percent of GRE test takers who received this question answered it cor-
rectly. Alternatively, if the P+ for a question is 14, that means that 14 percent of GRE
test takers who received this question answered it correctly. A question with a P+ of 89
may be interpreted as a relatively easy question, and a question with a P+ of 14 may be
interpreted as a difficult question.
To calculate your scores on Practice Test 2:
bAdd the number of correct answers in Sections 3 and 4 to obtain your raw Ver-
bal Reasoning score.
bAdd the number of correct answers in Sections 5 and 6 to obtain your raw
Quantitative Reasoning score.
bOnce you have calculated your raw scores, refer to the Practice Test 2 score
conversion table on pages 479–480. Find the scores on the 130–170 score scales
that correspond to your Verbal Reasoning and Quantitative Reasoning raw
scores. Note the scaled scores provided.
473
Evaluating Your Performance
This ebook was issued to HUAN-LIN CHANG, order #14803770223. Unlawful distribution of this ebook is prohibited.
Customer name HUAN-LIN CHANG , Order Id: 14803770223
Once you determine your scaled scores, you will need to evaluate your perform-
ance. To get a sense of how test takers are scoring on the Verbal Reasoning and Quan-
titative Reasoning measures of the actual test, you can review Verbal Reasoning and
Quantitative Reasoning percentile ranks on the GRE website at www.ets.org/gre
/percentile (PDF). A percentile rank for a score indicates the percentage of examinees
who took that test and received a lower score. Updated annually in July, this table
includes the Verbal Reasoning and Quantitative Reasoning scores on the 130–170 scale
in one-point increments and the corresponding percentile ranks. For each score you
earned on Practice Test 2, note the percent of GRE test takers who earned lower scores.
This is a reasonable indication of your rank among GRE revised General Test exami-
nees if you took Practice Test 2 under standard timed conditions.
474
GRE Practice Test 2
This ebook was issued to HUAN-LIN CHANG, order #14803770223. Unlawful distribution of this ebook is prohibited.
Customer name HUAN-LIN CHANG , Order Id: 14803770223
Answer Key
Section 3. Verbal Reasoning
Question
Number P+ Correct Answer
171Choice A: innocuous
268Choice C: banished from
341Choice C: comprehensive; Choice D: redundant
474Choice B: heed; Choice F: compromise
571Choice C: flawlessness; Choice D: wrong
655Choice B: problematic; Choice E: unsound; Choice H: discredited
728Choice A: imminent; Choice F: curtailed; Choice G: encouraging
838Choice A: row; Choice F: festering dispute over
974Choice B: The shape of the teeth indicates that sauropods were herbivorous.
10 54 Choice A: They are among the few fossils incorporating direct evidence of
sauropod behavior.
11 66 Choice C: argue against a particular position regarding sleep’s role in
memory
12 60 Choice A: There are some memory-consolidation processes that have
nothing to do with sleep.
13 73 Choice E: It concedes that the consolidation of declarative memory does not
depend entirely on one factor.
14 60 Choice B: identifies a specific function that sleep plays in the memory-
consolidation process
15 74 Choice A: eliminated AND Choice C: put to rest
16 41 Choice D: disagreement AND Choice E: divergence
17 27 Choice A: reined in AND Choice E: manacled
18 45 Choice A: bridling AND Choice D: inhibiting
19 56 Choice A: disaffection with AND Choice F: estrangement from
20 42 Choice B: If a planet’s orbit is disturbed, the planet can be drawn by gravity
toward the star it is orbiting.
21 35 Choice A: In the last five years, there has been no decline in the number of
workplace injuries leading to immediate admission to a hospital emergency
room.
22 51 Choice B: The possibility that the title page of a work may attribute works
written by other authors to the author of that work
AND
Choice C: The possibility that the author’s name printed on a title page is
fictitious
23 83 Choice A: The title page was prepared for printing in a hurried manner.
AND
Choice B: Material on the title page was included without the author’s
knowledge or approval.
24 37 Choice B: nuances
25 78 Choice C: It can allow the reader to discern in a work certain meanings that
the writer did not foresee.
475
Evaluating Your Performance
This ebook was issued to HUAN-LIN CHANG, order #14803770223. Unlawful distribution of this ebook is prohibited.
Customer name HUAN-LIN CHANG , Order Id: 14803770223
Answer Key
Section 4. Verbal Reasoning
Question
Number P+ Correct Answer
172Choice D: frustrating
250Choice C: venal
380Choice D: adequate to
459Choice E: superfluous
548Choice A: sham; Choice D: cloak
661Choice B: habit; Choice E: ploddingly
741Choice B: different in its effect; Choice F: entails; Choice G: permanent
840Choice C: collision; Choice E: orthodox; Choice G: clerical
974Choice C: presenting evidence to challenge an explanation and offering an
alternative explanation
10 84 Choice E: A half century of drought and falling groundwater levels caused a
certain population to abandon their settlements along a riverbank.
11 68 Choice D: local climatic variation in the environment of the southwestern
United States
12 48 Choice C: Moving a village because groundwater levels have changed over
the last generation
13 69 Choice A: It is similar in appearance to perennial ice.
AND
Choice C: It tastes saltier than perennial ice.
14 93 Choice E: small
15 56 Choice B: ecstatic AND Choice E: euphoric
16 75 Choice A: favor AND Choice B: aid
17 53 Choice B: astounding AND Choice E: stunning
18 59 Choice D: humdrum AND Choice F: dull
19 66 Choice A: Farmers focused primarily on growing wheat.
AND
Choice C: A relatively small portion of farmland was devoted to crops other
than wheat.
20 44 Choice D: vigorous
21 55 Choice D: The appearance in 1999 of many new Internet sites that relay
complaints directly to the Department of Transportation has made filing a
complaint about airlines much easier for consumers than ever before.
22 46 Choice C: arguing in support of one side in a controversy
23 42 Choice E: It was originally constructed in an architectural style that was
considered outmoded by the thirteenth or fourteenth century.
24 40 Choice A: They accurately reproduce the decoration on the choir’s original
lower flyers.
25 57 Choice C: Because of their speeded-up growth, lobsters now get large
enough to be legal catch before they reach reproductive maturity.
476
GRE Practice Test 2
This ebook was issued to HUAN-LIN CHANG, order #14803770223. Unlawful distribution of this ebook is prohibited.
Customer name HUAN-LIN CHANG , Order Id: 14803770223
Answer Key
Section 5. Quantitative Reasoning
Question
Number P+ Correct Answer
163Choice C: The two quantities are equal.
280Choice A: Quantity A is greater.
363Choice B: Quantity B is greater.
445Choice D: The relationship cannot be determined from the information
given.
567Choice B: Quantity B is greater.
673Choice B: Quantity B is greater.
748Choice A: Quantity A is greater.
859Choice C: The two quantities are equal.
935Choice C: The two quantities are equal.
10 60 Choice A:
11 84 17.5
12 54 Choice C: 5
13 76 Choice D:
3
14 65 Choice B:
7,
4,
2, 1, 13
AND
Choice D: 2, 3, 5, 15, 19, 22
AND
Choice E: 4, 5, 6, 24
15 66 Choice C: 6 to 5
16 51 Choice B:
2,500
n
17 51 Choice D: 18°
18 49 Choice B: 8
19 51 Choice A: 0.04
20 45 Choice C: 3 to 2
21 69 Choice A:
9
AND
Choice F: 3
22 42 Choice D:
23 41 Choice B: It contains one point.
24 40 Choice E:
25 62 13
477
Evaluating Your Performance
1
28
(x+1) y
xy x
2
This ebook was issued to HUAN-LIN CHANG, order #14803770223. Unlawful distribution of this ebook is prohibited.
Customer name HUAN-LIN CHANG , Order Id: 14803770223
Answer Key
Section 6. Quantitative Reasoning
Question
Number P+ Correct Answer
173Choice A: Quantity A is greater.
269Choice D: The relationship cannot be determined from the information
given.
364Choice B: Quantity B is greater.
441Choice C: The two quantities are equal.
536Choice D: The relationship cannot be determined from the information
given.
642Choice D: The relationship cannot be determined from the information
given.
772Choice C: The two quantities are equal.
828Choice C: The two quantities are equal.
936Choice B: Quantity B is greater.
10 69 Choice D: 1
11 69 Choice B: 4
12 78 156
13 66 Choice A: decreased by more than 5%
14 72 30
15 58 Choice B:
AND
Choice C: ab
16 46 Choice C: 6
17 90 Choice D: 28
18 82 Choice C: 24
19 56 Choice C: 150%
20 66 (or any equivalent fraction)
21 58 Choice B: 2,100
22 45 Choice A:
23 37 Choice E: d(1
0.01t) dollars
24 53 Choice B: 03
AND
Choice D: 6
25 32 Choice B:
478
GRE Practice Test 2
x
80
a
2
b
2
44
68
1
35
This ebook was issued to HUAN-LIN CHANG, order #14803770223. Unlawful distribution of this ebook is prohibited.
Customer name HUAN-LIN CHANG , Order Id: 14803770223
Score Conversion Table
Raw Score Verbal Reasoning Scaled Score Quantitative Reasoning Scaled Score
50 170 170
49 170 169
48 169 167
47 168 165
46 167 164
45 166 163
44 165 162
43 164 161
42 163 160
41 162 159
40 161 158
39 160 157
38 160 157
37 159 156
36 158 155
35 158 155
34 157 154
33 156 153
32 156 153
31 155 152
30 154 152
29 154 151
28 153 150
27 152 150
26 152 149
25 151 149
24 150 148
23 149 148
22 149 147
21 148 146
20 147 146
19 147 145
18 146 145
17 145 144
479
Evaluating Your Performance
This ebook was issued to HUAN-LIN CHANG, order #14803770223. Unlawful distribution of this ebook is prohibited.
Customer name HUAN-LIN CHANG , Order Id: 14803770223
Raw Score Verbal Reasoning Scaled Score Quantitative Reasoning Scaled Score
16 144 143
15 143 143
14 142 142
13 141 141
12 140 140
11 139 139
10 138 138
9 137 137
8 135 136
7 134 135
6 132 134
5 130 132
4 130 130
3 130 130
2 130 130
1 130 130
0 130 130
480
GRE Practice Test 2
This ebook was issued to HUAN-LIN CHANG, order #14803770223. Unlawful distribution of this ebook is prohibited.
Customer name HUAN-LIN CHANG , Order Id: 14803770223
Analytical Writing Sample Responses
and Reader Commentaries
SECTION 1
Analytical Writing
ANALYZE AN ISSUE
Some people believe that corporations have a responsibility to promote the well-being of the
societies and environments in which they operate. Others believe that the only responsibility
of corporations, provided they operate within the law, is to make as much money as possible.
Write a response in which you discuss which view more closely aligns with your own
position and explain your reasoning for the position you take. In developing and supporting
your position, you should address both of the views presented.
Score 6 Response*
It is not uncommon for some to argue that, in the world in which we live, corporations
have a responsibility to society and to the environment in which they operate.
Proponents of this view would argue that major environmental catastrophes (e.g.,
the oil spill in the Gulf) are key examples of the damage that can be wrought when
corporations are allowed to operate unchecked. Yet within that very statement lies a
contradiction that undermines this kind of thinking it is necessary for outside forces
to check the behavior of corporations, because we do not expect corporations to
behave in such a manner. In fact, the expectation is simply that corporations will follow
the law, and in the course of doing so, engage in every possible tactic to their advan -
tage in the pursuit of more and greater profit. To expect otherwise from corporations
is to fail to understand their puropose and their very structure.
The corporation arose as a model of business in which capital could be raised
through the contributions of stockholders; investors purchases shares in a company,
and their money is then used as the operating capital for the company. Shareholders
buy stock not because they are hoping to better make the world a better place or
because they have a desire to improve the quality of life but because they expect to see
a return in their investment in this company. The company may itself have generally
altruistic goals (perhaps it is a think tank that advises the government on how to
improve relations with the Middle East, or perhaps it is a company built around finding
alternative forms of energy), but the immediate expectation of the investor is that he
himself will see dividends, or profits, from the investment he has made. This is even
more true in the case of companies that are purely profit driven and which do not have
481
Analytical Writing Sample Responses
*NOTE: All responses are reproduced exactly as written, including errors, misspellings, etc., if
any.
This ebook was issued to HUAN-LIN CHANG, order #14803770223. Unlawful distribution of this ebook is prohibited.
Customer name HUAN-LIN CHANG , Order Id: 14803770223
goals that are particularly directed toward social improvement a description that
applies to the vast majority of corporations.
Is it a bad thing to have a corporation negatively affect the environment (and by
extentsion, its inhabitants)? To pump noxious fumes into the atmosphere as a
by-product of its manufacturing processes? Of course, and this is why agencies such
as the EPA were established and why governments federal, state, and local are
expected to monitor such companies to ensure that such practices fall within the
boundaries of legal expectations. Any and all corporations should be expected to
temper their pursuit of profit with the necessity of following those safeguards that have
been legislated as protections. But the assumption that corporations have an inherent
obligation or responsibility to go above and beyond that to actively PROMOTE the
environment and the well-being of society is absurd.
Engaging in practices to adhere to legal expectations to protect society and the
environment is costly to corporations. If the very purpose of a corporation is to
generate profits, and the obligation to adhere to safety expectations established by law
cuts into those profits, then to expect corporations to embrace such practices beyond
what is required is to presume that they willingly engage in an inherently self-
destructive process: the unnecessary lowering of profits. This is antithetical to the very
concept of the corporation. Treehuggers everywhere should be pleased that
environmental protections exist, but to expect corporations to “make the world a better
place” is to embrace altruism to the point that it becomes delusion.
This is not to say that we should reject efforts to hold corporations accountable. In
fact, the opposite is true we should be vigilant with the business world and maintain
our expectations that corporations do not make their profits at the EXPENSE of the
well-being of society. But that role must be fulfilled by a watchdog, not the corporation
itself, and those expectations must be imposed UPON the corporations, not expected
FROM them.
Reader Commentary
This response receives a 6 for developing an insightful position on the issue in accor-
dance with the assigned task, skillfully weaving a position that takes into consideration
both of the statements in the prompt. Beginning in the first paragraph, the writer
rejects the idea that corporations themselves “have a responsibility to promote the well-
being of the societies and environments in which they operate.” In the second para-
graph, the writer offers compelling reasons for this rejection by discussing the purpose
and structure of corporations. The writer then considers the role of government in pro-
moting corporations’ social and environmental responsibility, developing the position
fully. A cogent statement of the writer’s position appears at the conclusion of the
response: “we should be vigilant with the business world and maintain our expecta-
tions that corporations do not make their profits at the EXPENSE of the well-being of
society. But that role must be fulfilled by a watchdog, not the corporation itself.” The
response as a whole is logically organized, with each paragraph serving as a stepping
stone in the development of the writer’s position. It also demonstrates the writer’s
ability to convey ideas fluently and precisely, using effective vocabulary and sentence
variety. This sentence demonstrates the level of language facility seen throughout the
response: “If the very purpose of a corporation is to generate profits, and the obligation
to adhere to safety expectations established by law cuts into those profits, then to
expect corporations to embrace such practices beyond what is required is to presume
that they willingly engage in an inherently self-destructive process: the unnecessary
482
GRE Practice Test 2
This ebook was issued to HUAN-LIN CHANG, order #14803770223. Unlawful distribution of this ebook is prohibited.
Customer name HUAN-LIN CHANG , Order Id: 14803770223
lowering of profits.” Here the writer has skillfully maintained control of complex syn-
tax and diction while making a logically compelling point. The sentence demonstrates
the outstanding nature of this response.
Score 5 Response
In order to survive, corporations must make money. Successful corporations try and
make as much money as possible. Yet this incentive to make money does not mean
that a corporation can be a detriment to the society in which it operates. Corporations
have a duty and a responsability to ensure the well being of the society in which they
are a part.
Contributing to the well being of a society is actually benefical to a corporation
in many cases. One of these is making sure that workers are well taken care of.
Absenteeism and neglect while on duty are a big problem for corporations, as is
attracting the best workers, who hopefully will lower the risks caused by absenteeism
and neglect. One way that corporations can attract these workers is by offering them
generous benefits. If, for example, an employer includes with employment a good
health care plan, they will be able to attract better workers than one that does not, and
that will aid the corporation greatly. Health care plans provided by employers mean
that these people have at their disposal health coverage, which means that they have
the care they need if they get sick. This also might encourage preventive care,
something that has been shown to reduce the cost and risk of developing other major
ailments.
Another area where corporations providing support for themselves and society is
in the creation of human capital. Globalization and increased education means that
employers need a better educated workforce more than ever. One way that employers
can contribute to this is by sponsoring worker training programs, or paying for their
employees to return to school. This creates a more educated workforce for employers,
as well as may increase the loyalty of employees to an employer. An employee who
received an education sponsored by an employer may be thankful for receiving that
education, and may work harder for that employer. This creates a benefit for employers
and employees.
The main reason that corporations have a duty to contribute to the well being of
society is that they are a part of the society. Even though they have an economic desire
to make a profit, corporations also should think long term about actions they take
which could hurt their company. A good example of this is BP, after the recent oil spill
in the gulf. Their desire to make a profit meant that they did not keep up on all of their
safety regulations and standards, and the result of the then faulty equipment caused a
massive spill. This cost them huge amounts of money to clean up, as well as the fines
they had to pay for causing this. The biggest loss for BP however is that there brand
name will be associated in the US and abroad as the company that caused this giant oil
spill. As the spill was happening, many people boycotted the company, resulting in lost
potential revenue. They may realize that as they lose business to people upset by the
spill, that making sure a spill didn’t happen in the first place was cheaper.
Another reason corporations have to ensure the well-being of a society is that by
makign a society better off, a company may have more consumers. This is especially
true for corporations that sell goods for middle and upper class consumers. If a
corporation tries to bring people up and increase the overall economic well being of
society, they may find that more and more people have to ability to afford their goods.
483
Analytical Writing Sample Responses
This ebook was issued to HUAN-LIN CHANG, order #14803770223. Unlawful distribution of this ebook is prohibited.
Customer name HUAN-LIN CHANG , Order Id: 14803770223
This could generate huge new profits for this corporation, since their pool of potential
consumers has gone up considerably. Concentrating on the long term here means that
corporations can increase their pool of potential consumers.
By denying responsabilty to a society, a corporation is only looking at the possible
short term profits, not the potential long term ones. While in the short term it may work
for a corporation to ignore their societal responsability, it is advantageous in the long
term for the entire corporation to make sure society is getting better. The potential for
new markets, products, production processes and other beneficial factors that come
from promoting well being is quite large. This is something that corportions should be
ready and willing to take advantage of, and something that society should hold them
accountable for.
Reader Commentary
This strong response receives a 5 for its thoughtful, well-developed analysis of the
issue. In this case, the writer argues that corporations do indeed have a responsibility
to promote the well-being of the societies and environments in which they operate,
offering several reasons and well-chosen examples to explain why it is in the interests
of corporations to fulfill these responsibilities. The writer clearly follows the task direc-
tions by addressing the two views provided by the prompt, both explicitly in the open-
ing paragraph and more subtly throughout the response. While the writer clearly
signals at the beginning his or her alignment with the first position (“Corporations
have a duty and a responsability to ensure the well being of the society of which they
are a part”), the paragraphs that follow in fact acknowledge the writer’s opening state-
ment (“In order to survive, corporations must make money”). In areas such as
employee health care and education, as well as in relation to broader issues such as the
environment and the general level of prosperity in society, the writer argues that cor-
porations should strive to meet their social obligations because in the long term, it is
economically advantageous to do so. The various reasons and examples offered are
brought together to support a thoughtful position that implicitly suggests that the two
views are not as mutually exclusive as they might first appear. The response also
demonstrates considerable facility with language. There are some minor errors, but
overall the writer’s control of language is strong, demonstrating sentence variety and
appropriate use of vocabulary. The response lacks the superior fluency and precision of
a 6 but nevertheless conveys meaning clearly and well. Discernibly stronger than the
adequate level of analysis in a 4, the response has thoughtful, nuanced analysis of the
issue that earns it a score of 5.
Score 4 Response
While some people may believe that corporations have a responsibility to protect
society, others believe that the only purpose of a corporation is to make money. I agree
that making profits is important. In the grand scheme of things, though, all companies
have a responsibility to watch out for their customers. Their customers are how they
make their money. If they’re not watching out for their customers, they obviously will
see a drop in their profits.
Consider light bulbs. This is an invention that has all kinds of potential for serious
accidents. It is basically just a glass globe with electricity running through it! If a bulb
gets too hot, it could potentially start a fire. Similarly, if someone removed the glass
484
GRE Practice Test 2
This ebook was issued to HUAN-LIN CHANG, order #14803770223. Unlawful distribution of this ebook is prohibited.
Customer name HUAN-LIN CHANG , Order Id: 14803770223
from around the tungsten wire, you’d basically have an exposed electrical wire that
could hurt anyone who touched it. Makers of light bulbs know and understand all these
dangers. They want consumers to purchase their products, so the first and smartest
way to make that happen is to ensure that the products are safe and thus more
attractive to the customer base. If everyone who used light bulbs was afraid of getting
zapped profits would obviously go down and light bulbs would not be a very profitable
enterprise.
This same thinking applies to all major products. The automobile is one of the most
dangerous tools man uses. Tens of thousands of automobile drivers die every year in
accidents. Insuring that the vehicles contain designs and parts that promote customer
safety is a main focus of car manufacturers. Certain parts of of cars were built with
promoting driver’s well-being in mind. For instance, air bags, anti-lock braking
systems, online crash reporting. These features are considered standard now, and they
were all developed to increase the safety of consumers. These features were not cheap
to develop, but car manufacturers improved their profits anyway because they
developed products with public safety in mind, which is what customers expect. If this
symbiosis relationship wasn’t true, then we would still have cars without airbags or
even seatbelts. Worrying about the safety and actually improving it for customers is
not just a basic responsibility of corporations, but it drives their profits, too.
In conclusion, its pretty clear that a corporation’s desire to make more profits is
in line with a corporation’s responsibility to consumers. Increasing the focus on
consumers, worrying about taking care of them and the environment, can only lead
to bigger profits and success for corporations in the long run.
Reader Commentary
This adequate response follows the task directions and presents a clear position on the
issue. It supports and develops its position competently, using relevant examples. In
accordance with the assigned task, the response addresses both of the competing posi-
tions. Specifically, its position and the examples it develops argue that businesses can
care about both profits and ethical responsibility through the ways they develop prod-
ucts. The development of examples and ideas, while adequate, is not as thoughtful or
compelling as would be needed to earn higher scores. For instance, both of the exam-
ples the response uses are about product safety; the discussion of automobile design
does not advance the position much more than the prior discussion of lightbulb pro-
duction. Language control in the response is also competent. It demonstrates sufficient
control of the conventions of standard written English, and its main points are made
with acceptable clarity. The response features a few grammatical and mechanical
errors (e.g., “Certain parts of of cars . . . ” and “symbiosis relationship”) and some awk-
ward sentences. However, for the 4 range, GRE raters allow for minor errors in
responses like this one that holistically demonstrate sufficient clarity and control. Over-
all, then, this response demonstrates adequate development and control of language,
making the score of 4 appropriate.
485
Analytical Writing Sample Responses
This ebook was issued to HUAN-LIN CHANG, order #14803770223. Unlawful distribution of this ebook is prohibited.
Customer name HUAN-LIN CHANG , Order Id: 14803770223
Score 3 Response
Corporations can be viewed as both beneficial and bad. This statement addresses
both views that many people have about corporations. Views come from personal
experiences and is the reason why some people like corporations and why some
people do not.
Half of this argument deals with people that like the idea of corporations. Some
people believe that corporations help stmiulate societies and promote the well being of
society. These people are ones that have never encountered a corupt corporation. Just
like in any other aspects of life, people can get images of something as being good if
they only brush the outside. It isn’t until they are being faced with a problem within a
particular corporation where they either work for them or have just dealt with them.
Also, people that are benefited from corporations are obviously going to like the idea
of an corporation.
The other side of this argument is that many people believe corporations are just
money hungry. This can be seen in many corporations throughout America. Many
small business owners will side with this argument. The problem lies that this is
corporate America and the little businesses are being taking over by larger corpora -
tions. As Mark Twain once said, “The vast amount of money is only in a couple of
hands”. This statement still lies true today. In addition, corporations are large and with
that being said they lead to more lines of coruption. In small buisnesses, the owners
can oversee their store entirely. Can these corporation owners really oversee every -
thing that is going on? Ask any employee at a corporate office if they believe their
workplace is being ran how they think the corporation would want it. One is likely to
find the answer that it is not.
Just like any other issue there are two sides to the story. The problem with this issue
is that most will agree that corporations are only there to make money. They don’t care
about the people that are helping them make money. They only care at the end of the
day how much money they made.
Reader Commentary
This response receives a score of 3 primarily because it is limited in focus. Rather than
addressing the conflicting views of corporate responsibility given in the prompt, the
response instead incorrectly casts the two positions as “people that like the idea of cor-
porations” and people who “believe corporations are just money hungry.” The writer
proceeds to develop an explanation of these two positions, citing the various qualities
that lead each group of people to their beliefs, but the response concludes by declaring
that “there are two sides to the story” without adopting any position of its own. This
highlights another limitation of the response, the fact that it does not completely
address the specific task directions. Although the response does discuss the two oppos-
ing positions, it never discusses which view more closely aligns with the writer’s own.
The response does contain adequate organization, and the writing demonstrates a suf-
ficient control of language. Sentences such as “Just like in any other aspects of life,
people can get images of something as being good if they only brush the outside” are
typical of the writing in this response and, despite the presence of some errors, demon-
strate a sufficient control of the conventions of standard written English. However,
even though the response demonstrates some qualities of a 4, its problems with focus
and its failure to develop a clear position on the issue in accordance with the assigned
task mean that it merits a score of 3.
486
GRE Practice Test 2
This ebook was issued to HUAN-LIN CHANG, order #14803770223. Unlawful distribution of this ebook is prohibited.
Customer name HUAN-LIN CHANG , Order Id: 14803770223
487
Analytical Writing Sample Responses
Score 2 Response
I think corporations have a responsibility to not only follow the law but also work with
the societies and enviroments they are in. Our societies and environments in this age
are affected by corporations’ operations. An example of this is BP’s accident in the Gulf
of Mexico they might have been following the law and regulations, but once an
accident happens, our societies and environments are affected. Many fishermen and
businesses around the area have been affected by the accident. Now BP faces so many
liabilities and needs to pay money. As a result, they are loosing more than they made
in the past.
It is also important that corporations makes as much as money possible. If they do
well, there might be more employment opportunities for people and more taxes for the
city, states and federal which help our country’s economy better.
However, there are always choices that corporations can take and they can make
money by promoting the well-being of the societies and environments. It might cost
them more but it will also help to save thier expenses.
Reader Commentary
This seriously flawed response attempts to address the task directions by considering
both of the views presented in the prompt. The first paragraph seems to embrace the
first view given in the prompt when it asserts that “corporations have a responsibility
to . . . work with the societies and environments they are in.” The writer uses the exam-
ple of the BP oil spill in the Gulf of Mexico to demonstrate that “our societies and envi-
ronments in this age are affected by corporations’ operations,” but, apart from
restating these same claims, the paragraph provides no real support for this position.
Instead, the writer moves on to a discussion of the financial implications of the oil spill
for BP. The second paragraph then makes an abrupt transition to a discussion of the
prompt’s second view and again seems to embrace this position when it claims that “it
is also important that corporations makes as much as money possible.” This position is
supported with a single relevant but undeveloped reason. Although the writer attempts
to reconcile the two positions in the last paragraph by arguing that “there are always
choices that corporations can take and they can make money by promoting the well-
being of the societies and environments,” the response provides no support for this
position beyond its unsupported and contradictory claim that “it might cost them more
but it will also help to save their expenses.” Overall, then, this response provides few
examples in support of its claims. The response’s poor focus and its very limited sup-
port, then, warrant a score of 2.
This ebook was issued to HUAN-LIN CHANG, order #14803770223. Unlawful distribution of this ebook is prohibited.
Customer name HUAN-LIN CHANG , Order Id: 14803770223
Score 1 Response
It is certainly true that some people believe that corporations have a responsibility to
promote the well being of societies and environments. On the other hand, some other
people argue that the only responsibility of corporations, provided they operate within
the law, is to make as much money as possible. It is easy to see why it would be
difficult for some people to decide between these two positions.
The responsibility of all citizens of a society, including corporate citizens, is
ultimately to further the well being of the society as a whole. It takes little more than
examining the recent United States financial crisis to see the ill effects suffered by
society at large when corporations focus on maximizing profits.
Reader Commentary
This response earns a score of 1 because it provides little evidence of the ability to
develop an organized response. The first paragraph begins with a nearly word-for-word
restatement of the prompt, which increases the length of the response but does not
demonstrate the ability to develop a position on the issue in relation to the specific task
instructions. The final sentence in the first paragraph is analytically empty in that it
could be applied to any prompt that asks writers to discuss two competing positions.
The writer does nothing to relate that sentence to this specific prompt. The second
paragraph, then, is all that the writer has provided in terms of original analysis of the
issue. Although it does demonstrate understanding of the issue, it fits the “extremely
brief” description from the scoring guide description of a 1. Because of the extreme
brevity of its analysis, then, this response merits a score of 1.
488
GRE Practice Test 2
This ebook was issued to HUAN-LIN CHANG, order #14803770223. Unlawful distribution of this ebook is prohibited.
Customer name HUAN-LIN CHANG , Order Id: 14803770223
SECTION 2
Analytical Writing
ANALYZE AN ARGUMENT
The following appeared in a memorandum from the owner of Movies Galore, a chain of
video rental stores.
“In order to reverse the recent decline in our profits, we must reduce operating expenses at
Movies Galore’s ten video rental stores. Since we are famous for our special bargains, raising
our rental prices is not a viable way to improve profits. Last month our store in downtown
Marston significantly decreased its operating expenses by closing at 6:00 P.M. rather than
9:00 P.M. and by reducing its stock by eliminating all movies released more than five years
ago. Therefore, in order to increase profits without jeopardizing our reputation for offering
great movies at low prices, we recommend implementing similar changes in our other nine
Movies Galore stores.”
Write a response in which you discuss what questions would need to be addressed in order to
decide whether implementing the recommendation is likely to have the predicted result and
explain how the answers to those questions would help to evaluate the recommendation.
Score 6 Response
One question which needs to be addressed before implementing the recommendation
is whether there are not other ways to improve profits besides cutting operating
expenses. Without proof, the author decides, first, that there are only two viable
options for increasing the profits of Movies Galore: raising rental prices, and cutting
costs. He rules out the first course, and hence claims the second option must be
chosen. But it seems there may be alternative methods of increasing profits, such as
initiating advertising campaigns or closing unprofitable Movies Galore locations.
Even if it is granted that there are only two options for increasing profitability
cutting costs, and raising rental prices one might wonder why raising rental prices
is so unthinkable. The author implies that because Movies Galore is famous for special
bargains, raising the rental prices would eliminate this competitive advantage and
decrease profitability. However, in making this conclusion, he makes several
assumptions without considering questions that need to be addressed. First, he
assumes that there is no room to raise current prices and yet maintain lower prices
than competitors. One would need to ask if prices could be increased slightly, while
keeping them cheap. Even if there is no room for such a strategy, the author assumes
that Movies Galore’s reputation for bargain pricing would evaporate if they increased
their prices slightly. Perhaps such a reputation would be widespread enough to persist
despite a slight increase in prices. And thirdly, even if the reputation for bargains would
be eliminated by an increase in prices, the author assumes that Movies Galore cannot
489
Analytical Writing Sample Responses
This ebook was issued to HUAN-LIN CHANG, order #14803770223. Unlawful distribution of this ebook is prohibited.
Customer name HUAN-LIN CHANG , Order Id: 14803770223
change course and be successful in some other way. Perhaps it could instead become
known as the store with the friendliest employees. Perhaps it already is, and the author
is wrong to believe that a causal relationship between bargain prices and success
exists, when the real cause of Movies Galore’s good reputation is entirely independent
of its prices. The author needs to answer these questions to convince us that profits are
caused by bargains, and not by the other factors that may be involved.
Another question that needs to be raised is whether or not the downtown Marston
store is truly analogous to the other nine Movies Galore stores. The author seems to
assume that because the cost-cutting measures worked at the Marston location, it will
work at the others, but this is far from clear. Perhaps the patrons of the other Movies
Galore locations would resent such changes in the hours and stock of their local stores.
Perhaps the most important question that needs to be asked is whether the Marston
location’s changes truly increased profitability. The author writes that the Marston store
decreased operating expenses by closing earlier and cutting its stock, but he makes no
mention of increased profitability. It is quite possible that the Marston location’s profits
decreased as a result of their cost cutting, and this is a question that needs to be
addressed. The author then jumps to the conclusion that taking similar measures
would increase profitability at other locations, though such a connection has not even
been established at the Marston store.
Even if the cost-cutting measures increased profitability at the Marston store last
month (and a causal relationship, though presumably assumed, is still far from
evident), there is no guarantee that such measures would continue to increase
profitability over time. One would need to ask: Why not observe how the Marston
location’s action affect profitability over several months, before implementing such
sweeping changes at every store? A single month is a very short time span, and the
habits of customers may change slowly. As word gets around that the Marston store
has cut their hours and their selection, they may in fact jeopardize their reputation for
offering “great movies at low prices.” After all, the name of the franchise is Movies
Galore, and by drastically reducing the available selection, they may alienate their
customer base. If, as mentioned above, Movies Galore is famous for more than its
great bargains if customers prefer Movies Galore because of its selections, as well
then such a move may drastically reduce profits over time. It seems extremely rash to
implement such a new and relatively untried strategy at every Movies Galore location,
before the effects can be fully observed and interpreted.
Reader Commentary
This outstanding response clearly addresses the specific task directions and presents a
cogent, insightful analysis by discussing specific questions that need to be addressed in
order to analyze the argument presented. It is worth noting that although the direc-
tions ask for questions, it is not necessary that these be phrased in question form. The
response articulately discusses the information needed to evaluate whether the recom-
mendation is likely to have the predicted result and demonstrates how this information
would help to evaluate the recommendation. There are many examples of ways in
which the writer addresses questions without putting them in question form (e.g., “he
assumes that there is no room to raise current prices and yet maintain lower prices
than competitors. One would need to ask . . . ” or “it seems there may be alternative
methods of increasing profits”). Throughout this response, the writer provides the
cogent development typical of a 6 response. For example, the discussion of Movies
Galore’s reputation gets at a number of flaws in the argument’s reasoning, making
nuanced points (e.g., “Perhaps it could instead become known as the store with the
490
GRE Practice Test 2
This ebook was issued to HUAN-LIN CHANG, order #14803770223. Unlawful distribution of this ebook is prohibited.
Customer name HUAN-LIN CHANG , Order Id: 14803770223
friendliest employees. Perhaps it already is . . . ”) that create a compelling case for the
writer’s objections. Transitions are natural, and the paragraphs build on one another,
succinctly and completely developing the writer’s points. Because of its compelling and
insightful development and fluent and precise language, this response fits all of the bul-
let points for a score of 6.
Score 5 Response
Management’s prediction that declining profits could be reversed by reducing
operating hours and reducing stock seems to be rash since there is little evidence that
proper research has been conducted. It may be true that profits could be restored by
cutting operating costs, but management needs to ask whether making these changes
would have a negative impact on its best sources of revenue.
The management states that the downtown Marston store “significantly decreased
its operating expenses by closing at 6:00pm rather than 9:00pm.” It is reasonable to
think that closing at 6:00pm rather than 9:00pm would decrease operating expenses,
but the business is concerned with renting movies and these may be the busiest and
most profitable hours of operation. Could it be that most people renting movies have
normal working hours and have leisure time at night and to fill that time they turn to
renting movies? If management researches its daily rental history, it may discover that
its peak rental hours are between 6:00pm and 9:00pm. If this is the case, the store
could lose significant income or even go out of business altogether. If management
wants to reverse a decline in profits by cutting hours of operation and thereby reducing
expenses, it would be adventageous to determine through research which block of
time during the day is the least profitable and then cut those hours of operation. For
instance, if it is found that profits are lowest during the morning hours or around noon,
it would be better to close the store during those hours rather than during the hours
that bring in the greatest profits.
The management then states that operating expenses will also be cut “by
eliminating all movies released ore than five years ago.” Again, more research is
needed in order to determine if this would indeed help reverse the decline in profits
that Movies Galore is experiencing. Is it possible that the success of a movie rental
business is based on its ability to provide customers with a wide array of movie
selections, both new and old? It could be dangerous for this business to eliminate its
stock of older movies without first determining the percentages of income that come
from each product. Management should research its history of movie rentals in order
to determine if a significant percentage of its profits come from the rental of older
movies. Even if little profit does come from older movies, it may still be unwise to
eliminate the stock of older movies. If Movies Galore maintains a variety of movies,
a person searching for a current movie may decide to rent an older movie as well.
This may be especially relevent in the case of a new movie that is a sequal to an older
movie or part of a trilogy. Reducing movie variety may also damage the reputation of
the stores. The management states that Movie Galore already has a “reputation for
offering great movies.” If movie variety is suddenly reduced, the stores may gain a
negative reputation.
Overall, the management makes a prediction that is untrustworthy and potentially
damaging. More research should be conducted to see if indeed such changes to cut
operation costs will be effective, and if not, what should be done instead. If the proper
investigation is implemented by the management, Movies Galore stores may reverse
the recent decline of profits.
491
Analytical Writing Sample Responses
This ebook was issued to HUAN-LIN CHANG, order #14803770223. Unlawful distribution of this ebook is prohibited.
Customer name HUAN-LIN CHANG , Order Id: 14803770223
492
GRE Practice Test 2
Reader Commentary
This strong response presents a generally thoughtful and well-developed analysis of the
argument, and it follows the specific task directions clearly. The response approaches
the task by asking questions that seek to understand whether the management of
Movies Galore truly understands where its greatest profits are generated. It first ques-
tions whether closing the stores between 6:00 and 9:00 pm would be optimal since that
is when people with “normal working hours” might have the leisure to rent movies,
and, then, it questions whether eliminating movies released more than five years ago
would be prudent since it is possible that profits rely on carrying “a wide array of
movie selections, both new and old.” In both cases, the writer indicates the specific
kinds of information that management would need to gather (e.g., “If management
researches its daily rental history, it may discover that its peak rental hours are
between 6:00pm and 9:00pm”) in order to determine whether its prediction is valid.
Likewise, in both cases, the writer also analyzes the ways that this specific information
would impact the predicted result (e.g., “If this is the case, the store could lose signifi-
cant income or even go out of business altogether”). In general, then, development of
the points the writer makes is thorough, but it is not as compelling as that required for
a 6. Also, the response, on the whole, demonstrates facility with written English; how-
ever, it lacks the fluency necessary to merit a score of 6. In general, writing skills, in
spite of a few spelling errors and some repetitious sentence structuring (several sen-
tences in both the second and third paragraphs depend upon an “if . . . then” construc-
tion) are strong, as evidenced by the following characteristic sentence: “It is reasonable
to think that closing at 6:00pm rather than 9:00pm would decrease operating expenses,
but the business is concerned with renting movies and these may be the busiest and
most profitable hours of operation.” In terms of writing skill and analysis, then, this
response earns a score of 5.
Score 4 Response
The owner of Movies Galore is proposing a reduction in operating expenses in all of
its chain stores. This reduction would consist of closing the stores 3 hours early and
reducing its stock to include movies released only within the last 5 years. To determine
if this proposition would be effective in increasing profits across the chain, several
pieces of additional information would be needed.
First, it is necessary to ask if the Marston store is similar to the other Movies Galore
stores. If this is not the case, the proposition might have quite the opposite of the
intended effect. The performance of the other stores would be a critical piece to this
proposition since what works for one store, may not work for another.
Second, what effect did the reduction in operating expenses had on store profits
in Marston? Because the declines in profit are termed to be “recent” and that the
reduction of operating expenses happened within the last month, this is unclear. It is
possible that only a brief period of low profits, consistent with variablity in the market,
spurred the reduction and this actually caused a greater decrease in profits. It is also
possible that no improvements have been seen in profits since the reduction,
rendering it ineffective. Data regarding profits from several months before and several
months after the reduction in operating expenses would be necessary to determine if
this reduction was at all helpful.
This ebook was issued to HUAN-LIN CHANG, order #14803770223. Unlawful distribution of this ebook is prohibited.
Customer name HUAN-LIN CHANG , Order Id: 14803770223
Finally, to what degree did each of the two changes made, closing early and getting
rid of older movies, affect profits? To determine if such a change would be helpful, it is
important to understand how each variable contributed to the end result, assuming
that it was effective. Perhaps closing early resulted in such a decline in the operating
costs as employees did not have to be paid, that the reduction in their stock was
unnecessary. It could be the the stock reduction actually decreased profits but this was
masked by the increased profits caused by closing early. A more in depth analysis of
the variables involved is necessary.
To accept such an extreme change in the practices of these stores, the preceeding
recommendations should be followed. Specifically, the necessity of the reduction in
other stores shoudl be determined, data regarding the effectiveness of the reduction in
operating expenses in the Marston store should be analyzed, and an analysis of the
components of this reduction should be completed.
Reader Commentary
This adequate response presents a competent examination of the argument and con-
veys meaning with acceptable clarity. In accordance with the task directions, the
response raises appropriate questions that could help to evaluate the recommendation
and its predicted result. Unlike the thoughtful development of a 5-level response, how-
ever, this response develops its ideas (i.e., answers to the questions it raises) unevenly,
sometimes underdeveloping key claims. For example, the relatively brief second para-
graph supports the assertion that the downtown Marston store may not be comparable
to the chain’s other stores, but it does so with minimal reasoning. Other body para-
graphs more satisfactorily develop questions about the timing of the recommendation
and the profits that have actually been generated, while the conclusion merely recapit-
ulates the assertions made earlier. A basic organizational structure, aided by the use of
simple transitions between paragraphs and sufficient sentence variety within para-
graphs, are other qualities of this response that underscore its adequacy. The language
control is also adequate, demonstrating control, but not facility, with the conventions
of standard written English. There are some minor grammatical errors and typos (e.g.,
there is a tense error in paragraph 3: “what effect did the reduction in operating
expenses had on store profits in Marston?”; there is also vague diction in the same
paragraph: “Because the declines in profit are termed to be ‘recent’ and that the reduc-
tion of operating expenses happened within the last month, this is unclear”), but the
response manages to convey ideas with acceptable clarity overall. Because of its ade-
quate analytical development and language control, this response earns a score of 4.
Score 3 Response
It is imperative that “Movies Galore” must find a way to reduce operating expenses
without jepardizing its popularity with the customer. The option of reducing operating
hours and reducing its stock of avalible movies is a good start, however these two
ideas need some revising in order for them to be successful in turning the company’s
profits around.
The reduction of hours needs to be reversed. Instead of closing earlier they should
open later. People go to the video store to rent movies more frequently in the evening
hours than in the morning. In the morning is when most customers return movies. The
adjustment in hours can be structured so that the store opens later in the morning, and
493
Analytical Writing Sample Responses
This ebook was issued to HUAN-LIN CHANG, order #14803770223. Unlawful distribution of this ebook is prohibited.
Customer name HUAN-LIN CHANG , Order Id: 14803770223
costomers can simply return the movies in a drop box, allowing the store to remain
open later in the evening for people who want to rent movies.
Reducing the stock by “eliminating all movies released more than five years ago” is
a good way to reduce costs, but again it may hurt buisness. Unlike food moves don’t
go bad after a certain amount of time. In some cases it is quite the opposite, they
become classics. It would be bad business to assume that people will not want to rent
movies over five years old, and “Movies Galore” might actually lose customers if they
do so. Instead of eliminating an entire group of movies of a certain age, spread the
reduction of stock throughout the entire store, making it a more subtle reduction to the
consumer, but an effective cost-saver to the store.
The basic ideas of cutting stock and reducing operating hours do indicate saving
money, however only if it is done correctly with both the business and the consumer in
mind. It is obvious that “Movies Galore” has the consumer in mind because of its
refusal to jeopardize their reputation “for offering great movies at a low price.” So they
need to consider exactly how their proposals are going to impact the consumer and
whether or not they will actually lose business by putting these policies in place.
If teh management at “Movies Galore” uses good business sense then there is no
reason that their declining profits cannot be turned around, with little to no affect on
the consumer.
Reader Commentary
While this response conveys ideas with acceptable clarity, despite an occasional error,
it earns a score in the lower half because it mainly discusses tangential matters. The
introductory paragraph, for example, suggests that instead of presenting an examina-
tion of the prompt’s logic, the response will offer business advice to turn “the com-
pany’s profits around.” And that is what paragraphs 2 and 4 do: they avoid relevant
analysis and instead engage in analyzing tangential matters and generally agreeing
with the prompt. Paragraph 2, for example, implicitly agrees with the reduction in
hours but suggests that a better business tactic would be to switch the early closing for
a later closing and later opening. Paragraph 4 agrees with the overall recommendation
and concludes that Movies Galore is generally going in the right business direction.
What relevant analysis is present occurs in paragraph 3, which questions the wisdom
of reducing the stock of older movies since this might hurt rather than help profitabil-
ity. So, although the response exhibits competent control of the conventions of stan-
dard written English, it does not manage to exhibit adequate development of relevant
analysis. The fact that its relevant analysis is greatly outweighed by tangential material
and business advice indicates that the response is limited in addressing the specific
task directions. Thus, it merits a score of 3.
Score 2 Response
In order to reverse the decrease or decline in the profits it is very necessary to take
some steps which are benefical to both the customer and the seller. Thus to increase
the profits, offering movies at low prices can be one of the ways. This will not only
increase the purchasing ability of the customers but also bring about an increase in
the sales and the profits made by the company. Raising the rental prices of the videos
would not be a better option because this will not lead to an increase in the profits
made by the company. If the customers will get videos in lower price in comparsion
to the rent, they will prefer to purchase more videos then taking them on the rent.
494
GRE Practice Test 2
This ebook was issued to HUAN-LIN CHANG, order #14803770223. Unlawful distribution of this ebook is prohibited.
Customer name HUAN-LIN CHANG , Order Id: 14803770223
The increase in the profits can also be brought about by giving various exciting
offers at different occassions, for example- buy three and get one free video at the time
of Christmas , New Year etc can attract more and more customers towards the stores
and also bring about an increase in the profits earned by the stores.
Increase in the video sales can also be brought about by offering several discount
schemes when the hit and great movies are being released.Thus, in order to increase
the profits without jeopardizing the reputation of the stores , it is recommended to
implement the similar changes as mentioned above in all the other nine Movies Galore
stores.
Reader Commentary
This response clearly fits the second bulleted description of a 2 in the scoring guide. It
does not follow the directions of the assigned task. Instead of discussing questions that
need to be addressed in order to determine if the recommendation is likely to have the
predicted result, the writer merely posits potential strategies to increase profits. The
response also develops ideas poorly and contains serious errors in grammar, usage, and
mechanics, such as in this sentence: “If the customers will get videos in lower price in
comparsion to the rent, they will prefer to purchase more videos then taking them on
the rent.” However, it is the response’s overall disregard of the specific task directions
that earns it a score of 2.
Score 1 Response
Yes reducing the price of the movies would attract more coustmers.No one will be
ready to purchase a movie for a high price rather than that they could see the movie in
a theater.Watching a movie in the theater would be lesser than renting the movie.
It is stated that making special bargains and increasing the rental prices will not
result in yeilding any profits.
They had to shut down their store at 6:00pm instead of 9:00pm
Reader Commentary
This response is fundamentally deficient. Although the response arguably offers some
evidence that the writer understands the basic subject matter, it provides little evidence
of understanding the argument made in the prompt. The “Yes” that begins the first sen-
tence makes it seem as if the writer is responding to a claim made in the prompt, but
the fact that the remainder of the sentence makes a claim that never appears in the
prompt (i.e., “reducing the price of the movies would attract more coustmers”) sug-
gests, at best, a very limited understanding of the argument. Although the brief open-
ing paragraph is somewhat relevant to the argument’s assertion that “raising our rental
prices is not a viable way to improve profits,” it is not sufficient to demonstrate either
that the writer understands the argument or that the writer is able to develop an organ-
ized response. The second paragraph consists almost entirely of verbatim or poorly
paraphrased material from the prompt and, as such, provides no additional evidence
either that the writer understands the argument or that the writer has the ability to
develop an organized response. Thus, despite its relatively clear use of language, this
response demonstrates fundamental deficiencies in analytical writing that warrant
a score of 1.
495
Analytical Writing Sample Responses
This ebook was issued to HUAN-LIN CHANG, order #14803770223. Unlawful distribution of this ebook is prohibited.
Customer name HUAN-LIN CHANG , Order Id: 14803770223
Answers and Explanations
SECTION 3
Verbal Reasoning
25 Questions with Explanations
For questions 1 to 8, select one entry for each blank from the corresponding column
of choices. Fill all blanks in the way that best completes the text.
1. Although plant and animal species that become established in ecosystems where
they did not originate are sometimes referred to by the alarming term “invasive
species,” many such species are ________ in their new environments.
A
innocuous
B
conspicuous
C
robust
D
menacing
E
distinctive
Explanation
The sentence begins with “Although,” indicating that the correct answer will contrast
in tone with the “alarming term ‘invasive species.’” The only answer choice that pro-
vides the necessary contrast is “innocuous.” All the other choices are consistent with
being alarming.
Thus, the correct answer is innocuous (Choice A).
2. Far from being ________ the corporate world because of cutbacks, serious
researchers are playing a growing role in innovation at many firms.
A
lured to
B
enchanted with
C
banished from
D
protected by
E
immured in
Explanation
The words “Far from being” and the mention of “cutbacks” imply that the correct
answer will create a contrast with the idea that “serious researchers are playing a grow-
ing role in innovation.” Being “banished from” the corporate world and playing a grow-
ing role in it are strongly contrasted. Choices A, B, D, and E do not create any such
contrast.
Thus, the correct answer is banished from (Choice C).
496
GRE Practice Test 2
This ebook was issued to HUAN-LIN CHANG, order #14803770223. Unlawful distribution of this ebook is prohibited.
Customer name HUAN-LIN CHANG , Order Id: 14803770223
497
Answers and Explanations
3. The brief survey, published under the title The Work of Nature: How the Diversity
of Life Sustains Us, is surprisingly (i)________. Indeed it makes several longer
treatments of the effects of lost biodiversity seem (ii)________.
Blank (i) Blank (ii)
A
distorted
D
redundant
B
objective
E
pithy
C
comprehensive
F
premature
Explanation
In this question, it is hard to select an answer for either blank in isolation. The “brief
survey,” like any survey, could in fact be surprisingly “distorted,” “objective,” or “com-
prehensive.” The contrast in the following sentence with “longer treatments” suggests
that the brevity of the survey is important. Of the three choices, only “comprehensive”
is particularly unexpected of a brief survey.
Provisionally accepting “comprehensive” makes it easier to analyze the second
blank. If the short survey is surprisingly comprehensive, then longer treatments may
not convey any useful additional information, making them “redundant.” Longer works
are unlikely to seem “pithy” in comparison to shorter ones, and “premature” makes
little sense in this context.
Reading the sentence again with “comprehensive” and “redundant” filling the
blanks confirms that these two choices result in a coherent whole.
Thus, the correct answer is comprehensive (Choice C) and redundant (Choice D).
4. The government has no choice but to (i)________ the incessant demands for land
reform, and yet any governmental action that initiated land reform without
requisite attention to agrarian reform would (ii)________ the overall goal of
economic modernization.
Blank (i) Blank (ii)
A
anticipate
D
delineate
B
heed
E
condone
C
silence
F
compromise
Explanation
The sentence informs us that the government has an “overall goal,” and the use of “and
yet” indicates that initiating land reform without attending to agrarian reform would
have some negative consequence for that goal. Thus, the only answer choice that fits
the second blank is “compromise.” The use of “and yet” also implies that the govern-
ment’s response to the demands for land reform must be in line with initiating such
reform, so Choice C, “silence,” is incorrect. The description of the demands as “inces-
sant” implies that the demands already exist, so “anticipate” is incorrect. Only “heed”
describes an appropriate response.
Thus, the correct answer is heed (Choice B) and compromise (Choice F).
This ebook was issued to HUAN-LIN CHANG, order #14803770223. Unlawful distribution of this ebook is prohibited.
Customer name HUAN-LIN CHANG , Order Id: 14803770223
5. Certain music lovers yearn for (i)________, but when it is achieved, there is
something missing; perhaps they feel uncomfortable in a world where nothing
discernible is (ii)________.
Blank (i) Blank (ii)
A
novelty
D
wrong
B
beauty
E
visionary
C
flawlessness
F
changed
Explanation
The structure of the sentence alerts us that it will describe a kind of contradiction or
paradox: music lovers want something, but when they get what they want, they dis-
cover some cause for dissatisfaction. Among the answer choices, the only two words
that produce such a paradox are “flawlessness” and “wrong” the music lovers long
for flawlessness but are unsatisfied with a world in which nothing is wrong.
Thus, the correct answer is flawlessness (Choice C) and wrong (Choice D).
6. Putting a cash value on the ecological services provided by nature such as the
water filtration “service” provided by a forested watershed has, historically,
been a (i)________ process. Early attempts at such valuation resulted in
impressive but (ii)________ figures that were seized on by environmental
advocates, and then, when these figures were later (iii)________, they were used
by opponents to tar the whole idea.
Blank (i) Blank (ii) Blank (iii)
A
dispassionate
D
redundant
G
ignored
B
problematic
E
unsound
H
discredited
C
straightforward
F
understated
I
confirmed
Explanation
The correct response for the first blank cannot be determined without considering the
second sentence. The correct choice for the second blank, however, can be determined
more readily. Neither Choice D, “redundant,” nor Choice F, “understated,” makes sense
when coupled with the preceding “impressive but.” Since the figures were “used by
opponents to tar the whole idea,” Choice E, “unsound,” is the word that makes the
most sense in this context.
Once “unsound” is selected for the second blank, it follows that “confirmed” cannot
be correct for the third blank. And if the figures were “used by opponents,” then they
cannot have been “ignored.” Since the figures were unsound, it is natural that they
would later be “discredited.”
Now that the figures have been characterized as unsound and discredited, it is pos-
sible to identify the correct response for the first blank. From the second sentence it is
clear that the process of putting a cash value on the ecological services provided by
nature is neither “dispassionate” nor “straightforward.” It is instead “problematic.”
Thus, the correct answer is problematic (Choice B); unsound (Choice E); and dis-
credited (Choice H).
498
GRE Practice Test 2
This ebook was issued to HUAN-LIN CHANG, order #14803770223. Unlawful distribution of this ebook is prohibited.
Customer name HUAN-LIN CHANG , Order Id: 14803770223
499
Answers and Explanations
7. Only with the discovery of an ozone hole over Antarctica in 1985 did chemical
companies finally relinquish their opposition to a ban on chlorofluorocarbons
(CFCs), which destroy ozone. The discovery suggested that strong political action
to halt production of CFCs might be (i)________, and fortunately, the chemical
industry no longer felt compelled to oppose such action: although companies had
recently (ii)________ their research into CFC substitutes, studies they had
initiated years earlier had produced (iii)________ results.
Blank (i) Blank (ii) Blank (iii)
A
imminent
D
corroborated
G
encouraging
B
imprudent
E
publicized
H
inconclusive
C
premature
F
curtailed
I
unsurprising
Explanation
According to the first sentence, chemical companies opposed a ban on CFCs and then
changed their stance in 1985 with the discovery of an ozone hole.
In the first blank, only “imminent” is compatible with the discovery of an ozone
hole linked to CFCs: such a discovery would suggest that “strong political action” is
required, not that it is “imprudent” or “premature.”
What follows the colon in the second sentence explains why the chemical industry
“no longer felt compelled to oppose” a ban on CFCs. In order for the industry to drop
its opposition, the outcome of the studies into CFC substitutes must have been positive.
Among the choices for the third blank, only “encouraging” has a sufficiently positive
connotation.
Finally, the word “although” after the colon indicates that the second blank should
contrast with the third blank in some way. Since the completed third blank now indi-
cates that studies of CFC substitutes have been successful, “curtailed” makes the most
sense in the second blank. “Corroborated” and “publicized” do not contrast appropri-
ately with the success of the studies.
Thus, the correct answer is imminent (Choice A); curtailed (Choice F); and
encouraging (Choice G).
8. The incipient (i)________ regarding taxes could affect trade between the two
countries much more than the (ii)________ banana imports, which has been
going on for years. Unfortunately, the trade regulators seem to be ignoring both
disagreements.
Blank (i) Blank (ii)
A
row
D
profitable dealing in
B
accord
E
predicament regarding
C
investigation
F
festering dispute over
Explanation
The words “both disagreements” at the end of the second sentence indicate that both
blanks should be filled with words or phrases that are synonyms for “disagreement.”
“Row” and “festering dispute over” are the only choices that describe kinds of dis-
agreement. (Note that the word “row” has many meanings, one of which is “a quarrel.”)
Thus, the correct answer is row (Choice A) and festering dispute over (Choice F).
This ebook was issued to HUAN-LIN CHANG, order #14803770223. Unlawful distribution of this ebook is prohibited.
Customer name HUAN-LIN CHANG , Order Id: 14803770223
500
GRE Practice Test 2
For each of questions 9 to 14, select one answer choice unless otherwise instructed.
Questions 9 and 10 are based on the following reading passage.
Fossil bones of the huge herbivorous dinosaurs known as sauropods were first discov-
ered and studied between 1840 and 1880, providing evidence for the gargantuan
dimensions of the adults. The shape of sauropod teeth suggested what they ate. But
aside from trackways, or series of fossilized footprints which established that sauro -
pods at least occasionally lived in herds fossils incorporating direct evidence of other
behavior, such as reproductive behavior, have been almost nonexistent. Because no
modern land animals even approach sauropod size, scientists have also lacked a living
analogue to use as a guide to possible sauropod behavior. Until the recent discovery of
fossilized sauropod nesting grounds, scientists were thus uncertain whether sauropods
laid eggs or gave birth to live young.
Description
The passage outlines what was learned about sauropods after the discovery of their fos-
silized bones in the nineteenth century, including what has been inferred about their
behavior from the fossil record.
For the following question, consider each of the choices separately and select all that
apply.
9. Which of the following can be inferred from the passage regarding the evidence
provided by sauropod teeth?
A
The teeth allow inferences to be made about sauropod social behavior.
B
The shape of the teeth indicates that sauropods were herbivorous.
C
The teeth have no resemblance to those of any modern land animal.
Explanation
Choice B is the only correct answer.
Choice A is incorrect: the passage mentions that fossilized footprints permit the
inference that sauropods exhibited herd behavior, but there is no indication that this or
any other social behavior can be inferred from sauropod teeth.
Choice B is correct: the passage states that sauropods were herbivorous (feeding
on plants) and that “the shape of sauropod teeth suggested what they ate.”
Choice C is incorrect: the passage says that there are no modern land animals sim-
i lar in size to sauropods, not that there are no such animals with similar teeth.
line
5
10
This ebook was issued to HUAN-LIN CHANG, order #14803770223. Unlawful distribution of this ebook is prohibited.
Customer name HUAN-LIN CHANG , Order Id: 14803770223
For the following question, consider each of the choices separately and select all that
apply.
10. Which of the following can be inferred from the passage regarding the recently
discovered fossilized sauropod nesting grounds?
A
They are among the few fossils incorporating direct evidence of sauropod
behavior.
B
They confirm the evidence provided by trackways about sauropod
behavior.
C
They have forced a reevaluation of theories regarding the nature of
sauropod herd behavior.
Explanation
Choice A is the only correct answer.
Choice A is correct: the last sentence of the passage implies that the discovery of
fossilized sauropod nesting grounds resolved the question of whether sauropods
laid eggs or gave birth to live young and therefore provided evidence of sauropods’
reproductive behavior. Until this discovery, except for the trackways that showed herd
behavior, “fossils incorporating direct evidence” of sauropod behavior were “almost
nonexistent” (lines 5–6).
Choice B is incorrect: according to the passage, trackways provided evidence of
herd behavior, whereas fossilized nesting grounds provided evidence concerning repro-
ductive behavior. There is no information in the passage to suggest that the nesting
grounds confirmed evidence provided by the trackways, or even that the trackway evi-
dence needed confirming.
Choice C is incorrect: the passage presents fossilized nesting grounds as providing
evidence about reproductive behavior, not herd behavior. Nothing in the passage sug-
gests that there are theories of the nature of sauropod herd behavior that have been
reevaluated.
501
Answers and Explanations
This ebook was issued to HUAN-LIN CHANG, order #14803770223. Unlawful distribution of this ebook is prohibited.
Customer name HUAN-LIN CHANG , Order Id: 14803770223
502
GRE Practice Test 2
Questions 11 to 14 are based on the following reading passage.
Some researchers contend that sleep plays no role in the consolidation of declarative
memory (i.e., memory involving factual information). These researchers note that peo-
ple with impairments in rapid eye movement (REM) sleep continue to lead normal
lives, and they argue that if sleep were crucial for memory, then these individuals
would have apparent memory deficits. Yet the same researchers acknowledge that the
cognitive capacities of these individuals have never been systematically examined, nor
have they been the subject of studies of tasks on which performance reportedly
depends on sleep. Even if such studies were done, they could only clarify our under-
standing of the role of REM sleep, not sleep in general.
These researchers also claim that improvements of memory overnight can be
ex plained by the mere passage of time, rather than attributed to sleep. But recent
studies of memory performance after sleep including one demonstrating that sleep
stabilizes declarative memories from future interference caused by mental activity
during wakefulness make this claim unsustainable. Certainly there are memory
consolidation processes that occur across periods of wakefulness, some of which
neither depend on nor are enhanced by sleep. But when sleep is compared with wake-
fulness, and performance is better after sleep, then some benefit of sleep for memory
must be acknowledged.
Description
The passage presents and then rebuts two arguments made by researchers who ques-
tion the contribution of sleep to the consolidation of declarative memory (memory
involving factual information). The first argument is that people with impairments to
REM sleep continue to lead normal lives. In response, the passage says that these
researchers themselves acknowledge the absence of systematic study of such individu-
als’ cognitive abilities, study that would be necessary in order to fully support the
researchers’ claim. The passage also points out that the researchers’ claim applies only
to REM sleep rather than to sleep in general. The second claim is that improvements
of memory that occur overnight might be explained merely by the passage of time.
In response, the passage cites research findings that demonstrate the role of sleep in
stabilizing declarative memory.
11. The primary purpose of the passage is to
A
present the evidence that supports a particular claim regarding REM sleep
and memory
B
describe how various factors contribute to the effect of sleep on memory
C
argue against a particular position regarding sleep’s role in memory
D
summarize the most prevalent theory regarding sleep and memory
E
defend the importance of the consolidation of declarative memory
Explanation
As described above, the purpose of the passage as a whole is to argue against the view
held by some researchers that sleep plays no role in the consolidation of declarative
memory. Therefore, Choice C is correct. Choice A is incorrect: the passage does men-
tion REM sleep twice in the first paragraph, but its primary purpose is not to examine
REM sleep in particular, and it does not present evidence related to REM sleep. Choice
B is incorrect: the passage is concerned with the effect of sleep on memory, but not
with any factors that contribute to that effect. Choice D is incorrect: the passage does
line
5
10
15
This ebook was issued to HUAN-LIN CHANG, order #14803770223. Unlawful distribution of this ebook is prohibited.
Customer name HUAN-LIN CHANG , Order Id: 14803770223
not summarize a theory. Instead, it cites a claim and then assesses and rejects that
claim. Choice E is incorrect: although the passage is about the consolidation of declar-
ative memory, it does nothing to defend its importance.
12. According to the author of the passage, which of the following generalizations
about memory and sleep is true?
A
There are some memory-consolidation processes that have nothing to do
with sleep.
B
Sleep is more important to the consolidation of declarative memory than
to the consolidation of other types of memory.
C
REM sleep is more important to memory consolidation than is non-REM
sleep.
D
There are significant variations in the amount of sleep that people require
for the successful consolidation of memory.
E
It is likely that memory is more thoroughly consolidated during
wakefulness than during sleep.
Explanation
The passage states that “there are memory-consolidation processes that occur across
periods of wakefulness.” Accordingly, Choice A is correct. Choices B, C, and D are
incorrect: the passage does not discuss types of memory other than consolidative mem-
ory, the relative importance to consolidative memory of REM and non-REM sleep, or
differences among individuals in the amount of sleep they require. Choice E is also
incorrect: the passage suggests that the truth is the opposite of what this answer choice
states. The last sentence of the passage indicates that performance on memory tasks
has been found to be better after sleep than after periods of wakefulness.
13. Which of the following best describes the function of the sentence in lines 14–16
(“Certainly . . . sleep”)?
A
It provides the reasoning behind a claim about the role of sleep in
memory consolidation.
B
It explains why a previous claim about sleep and memory is
unsustainable.
C
It demonstrates why wakefulness is central to the process of declarative
memory consolidation.
D
It emphasizes the limited role sleep plays in the process of declarative
memory consolidation.
E
It concedes that the consolidation of declarative memory does not depend
entirely on one factor.
Explanation
The cited sentence begins with the word “Certainly,” a clue that the sentence will con-
cede that the researchers are not entirely wrong: in this instance, they are not wrong
about memory consolidation occurring during periods of wakefulness. Thus, Choice E
is correct. Choice A is incorrect: the sentence deals with memory consolidation during
wakefulness, not with the role of sleep in memory consolidation. Choice B is incorrect:
the sentence does follow an assertion that the researchers’ claim is unsustainable, but
it does not explain why it is unsustainable. Choice C is incorrect: the sentence does not
demonstrate anything. It acknowledges that memory consolidation occurs during
503
Answers and Explanations
This ebook was issued to HUAN-LIN CHANG, order #14803770223. Unlawful distribution of this ebook is prohibited.
Customer name HUAN-LIN CHANG , Order Id: 14803770223
wakeful periods but does not identify wakefulness as central to the process. Choice D is
incorrect: while the sentence does acknowledge that some memory-consolidation
processes are not dependent on sleep, it does not go so far as to claim that sleep plays
a limited role in memory consolidation generally.
14. The importance of the study mentioned in lines 12–14 is that it
A
reveals the mechanism by which declarative memory is stabilized during
sleep
B
identifies a specific function that sleep plays in the memory-consolidation
process
C
demonstrates that some kinds of mental activity can interfere with
memory consolidation
D
suggests that sleep and wakefulness are both important to memory
consolidation
E
explains how the passage of time contributes to memory consolidation
Explanation
The question asks what “the importance of the study mentioned in lines 12–14” is. The
study is described as having shown that sleep stabilizes declarative memory from
future interference caused by mental activity during wakefulness. This protection of
memory from interference is the “specific function” played by sleep mentioned in
Choice B. Therefore, Choice B is correct. Choice A is incorrect: there is no description
of any mechanism, or specific process, by which declarative memory is stabilized.
Although Choices C, D, and E each involve issues connected with the study, those con-
nections are all tangential.
For questions 15 to 19, select the two answer choices that, when used to complete
the sentence, fit the meaning of the sentence as a whole and produce completed
sentences that are alike in meaning.
15. In American Indian art, the supposed distinction between modern and traditional
was fabricated by critics, and when artists have control over interpretation of
their own work, the distinction appears, happily, to have been ________.
A
eliminated
B
reinforced
C
put to rest
D
intensified
E
recognized
F
established
Explanation
By characterizing the distinction as “supposed” and “fabricated,” the sentence indicates
that the distinction has no basis in reality. Accordingly, when the sentence reports a
happy outcome, this must mean that the distinction has been abandoned or rejected.
Only “eliminated” and “put to rest” convey that sense; all the other answer choices sug-
gest that the distinction is maintained, or even strengthened.
Thus, the correct answer is eliminated (Choice A) and put to rest (Choice C).
504
GRE Practice Test 2
This ebook was issued to HUAN-LIN CHANG, order #14803770223. Unlawful distribution of this ebook is prohibited.
Customer name HUAN-LIN CHANG , Order Id: 14803770223
16. Notwithstanding their ________ regarding other issues, township residents have
consistently passed the board of education’s annual budget.
A
accord
B
indecision
C
consensus
D
disagreement
E
divergence
F
enthusiasm
Explanation
By using the word “Notwithstanding,” the sentence sets up a contrast between the
township residents’ behavior regarding the “other issues” and their behavior regarding
the board’s annual budget, which they have “consistently passed.” “Accord” and “con-
sensus” are similar in meaning but do not provide the required contrast. Only “dis-
agreement” and “divergence” provide the necessary contrast and lead to two sentences
nearly alike in meaning. “Indecision” fits the context, but there is no other word among
the possible choices that matches it closely.
Thus, the correct answer is disagreement (Choice D) and divergence (Choice E).
17. Some of the company’s supporters charged that the negative report had been
motivated by a broader political assault on the company that was designed to
help market rivals who would like to see the company ________.
A
reined in
B
bolstered
C
indemnified
D
propped up
E
manacled
F
lionized
Explanation
The “market rivals” would clearly like to see the company experience some negative
outcome. Only “reined in” and “manacled” describe such an outcome; the other choices
all describe positive results for the company.
Thus, the correct answer is reined in (Choice A) and manacled (Choice E).
505
Answers and Explanations
This ebook was issued to HUAN-LIN CHANG, order #14803770223. Unlawful distribution of this ebook is prohibited.
Customer name HUAN-LIN CHANG , Order Id: 14803770223
18. Skeptics contend that any scheme for charging visitors to Web sites that rewards
the vendor adequately would require steep prices, ________ the kind of frequent,
casual use of Web sites that surfers now take for granted.
A
bridling
B
exciting
C
forbidding
D
inhibiting
E
provoking
F
reversing
Explanation
The sentence concerns skeptics’ reaction to a plan to generate revenue by charging vis-
itors to Web sites. To justify the skeptics’ reaction, the “steep prices” must be associated
with a decrease in visitor volume. Only “bridling” and “inhibiting” are consistent with
this logic and result in sentences nearly alike in meaning. “Forbidding” is too strong:
steep prices might dissuade a casual visitor, but they would not forbid one. Although
“exciting” and “provoking” can be similar in meaning, they do not fit the logic of the
sentence.
Thus, the correct answer is bridling (Choice A) and inhibiting (Choice D).
19. It seems obvious that Miles Davis’ ________ the Juilliard School, which resulted
in his decision to drop out, was based on the school’s training of musicians for a
kind of music that he did not want to play.
A
disaffection with
B
dislocation of
C
disentanglement from
D
subversion of
E
displacement of
F
estrangement from
Explanation
The sentence asserts a logical relationship between Davis’ attitude toward the school
(as indicated in the blank) and his “decision to drop out.” Only “disaffection with” and
“estrangement from” are consistent with a decision to drop out and result in sentences
nearly alike in meaning.
Thus, the correct answer is disaffection with (Choice A) and estrangement from
(Choice F).
506
GRE Practice Test 2
This ebook was issued to HUAN-LIN CHANG, order #14803770223. Unlawful distribution of this ebook is prohibited.
Customer name HUAN-LIN CHANG , Order Id: 14803770223
507
Answers and Explanations
For each of questions 20 to 25, select one answer choice unless otherwise instructed.
Question 20 is based on the following reading passage.
Astronomers found a large body orbiting close to the star Upsilon Andromedae. The
standard theory of planet formation holds that no planet that large could be formed so
close to a star, leading to the suggestion that the body is a companion star. A subse-
quent discovery puts that suggestion in doubt: two other large bodies were found orbit-
ing close to Upsilon Andromedae, and the standard theory of companion stars allows
for at most one companion star.
20. Which of the following, if true, most helps to resolve the status of the orbiting
body without casting doubt on the two standard theories mentioned?
A
The smaller a planet orbiting a star is, and the farther away it is from the
star, the less likely it is to be discovered.
B
If a planet’s orbit is disturbed, the planet can be drawn by gravity toward
the star it is orbiting.
C
The largest of the bodies orbiting Upsilon Andromedae is the farthest
away from the star, and the smallest is the nearest.
D
It is likely that there are many stars, in addition to Upsilon Andromedae
and the Sun, that are orbited by more than one smaller body.
E
In most cases of companion stars, the smaller companion is much fainter
than the larger star.
Explanation
The passage outlines a conflict between two standard theories one of planet forma-
tion, the other of companion stars and observations of one large body, and later two
others, orbiting close to a star. The question asks what would resolve this conflict with-
out casting doubt on either one of the theories.
Choice B is correct: if, as it asserts, it is possible for a planet to be formed relatively
far from a star and later move closer to it, then the observed large bodies found close
to Upsilon Andromedae can be planets without casting doubt on the standard theory of
planet formation. This explanation also leaves the standard theory of companion stars
intact.
Choice A is incorrect because it describes difficulties with discovering a small
planet far from a star, not anything pertaining to a large body near a star. Choice C is
incorrect as well, since whatever the relative size and position of the three bodies may be,
all three appear to be too close according to the standard theories. Choice D is incor-
rect because the pervasiveness of stars with multiple orbiting bodies has nothing to do
with the status of the large bodies discussed in the passage. Choice E is similarly irrel-
evant and thus incorrect: information about the brightness of a star relative to its com-
panion star does not help clarify the status of the large bodies discussed in the passage.
This ebook was issued to HUAN-LIN CHANG, order #14803770223. Unlawful distribution of this ebook is prohibited.
Customer name HUAN-LIN CHANG , Order Id: 14803770223
Question 21 is based on the following reading passage.
In Gilavia, the number of reported workplace injuries has declined 16 percent in the
last five years. However, perhaps part of the decline results from injuries going unre-
ported: many employers have introduced safety-incentive programs, such as prize
drawings for which only employees who have a perfect work-safety record are eligible.
Since a workplace injury would disqualify an employee from such programs, some
employees might be concealing injury, when it is feasible to do so.
21. Which of the following, if true in Gilavia, most strongly supports the proposed
explanation?
A
In the last five years, there has been no decline in the number of workplace
injuries leading to immediate admission to a hospital emergency room.
B
Employers generally have to pay financial compensation to employees who
suffer work-related injuries.
C
Many injuries that happen on the job are injuries that would be impossible
to conceal and yet would not be severe enough to require any change to
either the employee’s work schedule or the employee’s job responsibilities.
D
A continuing shift in employment patterns has led to a decline in the
percentage of the workforce that is employed in the dangerous occupations
in which workplace injuries are likely.
E
Employers who have instituted safety-incentive programs do not in general
have a lower proportion of reported workplace injuries among their
employees than do employers without such programs.
Explanation
The question asks what would support the claim that the decline in reported workplace
injuries in Gilavia may be the result of incentives for workers to not report those
injuries that they can conceal. If the number of injuries that cannot be concealed
such as injuries requiring immediate emergency care has not declined in the same
period, that could help bolster the claim that the decline in overall reported injuries
may be a result of concealable injuries going unreported rather than an actual decline
in workplace injuries in general, so Choice A is correct.
If employers have to provide financial compensation to employees injured on the
job, employees would have an incentive to report injuries. More reported injuries
would not support the author’s argument, making Choice B incorrect. Choice C is
incorrect because the fact that some injuries that cannot be concealed do not result in
lost time or changed responsibilities has nothing to do with whether concealable
injuries are going unreported. While a decline in dangerous occupations could well
result in a decrease in workplace injuries, this fact would challenge the author’s
argument, not support it, so Choice D is incorrect. Similarly, if employers with safety-
incentive programs do not see any drop in reported injuries compared to employers
without such programs, the author’s argument would be weakened, not supported,
making Choice E incorrect.
508
GRE Practice Test 2
This ebook was issued to HUAN-LIN CHANG, order #14803770223. Unlawful distribution of this ebook is prohibited.
Customer name HUAN-LIN CHANG , Order Id: 14803770223
Questions 22 and 23 are based on the following reading passage.
The attribution of early-nineteenth-century English fiction is notoriously problematic.
Fewer than half of new novels published in Britain between 1800 and 1829 had the
author’s true name printed on the title page. Most of these titles have subsequently
been attributed, either through the author’s own acknowledgment of a previously
anonymous or pseudonymous work or through bibliographical research. One impor-
tant tool available to researchers is the list of earlier works “by the author” often found
on title pages. But such lists are as likely to create new confusion as they are to solve
old problems. Title pages were generally prepared last in the publication process, often
without full authorial assent, and in the last-minute rush to press, mistakes were fre-
quently made.
Description
The passage discusses the reasons why identifying the authors of early-nineteenth-
century British fiction poses significant challenges. The passage explains that few
authors during this period used their real names and goes on to describe how title
pages can facilitate but also hamper efforts to attribute these works.
For the following question, consider each of the choices separately and select all that
apply.
22. The passage suggests that which of the following factors contributes to the
“notoriously problematic” (line 1) nature of authorial attribution in early-
nineteenth-century English fiction?
A
The unwillingness of any writers to acknowledge their authorship of
works that were originally published anonymously or pseudonymously
B
The possibility that the title page of a work may attribute works written by
other authors to the author of that work
C
The possibility that the author’s name printed on a title page is fictitious
Explanation
Choices B and C are correct.
Choice A is incorrect: the passage mentions that the attribution of early-nineteenth-
century fiction was sometimes achieved when the author came forward to acknowl-
edge a previously anonymous work (lines 4–5), so Choice A can be eliminated.
Choice B is correct: in lines 5–7, the passage mentions that “one important tool
available to researchers is the list of earlier works ‘by the author’ often found on title
pages,” but goes on to say that these title pages were prepared hastily and “frequently”
contained mistakes (lines 8–10). Since the mistake most likely to “create new con -
fusion” would be the inclusion of works not written by the author, Choice B may be
inferred.
Choice C is correct: in lines 2–3, the passage states, “Fewer than half of the new
novels published in Britain . . . had the author’s true name printed on the title page.”
Line 5 suggests that pseudonyms fictitious names were commonly used. Hence,
Choice C may be inferred.
509
Answers and Explanations
line
5
10
This ebook was issued to HUAN-LIN CHANG, order #14803770223. Unlawful distribution of this ebook is prohibited.
Customer name HUAN-LIN CHANG , Order Id: 14803770223
For the following question, consider each of the choices separately and select all that
apply.
23. The passage suggests that which of the following is frequently true of the title
pages of early-nineteenth-century English novels?
A
The title page was prepared for printing in a hurried manner.
B
Material on the title page was included without the author’s knowledge or
approval.
C
Information on the title page was deliberately falsified to make the novel
more marketable.
Explanation
Choices A and B are correct.
Choice A is correct: the passage mentions that title pages were prepared last and
that mistakes often occurred “in the last-minute rush to press” (line 9). This indicates
that title pages were often prepared for printing in a hurried manner; hence, Choice A
can be inferred.
Choice B is correct: the passage includes the detail that title pages were often pre-
pared for printing “without full authorial assent” (line 9); hence, Choice B can be
inferred.
Choice C is incorrect: nowhere does the passage speculate about commercial
motives for falsifying information on title pages. Choice C, therefore, cannot be
inferred.
Questions 24 and 25 are based on the following reading passage.
The more definitions a given noun has, the more valuable is each one. Multiple defini-
tions, each subtly different from all the others, convey multiple shades of meaning.
They expand the uses of the word; language is enriched, thought is widened, and inter-
pretations increase or dilate to fill the potentialities of association. The very impossi-
bility of absoluteness in the definition of certain nouns adds to the levels of
connotation they may reach. The inner life of a writer often says more than most read-
ers can know; the mind of a reader can discover truths that go beyond the intent or
perhaps even the comprehension of the writer. And all of it finds expression because a
word can mean many things.
Description
The passage claims that the capacity of words to have multiple meanings can greatly
enhance the resources of a language. The passage describes how this increases possi-
bilities for interpretation and the expression of ideas, thus enriching the relationship
between readers and writers.
24. In the context in which it appears, “shades” (line 2) most nearly means
A
reminders
B
nuances
C
obscurities
D
coverings
E
degrees
510
GRE Practice Test 2
line
5
This ebook was issued to HUAN-LIN CHANG, order #14803770223. Unlawful distribution of this ebook is prohibited.
Customer name HUAN-LIN CHANG , Order Id: 14803770223
Explanation
In the context in which it appears, “shades” is used to refer to the subtle distinctions in
meaning that are made possible by “multiple definitions, each subtly different from all
the others.” Since the sentence deals with multiple definitions conveying meaning,
Choice C (“obscurities”) and the more literal Choice D (“coverings”) may be eliminated.
In deciding between Choice B and Choice E, one should bear in mind that the sentence
focuses on subtle differences in meaning as opposed to different degrees of emphasis
for the same meaning. Choice B, “nuances,” best captures this sense of slight varia-
tions in meaning and is therefore the correct answer.
25. The passage suggests that a writer’s use of nouns that have multiple definitions
can have which of the following effects on the relationship between writer and
reader?
A
It can encourage the reader to consider how the writer’s life might have
influenced the work.
B
It can cause the reader to become frustrated with the writer’s failure to
distinguish between subtle shades of meaning.
C
It can allow the reader to discern in a work certain meanings that the
writer did not foresee.
D
It allows the writer to provide the reader with clues beyond the word itself
in order to avoid ambiguity.
E
It allows the writer to present unfamiliar ideas to the reader more
efficiently.
Explanation
Lines 7–8 clearly indicate that multiple meanings of words enable readers to “discover
truths that go beyond the intent or perhaps even the comprehension of the writer”;
hence, Choice C is the correct answer. Choices A, B, D, and E all deal with topics that
are not mentioned in the passage: the writer’s life, the reader’s frustration, the avoid-
ance of ambiguity, and the question of how efficiently multiple definitions can aid in
the presentation of unfamiliar ideas.
511
Answers and Explanations
This ebook was issued to HUAN-LIN CHANG, order #14803770223. Unlawful distribution of this ebook is prohibited.
Customer name HUAN-LIN CHANG , Order Id: 14803770223
SECTION 4
Verbal Reasoning
25 Questions with Explanations
For questions 1 to 8, select one entry for each blank from the corresponding column
of choices. Fill all blanks in the way that best completes the text.
1. The unexplained digressions into the finer points of quantum electrodynamics
are so ________ that even readers with a physics degree would be wise to keep a
textbook handy to make sense of them.
A
uninteresting
B
controversial
C
unsophisticated
D
frustrating
E
humorless
Explanation
An initial reading of this sentence might suggest that the blank should be filled with a
word like “complex” that indicates how hard it is to “make sense of” the digressions.
However, there is no such word among the answer choices. Focusing on the second
half of the sentence suggests a different interpretation. According to the sentence, it
would be “wise to” make sense of the digressions, and a textbook would help the reader
to do so. If the digressions are “uninteresting,” “unsophisticated,” or “humorless,” the
sentence provides no reason to think it would be wise to make sense of them, and if
they are “controversial,” it provides no reason to think that a textbook would help. Only
if the digressions are “frustrating” does the sentence make a coherent whole.
Thus, the correct answer is frustrating (Choice D).
2. The belief that politicians might become ________ after their election to office led
to the appointment of ethics officers at various levels of government.
A
scrupulous
B
entrenched
C
venal
D
puzzled
E
artificial
Explanation
If a certain belief led to the appointment of ethics officers, that belief must concern
some ethical issue. Of the choices provided, only “venal” fits that context. Although sev-
eral of the other choices are not necessarily positive characteristics, none of them
involves ethics.
Thus, the correct answer is venal (Choice C).
512
GRE Practice Test 2
This ebook was issued to HUAN-LIN CHANG, order #14803770223. Unlawful distribution of this ebook is prohibited.
Customer name HUAN-LIN CHANG , Order Id: 14803770223
3. Even the charisma and technical prowess of two fine actors are not ________ the
task of fully invigorating a gray domestic drama with a tired tale to tell.
A
required for
B
interested in
C
preferred for
D
adequate to
E
inferior to
Explanation
The “task” described in the second half of the sentence clearly presents the actors with
a challenge. The “even the” followed by positive characteristics indicates that the actors
did not meet that challenge; “adequate to” is the only answer choice that conveys this
sense.
Thus, the correct answer is adequate to (Choice D).
4. There may be a threshold below which blood pressure reductions become
________ given that a long-running study showed no decreased heart risk for
drops in blood pressure below a certain point.
A
worthwhile
B
indiscernible
C
arduous
D
significant
E
superfluous
Explanation
The portion of the sentence that begins with “given that” provides a reason for a con-
clusion reached in the first part of the sentence. Since the study “showed no decreased
heart risk for drops in blood pressure below a certain point,” that point may be a
threshold below which reductions in blood pressure provide no benefit; that is, they
may be “superfluous.”
Thus, the correct answer is superfluous (Choice E).
513
Answers and Explanations
This ebook was issued to HUAN-LIN CHANG, order #14803770223. Unlawful distribution of this ebook is prohibited.
Customer name HUAN-LIN CHANG , Order Id: 14803770223
514
GRE Practice Test 2
5. Unlike the problems in recent financial scandals, issues raised by the regulators
in this case appear largely to pertain to unwieldy accounting rules that are open
to widely divergent interpretations not to (i)________ transactions designed to
(ii)________ corporate malfeasance.
Blank (i) Blank (ii)
A
sham
D
cloak
B
unpremeditated
E
ameliorate
C
justifiable
F
illuminate
Explanation
The “Unlike” at the beginning of the sentence and the “not to” that follows the dash set
up a contrast between the relatively innocent problems in the current case and the
issues involved in the “recent financial scandals.” Clearly, these latter issues must have
involved wrongdoing. Looking at the second blank, only transactions designed to
“cloak” corporate malfeasance would qualify: both ameliorating and illuminating
malfeasance are positive actions. For the first blank, only “sham” fits; “unpremedi-
tated” or “justifiable” transactions could not be designed to cloak malfeasance.
Thus, the correct answer is sham (Choice A) and cloak (Choice D).
6. Everyone has routines that govern their work. The myth is that artists are
somehow different, that they reject (i)________, but of course that’s not true: most
artists work as the rest of us do, (ii) ________, day by day, according to their own
customs.
Blank (i) Blank (ii)
A
latitude
D
impetuously
B
habit
E
ploddingly
C
materialism
F
sporadically
Explanation
The passage conveys the sense that artists are like everyone else in that they have “rou-
tines that govern their work.” This view is contrasted with a myth that artists are
“somehow different.” In the first blank, only “habit” is something whose rejection pre -
sents a contrast with being governed by work routines. Rejecting “latitude” might well
match being governed by work routines, and though “materialism” is sometimes
rejected by artists, it is not relevant to having work routines. The second blank
describes how artists “work as the rest of us do”; only “ploddingly” is consistent with
the emphasis on routines and “day by day” work.
Thus, the correct answer is habit (Choice B) and ploddingly (Choice E).
This ebook was issued to HUAN-LIN CHANG, order #14803770223. Unlawful distribution of this ebook is prohibited.
Customer name HUAN-LIN CHANG , Order Id: 14803770223
515
Answers and Explanations
7. Repression of painful memories is sometimes called “willed forgetting.” Yet true
forgetting is (i)________ than the phenomenon of repressed memory. In spite of
the effort that it (ii)________, repressing unwanted memories is less (iii) ________
than truly forgetting them, for repressed memories are prone to come back.
Blank (i) Blank (ii) Blank (iii)
A
less controlled
D
eases
G
permanent
B
different in its effect
E
conveys
H
arduous
C
far more common
F
entails
I
immediate
Explanation
This question is best answered by first completing the third blank.
The third sentence sets up a comparison between repressing memories and forget-
ting them. The word “for” indicates that the last part of the sentence “repressed
memories are prone to come back” presents the basis of that comparison. Choice G,
“permanent,” is the only choice that is related to the tendency to come back.
Working backward, the sentence begins with “In spite of,” suggesting that the
correct choice for the second blank is contrary to what one might expect. One would
ordinarily expect that something entailing effort would be more rather than less per-
manent. Neither “eases” nor “conveys” sets up such an expectation.
Filling the second and third blanks makes it possible to fill the first blank. Nothing
in the completed text suggests that true forgetting is “more common” or “less con-
trolled” than the repression of painful memories, but it does suggest that true forget-
ting is different in its effect it is more permanent. Thus, Choice B, “different in its
effect,” is correct.
Thus, the correct answer is different in its effect (Choice B), entails (Choice F),
and permanent (Choice G).
8. Rather than viewing the Massachusetts Bay Colony’s antinomian controversy as
the inevitable (i)________ of the intransigent opposing forces of radical and
(ii)________ beliefs, male and female piety, (iii)________ and secular power, and
the like, as other critics have, Winship argues that the crisis was not “fixed and
structural.”
Blank (i) Blank (ii) Blank (iii)
A
dissolution
D
revolutionary
G
clerical
B
melding
E
orthodox
H
civil
C
collision
F
questionable
I
cerebral
Explanation
The words “Rather than” indicate that the other critics, unlike Winship, think of the
controversy as “fixed and structural.” Since both “dissolution” and “melding” of
“intransigent opposing forces” would tend to lessen the controversy, only “collision”
(Choice C) fits the first blank. The second and third blanks appear in a series of exam-
ples of such opposing forces; only “orthodox” contrasts with “radical” in the second
blank and only “clerical” contrasts with “secular” in the third blank.
Thus, the correct answer is collision (Choice C), orthodox (Choice E), and clerical
(Choice G).
This ebook was issued to HUAN-LIN CHANG, order #14803770223. Unlawful distribution of this ebook is prohibited.
Customer name HUAN-LIN CHANG , Order Id: 14803770223
For each of questions 9 to 14, select one answer choice unless otherwise instructed.
Questions 9 to 12 are based on the following reading passage.
Until recently, many anthropologists assumed that the environment of what is now the
southwestern United States shaped the social history and culture of the region’s indige-
nous peoples. Building on this assumption, archaeologists asserted that adverse envi-
ronmental conditions and droughts were responsible for the disappearances and
migrations of southwestern populations from many sites they once inhabited.
However, such deterministic arguments fail to acknowledge that local environmen-
tal variability in the Southwest makes generalizing about that environment difficult. To
examine the relationship between environmental variation and sociocultural change in
the Western Pueblo region of central Arizona, which indigenous tribes have occupied
continuously for at least 800 years, a research team recently reconstructed the climatic,
vegetational, and erosional cycles of past centuries. The researchers found it impossi-
ble to provide a single, generally applicable characterization of environmental condi-
tions for the region. Rather, they found that local areas experienced different patterns
of rainfall, wind, and erosion, and that such conditions had prevailed in the Southwest
for the last 1,400 years. Rainfall, for example, varied within and between local valley
systems, so that even adjacent agricultural fields can produce significantly different
yields.
The researchers characterized episodes of variation in southwestern environments
by frequency: low-frequency environmental processes occur in cycles longer than one
human generation, which generally is considered to last about 25 years, and high-
frequency processes have shorter cycles. The researchers pointed out that low-frequency
processes, such as fluctuations in stream flow and groundwater levels, would not usu-
ally be apparent to human populations. In contrast, high-frequency fluctuations such
as seasonal temperature variations are observable and somewhat predictable, so that
groups could have adapted their behaviors accordingly. When the researchers com-
pared sequences of sociocultural change in the Western Pueblo region with episodes of
low- and high-frequency environmental variation, however, they found no simple cor-
relation between environmental process and sociocultural change or persistence.
Although early Pueblo peoples did protect themselves against environmental risk and
uncertainty, they responded variously on different occasions to similar patterns of high-
frequency climatic and environmental change. The researchers identified seven major
adaptive responses, including increased mobility, relocation of permanent settlements,
changes in subsistence foods, and reliance on trade with other groups. These findings
suggest that groups’ adaptive choices depended on cultural and social as well as envi-
ronmental factors and were flexible strategies rather than uncomplicated reactions to
environmental change. Environmental conditions mattered, but they were rarely, if ever,
sufficient to account for sociocultural persistence and change. Group size and compo-
sition, culture, contact with other groups, and individual choices and actions were
barring catastrophes such as floods or earthquakes more significant for a population’s
survival than were climate and environment.
Description
The passage describes research that bears on a presumed historical relationship
between environmental variation and sociocultural change among indigenous people
of the southwestern United States. The author mentions in the first paragraph that
many anthropologists believed until recently that environmental variations explain
516
GRE Practice Test 2
line
5
10
15
20
25
30
35
40
This ebook was issued to HUAN-LIN CHANG, order #14803770223. Unlawful distribution of this ebook is prohibited.
Customer name HUAN-LIN CHANG , Order Id: 14803770223
changes in the human populations of the region. The passage then goes on to point out
studies that show problems with this explanation, including the lack of generally appli-
cable characterizations of the environment in the region and lack of correlation
between environmental changes and sociocultural changes. In the final paragraph the
author mentions an alternative explanation in researchers’ findings suggesting that
responses to environmental changes varied according to differing factors such as group
size and composition, culture, contact with other groups, and individual choices.
9. The passage is primarily concerned with
A
explaining why certain research findings have created controversy
B
pointing out the flaws in a research methodology and suggesting a
different approach
C
presenting evidence to challenge an explanation and offering an
alternative explanation
D
elucidating the means by which certain groups have adapted to their
environment
E
defending a long-held interpretation by presenting new research findings
Explanation
As the description above indicates, Choice C is the best answer: the passage introduces
an explanation, presents evidence that challenges it, and offers an alternative explana-
tion. The passage does not mention the creation of controversy or discuss flaws in
research methodology; therefore, Choices A and B are incorrect. Although the passage
reports findings that different groups used different adaptive responses to environ-
mental conditions, there is no focus on the adaptations used by particular groups, so
Choice D is incorrect. The passage presents recent research findings but not in defense
of a long-held interpretation; therefore, Choice E is incorrect.
10. Which of the following findings would most strongly support the assertion made
by the archaeologists mentioned in line 3?
A
A population remained in a certain region at least a century after erosion
wore away much of the topsoil that sustained grass for their grazing
animals.
B
The range of a certain group’s agricultural activity increased over a
century of gradual decrease in annual rainfall.
C
As winters grew increasingly mild in a certain region, the nomadic
residents of the region continued to move between their summer and
winter encampments.
D
An agricultural population began to trade for supplies of a grain instead of
producing the grain in its own fields as it had in the past.
E
A half century of drought and falling groundwater levels caused a certain
population to abandon their settlements along a riverbank.
Explanation
The archaeologists mentioned in line 3 asserted that adverse environmental conditions
caused southwestern populations to move or disappear. The question asks which find-
ing would support this assertion.
Choices A, B, and C all describe populations that did not move away or disappear
in the face of environmental changes, and hence are all incorrect. Choice D is incorrect
517
Answers and Explanations
This ebook was issued to HUAN-LIN CHANG, order #14803770223. Unlawful distribution of this ebook is prohibited.
Customer name HUAN-LIN CHANG , Order Id: 14803770223
because it does not mention a change in environmental conditions and therefore can-
not support an assertion about the effects of changing environmental conditions.
Choice E is the best answer: it mentions an adverse environmental change (a long
drought) that caused a population to leave the site it had inhabited, which would sup-
port the archaeologists’ assertion that such environmental changes caused such popu-
lation changes.
11. The fact that “adjacent agricultural fields can produce significantly different
yields” (lines 16–17) is offered as evidence of the
A
unpredictability of the climate and environment of the southwestern
United States
B
difficulty of producing a consistent food supply for a large population in
the Western Pueblo region
C
lack of water and land suitable for cultivation in central Arizona
D
local climatic variation in the environment of the southwestern United
States
E
high-frequency environmental processes at work in the southwestern
United States
Explanation
Choice D is the correct answer: the second paragraph says rainfall variations between
local valleys cause different agricultural yields between adjacent fields and gives this
as an example of how climate is not uniform within the Southwest but rather can vary
significantly from place to place. Choice A is incorrect: while such variability might
give rise to unpredictability, that is not how the difference in agricultural yields is being
used as evidence in the passage. Choices B and C are incorrect: the passage does not
make or report a claim about feeding large populations, nor does it assert that central
Arizona lacks land suitable for cultivation. Choice E is incorrect: a discussion of high-
and low-frequency processes occurs in the third paragraph, but the author does not
present geographic differences in rainfall and agricultural yield as either a high- or a
low- frequency environmental process.
12. It can be inferred from the passage that which of the following activities is NOT
an example of a population responding to high-frequency environmental
processes?
A
Developing watertight jars in which to collect and store water during the
rainy season
B
Building multistory dwellings in low-lying areas to avoid the flash
flooding that occurs each summer
C
Moving a village because groundwater levels have changed over the last
generation
D
Trading with other groups for furs from which to make winter clothes.
E
Moving one’s herds of grazing animals each year between summer and
winter pastures
Explanation
The phrasing of the question indicates that all but one of the answer choices are exam-
ples of a population responding to a high-frequency environmental process. You are
asked to choose the one answer choice that does not provide such an example. Choices
518
GRE Practice Test 2
This ebook was issued to HUAN-LIN CHANG, order #14803770223. Unlawful distribution of this ebook is prohibited.
Customer name HUAN-LIN CHANG , Order Id: 14803770223
A, B, D, and E are incorrect because they all present responses to high-frequency envi-
ronmental processes: developing water-storage jars to adapt to seasonal rainfall varia-
tions, adapting dwellings in response to seasonal flooding, trading to acquire clothing
in adaptation to seasonal temperature variations, and moving grazing herds seasonally.
Choice C is the best answer: the passage mentions fluctuations in ground water levels
as a low-frequency process (lines 21–22); moving a village because of a change that
takes place over the course of a generation is not a response to a high-frequency
process.
Questions 13 and 14 are based on the following reading passage.
Arctic sea ice comes in two varieties. Seasonal ice forms in winter and then melts in
summer, while perennial ice persists year-round. To the untrained eye, all sea ice looks
similar, but by licking it, one can estimate how long a particular piece has been float-
ing around. When ice begins to form in seawater, it forces out salt, which has no place
in the crystal structure. As the ice gets thicker, the rejected salt collects in tiny pockets
of brine too highly concentrated to freeze. A piece of first-year ice will taste salty. Even-
tually, if the ice survives, these pockets of brine drain out through fine, veinlike chan-
nels, and the ice becomes fresher; multiyear ice can even be melted and drunk.
Description
The passage describes two varieties of Arctic sea ice and explains how the freezing
process causes seasonal ice to taste much saltier than perennial ice.
For the following question, consider each of the choices separately and select all that
apply.
13. The passage mentions which of the following as being a characteristic of seasonal
ice?
A
It is similar in appearance to perennial ice.
B
It is typically filled with fine, veinlike channels.
C
It tastes saltier than perennial ice.
Explanation
Choices A and C are correct.
Choice A is correct: the passage states that “to the untrained eye, all sea ice looks
similar” (lines 2–3).
Choice B is incorrect: it is clear that perennial ice contains fine, veinlike channels,
but the passage does not mention whether seasonal ice contains them.
Choice C is correct: in lines 6–8, the passage establishes that first-year ice tastes
salty but eventually gets fresher if the ice survives.
519
Answers and Explanations
line
5
This ebook was issued to HUAN-LIN CHANG, order #14803770223. Unlawful distribution of this ebook is prohibited.
Customer name HUAN-LIN CHANG , Order Id: 14803770223
520
GRE Practice Test 2
14. In the context in which it appears, “fine” (line 7) most nearly means
A
acceptable
B
elegant
C
precise
D
pure
E
small
Explanation
“Fine” appears in the context of an explanation of how the brine drains out; in such
a context, it must be being used to describe a physical characteristic of the channels.
In addition, the word “Eventually” implies that the draining is a slow process. Only
Choice E, “small,” helps to explain why the process is slow and is therefore the best
choice. None of the other choices contributes to the explanation.
For questions 15 to 18, select the two answer choices that, when used to complete
the sentence, fit the meaning of the sentence as a whole and produce completed
sentences that are alike in meaning.
15. It would have been disingenuous of the candidate to appear ________ when her
opponent won the election, but she congratulated the victor nonetheless.
A
gracious
B
ecstatic
C
crestfallen
D
indifferent
E
euphoric
F
disgruntled
Explanation
To answer the question, one must understand what sort of reaction on the part of a los-
ing candidate would appear “disingenuous.” Certainly “ecstatic” and “euphoric” reac-
tions would be highly disingenuous or insincere. “Gracious” also fits the blank, but
there is no other word offered that is nearly alike in meaning.
Thus, the correct answer is ecstatic (Choice B) and euphoric (Choice E).
16. As market forces penetrate firms and bid up the value of attributes of labor that
are more measurable than is the knowledge born of experience, it can be
expected that trends in wages will not ________ those whose main value lies in
such experiential knowledge.
A
favor
B
aid
C
affect
D
forsake
E
betray
F
differentiate
This ebook was issued to HUAN-LIN CHANG, order #14803770223. Unlawful distribution of this ebook is prohibited.
Customer name HUAN-LIN CHANG , Order Id: 14803770223
Explanation
The sentence states that market forces are bidding up the value of certain attributes of
labor that are “more measurable than is the knowledge born of experience.” The blank
has to do with trends in wages for those whose main value in the labor force lies in
“experiential knowledge.” Since experiential knowledge appears to be losing value in
the bidding war for labor, the blank needs to be filled in a way that leads to something
negative. Given the “not” that precedes the blank, “favor” and “aid” make for such an
outcome and result in sentences alike in meaning.
Thus, the correct answer is favor (Choice A) and aid (Choice B)
17. The point we might still take from the First World War is the old one that wars
are always, as one historian aptly put it, ________: they produce unforeseeable
results.
A
unsurprising
B
astounding
C
conventional
D
ruinous
E
stunning
F
devastating
Explanation
The colon after the blank indicates a definitional relationship between the blanked
word and the phrase that follows the colon. The two answer choices for which “they
produce unforeseeable results” would most clearly serve as a definition are “astound-
ing” and “stunning.” While “ruinous” and “devastating” might be adjectives describing
the effects of war, they clearly do not fit the logical structure of this sentence, since they
are not by definition “unforeseeable.”
Thus, the correct answer is astounding (Choice B) and stunning (Choice E).
18. This is the kind of movie stuffed with intimations of faraway strife and
people in suits talking frantically on cell phones and walkie-talkies that is
conven tionally described as a political thriller, but the film is as apolitical as
it is ________.
A
intense
B
unprecedented
C
subtle
D
humdrum
E
refined
F
dull
Explanation
The sentence suggests that the film is not well described by the conventional term
“political thriller.” The film is not political but rather apolitical, and the phrase “as apo-
litical as it is . . .” sets up a parallel between “apolitical” and the blanked word; therefore,
the blanked word should go against the term “political thriller” in the same way that
“apolitical” does. “Humdrum” and “dull” are the opposite of “thrilling” and are there-
fore the best choices.
Thus, the correct answer is humdrum (Choice D) and dull (Choice F).
521
Answers and Explanations
This ebook was issued to HUAN-LIN CHANG, order #14803770223. Unlawful distribution of this ebook is prohibited.
Customer name HUAN-LIN CHANG , Order Id: 14803770223
For each of questions 19 to 25, select one answer choice unless otherwise instructed.
Questions 19 and 20 are based on the following reading passage.
Historians credit repeated locust invasions in the nineteenth century with reshaping
United States agriculture west of the Mississippi River. Admonished by government
entomologists, farmers began to diversify. Wheat had come to nearly monopolize the
region, but it was particularly vulnerable to the locusts. In 1873, just before the locusts’
most withering offensive, nearly two-thirds of Minnesota farmland was producing
wheat; by the invasions’ last year, that fraction had dropped to less than one-sixth.
Farmers learned that peas and beans were far less vulnerable to the insects, and corn
was a more robust grain than wheat. In addition to planting alternative crops, many
farmers turned to dairy and beef production. Although pastures were often damaged
by the locusts, these lands were almost always left in better shape than the crops were.
Description
The passage explains how the damage caused by repeated invasions of locusts in the
nineteenth century caused farmers west of the Mississippi River to diversify. Since
wheat, the dominant crop in the region, was especially susceptible to damage from
locusts, it made sense for farmers to lower their wheat production and raise their pro-
duction of other crops and animals less vulnerable to locust invasions.
For the following question, consider each of the choices separately and select all that
apply.
19. According to the passage, before the recommendations by the government
entomologists, which of the following was true about farming west of the
Mississippi River?
A
Farmers focused primarily on growing wheat.
B
Peas and beans had not yet been planted in the region.
C
A relatively small portion of farmland was devoted to crops other than
wheat.
Explanation
Choices A and C are correct.
Choice A is correct: according to the passage, “wheat had come to nearly monopo-
lize the region” prior to the recommendations of government entomologists.
Choice B is incorrect: although wheat was the dominant crop, there is no indica-
tion that peas and beans had not been planted in the region prior to the admonish-
ments of government entomologists.
Choice C is correct: given that wheat was the dominant crop, only a relatively
small portion of farmland could have been devoted to other crops.
522
GRE Practice Test 2
line
5
10
This ebook was issued to HUAN-LIN CHANG, order #14803770223. Unlawful distribution of this ebook is prohibited.
Customer name HUAN-LIN CHANG , Order Id: 14803770223
20. In the context in which it appears, “robust” (line 8) most nearly means
A
crude
B
demanding
C
productive
D
vigorous
E
rich
Explanation
In discussing the advantages of less vulnerable crops, the author describes corn as
“robust.” Of the choices presented, “vigorous” is most similar in meaning to “robust.”
Neither “crude” nor “demanding” is an advantage, and although being “productive” or
“rich” might be desirable, neither matches the meaning of “robust” in this context.
Therefore, Choice D is the correct answer.
Question 21 is based on the following reading passage.
In 1998 the United States Department of Transportation received nearly 10,000 con-
sumer complaints about airlines; in 1999 it received over 20,000. Moreover, the num-
ber of complaints per 100,000 passengers also more than doubled. In both years the
vast majority of complaints concerned flight delays, cancellations, mishandled bag-
gage, and customer service. Clearly, therefore, despite the United States airline indus-
try’s serious efforts to improve performance in these areas, passenger dissatisfaction
with airline service increased significantly in 1999.
21. Which of the following, if true, most seriously weakens the argument?
A
Although the percentage of flights that arrived on time dropped slightly
overall, from 77 percent in 1998 to 76 percent in 1999, some United States
airlines’ 1999 on-time rate was actually better than their 1998 on-time
rate.
B
The number of passengers flying on United States airlines was
significantly higher in 1999 than in 1998.
C
Fewer bags per 1,000 passengers flying on United States airlines were lost
or delayed in 1999 than in 1998.
D
The appearance in 1999 of many new Internet sites that relay complaints
directly to the Department of Transportation has made filing a complaint
about airlines much easier for consumers than ever before.
E
Although the number of consumer complaints increased for every major
United States airline in 1999, for some airlines the extent of the increase
was substantial, whereas for others it was extremely small.
Explanation
The passage describes two different year-over-year increases in airline passenger com-
plaints: both the absolute number of complaints and the rate of complaints more than
doubled from 1998 to 1999. From these facts, the author of the passage concludes that
passenger dissatisfaction with airline service significantly increased in the same period.
Choice D is the correct answer: it weakens the argument because it presents a sce-
nario in which the increase in complaints and in the rate of complaints could merely be
the result of an easier means of filing complaints, not an actual increase in passenger
dissatisfaction.
523
Answers and Explanations
This ebook was issued to HUAN-LIN CHANG, order #14803770223. Unlawful distribution of this ebook is prohibited.
Customer name HUAN-LIN CHANG , Order Id: 14803770223
Choice A and Choice C are incorrect because neither presents a scenario that bears
directly on the claim that passenger dissatisfaction increased from 1998 to 1999.
Choice E is incorrect: the fact that some airlines experienced a smaller increase than
others does not change the fact that all airlines experienced an increase and thus can-
not weaken the argument. Choice B could be correct if the passage discussed only the
change in the absolute number of complaints, since more passengers could account for
more complaints without entailing an increase in dissatisfaction. But the passage also
says that the rate of complaints increased, making Choice B incorrect.
Questions 22 to 24 are based on the following reading passage.
Nineteenth-century architect Eugène-Emmanuel Viollet-le-Duc contended that Paris’s
Notre-Dame cathedral, built primarily in the late twelfth century, was supported from
the very beginning by a system of flying buttresses a series of exterior arches (flyers)
and their supports (buttresses) which permitted the construction of taller vaulted
buildings with slimmer walls and interior supports than had been possible previously.
Other commentators insist, however, that Notre-Dame did not have flying buttresses
until the thirteenth or fourteenth century, when they were added to update the build-
ing aesthetically and correct its structural flaws. Although post-twelfth-century modi-
fications and renovations complicate efforts to resolve this controversy all
pre-fifteenth-century flyers have been replaced, and the buttresses have been rebuilt
and/or resurfaced it is nevertheless possible to tell that both the nave and the choir,
the church’s two major parts, have always had flying buttresses. It is clear, now that
nineteenth-century paint and plaster have been removed, that the nave’s lower but-
tresses date from the twelfth century. Moreover, the choir’s lower flyers have chevron
(zigzag) decoration. Chevron decoration, which was characteristic of the second half of
the twelfth century and was out of favor by the fourteenth century, is entirely absent
from modifications to the building that can be dated with confidence to the thirteenth
century.
Description
The passage describes a disagreement about when Notre-Dame cathedral was sup-
ported by flying buttresses, with Viollet-le-Duc arguing that buttresses were present
from the cathedral’s construction in the late twelfth century and others claiming the
buttresses were built later. The author of the passage goes on to present evidence that
suggests that Viollet-le-Duc’s argument is correct.
22. The passage is primarily concerned with
A
tracing the development of a controversy
B
discussing obstacles to resolving a controversy
C
arguing in support of one side in a controversy
D
analyzing the assumptions underlying the claims made in a controversy
E
explaining why evidence relevant to a controversy has been overlooked
Explanation
As the description above indicates, Choice C is correct: the passage supports one side
in a controversy. Choice A is incorrect because while the passage describes a contro-
versy, it makes no mention of how that controversy developed. The passage also does
not discuss any obstacles to resolving the controversy, any assumptions underlying the
524
GRE Practice Test 2
line
5
10
15
This ebook was issued to HUAN-LIN CHANG, order #14803770223. Unlawful distribution of this ebook is prohibited.
Customer name HUAN-LIN CHANG , Order Id: 14803770223
525
Answers and Explanations
claims in the controversy, or any reasons why pertinent evidence may have been over-
looked, so Choice B, Choice D, and Choice E are all incorrect.
23. The claim of the “other commentators” (line 6) suggests that they believe which
of the following about Notre-Dame?
A
It was the inspiration for many vaulted cathedrals built in the thirteenth
and fourteenth centuries.
B
Its design flaws were not apparent until flying buttresses were added in
the thirteenth or fourteenth century.
C
Its flying buttresses are embellished with decoration characteristic of the
thirteenth and fourteenth centuries.
D
It had been modified in some respects before flying buttresses were added
in the thirteenth or fourteenth century.
E
It was originally constructed in an architectural style that was considered
outmoded by the thirteenth or fourteenth century.
Explanation
The passage states that the “other commentators” claim that Notre-Dame first received
flying buttresses when it was updated for aesthetic and structural reasons in the thir-
teenth or fourteenth century. This claim thus suggests that the aesthetics of Notre-
Dame were then seen as out of date, making Choice E correct. Choice A is incorrect
because the passage does not include any information about other cathedrals, let alone
attribute a view of them to the other commentators. While the other commentators do
suggest that the design of Notre-Dame was seen as flawed in the thirteenth or four-
teenth century, they say that flying buttresses were added to correct these flaws, not
that the flaws became apparent after the addition of the flying buttresses, which makes
Choice B incorrect. Choice C is incorrect because the passage does not attribute any
views of the embellishments on the flying buttresses to the other commentators; simi-
larly, Choice D is incorrect because the passage does not describe the other commenta-
tors as discussing any modifications prior to the thirteenth or fourteenth century.
24. The author’s argument concerning Notre-Dame’s flying buttresses depends on
which of the following assumptions about the choir’s lower flyers?
A
They accurately reproduce the decoration on the choir’s original lower
flyers.
B
They have a type of decoration used exclusively for exterior surfaces.
C
They were the models for the choir’s original upper flyers.
D
They were the models for the nave’s original lower flyers.
E
They were constructed after the nave’s flyers were constructed.
Explanation
The author supports the claim that flying buttresses were present on Notre-Dame from
the twelfth century by noting that the choir’s lower flyers feature a chevron decoration
that was characteristic of the twelfth century. But since all flyers constructed prior to
the fifteenth century have been replaced, the chevron decorations can indicate only
that flyers were present in the twelfth century if those decorations accurately repro-
duce the decorations that existed on the original flyers. Thus, Choice A is the correct
answer.
Choice B is incorrect: whether chevron decorations are used only on the exterior is
This ebook was issued to HUAN-LIN CHANG, order #14803770223. Unlawful distribution of this ebook is prohibited.
Customer name HUAN-LIN CHANG , Order Id: 14803770223
not a point of dispute in the passage. Choices C, D, and E are all incorrect: no part of
the argument turns on any claim about the choir’s upper flyers, the nave’s lower flyers,
or the sequence in which the choir’s and the nave’s flyers were constructed.
Question 25 is based on the following reading passage.
The average temperature of the lobster-rich waters off the coast of Foerkland has been
increasing for some years. In warmer water, lobsters grow faster. In particular, lobster
larvae take less time to reach the size at which they are no longer vulnerable to preda-
tion by young cod, the chief threat to their survival. Consequently, the survival rate of
lobster larvae must be going up, and the lobster population in Foerkland’s coastal
waters is bound to increase.
25. Which of the following, if true, most seriously weakens the argument?
A
There are indications that in recent years the fishing fleet operating off the
coast of Foerkland has been taking cod at an unsustainably high rate.
B
The increase in water temperatures off Foerkland has not been as
pronounced as the increase in average soil temperatures in Foerkland.
C
Because of their speeded-up growth, lobsters now get large enough to be
legal catch before they reach reproductive maturity.
D
Even though lobsters grow faster in warmer waters, warmer waters have
no effect on the maximum size to which a lobster can eventually grow.
E
Cod are a cold-water species, and the increasing water temperatures have
caused a northward shift in Foerkland’s cod population.
Explanation
The argument in the passage concludes that the survival rate of lobster larvae is
increasing and that the lobster population will increase. The basis for the conclusion is
that the change in the water temperature, by speeding the growth of lobster larvae, has
made them less vulnerable to predation by cod. However, Choice C points to a way that
the faster growth of individual lobsters could create a threat to the population: lobsters
that have not yet reproduced might be large enough to be legally caught. Thus, Choice C
weakens the argument and is the correct answer.
Among the other choices, Choice A tends, if anything, to support the passage’s con-
clusion, by suggesting further reduction in the risks of predation by cod. The other
choices have no clear bearing on the argument.
526
GRE Practice Test 2
This ebook was issued to HUAN-LIN CHANG, order #14803770223. Unlawful distribution of this ebook is prohibited.
Customer name HUAN-LIN CHANG , Order Id: 14803770223
Answers and Explanations
527
SECTION 5
Quantitative Reasoning
25 Questions with Explanations
AQuantity A is greater.
BQuantity B is greater.
CThe two quantities are equal.
DThe relationship cannot be determined from the information given.
Quantity A Quantity B
1.
1
3
1
4
4
3
ABCD
Explanation
In this question, you are asked to compare with Recall that if ais a
1
34
.
1
43
nonzero number, then and Using these rules of exponents, you
11
1
a==a.
1
aa
can see that
1
3114
1
=(3 ) =(4) =
11
冢冣
4433
Thus, and the correct answer is Choice C.
1
34
=,
1
43
andx<1x0
Quantity A Quantity B
2.
2
x+1
3
x+1ABCD
Explanation
In this question, you are given that and and you are asked tox<1x0,
compare with One way to approach this problem is to set up a
23
x+1x+1.
comparison between the two quantities using a placeholder symbol to represent
the relationship between them as follows.
23
x+1?x+1
| |
Then simplify the comparison.
Step 1: Subtract 1 from both sides to get
23
x?x
| |
Step 2: Since you can divide both sides by the positive quantity to get
2
x0, x
1?x
| |
This ebook was issued to HUAN-LIN CHANG, order #14803770223. Unlawful distribution of this ebook is prohibited.
Customer name HUAN-LIN CHANG , Order Id: 14803770223
GRE Practice Test 2
528
Since you are given that or you can conclude that the placeholder
x<1, 1 >x,?
| |
in the simplified comparison represents greater than (>). Note that the1?x
| |
strategy of simplifying the comparison requires you to consider whether the
steps in the simplification are reversible. This is because you must arrive at a
conclusion about the initial comparison, not the simplified comparison. If you
follow the simplification steps in reverse, you can see that the placeholder in
each step remains unchanged: implies because multiplying by the
23
1>xx>x
positive number retains the inequality greater than (>). Also, implies
223
xx>x
Therefore, Quantity A is greater than Quantity B, and the correct
23
x+1>x+1.
answer is Choice A.
x>0
Quantity A Quantity B
3. 0.5% of x1x
2
ABCD
Explanation
In this question, you are given that , and you are asked to compare 0.5% ofx>0
xwith Recall that of a number is the same as 50% of the number.
11
x.
22
Therefore, is equal to 50% of x. Since it follows that 50% of xis greater
1xx>0,
2
than 0.5% of x. The correct answer is Choice B.
You could also make the comparison by rewriting 0.5% of xas a fraction of x
and then comparing the result with Rewrite 0.5% of xas a fraction of xas
1x.
2
follows.
0.5 5
x=x
100 1,000
Since it follows that is less than and Quantity B is greater than
51
x>0, xx,
1,000 2
Quantity A. The correct answer is Choice B.
The median income of a group of College Cgraduates
six months after graduation was $3,000 higher than the
median income of a group of College Dgraduates six
months after graduation.
Quantity A Quantity B
4. The 75th percentile of the
incomes of the group of
College Cgraduates six
months after graduation
The 75th percentile of the
incomes of the group of
College Dgraduates six
months after graduation
ABCD
Explanation
In this question, you are asked to compare the 75th percentiles of the incomes
of two groups of college graduates six months after graduation. The only
This ebook was issued to HUAN-LIN CHANG, order #14803770223. Unlawful distribution of this ebook is prohibited.
Customer name HUAN-LIN CHANG , Order Id: 14803770223
Answers and Explanations
529
information you are given is that the median income of the group in Quantity A
is $3,000 greater than the median income of the group in Quantity B.
Recall that the median of a group of numbers is the middle number (or the
average of the two middle numbers) when the numbers are listed from least to
greatest. The median is also equal to the 50th percentile. The median does not
indicate anything about the spread of the numbers in the group. In particular,
for each group of incomes, you do not know how much greater than the median
the 75th percentile of the group of incomes is, nor do you know the relationship
between the 75th percentiles of the two groups. Since the relationship between
Quantity A and Quantity B cannot be determined, the correct answer is Choice D.
Frequency
15
10
5
0123456
24710
16
11
The graph above shows the frequency distribution of 50
integer values varying from 1 to 6.
Quantity A Quantity B
5. The average (arithmetic
mean) of the 50 values
The median of the 50
values
ABCD
Explanation
In this question, you are given a graph of the frequency distribution of 50 integer
values and are asked to compare the average (arithmetic mean) with the median
of the distribution.
In general, the median of a group of nvalues, where nis even, is obtained by
ordering the values from least to greatest and then calculating the average
(arithmetic mean) of the two middle values. So, for the 50 values shown in the
graph, the median is the average of the 25th and 26th values, both of which are
equal to 5. Therefore, the median of the 50 values is 5.
Once you know that the median of the 50 values is 5, the comparison
simplifies to comparing the average of the 50 values with 5. You can make this
comparison without actually calculating the average by noting from the graph
that of the 50 values,
11 values are 1 unit above 5,
16 values are equal to 5,
10 values are 1 unit below 5, and
13 values are more than 1 unit below 5.
Since the part of the distribution that is below 5 contains 23 values13 of
which are more than 1 unit below 5and the part of the distribution that is
above 5 contains 11 valuesnone of which is more than 1 unit above 5the
average (arithmetic mean) of the 50 values must be less than 5. The correct
answer is Choice B.
Alternatively, you can calculate the average of the 50 values as follows.
(1)(2) +(2)(4) +(3)(7) +(4)(10) +(5)(16) +(6)(11) 217
=
50 50
This ebook was issued to HUAN-LIN CHANG, order #14803770223. Unlawful distribution of this ebook is prohibited.
Customer name HUAN-LIN CHANG , Order Id: 14803770223
GRE Practice Test 2
530
Thus the average of the 50 values, or 4.34, is less than the median of the 50
217,
50
values, 5. The correct answer is Choice B.
3
P
R
Q
S
5
52
Quantity A Quantity B ABCD
6. The area of triangle PQR The area of triangle PSR
Explanation
In this question, you are asked to compare the area of triangle PQR with the
area of triangle PSR. Note that both triangles are right triangles and that line
segment PR is the hypotenuse of both triangles. Recall that the area of a triangle
is equal to one-half the product of a base and the height corresponding to the
base. Also, for any right triangle, the lengths of the two legs of the triangle are a
base and the corresponding height.
The area of triangle PQR : In the figure, it is given that the length of leg PQ is
and the length of leg QR is Therefore, you can conclude that the area of
25 5.
冪冪
triangle PQR is or 5.
1
(
25
)(
5
)
,
冪冪
2
The area of triangle PSR : To calculate the area of triangle PSR, you need to
know the lengths of the legs PS and RS. From the figure, you know that the
length of RS is 3, but you do not know the length of PS. How can you determine
the length of PS ? If, in addition to the length of RS, you knew the length of
hypotenuse PR, you could use the Pythagorean theorem to determine the length
of PS. So, to find the length of PS, you first need to find the length of hypotenuse
PR.
Recall that PR is also the hypotenuse of triangle PQR. The lengths of legs PQ
and QR of triangle PQR are and respectively. By the Pythagorean
25 5,
冪冪
theorem,
22 2
(PR)=(PQ)+(QR)
22
=25 +5
(
)(
)
=20 +5
=25
Thus, the length of PR is or 5.
25,
This ebook was issued to HUAN-LIN CHANG, order #14803770223. Unlawful distribution of this ebook is prohibited.
Customer name HUAN-LIN CHANG , Order Id: 14803770223
Answers and Explanations
531
Returning to triangle PSR, you now know that the length of hypotenuse PR
is 5 and the length of leg RS is 3. Therefore, by the Pythagorean theorem,
222
3+(PS)=5
2
9+(PS)=25
2
(PS)=25 9
2
(PS)=16
and the length of PS is 4.
Since legs PS and RS have lengths 4 and 3, respectively, the area of triangle
PSR is or 6. Recall that you have already determined that the area of
1(4)(3),
2
triangle PQR is 5. So Quantity B, the area of triangle PSR, is greater than
Quantity A, the area of triangle PQR, and the correct answer is Choice B.
Quantity A Quantity B
7. The sum of the odd
integers from 1 to 199
The sum of the even
integers from 2 to 198
ABCD
Explanation
In this question, you are asked to compare the sum of the odd integers from 1 to
199 with the sum of the even integers from 2 to 198. Both of these sums involve
many integers. How many integers are in each sum? Note that there are 200
integers from 1 to 200, where 100 of them are even and 100 of them are odd.
The 100 odd integers are precisely the odd integers in Quantity A, whereas the
100 even integers include one more integer, 200, than the even integers in
Quantity B. So Quantity A is the sum of 100 integers and Quantity B is the sum
of 99 integers.
It would be very time-consuming to write out all the terms in each sum and
add them together. Therefore, it is reasonable to find a more efficient way to
calculate the sums or to find a way to compare the sums without actually
calculating them. To find a more efficient way to calculate the two sums, it is
often useful to look for ways to rearrange the terms in the sum so that they can
be added more easily. You can begin by writing a few terms from the beginning
and the end of the sum.
For the sum of the 100 odd integers from 1 to 199, you could write
1+3+5+...+195 +197 +199
You can pair the odd integers in the sum and add the two integers in each pair
as follows.
1 + 3 + 5 + . . . + 195 + 197 + 199
200
200
200
Note that the sum of the integers in each of the three pairs shown is 200. You
can continue pairing terms in the sum in this way until all 100 terms have been
rearranged in 50 pairs, where the sum of each pair is 200. It follows that
1+3+5+...+195 +197 +199 =(1 +199) +(3 +197) +(5 +195) +...+(99 +101)
=50(200)
=10,000
This ebook was issued to HUAN-LIN CHANG, order #14803770223. Unlawful distribution of this ebook is prohibited.
Customer name HUAN-LIN CHANG , Order Id: 14803770223
GRE Practice Test 2
532
Now consider the sum of the 99 even integers from 2 to 198. For this sum, you
could write
2+4+6+...+194 +196 +198
In this sum, note that
the sum of the 1st and 99th terms is 2 +198 =200
the sum of the 2nd and 98th terms is 4 +196 =200
You can continue pairing terms in this way until 98 of the 99 terms in the sum
have been rearranged into 49 pairs and the 50th term is unpaired. Note that the
unpaired term is 100 (the 50th positive even integer). It follows that
2+4+...+98 +100 +102 +...+196 +198 =(2 +198) +(4 +196) +...+(98 +102) +100
=49(200) +100
=9,900
Therefore, Quantity A, 10,000, is greater than Quantity B, 9,900, and the correct
answer is Choice A.
Alternatively, you can try to compare the two sums without actually
calculating them. Recall that Quantity A is the sum of the 100 odd integers from
1 to 199, and Quantity B is the sum of the 99 even integers from 2 to 198. Write
each sum with the terms in increasing order, as follows, pairing the nth term in
Quantity B with the nth term in Quantity A and noting that there is no term in
Quantity B that is paired with the 100th term, 199, in Quantity A.
Quantity A: 1 +3+5+...+193 +195 +197 +199
Quantity B: 2 +4+6+...+194 +196 +198
Note that each of the 99 terms in Quantity B is 1 more than its paired term in
Quantity A, so Quantity B is 99 more than the sum of all the terms in Quantity A
excluding the last term, 199. Consequently, Quantity A is more than199 99
Quantity Bthat is, 100 more than Quantity Band the correct answer is
Choice A.
sand tare positive integers, and
st
32 =2.
Quantity A Quantity B
8. s
t
1
5
ABCD
Explanation
In this question, you are given that sand tare positive integers and that
st
32 =2,
and you are asked to compare with Since the expression involves the
s1s
.
t5t
variables sand t, you need to look for a relationship between sand tusing the
equation
st
32 =2.
If the two bases in this equation were equal, then the exponents would be
equal. However, one of the bases is 32 and the other is 2. This suggests making
the two bases equal by rewriting 32 as a power of 2 if it is possible to do so. In
fact, Therefore, and the equation can be
5s5s5sst
32 =2. 32=(2 ) =2, 32=2
rewritten as In the rewritten equation, the bases are equal, so you can
5st
2=2.
conclude that 5s=t.
This ebook was issued to HUAN-LIN CHANG, order #14803770223. Unlawful distribution of this ebook is prohibited.
Customer name HUAN-LIN CHANG , Order Id: 14803770223
Answers and Explanations
533
Since it follows that Quantity A is equal to Quantity B, and the
s1
5s=t,=.
t5
correct answer is Choice C.
In a quality-control test, 50 boxeseach containing 30
machine partswere examined for defective parts. The
number of defective parts was recorded for each box,
and the average (arithmetic mean) of the 50 recorded
numbers of defective parts per box was 1.12. Only one
error was made in recording the 50 numbers: “1”
defective part in a certain box was incorrectly recorded
as “10”.
Quantity A Quantity B
9. The actual average number
of defective parts per box
0.94 ABCD
Explanation
In this question, you are given that the number of defective parts in each of 50
boxes was recorded and that the average of the 50 recorded numbers was 1.12.
You are also given that an error was made in recording one of the 50 numbers
the number 10 was recorded instead of the number 1so the actual number of
defective parts in this box is 9 less than the recorded number. Then you are
asked to compare the actual average number of defective parts per box with 0.94.
To determine the actual average number of defective parts per box, first note
that the sum of the 50 recorded numbers equals the average of the 50 recorded
numbers times 50that is, or 56.
(1.12)(50),
Now you know that for 49 of the 50 boxes, the actual number of defective
parts is equal to the recorded number; and for one box, the actual number is 9
less than the recorded number. From this you can conclude that the sum of the
50 actual numbers is equal to the sum of the 50 recorded numbers minus 9. So
the sum of the actual numbers of defective parts is or 47.
56 9,
Therefore, the actual average number of defective parts per box is or
47,
50
0.94. Quantity A is equal to Quantity B, and the correct answer is Choice C.
10. In year Y, the population of Colorado was approximately half that of New
Jersey, and the land area of Colorado was approximately 14 times that of
New Jersey. The population density (number of persons per unit of land
area) of Colorado in year Ywas approximately how many times the
population density of New Jersey?
A1
28
B1
14
C1
7
D1
4
E1
2
This ebook was issued to HUAN-LIN CHANG, order #14803770223. Unlawful distribution of this ebook is prohibited.
Customer name HUAN-LIN CHANG , Order Id: 14803770223
GRE Practice Test 2
534
Explanation
The information given in the question can be rewritten algebraically as follows.
1
population of Colorado (population of New Jersey)
冢冣
2
land area of Colorado (14) (land area of New Jersey)
Using the information given and the fact that population density is the number
of persons per unit of land area, you can express the population density of
Colorado in terms of the population density of New Jersey as follows.
population of Colorado
population density of Colorado =land area of Colorado
1(population of New Jersey)
冢冣
2
(14) (land area of New Jersey)
1 1 population of New Jersey
⳯⳯
冢冣 冢 冣 冢
2 14 land area of New Jersey
1
(population density of New Jersey)
冢冣
28
Thus, the population density of Colorado was approximately times the
1
28
population density of New Jersey. The correct answer is Choice A.
For the following question, enter your answer in the box.
35°
(2yk
m
11. In the figure above, line kis parallel to line m. What is the value of y?
y=
Explanation
In the figure, the angles with measures and are between parallel lines(2y)35
kand m, and they are on opposite sides of the line that crosses kand m.
Therefore, you can conclude that these two angles are congruent. So 2y=35,
and The correct answer is 17.5.
35
y==17.5.
2
This ebook was issued to HUAN-LIN CHANG, order #14803770223. Unlawful distribution of this ebook is prohibited.
Customer name HUAN-LIN CHANG , Order Id: 14803770223
Answers and Explanations
535
12. The numbers in data set Shave a standard deviation of 5. If a new data set
is formed by adding 3 to each number in S, what is the standard deviation
of the numbers in the new data set?
A2
B3
C5
D8
E15
Explanation
Recall that the standard deviation of the numbers in a data set is a measure of
the spread of the numbers about the mean of the numbers. The new data set is
formed by adding the same number, 3, to each number in data set S. Thus, the
mean of the numbers in the new data set is 3 more than the mean of the
numbers in S, but the spread of the numbers in the new data set about the mean
of the numbers in the new data set is the same as the spread of the numbers in
Sabout the mean of the numbers in S. Because the standard deviation of the
numbers in Sis 5, the standard deviation of the numbers in the new data set is
also 5. The correct answer is Choice C.
13. If which of the following could be the value of y?
2y33y
=,
y2
A4
B1
C1
D3
E5
Explanation
One approach to answer the question is to solve the equation for yas follows.
2(2y3) =y(3 y)
2
4y6=3yy
2
y+y6=0
(y+3)(y2) =0
Since a product equals 0 only if at least one of the factors equals 0,
y+3=0ory2=0
y=−3ory=2
Thus, there are two values of ythat satisfy the equation, and 2. The value33
is Choice D, and the value 2 is not among the answer choices. The correct
answer is Choice D.
This ebook was issued to HUAN-LIN CHANG, order #14803770223. Unlawful distribution of this ebook is prohibited.
Customer name HUAN-LIN CHANG , Order Id: 14803770223
GRE Practice Test 2
536
Another approach is to determine, for each answer choice, whether the
equation holds. To do this, you can substitute the answer choice for yin the
equation replace the equals sign in the equation by the place-
2y33y
=,
y2
holder symbol , and then simplify to see whether the two expressions are in
?
=
fact equal.
For Choice A, substituting in the equation and replacing
2y33y
y=4=
y2
the equals sign with the placeholder symbol yields the relationship
?
==
which can be simplified as follows.
?
2(4) 334
=,
42
?
2(4) 334
=
42
?
831
=
42
?
51
=−
42
Since , you can conclude that the placeholder symbol does not represent
51
42
equality, and therefore the equation does not hold for y=4.
If you continue evaluating the answer choices, you will find that the correct
answer is Choice D, To see that the equation is true when
2y33y
3. =
y2
substitute in the equation and replace the equals sign with they=−3, y=−3=
placeholder symbol . This yields the relationship This
??
2(3) 33(3)
==.
32
relationship can be simplified as follows.
?
2(3) 33(3)
=
32
?
633+3
=
32
?
96
=
32
?
3=3
Since you can conclude that the placeholder symbol represents equality,3 =3,
and therefore the equation holds for The correct answer is Choice D.y=−3.
For the following question, select all the answer choices that apply.
14. List Kconsists of the numbers 0, 5, and 10. Which of the following10, 5,
lists of numbers have the same range as the numbers in list K?
Indicate all such lists.
A0, 1, 1515, 1,
B1, 137, 4, 2,
C0, 1, 2, 5, 8, 10
D2, 3, 5, 15, 19, 22
E4, 5, 6, 24
This ebook was issued to HUAN-LIN CHANG, order #14803770223. Unlawful distribution of this ebook is prohibited.
Customer name HUAN-LIN CHANG , Order Id: 14803770223
Answers and Explanations
537
Explanation
Recall that the range of a list of numbers is defined as the difference between the
greatest number and the least number in the list. The greatest number in list K
is 10 and the least number is Therefore, the range of the numbers in list K10.
is So, to answer the question, you need to consider10 (10) =10 +10 =20.
each list of numbers given in the choices and determine whether that list of
numbers has a range of 20.
Note that in each of the choices, the numbers are listed in order from least
to greatest. Therefore, you need to look only at the first number and last number
in each list to determine which lists have a range of 20. The ranges can be
calculated quickly as follows.
Choice A: The greatest number is 15 and the least number is therefore,15;
the range is 15 (15) =15 +15 =30.
Choice B: The greatest number is 13 and the least number is therefore,7;
the range is 13 (7) =13 +7=20.
Choice C: The greatest number is 10 and the least number is 0; therefore, the
range is 10 0=10.
Choice D: The greatest number is 22 and the least number is 2; therefore, the
range is 22 2=20.
Choice E: The greatest number is 24 and the least number is 4; therefore, the
range is 24 4=20.
In each of Choices B, D, and E, the range is 20. The correct answer consists
of Choices B, D, and E.
15. Aisha’s income in 2004 was 20 percent greater than her income in 2003.
What is the ratio of Aisha’s income in 2004 to her income in 2003 ?
A1to5
B5to6
C6to5
D5to1
E20 to 1
Explanation
Because Aisha’s income in 2004 was 20% greater than her income in 2003, her
income in 2004 was equal to
(100% of her income in 2003) +(20% of her income in 2003)
which is 120% of her income in 2003. Therefore, the ratio of her income in 2004
to her income in 2003 is 120 to 100, which is equivalent to 6 to 5. The correct
answer is Choice C.
Alternatively, to say that Aisha’s income in 2004 was 20% greater than her
income in 2003 is the same as saying that her income increased by Therefore,
1.
5
her income in 2004 was of her income in 2003, and the ratio of her income in
6
5
2004 to her income in 2003 is 6 to 5. The correct answer is Choice C.
This ebook was issued to HUAN-LIN CHANG, order #14803770223. Unlawful distribution of this ebook is prohibited.
Customer name HUAN-LIN CHANG , Order Id: 14803770223
GRE Practice Test 2
538
16. Jacob’s weekly take-home pay is ndollars. Each week he uses dollars for
4n
5
expenses and saves the rest. At those rates, how many weeks will it take
Jacob to save $500, in terms of n?
A500
n
B2,500
n
Cn
625
Dn
2,500
E625n
Explanation
It may be helpful to consider how you would determine the number of weeks it
would take Jacob to save $500 if you knew how much he saved each week. For
example, suppose Jacob saved $25 each week. At that rate, it is easy to see that it
would take him or 20, weeks to save $500. Using this example, you500 25,
can see that the number of weeks it will take Jacob to save $500 is equal to 500
divided by the amount he saves each week.
Now use the information given in the question to determine an algebraic
expression representing the amount Jacob saved each week. In the question, you
are given that Jacob’s weekly expenses are Therefore, the amount he
4ndollars.
5
saves each week is equal to his weekly take-home pay minus his weekly
expenses, or 4n5n4n5n4nn
n−=−= =dollars.
555 5 5
Recall that you had already concluded that the number of weeks it will take
Jacob to save $500 is equal to 500 divided by the amount he saves each week. So
the number of weeks it will take Jacob to save $500 is
The correct answer is Choice B.
n5 2,500
500 =500 =.
5nn
Questions 17 to 20 are based on the following data.
LENGTH OF UNEMPLOYMENT FOR WORKERS IN REGION X FOR TWO INDUSTRIES, 2003
Note: The circle graphs show the distributions of workers who were unemployed for at
least 1 week in 2003, by length of unemployment, rounded to the nearest week.
Manufacturing Industry
Total: 10 million
Service Industry
Total: 8 million
1 to 4
weeks
56%
1 to 4
weeks
40%
16%
5 to 10 weeks
20%
18%
10%
12%
14%
9%
5%
11 to 14 weeks
15 to 25 weeks
26 weeks or more
This ebook was issued to HUAN-LIN CHANG, order #14803770223. Unlawful distribution of this ebook is prohibited.
Customer name HUAN-LIN CHANG , Order Id: 14803770223
Answers and Explanations
539
17. In the circle graphs, the degree measure of the central angle of the sector
representing the number of workers unemployed for 11 to 14 weeks is how
much greater in the manufacturing industry graph than in the service
industry graph?
A5
B10
C15
D18
E20
Explanation
Recall that in a circle graph, the degree measure of the central angle of a sector
representing npercent of the data is equal to npercent of 360.
The degree measure of the sector representing the number of workers
unemployed for 11 to 14 weeks is 10% of or for the manufacturing360,36,
industry graph and is 5% of or for the service industry graph. Since360,18,
the measure of the central angle of that sector in the3618=18,
manufacturing industry graph is greater than the measure of the central18
angle of the corresponding sector in the service industry graph. The correct
answer is Choice D.
18. Which of the following could be the median length of unemployment, in
weeks, for manufacturing industry workers who were unemployed for at
least 1 week?
A4
B8
C12
D16
E20
Explanation
Note that the sectors in the manufacturing industry circle graph separate the
unemployed manufacturing industry workers into five groups by length of
unemployment; also, the percent of workers within each of the five groups is
given. Also note that there are 10 million lengths of unemployment, one length
for each of the 10 million workers. Since the lengths are rounded to whole
numbers of weeks, most of the 10 million lengths must be repetitions.
The median length of unemployment is the average of the two middle
lengths when the lengths are listed in order from least to greatest; that is, the
median is the number at which 50% of the lengths have been listed.
To find the median length, first note that the group with the shortest
unemployment lengths, 1 to 4 weeks, accounts for the first 40% of the lengths in
the ordered list. Then, because the group with the next longer lengths, 5 to 10
weeks, accounts for the next 20% of the lengths in the list, the number at which
50% of the lengths have been listed is in this group. So the median length is in
the 5-to-10 week interval. Among the answer choices, the only choice that is in
the 5-to-10 week interval is Choice B, 8. The correct answer is Choice B.
This ebook was issued to HUAN-LIN CHANG, order #14803770223. Unlawful distribution of this ebook is prohibited.
Customer name HUAN-LIN CHANG , Order Id: 14803770223
GRE Practice Test 2
540
19. If one of the workers in the manufacturing and service industries who were
unemployed for at least 1 week will be randomly selected, what is the
probability that the person selected will be a service industry worker who
was unemployed for 26 weeks or more?
A0.04
B0.09
C0.21
D0.40
E0.90
Explanation
The probability that the person selected will be a service industry worker who
was unemployed for 26 weeks or more is equal to the following fraction.
the number of service industry workers who were unemployed 26 weeks or more
the number of workers in the two industries who were unemployed at least 1 week
The number of workers in the two industries who were unemployed for at least
1 week is the sum of the total numbers of workers represented by the two
graphs, or 10 million +8 million =18 million.
According to the graph for the service industry, the number of workers who
were unemployed for 26 weeks or more is 9% of 8 million, or 0.72 million.
Therefore, the desired probability is The correct
0.72 million 72
==0.04.
18 million 1,800
answer is Choice A.
20. The ratio of the number of manufacturing industry workers who were
unemployed for 5 to 10 weeks to the number of service industry workers
who were unemployed for 5 to 10 weeks is closest to which of the
following?
A5to4
B6to5
C3to2
D5to2
E7to6
Explanation
According to the graphs, the number of manufacturing industry workers who
were unemployed for 5 to 10 weeks is 20% of 10 million, or 2 million; and the
number of service industry workers who were unemployed for 5 to 10 weeks is
16% of 8 million, or 1.28 million. Thus, the ratio of the two numbers is 2 to
1.28.
To answer the question, you must now determine which of the answer
choices is closest to the ratio 2 to 1.28. A good way to compare ratios is to
express each ratio as a fraction and then as a decimal, and then to compare the
decimals. The ratio 2 to 1.28 can be expressed as the fraction which is
2,
1.28
equal to the decimal 1.5625.
This ebook was issued to HUAN-LIN CHANG, order #14803770223. Unlawful distribution of this ebook is prohibited.
Customer name HUAN-LIN CHANG , Order Id: 14803770223
Answers and Explanations
541
Now look at the answer choices. As you go through the answer choices, keep
in mind that you are trying to determine which is closest to 1.5625.
Choice A: 5 to 4 can be expressed as which is equal to 1.25.
5,
4
Choice B: 6 to 5 can be expressed as which is equal to 1.2.
6,
5
Choice C: 3 to 2 can be expressed as which is equal to 1.5. Note that this
3,
2
ratio is close to 1.5625.
Choice D: 5 to 2 can be expressed as which is equal to 2.5.
5,
2
Choice E: 7 to 6 can be expressed as which is approximately equal to 1.17.
7,
6
Of the five choices, the ratio in Choice C is closest to 1.5625. The correct
answer is Choice C.
For the following question, select all the answer choices that apply.
21. If which of the following could be the value of t?t+3>5,
Indicate all such values.
A9
B6
C2
D0
E2
F3
Explanation
One way to approach this question is to substitute each of the answer choices
into the inequality and determine which ones satisfy the inequality. If you do
this, you will see that Choice A, and Choice F, 3, satisfy the inequality, but9,
the other answer choices do not. The correct answer consists of Choices A and F.
An algebraic approach to the question is to note that the inequality
is satisfied whenever or , that is, whenever ort+3>5t+3>5t+3<−5t>2
Therefore, all values of tgreater than 2 or less than satisfy thet<−8. 8
inequality The only answer choices that meet those conditions aret+3>5.
Choice A, and 3, Choice F. The correct answer consists of Choices A and F.9,
This ebook was issued to HUAN-LIN CHANG, order #14803770223. Unlawful distribution of this ebook is prohibited.
Customer name HUAN-LIN CHANG , Order Id: 14803770223
GRE Practice Test 2
542
22. The operation is defined for all integers xand yas If xxy=xy y.
and yare positive integers, which of the following CANNOT be zero?
Axy
Byx
C(x1) y
D(x+1) y
Ex(y1)
Explanation
In the formula the variables xand yare placeholders that can bexy=xy y,
replaced by integers or by expressions representing integers. Here are two
examples.
If xis replaced by 3 and yis replaced by 4, then the formula gives
34=(3)(4) 4=12 4=8
If xis replaced by and yis replaced by 2, then the formula givesx1
(x1) 2=((x1)(2)) 2=2x22=2x4
Scanning the answer choices, you can see that all of them are of the form
“first expression” “second expression”
For each answer choice, you must determine whether the answer choice can be
equal to 0 for some positive integers xand y. Are there positive integers xand y
for which the answer choice is equal to 0 ? If not, then that answer choice is the
correct answer.
Choice A: Using the formula, try to find positive integers xand yforxy.
which , that is, for which To solve this equation, note thatxy=0xy y=0.
factoring yout of the left-hand side of the equation gives the equationxy y=0
So now you must find positive integers xand ysuch that the(x1)y=0.
product of the two numbers and yis 0. Since the product of two numbersx1
is 0 only if at least one of the numbers is 0, it follows that the product of x1
and ywill be 0 if no matter what the value of yis. For example, ifx=1, x=1
and then and both xand yare positivey=2, xy=12=(1)(2) 2=0,
integers. Therefore, Choice A is not correct, since there are positive integers x
and yfor which xy=0.
Choice B: This is similar to Choice A, except the xand yareyx.
interchanged. Therefore, you might try the example in Choice A but with the
values of xand yinterchanged: and Using the formula,y=1x=2.
Therefore, Choice B is not correct, since there areyx=yx x=(1)(2) 2=0.
positive integers xand yfor which yx=0.
Choice C: Using the formula, try to find positive integers xand y(x1) y.
for which , that is, for which Factoring yout of the(x1) y=0(x1)yy=0.
left-hand side of the equation yields(x1)yy=0(x11)y=(x2)y=0.
Here the product of the two numbers and yis 0. So the product will be 0 ifx2
no matter what the value of yis. For example, if and thenx=2, x=2y=10,
and both xand yare positive(x1) y=(2 1) 10 =110 =(1)(10) 10 =0,
This ebook was issued to HUAN-LIN CHANG, order #14803770223. Unlawful distribution of this ebook is prohibited.
Customer name HUAN-LIN CHANG , Order Id: 14803770223
Answers and Explanations
543
integers. Therefore, Choice C is not correct, since there are positive integers x
and yfor which (x1) y=0.
Choice D: Using the formula, try to find positive integers xand y(x+1) y.
for which , that is, for which Factoring yout of the(x+1) y=0(x+1)yy=0.
left-hand side of the equation yields Here(x+1)yy=0(x+11)y=xy =0.
the product of xand yis 0, so or Since both xand ymust be positivex=0y=0.
but 0 is not positive, it follows that there are no positive integers xand yfor
which The correct answer is Choice D.(x+1) y=0.
Choice E: cannot be correct, since Choice D is correct, but Choicex(y1)
E is considered here for completeness. Using the formula, try to find positive
integers xand yfor which , that is, for whichx(y1) =0x(y1) (y1) =0.
Factoring out of the left-hand side of the equationy1x(y1) (y1) =0
yields Here the product of the two numbers and is 0.(x1)(y1) =0. x1y1
So the product will be 0 if or no matter what the value of the otherx=1y=1,
variable is. For example, if and thenx=20 y=1, x(y1) =20 (1 1) =
and both xand yare positive integers. Therefore, Choice20 0=(20)(0) 0=0,
E is not correct, since there are positive integers xand yfor which x(y1) =0.
23. P, Q , and Rare three points in a plane, and Rdoes not lie on line PQ.
Which of the following is true about the set of all points in the plane that
are the same distance from all three points?
AIt contains no points.
BIt contains one point.
CIt contains two points.
DIt is a line.
EIt is a circle.
Explanation
First consider just two of the three points, say Pand Q, and the set of points in
the plane that are the same distance from them. Clearly the midpoint of line
segment PQ is such a point. Are there others? You may recall from geometry
that the points on the line that bisects PQ and is perpendicular to PQ are all the
points that are equidistant from Pand Q. Similarly, the points in the plane that
lie on the perpendicular bisector of line segment PR are all the points that are
equidistant from points Pand R.
Because Rdoes not lie on line PQ, line segments PQ and PR do not lie on
the same line, and so their respective perpendicular bisectors are not parallel.
Therefore, you can conclude that the two perpendicular bisectors intersect at a
point. The point of intersection is on both perpendicular bisectors, so it is
equidistant from Pand Qas well as from Pand R. Therefore, the point of
intersection is equidistant from all three points. Are there any other points that
are equidistant from P, Q , and R? If there were, they would be on both
perpendicular bisectors, but in fact only one point lies on both lines. The correct
answer is Choice B.
This ebook was issued to HUAN-LIN CHANG, order #14803770223. Unlawful distribution of this ebook is prohibited.
Customer name HUAN-LIN CHANG , Order Id: 14803770223
GRE Practice Test 2
544
24. If which of the following inequalities must be true?x<y<0,
Ay+1<x
By1<x
C
2
xy <x
D
2
xy <y
E
2
xy <x
Explanation
The conditions stated in the question, tell you that xand yare negativex<y<0,
numbers and that yis greater than x. Keep this in mind as you evaluate each of
the inequalities in the answer choices, to see whether the inequality must be
true.
Choice A: According to the conditions given in the question, yisy+1<x.
greater than x. Since yis greater than xand is greater than y, it followsy+1
that is greater than x. So it cannot be true that Therefore, Choicey+1y+1<x.
A is not the correct answer.
Choice B: While it is true that both xand are less than y,ity1<x.y1
may not be true that Consider what happens if and Iny1<x.y=−2x=−7.
this case, the inequality becomes which is false. Therefore,y1<x3<−7,
Choice B is not the correct answer.
Choice C: Note that is positive and xis negative, so is negative.
22 2
xy <x.yxy
Is the negative number less than the negative number x? It depends on
2
xy
whether or Consider what happens if and where
1
22
y>1y<1. x=−4y=− ,
2
In this case, the inequality becomes which is false.
22
y<1. xy <x1<−4,
Therefore, Choice C is not the correct answer.
Choice D: Since yis a negative number, multiplying both sides of the
2
xy <y.
inequality by yreverses the inequality, resulting in the inequality So
2
x<yxy>y.
it cannot be true that Therefore, Choice D is not the correct answer.
2
xy <y.
Since Choices A through D have been eliminated, the correct answer is
Choice E. You can show that the inequality in Choice E, must be true as
2
xy <x,
follows: Multiply both sides of the given inequality by xto obtain thex<y
inequality reversing the direction of the inequality because xis negative.
2
x>xy,
Therefore, the inequality must be true, and the correct answer is
2
xy <x
Choice E.
For the following question, enter your answer in the box.
25. What is the length of a diagonal of a rectangle that has width 5 and
perimeter 34 ?
Explanation
In this question, you are given that a rectangle has width 5 and perimeter 34,
and you are asked to find the length of a diagonal of the rectangle. Let Land W
represent the length and width of the rectangle, respectively, and let Drepresent
the length of a diagonal. Note that you are not given Lbut you are given that
This ebook was issued to HUAN-LIN CHANG, order #14803770223. Unlawful distribution of this ebook is prohibited.
Customer name HUAN-LIN CHANG , Order Id: 14803770223
Answers and Explanations
545
and that the perimeter is 34. Because the perimeter is equal toW=5
or you can determine Las follows.L+L+W+W,2(L+W),
2(L+5) =34
L+5=17
L=12
The following figure shows a rectangle of length 12, width 5, and diagonal of
length D.
12
D5
From the figure, you can see that the diagonal is the hypotenuse of a right
triangle with legs of length 5 and 12. Therefore, by the Pythagorean theorem,
222
5+12 =D
2
25 +144 =D
2
169 =D
13 =D
The length of the diagonal is 13, so the correct answer is 13.
This ebook was issued to HUAN-LIN CHANG, order #14803770223. Unlawful distribution of this ebook is prohibited.
Customer name HUAN-LIN CHANG , Order Id: 14803770223
GRE Practice Test 2
546
SECTION 6
Quantitative Reasoning
25 Questions with Explanations
AQuantity A is greater.
BQuantity B is greater.
CThe two quantities are equal.
DThe relationship cannot be determined from the information given.
A circle is inscribed in a square with sides of length 5.
Quantity A Quantity B
1. The circumference of the
circle
15 ABCD
Explanation
In this question, you are given that a circle is inscribed in a square with sides of
length 5 and are asked to compare the circumference of the circle with 15. Since
the circle is inscribed in the square, the diameter of the circle is equal to the
length of a side of the square, or 5. Thus, the circumference of the circle is 5p.
Because is greater than 3, it follows that is greater than 15. Therefore,p5p
Quantity A is greater than Quantity B, and the correct answer is Choice A.
2u+v=14
uv =0
Quantity A Quantity B
2. uv
ABCD
Explanation
In this question, you are asked to compare uwith v, given that and2u+v=14
uv =0.
Consider the equation Since a product can equal 0 only if at least oneuv =0.
of the factors in the product equals 0, you know that or or both. Butu=0v=0,
since you are also given that it follows that uand vcannot both2u+v=14,
equal 0.
Knowing that either or you can substitute 0 into the equationu=0v=0,
for either uor vin order to determine the relationship between2u+v=14
uand vif it is possible to do so from the information given. If thenu=0,
simplifies to In this case, uis less than v. However, if2u+v=14 v=14. v=0,
then simplifies to or And in this case, uis greater2u+v=14 2u=14, u=7.
than v.
In the first case, , and in the second case, Therefore, theu<vu
>v.
relationship between the two quantities uand vcannot be determined from the
information given, and the correct answer is Choice D.
This ebook was issued to HUAN-LIN CHANG, order #14803770223. Unlawful distribution of this ebook is prohibited.
Customer name HUAN-LIN CHANG , Order Id: 14803770223
Answers and Explanations
547
Quantity A Quantity B
3.
2,000
950
6,000
10 ABCD
Explanation
In this question, you are asked to compare the quantity with the quantity
2,000
950
Note that both quantities are written in the form “base to a power.” If the
6,000
10 .
bases were equal, you would be able to compare the quantities by comparing the
powers. Because powers of 10 are easier to work with than powers of 950, it is
reasonable to try to compare the quantities by rewriting the quantity as a
2,000
950
power of 10. Unfortunately, there is no obvious way to do that. However, if you
can approximate 950 by a power of 10, you may then be able to use the
approximation to compare the quantity with the quantity
2,000 6,000
950 10 .
Note that 950 is close to, but a little less than, 1,000, or Raising
3
10 .
both sides of the inequality to the power 2,000 gives the inequality
3
950 <10
Since you can conclude that
2,000 3 2,000 3 2,000 6,000
950 <(10 ) . (10 ) =10 ,
Thus, Quantity A is less than Quantity B, and the correct
2,000 6,000
950 <10 .
answer is Choice B.
Set Aconsists of 40 integers, and set Bconsists of 150
integers. The number of integers that are in both set A
and set Bis 20.
Quantity A Quantity B
4. The total number of integers
that are in set A or set B,
or both
170 ABCD
Explanation
In this question, you are given that the number of integers in set Ais 40, the
number of integers in set Bis 150, and the number of integers that are in both A
and Bis 20. You are asked to compare the total number of integers that are in
set Aor set B, or both, with 170.
This is the type of question for which a Venn diagram is usually helpful to
represent the information given. The following Venn diagram is a representation
of the integers in sets Aand B.
A
40
B
150
Note that there is no number in the shaded region of the diagramthe region
representing the integers in both Aand B. In fact, the number of integers in both
Aand Bis included in both the number of integers in Aand the number of
This ebook was issued to HUAN-LIN CHANG, order #14803770223. Unlawful distribution of this ebook is prohibited.
Customer name HUAN-LIN CHANG , Order Id: 14803770223
GRE Practice Test 2
548
integers in B. It is a good idea, therefore, to redraw the Venn diagram so that the
numbers are separated into three categories: the integers in Aonly, the integers
in Bonly, and the integers in both Aand B. The revised Venn diagram follows.
A only
20
B only
130
20
A and B
Observe that summing the numbers of integers in set Aonly, set Bonly, and both
Aand Byields the total number of integers that are in set Aor set B, or both.
Therefore, Quantity A is or 170, and the correct answer is20 +130 +20,
Choice C.
Another approach is to realize that if you listed the integers in set Aand the
integers in set B, you would have listed the integers that are in both Aand B
twice and all of the other integers once. So the total number of integers in set A
or set B, or both, is equal to
(number in set A)+(number in set B)(number in both sets)
Thus, the number of integers in set Aor set B, or both, is or 170,40 +150 20,
and the correct answer is Choice C.
xis a negative integer.
Quantity A Quantity B
5.
x
2
x+1
3ABCD
Explanation
In this question, you are asked to compare with given that xis a negative
xx+1
23,
integer. One way to approach this problem is to plug a value of xin both
expressions and compare the results.
You are given that xis a negative integer, so the greatest integer you can plug
in for xis 1.
For it follows that and
1
x1x+11+10
x=−1, 2 =2=3=3=3=1.
2
In this case, is less than However, to conclude that Quantity B is
xx+1
23.
greater, it is not sufficient for to be less than for one particular value of x;
xx+1
23
the relationship would need to be true for all negative integer values of x.To
analyze this relationship further, plug in another value of x, for example, 2.
For it follows that and
11 1
x2x+12+11
x=−2, 2 =2== 3=3=3=.
2
24 3
Again, is less than but note that these values are closer together than
xx+1
23,
the previous values of and It appears that the relationship between the
xx+1
23.
quantities may differ for smaller values of x, so now try plugging in for x.3
For it follows that and
11 11
x3x+13+12
x=−3, 2 =2== 3=3=3==.
32
28 39
This ebook was issued to HUAN-LIN CHANG, order #14803770223. Unlawful distribution of this ebook is prohibited.
Customer name HUAN-LIN CHANG , Order Id: 14803770223
Answers and Explanations
549
In this case, is greater than
xx+1
23.
Since is less than for and is greater than for
xx+1xx+1
23x=−12 3 x=−3,
the relationship between these two quantities cannot be determined from the
information given. The correct answer is Choice D.
Since both quantities are algebraic expressions, another way to approach the
comparison is to set up a placeholder relationship, denoted by , between the
?
| |
two quantities and then to simplify to see what conclusions you can draw. As
you simplify and draw conclusions, keep in mind that xis a negative integer.
xx+1
2?3
| |
xx
2 ? 3(3 )
| |
x
2?3
x
| |
3
x
2?3
冢冣
| |
3
For any value of x(including negative integer values of x), the value of is
x
3
positive, so dividing by does not affect any inequality that might be
x
3
represented by the placeholder. Since each step in this simplification is
reversible, the simplification reduces the problem to comparing with 3,
x
2
冢冣
3
given that xis a negative integer. Note that where so the
xn
23
=,n=−x;
冢冣 冢冣
32
problem can be reduced further to comparing with 3, given that nis a
n
3
冢冣
2
positive integer.
Because is greater than 1, the value of becomes greater as nbecomes
n
33
冢冣
22
larger. For small values of n, is less than 3, but for large values of n,is
nn
33
冢冣 冢冣
22
greater than 3. Therefore, the relationship between Quantity A and Quantity B
cannot be determined from the given information, and the correct answer is
Choice D.
(x+3)( y4) =0
Quantity A Quantity B
6. xy 12 ABCD
Explanation
In this question, you are given that and you are asked to(x+3)(y4) =0,
compare the product with Since and the product of twoxy 12. (x+3)(y4) =0
numbers can equal 0 only if at least one of the numbers in the product equals 0,
you can conclude that or or both.x=−3y=4,
Consider the case When you can choose any number as thex=−3. x=−3,
value of yand the equation will be satisfied. Depending on the(x+3)(y4) =0
particular value of yyou choose, the product may be greater than, less than,xy
or equal to For example, if then is greater than and if12. y=1, xy =−312;
then is less thany=10, xy =−30 12.
This ebook was issued to HUAN-LIN CHANG, order #14803770223. Unlawful distribution of this ebook is prohibited.
Customer name HUAN-LIN CHANG , Order Id: 14803770223
GRE Practice Test 2
550
Since different examples for the value of yyield different relationships
between Quantities A and B, the relationship cannot be determined from the
information given. The correct answer is Choice D.
Geoff used $630 to buy a new guitar. This amount was
15 percent of his earnings last summer.
Quantity A Quantity B
7. The amount of Geoff’s
earnings last summer not
used to buy the new guitar
$3,570 ABCD
Explanation
In this question, you are asked to compare the amount of Geoff’s earnings last
summer not used to buy a new guitar with the amount $3,570. You are given
that Geoff used 15% of his earnings last summer, or $630, to buy the new guitar.
So the relationship between Geoff’s earnings last summer and the amount he
spent to buy the guitar can be expressed by the equation
(0.15) (Geoff’s earnings last summer) =$630
Therefore, you can conclude that Geoff’s earnings last summer totaled or
$630,
0.15
$4,200. Since Geoff earned $4,200 last summer and spent $630 of his earnings to
buy the guitar, the amount he did not spend to buy the guitar was $4,200 $630,
or $3,570. Therefore, Quantity A is equal to Quantity B, and the correct answer
is Choice C.
Set Sconsists of 5 objects.
Quantity A Quantity B
8. The number of subsets
of set Sthat consist of
1 object
The number of subsets
of set Sthat consist of
4 objects
ABCD
Explanation
In this question, you are given that set Sconsists of 5 objects and are asked to
compare the number of subsets of set Sthat consist of 1 object with the number
of subsets of set Sthat consist of 4 objects. Recall that a set Ris a subset of set
Sif all of the members of Rare also members of S.
Note that if you select 1 object from set S, there are 4 objects in Sthat you
have not selected; that is to say, each subset of Swith 1 object corresponds to a
subset of Swith 4 objects. Therefore, the number of subsets of Swith 1 object is
equal to the number of subsets of Swith 4 objects. Since Quantity A is equal to
Quantity B, the correct answer is Choice C.
Another approach to solving this problem is to consider a particular set of 5
objects and determine all of the subsets consisting of 1 object and all of the
subsets consisting of 4 objects. For example, let There are 5S=a,b,c,d,e.
{}
subsets of Sconsisting of 1 object: and there are 5 subsets of Sa ,b,c,d,e;
{}{}{}{} {}
consisting of 4 objects: {a,b,c,d},{a,b,c,e},{a,b,d,e},{a,c,d,e}, and {b,c,d,e}.
This ebook was issued to HUAN-LIN CHANG, order #14803770223. Unlawful distribution of this ebook is prohibited.
Customer name HUAN-LIN CHANG , Order Id: 14803770223
Answers and Explanations
551
Clearly, the particular 5 objects in Sdo not change the fact that the number of
subsets of Sconsisting of 1 object is equal to the number of subsets of S
consisting of 4 objects. The correct answer is Choice C.
A
B
C
45° 60°
2
Quantity A Quantity B
9. The length of line segment AC 3ABCD
Explanation
In this question, you are asked to compare the length of line segment AC with 3.
Note that in the figure, the vertical line segment divides triangle ABC into
two right triangles. Based on the fact that the sum of the measures of the angles
in a triangle is you can conclude that the triangle to the left of the vertical180,
line is a - - right triangle and the triangle to the right of the vertical line454590
is a - - right triangle. The following figure shows all of these angle306090
measures, along with a new label Dat the vertex of the right angles.
AC
B
D
45°
30°
2
45° 90° 90° 60°
Note that the length of AC is equal to the length of AD plus the length of DC.
Also note that AD is a leg of the - - triangle and DC is a leg of the454590
- - triangle.306090
In the figure, you are given that the length of BC, the hypotenuse of the
- - triangle, is 2. No other lengths are given. Recall that if the length of306090
the hypotenuse of a - - triangle is 2, then the length of the side opposite306090
the angle is 1, and the length of the side opposite the angle is So, in
30603.
the - - triangle BDC, the length of DC (the side opposite the angle) is30609030
1, and the length of BD (the side opposite the angle) is
603.
Now consider the - - triangle ABD. Since this is an isosceles right454590
triangle, its legs, AD and BD, have equal length. Since the length of BD is the
3,
length of AD is also 3.
As noted above, the length of AC is equal to the length of AD plus the length
of DC. Since the length of AD is and the length of DC is 1, it follows that the
3
length of AC is equal to 3 +1.
This ebook was issued to HUAN-LIN CHANG, order #14803770223. Unlawful distribution of this ebook is prohibited.
Customer name HUAN-LIN CHANG , Order Id: 14803770223
GRE Practice Test 2
552
Recall that in the question you were asked to compare the length of AC with
3. Because is less than 2, it follows that the length of AC, which is equal to
3
is less than or 3. Hence, Quantity B is greater than Quantity A, and3 +1, 2 +1,
the correct answer is Choice B.
5
–5
x
k
y
O
10. What is the slope of line kin the xy-plane above?
A5
B1
C0
D1
E5
Explanation
Recall that if a line passes through the points with coordinates and(x,y)
11
, where then the slope of the line is(x,y)xx,
22 1 2
yy
21
xx
21
From the graph of line kin the xy-plane, you can conclude that the x-intercept of
line khas coordinates and the y-intercept of line khas coordinates(5, 0) (0, 5).
Thus, the slope of line kis
505
==1
055
The correct answer is Choice D.
b3, b1, b+2, b+3, b+4
11. The median of the five terms listed above is 5, where bis a constant. What
is the average (arithmetic mean) of the five terms?
A3
B4
C5
D6
E7
This ebook was issued to HUAN-LIN CHANG, order #14803770223. Unlawful distribution of this ebook is prohibited.
Customer name HUAN-LIN CHANG , Order Id: 14803770223
Answers and Explanations
553
Explanation
To calculate the average of the five terms, you first need to use the information
given in the question to determine the value of b.
You are given that 5 is the median of the five terms b3, b1, b+2, b+3,
and where bis a constant. The median of five terms is the middle, orb+4,
third, term when the terms are listed in increasing order. Observe that the five
terms are already given in increasing order. Since is the third term, youb+2
can conclude that and thusb+2=5b=3.
Since it follows that the values of the five terms are 0, 2, 5, 6, and 7,b=3,
and the average of the five terms is The correct answer
0+2+5+6+720
==4.
55
is Choice B.
For the following question, enter your answer in the box.
15
9
12 8
12. What is the area of the region shown above?
Explanation
In this question, you are asked to determine the area of the given region. One
approach to solving this problem is to split the region into two rectangles,
rectangle Rand rectangle S, as follows.
Rectangle R
Rectangle S
12
8
15
9
The area of the region is the sum of the areas of the two rectangles. The area of
rectangle Ris or 120. Since the width of rectangle Sis or 4, the(15)(8), 12 8,
area of rectangle Sis or 36. Thus, the area of the region is or(9)(4), 120 +36,
156. The correct answer is 156.
This ebook was issued to HUAN-LIN CHANG, order #14803770223. Unlawful distribution of this ebook is prohibited.
Customer name HUAN-LIN CHANG , Order Id: 14803770223
GRE Practice Test 2
554
Another approach to solving this problem is to form the region by removing
a small rectangle from a rectangle with length 15 and width 12, as follows.
8
9
15
12
Small rectangle
The area of the region is the area of the rectangle with length 15 and width 12
minus the area of the small rectangle. The area of the rectangle with length 15
and width 12 is or 180. Since the length of the small rectangle is(15)(12), 15 9,
or 6, and the width of the small rectangle is or 4, the area of the small12 8,
rectangle is or 24. Thus, the area of the region is or 156. The(6)(4), 180 24,
correct answer is 156.
13. During a one-year study, biologists observed the number of fish in a certain
pond as well as the percent of the fish that were catfish. At the beginning of
the year, there were 300 fish in the pond, of which 15 percent were catfish;
and at the end of the year, there were 400 fish in the pond, of which 10
percent were catfish. From the beginning of the year to the end of the year,
the number of catfish in the pond
Adecreased by more than 5%
Bdecreased by 5%
Cdid not change
Dincreased by 5%
Eincreased by more than 5%
Explanation
The answer choices indicate that the question is asking about the percent change
in the number of catfish. The number of catfish in the pond at the beginning of
the year was 15% of 300, or which is 45. The number of catfish in(0.15)(300),
the pond at the end of the year was 10% of 400, or which is 40.(0.10)(400),
Thus, the number of catfish decreased by 5.
The percent by which the number of catfish decreased from the beginning of
the year to the end of the year is
the decrease in the number of catfish over the year 5
(100%) =(100%) 11%
冢冣
the number of catfish at the beginning of the year 45
Thus, the number of catfish decreased by about 11%. This is a decrease of more
than 5%, so the correct answer is Choice A.
This ebook was issued to HUAN-LIN CHANG, order #14803770223. Unlawful distribution of this ebook is prohibited.
Customer name HUAN-LIN CHANG , Order Id: 14803770223
Answers and Explanations
555
For the following question, enter your answer in the box.
14. On a radio tower, a red light flashes every 6 seconds and a blue light flashes
every 10 seconds. If both lights flash together at a certain time, how many
seconds later will both lights flash together the next time?
seconds
Explanation
One way to approach this question is to look at the “flash times” for both lights
to see what times they have in common. The following lists show the flash times
for both lights as the numbers of seconds after the time at which both lights
flashed together.
Red light: 6, 12, 18, 24, 30, 36, 42, 48, 54, 60, . . .
Blue light: 10, 20, 30, 40, 50, 60, 70, 80, 90, . . .
Note that 30 is the first number that is common to both lists. Therefore, if both
lights flash together, they will flash together again 30 seconds later. The correct
answer is 30.
Alternatively, you may realize that if the lights flash together, the number of
seconds that will elapse before they flash together the next time is the least
common multiple of 6 and 10. To find the least common multiple of 6 and 10,
begin by writing each integer as the product of its prime factors.
6=(2)(3)
10 =(2)(5)
Since 2 is a factor in both products, but 3 and 5 are factors of only one of the
products, the least common multiple of 6 and 10 is or 30. Therefore, if(2)(3)(5),
both lights flash together, the next time they will flash together is 30 seconds
later. The correct answer is 30.
For the following question, select all the answer choices that apply.
15. If which of the following numbers must be positive?a<b<0,
Indicate all such numbers.
Aab
B
22
ab
Cab
D
2
ab
E
22
ab+ab
Explanation
In this question, you are given that and are asked to determine whicha<b<0
of the answer choices must be positive. Note that the condition meansa<b<0
that aand bare negative and that a<b.
Choice A: In the question, it is given that Subtracting bfromab.a<b.
both sides of the inequality gives the inequality Therefore,a<bab<0. ab
must be negative.
This ebook was issued to HUAN-LIN CHANG, order #14803770223. Unlawful distribution of this ebook is prohibited.
Customer name HUAN-LIN CHANG , Order Id: 14803770223
GRE Practice Test 2
556
Choice B: Since aand bare negative, you can square both sides of
22
ab.
the inequality to get the inequality Then you can subtract from
22 2
a<ba>b.b
both sides of the inequality to conclude that So must
22 22 22
a>bab>0. ab
be positive. Alternatively, note that can be factored as The
22
ab(ab)(a+b).
factor is Choice A, which must be negative, and the factor is the sumaba+b
of two negative numbers, which also must be negative. Thus, is the
22
ab
product of two negative numbers, so it must be positive.
Choice C: ab. Because aand bare negative, you can conclude that their
product ab must be positive.
Choice D: Because can be written as which is the product
22
ab.ab (a)(a)(b),
of three negative numbers, you can conclude that must be negative.
2
ab
Choice E: By the reasoning in the explanation of Choice D, Choice
22
ab+ab .
E is the sum of two negative numbers. Therefore, you can conclude that
22
ab+ab
must be negative.
Choices B and C must be positive, and Choices A, D, and E must be negative.
The correct answer consists of Choices B and C.
x
x
16. A flat rectangular picture, represented by the unshaded region in the figure
above, is mounted in a flat rectangular frame, represented by the shaded
region. The frame is 1 inch wide on all sides. For what value of x, in inches,
is the area of the frame equal to the area of the picture?
A4
B5
C6
D7
E8
Explanation
In this question, you are asked to determine the value of xfor which the area of
the frame is equal to the area of the picture. To do this, you need to express both
the area of the frame and the area of the picture in terms of xand then find the
value of xfor which the two expressions are equal.
The area of the picture is the area of the inner rectangle, and the area of the
frame is the area of the outer rectangle minus the area of the inner rectangle.
Since the area of a rectangle is the length times the width, you need to know the
length and width of the inner and outer rectangles.
In the figure, you are given that the length of the inner rectangle is xinches,
but the width is not given. However, since you know that the width of the frame
This ebook was issued to HUAN-LIN CHANG, order #14803770223. Unlawful distribution of this ebook is prohibited.
Customer name HUAN-LIN CHANG , Order Id: 14803770223
Answers and Explanations
557
is 1 inch, it follows that the width of the inner rectangle is equal to the width of
the outer rectangle minus 2 inches, or inches. Thus, the area of the innerx2
rectangle is square inches.x(x2)
In the figure, you are given that the width of the outer rectangle is xinches,
but the length is not given. However, since you know that the width of the frame
is 1 inch, it follows that the length of the outer rectangle is equal to the length of
the inner rectangle plus 2 inches, or inches. Thus, the area of the outerx+2
rectangle is square inches.x(x+2)
Since the area of the frame is the area of the outer rectangle minus the area
of the inner rectangle, the area of the frame is
square inches.
22
x(x+2) x(x2) =x+2xx+2x=4x
Now you are ready to set up the equation. Set the expression for the area of
the picture equal to the expression for the area of the frame and solve the
resulting equation for x, as follows.
x(x2) =4x
2
x2x=4x
2
x6x=0
x(x6) =0
There are two solutions to the equation, and Since xrepresents thex=0x=6.
length of a picture, in inches, the solution does not make sense in thisx=0
context. Therefore, when the area of the picture equals the area of thex=6,
frame. The correct answer is Choice C.
Questions 17 to 20 are based on the following data.
PERCENT OF THE 300 PEOPLE IN GROUP 1 AND THE 400 PEOPLE
IN GROUP 2 WHO HAVE SELECTED AILMENTS
Respiratory Ailment
Percent of People
in Group 1 Who
Have Ailment
Percent of People
in Group 2 Who
Have Ailment
Allergic sensitivity to endotoxins 14% 21%
Asthma (allergic) 3% 4%
Asthma (nonallergic) 2% 3%
Hay fever 4% 10%
Sneezing and itchy eyes 8% 11%
Wheezing (allergic) 5% 6%
Wheezing (nonallergic) 2% 5%
This ebook was issued to HUAN-LIN CHANG, order #14803770223. Unlawful distribution of this ebook is prohibited.
Customer name HUAN-LIN CHANG , Order Id: 14803770223
GRE Practice Test 2
558
17. The number of people in group 2 who have hay fever is how much greater
than the number of people in group 1 who have hay fever?
A37
B35
C32
D28
E24
Explanation
In group 1, there are 300 people, 4% of whom have hay fever. Therefore, in
group 1, there are or 12 people, who have hay fever. In group(0.04)(300) people,
2, there are 400 people, 10% of whom have hay fever. Therefore, in group 2,
there are or 40 people, who have hay fever. Since(0.10)(400) people,
it follows that there are 28 more people in group 2 who have hay40 12 =28,
fever than there are in group 1. The correct answer is Choice D.
18. For the seven ailments, what is the median of the numbers of people in
group 2 who have the ailments?
A20
B22
C24
D26
E28
Explanation
The median of the seven numbers of people in group 2 is the middle number
when the numbers are listed in increasing order. To find the middle number
easily, first order from least to greatest the percents of people in group 2 who
have the seven ailments, then find the median of the percents, and finally
calculate the number of people corresponding to that percent.
The following table shows the percents of people in group 2 who have the
seven ailments, ordered from least to greatest.
Respiratory Ailment Percent of People in Group 2
Who Have Ailment
Asthma (nonallergic) 3%
Asthma (allergic) 4%
Wheezing (nonallergic) 5%
Wheezing (allergic) 6%
Hay fever 10%
Sneezing and itchy eyes 11%
Allergic sensitivity to endotoxins 21%
This ebook was issued to HUAN-LIN CHANG, order #14803770223. Unlawful distribution of this ebook is prohibited.
Customer name HUAN-LIN CHANG , Order Id: 14803770223
Answers and Explanations
559
The median percent, which is the fourth number in the list, is 6%. Thus, the
median number of people in group 2 who have the ailments is or 24,(0.06)(400),
and the correct answer is Choice C.
19. The number of people in group 1 who have the ailment wheezing (allergic)
is what percent greater than the number of people in group 1 who have the
ailment wheezing (nonallergic)?
A50%
B75%
C150%
D200%
E300%
Explanation
In group 1, there are 300 people, 5% of whom have allergic wheezing and 2% of
whom have nonallergic wheezing. That is, or 15 people, have(0.05)(300) people,
allergic wheezing and or 6 people, have nonallergic wheezing.(0.02)(300) people,
Therefore, in group 1, the number of people who have allergic wheezing exceeds
the number who have nonallergic wheezing by 9, which is greater
9(100%)
冢冣
6
than 6, or 150% greater than 6. The correct answer is Choice C.
For the following question, enter your answer in the boxes.
20. What is the ratio of the number of people in group 2 with the ailment sneezing
and itchy eyes to the total number of people in both groups with the ailment
sneezing and itchy eyes?
Give your answer as a fraction.
Explanation
In group 2, there are 400 people, 11% of whom have sneezing and itchy eyes.
Therefore, in group 2, there are or 44 people, who have(0.11)(400) people,
sneezing and itchy eyes. In group 1, there are 300 people, 8% of whom have
sneezing and itchy eyes. Therefore, in group 1, there are or 24(0.08)(300) people,
people, who have sneezing and itchy eyes. So the total number of people in both
groups who have sneezing and itchy eyes is or 68. Thus, the ratio of the24 +44,
number of people in group 2 who have sneezing and itchy eyes to the total
number of people in both groups who have sneezing and itchy eyes is The
44.
68
correct answer is (or any equivalent fraction).
44
68
This ebook was issued to HUAN-LIN CHANG, order #14803770223. Unlawful distribution of this ebook is prohibited.
Customer name HUAN-LIN CHANG , Order Id: 14803770223
GRE Practice Test 2
560
21. Of the people in a certain survey, 58 percent were at most 40 years old and
70 percent were at most 60 years old. If 252 of the people in the survey
were more than 40 years old and at most 60 years old, what was the total
number of people in the survey?
A1,900
B2,100
C2,400
D2,700
E3,000
Explanation
In this question, it is given that of the people surveyed, 58% were at most 40
years old and 70% were at most 60 years old. Therefore, or 12%, of70% 58%,
the people surveyed were more than 40 years old and at most 60 years old, and
you are given that 252 people are in this group. Let xbe the total number of
people in the survey. Then 12% of xis 252, that is, , and so0.12x=252
Therefore, the total number of people in the survey was 2,100,
252
x==2,100.
0.12
and the correct answer is Choice B.
22. If which of the following is equal to 1.25 percent of x?x>0,
Ax
80
Bx
8
Cx
4
D5x
8
E3x
4
Explanation
Since the answer choices are fractional expressions, to answer the question you
need to convert 1.25% of xto a fractional expression. Note that 1.25% is
equivalent to the decimal 0.0125. Converting the decimal 0.0125 to a fraction
and simplifying gives Thus, 1.25% of xis equal to of x,
125 1 1
0.0125 ==.
10,000 80 80
or and the correct answer is Choice A.
x,
80
This ebook was issued to HUAN-LIN CHANG, order #14803770223. Unlawful distribution of this ebook is prohibited.
Customer name HUAN-LIN CHANG , Order Id: 14803770223
Answers and Explanations
561
23. Alice earns ddollars and has tpercent of what she earns deducted for taxes.
How much of what she earns does Alice have left after taxes?
Adollarsd(1 100t)
Bdollarsd(1 10t)
Cdollarsd(1 t)
Ddollarsd(1 0.1t)
Edollarsd(1 0.01t)
Explanation
Recall that tpercent can be expressed as or 0.01t. Therefore, the amount
t,
100
that Alice has deducted for taxes, which is tpercent of ddollars, can be
expressed as 0.01td dollars. The amount that Alice has left after taxes is the
amount that she earns minus the amount that she has deducted for taxes, or
dollars. Note that is an algebraic expression with two terms,d0.01td d 0.01td
each containing das a factor. Factoring out dfrom each term results in the
algebraic expression The correct answer is Choice E.d(1 0.01t).
For the following question, select all the answer choices that apply.
24. A student made a conjecture that for any integer n, the integer is a4n+3
prime number. Which of the following values of ncould be used to disprove
the student’s conjecture?
Indicate all such values.
A1
B3
C4
D6
E7
Explanation
Recall that a prime number is an integer greater than 1 that has no positive
divisors other than 1 and itself.
The answer choices for this question are integer values of n. Any of the
answer choices for which the integer is not a prime number could be4n+3
used to disprove the conjecture that for any integer n, the integer is a4n+3
prime number.
To answer this question, you must determine for each of the answer choices
whether the integer is a prime number. The evaluations are as follows:4n+3
Choice A: For the integer is or 7, which is a primen=1, 4n+34(1)+3,
number.
Choice B: For the integer is or 15, which is not an=3, 4n+34(3)+3,
prime number.
Choice C: For the integer is or 19, which is a primen=4, 4n+34(4)+3,
number.
Choice D: For the integer is or 27, which is not an=6, 4n+3 4(6) +3,
prime number.
This ebook was issued to HUAN-LIN CHANG, order #14803770223. Unlawful distribution of this ebook is prohibited.
Customer name HUAN-LIN CHANG , Order Id: 14803770223
GRE Practice Test 2
562
Choice E: For the integer is or 31, which is a primen=7, 4n+34(7)+3,
number.
Therefore, of the answer choices, only Choices B and D, that is, andn=3
result in integers that are not prime numbers. Thus, the correctn=6, 4n+3
answer consists of Choices B and D.
25. Eight points are equally spaced on a circle. If 4 of the 8 points are to be
chosen at random, what is the probability that a quadrilateral having the 4
points chosen as vertices will be a square?
A1
70
B1
35
C1
7
D1
4
E1
2
Explanation
For questions involving geometry, it is often helpful to draw a figure representing
the information in the question as accurately as possible. The figure below shows
a circle with 8 equally spaced points, labeled Athrough H, and quadrilateral
BCDH, which is one of the many quadrilaterals that have 4 of the 8 equally
spaced points as vertices.
C
G
H
BD
AE
F
The probability that a quadrilateral having the 4 points chosen as vertices will be
a square is equal to the following fraction.
the number of squares that can be drawn using 4 of the 8 points as vertices
the number of quadrilaterals that can be drawn using 4 of the 8 points as vertices
To calculate the desired probability, you need to determine the number of
squares and the number of quadrilaterals that can be drawn using 4 of the 8
points as vertices.
To determine the number of quadrilaterals, first note that since the 8 points
lie on a circle, every subset of 4 of the 8 points determines a unique
quadrilateral. Therefore, the number of quadrilaterals that can be drawn using 4
This ebook was issued to HUAN-LIN CHANG, order #14803770223. Unlawful distribution of this ebook is prohibited.
Customer name HUAN-LIN CHANG , Order Id: 14803770223
Answers and Explanations
563
of the 8 points as vertices is equal to the number of ways of choosing 4 points
from the 8 points shown. The number of ways of choosing 4 points from the 8
points shown (also called the number of combinations of 8 objects taken 4 at a
time) is equal to You can calculate the value of this expression as
8! .
4!(8 4)!
follows.
8! (8)(7)(6)(5)(4!)
=
4!(8 4)! (4)(3)(2)(1)(4!)
(8)(7)(6)(5)
=(4)(3)(2)
=70
Thus, there are 70 quadrilaterals that can be drawn using 4 of the 8 points as
vertices.
Because the points are equally spaced around the circle, there are only 2
squares that can be drawn using 4 of the 8 points as vertices, namely ACEG and
BDFH, as shown in the following figures.
C
G
H
BD
A
E
F
C
G
H
BD
A
E
F
Therefore, the probability that the quadrilateral will be a square is or
21
,,
70 35
and the correct answer is Choice B.
This ebook was issued to HUAN-LIN CHANG, order #14803770223. Unlawful distribution of this ebook is prohibited.
Customer name HUAN-LIN CHANG , Order Id: 14803770223
POWERPREP
®
II
Version 2.0
Official Software from the Test Maker
Preparation for the computer-based GRE®revised General Test
The software includes a comprehensive review of each section of
the GRE®revised General Test as well as a Test Preview Tool and
two practice tests to help you get ready for test day.
Installation Procedures
Download the free software at www.ets.org/gre/powerprep2/download.
Technical Assistance
Frequently asked questions about the POWERPREP®II software can be found at
www.ets.org/gre/powerprep2faqs.
If you need further assistance, send a description of your problem to
GRETechSupport@ets.org or call 1-914-747-9075, Monday through Friday,
8 a.m.–11 p.m.; Saturday, 11:30 a.m.–11 p.m. (Eastern Time). Emails sent to the
download support hotline will be answered within one business day of receipt.
Additional GRE revised General Test Preparation Material
Want additional practice? Visit www.ets.org/gre/prepare.
Minimum System Requirements
Microsoft®XP, Vista, or 7; Microsoft®Internet Explorer version
6, 7, or 8; Pentium IV 1.8 GHz processor; 1 GB of RAM; 150 MB of hard disk space;
video card with 1024 x768 32-bit color depth resolution; CRT 17-inch or LCD 14-inch
display monitor; Windows user account with local administrative privileges and CD-
ROM drive.
Note: Internet connectivity (cable modem, DSL or better) is required to launch the
GRE website and/or download the installation package. The stand-alone software
does not require an Internet connection.
Copyright 2013 by Educational Testing Service. All rights reserved. ETS, the ETS logo,
LISTENING. LEARNING. LEADING., GRE, and POWERPREP are registered trademarks
of Educational Testing Service (ETS) in the United States and other countries. All other
trademarks are property of their respective owners.
This ebook was issued to HUAN-LIN CHANG, order #14803770223. Unlawful distribution of this ebook is prohibited.
Customer name HUAN-LIN CHANG , Order Id: 14803770223

Navigation menu